Sunteți pe pagina 1din 167

LIVRO 6

LINGUAGENS CDIGOS E SUAS TECNOLOGIAS

A empresa moderna compete em dois mundos: um fsico (a


praa, ou marketplace) e um mundo digital de informao (o
espao mercadolgico, ou marketspace). As empresas no
devem preocupar-se com a criao de um web site vistoso,
mas sim de uma grande comunidade online e com o capital de
relacionamento. Coraes, e no olhos, so o que conta.
Dentro de uma dcada, a maioria dos produtos ser vendida
no espao mercadolgico. Uma nova fronteira de comrcio a
marketface a interface entre o marketplace e
o marketspace.
(...)
Publicidade, promoo, relaes pblicas etc. exploram
mensagens unidirecionais, de um-para-muitos e de tamanho
nico, dirigidas a consumidores sem rosto e sem poder. As
comunidades online perturbam drasticamente esse modelo.
Os consumidores com frequncia tm acesso a informaes
sobre os produtos, e o poder passa para o lado deles. So eles
que controlam as regras do mercado, no voc. Eles escolhem
o meio e a mensagem. Em vez de receber mensagens enviadas
por profissionais de relaes pblicas, eles criam a opinio
pblica online.
Os marqueteiros esto perdendo o controle, e isso muito
bom.
(Don Tapscott. O fim do marketing. INFO, So Paulo, Editora
Abril, janeiro 2011, p. 22.)

LNGUA PORTUGUESA
1. (INSPER-2012)

O que motivou o apito do juiz foi


a) a necessidade de empregar a nclise para seguir a normapadro.
b) o uso de um objeto direto no lugar de um objeto indireto.
c) a opo pelo pronome pessoal oblquo o em vez de a.
d) a obrigatoriedade da mesclise nessa construo lingustica.
e) a transgresso s regras de concordncia nominal
relacionadas ao pronome.
RESPOSTA : B
COMENTRIO: No primeiro quadro, observa-se falha
gramatical na fala da cobra que usa o pronome oblquo o em
vez de lhe. O verbo escapar transitivo indireto, exigindo
assim o pronome com essa funo, o que valida opo (B).
2.
(UNESP-2012) Instruo: A questo de nmero XX toma por
base um artigo de Don Tapscott (1947-).
O fim do marketing
A empresa vende ao consumidor com a web no mais
assim.
Com a internet se tornando onipresente, os Quatro Ps do
marketing produto, praa, preo e promoo no
funcionam mais. O paradigma era simples e unidirecional: as
empresas vendem aos consumidores. Ns criamos produtos;
fixamos preos; definimos os locais onde vend-los; e fazemos
anncios. Ns controlamos a mensagem. A internet
transforma todas essas atividades.
(...)
Os produtos agora so customizados em massa, envolvem
servios e so marcados pelo conhecimento e os gostos dos
consumidores. Por meio de comunidades online, os
consumidores hoje participam do desenvolvimento do
produto. Produtos esto se tornando experincias. Esto
mortas as velhas concepes industriais na definio e
marketing de produtos.
(...)
Graas s vendas online e nova dinmica do mercado, os
preos fixados pelo fornecedor esto sendo cada vez mais
desafiados. Hoje questionamos at o conceito de preo,
medida que os consumidores ganham acesso a ferramentas
que lhes permitem determinar quanto querem pagar. Os
consumidores vo oferecer vrios preos por um produto,
dependendo de condies especficas. Compradores e
vendedores trocam mais informaes e o preo se torna
fluido. Os mercados, e no as empresas, decidem sobre os
preos de produtos e servios.
(...)

(UNESP-2012) Ns criamos produtos; fixamos preos;


definimos os locais onde vend-los; e fazemos anncios. Ns
controlamos a mensagem.
Nas oraes que compem os dois perodos transcritos, os
termos destacados exercem a funo de
A) sujeito.
B) objeto direto.
C) objeto indireto.
D) predicativo do sujeito.
E) predicativo do objeto.
RESPOSTA: B
COMENTRIO: Todos os termos destacados exercem a funo
de objeto direto. Com efeito, ligam-se diretamente (ou seja,
sem a presena de uma preposio) a verbos transitivos
diretos cujo sentido completam: criar, fixar, definir, fazer,
controlar. A comprovao prtica que esses termos
poderiam ser substitudos pelo pronome pessoal tono os
(produtos, preos, os locais, anncios) ou a (a mensagem).
Verifica-se tambm que, se as oraes em questo fossem
colocadas na voz passiva, todos esses termos passariam da
funo de objeto direto para a de sujeito da orao.
Leia o texto para responder s questes de nmeros 3 a 4.
O padeiro
Levanto cedo, fao a higiene pessoal, ponho a chaleira no fogo
para fazer caf e abro a porta do apartamento mas no
encontro o po costumeiro. No mesmo instante me lembro de
ter lido alguma coisa nos jornais da vspera sobre a greve do
po dormido. De resto no bem uma greve, um lockout,
greve dos patres, que suspenderam o trabalho noturno;

acham que obrigando o povo a tomar seu caf da manh com


po dormido conseguiro no sei bem o que do governo.
Est bem. Tomo meu caf com po dormido, que no to
ruim assim. E enquanto tomo caf vou me lembrando de um
homem modesto que conheci antigamente. Quando vinha
deixar po porta do apartamento ele apertava a campainha,
mas, para no incomodar os moradores, avisava gritando:
No ningum, o padeiro!
Interroguei-o uma vez: como tivera a ideia de gritar aquilo?
Ele abriu um sorriso largo. Explicou que aprendera aquilo de
ouvido. Muitas vezes lhe acontecera bater a campainha de
uma casa e ser atendido por uma empregada ou por uma
outra pessoa qualquer, e ouvir uma voz que vinha l de dentro
perguntando quem era; e ouvir a pessoa que o atendera dizer
para dentro: no ningum, no senhora, o padeiro.
Assim ficara sabendo que no era ningum...
Ele me contou isso sem mgoa nenhuma, e se despediu ainda
sorrindo. Eu no quis det-lo para explicar que estava falando
com um colega, ainda menos importante. Naquele tempo eu
tambm, como os padeiros, fazia trabalho noturno.
Era pela madrugada que deixava a redao do jornal, quase
sempre depois de uma passagem pela oficina - e muitas vezes
saa j levando na mo um dos exemplares rodados, o jornal
ainda quentinho da mquina, como po sado do forno.
Ah, eu era rapaz, eu era rapaz naquele tempo! E s vezes me
julgava importante porque no jornal que levava para casa,
alm de reportagens ou notas que eu escrevera sem assinar, ia
uma crnica ou artigo com o meu nome. O jornal e o po
estavam bem cedinho na porta de cada lar; e dentro do meu
corao eu recebi uma lio daquele homem entre todos til e
entre todos alegre; no ningum, o padeiro!.
E assoviava pelas escadas.
(Rubem Braga, Ai de ti, Copacabana. Rio de Janeiro: Editora do
Autor, 1960. Adaptado)
3.

(D) afirmao, pois, ao sorrir, o padeiro mostrava no se


magoar ao ouvir que no era ningum.
(E) causa, pois aprendera que no era ningum com as
pessoas que o atendiam nos apartamentos.
RESPOSTA: A
COMENTRIO: A expresso sem mgica indica circunstncia
de modo e reproduz o sentimento do padeiro ao explicar a
razo do alerta que fazia sempre que tocava a campainha para
anunciar a entrega do po.
5.

RESPOSTA: A
COMENTRIO: Na opo (A), o termo verbal avisaram
apresenta transitividade direta e indireta, pois a orao
subordinada substantiva de que era necessrio exercer a
funo de objeto indireto e (n)a de objeto direto. Nas
demais opes os pronomes deveriam ser substitudos por
me, a, nos e no, sequencialmente, para se adaptarem
s regras da gramtica normativa.
(IFPE-2012)
De um jogador brasileiro a um tcnico espanhol
Joo Cabral de Melo Neto
No a bola alguma carta
que se leva de casa em casa:
antes telegrama que vai
de onde o atiram ao onde cai.
Parado, o brasileiro a faz
ir onde h-de, sem leva e traz;
com aritmticas de circo
ele a faz ir onde preciso;
em telegrama, que sem tempo
ele a faz ir ao mais extremo.
No corre: ele sabe que a bola,
Telegrama, mais que corre voa.
(Disponvel
em:
<http://www.revista.agulha.nom.br/futebol.html#jogador>
Acesso em: 12 out. 2011.)

(IFSP-2012) Considere o trecho em que a expresso em


destaque exerce a funo de agente da passiva.
Muitas vezes lhe acontecera bater a campainha de uma casa e
ser atendido por uma empregada ou por uma outra pessoa
qualquer, e ouvir uma voz...
Assinale a alternativa em que o trecho em destaque exerce a
mesma funo sinttica.
(A) Esta a avenida por onde passaro as escolas de samba.
(B) Ele fez tudo isso por voc, a quem admira muito.
(C) Incomodou-o por semanas um problema que parecia sem
soluo.
(D) O vestido, que havia sido feito por um renomado estilista,
impressionou a todos.
(E) Ela sentiu-se arrependida por ter respondido de forma
indelicada ao funcionrio.

6.

RESPOSTA: D
COMENTRIO: apenas em C o segmento sublinhado agente
da passiva, pois indica o executor da ao sofrida pelo sujeito
da orao.
4.

(IFSP-2012) Assinale a alternativa em que o pronome em


destaque est empregado de acordo com a norma padro.
(A) Avisaram-na de que era necessrio reconhecer firma dos
documentos.
(B) Pediu para mim enviar-lhe as faturas do ltimo ms.
(C) Como gostou da bolsa, comprou ela sem pensar duas
vezes.
(D) Eu e minha irm se encontraremos com amigos no sbado.
(E) Recrutaram-o para realizar tarefas pouco prazerosas.

(IFPE-2012) Quanto aos aspectos morfossintticos do texto,


assinale a alternativa correta.
a) O sujeito das duas primeiras estrofes indeterminado,
como se verifica pelos verbos se leva e atiram.
b) O predicado em No a bola alguma carta e antes
telegrama... verbal, pois os verbos indicam o estado da
bola.
c) O sujeito simples brasileiro da terceira estrofe retomado
nas demais estrofes pelo pronome ele.
d) O predicado da orao Ele a faz ir, na quarta e quinta
estrofes, verbo-nominal, pois indica ao e descreve a bola.
e) O substantivo telegrama, no ltimo verso do poema, um
adjunto adnominal de bola.

(IFSP-2012) Em Ele me contou isso sem mgoa nenhuma, e


se despediu ainda sorrindo. o trecho em destaque expressa
a circunstncia de
(A) modo, pois revela a maneira como o padeiro se comportou
diante da situao.
(B) negao, pois o padeiro no se ofendia com o tratamento
recebido dos moradores.
(C) meio, pois o cronista deseja enfatizar a modstia e a
inteligncia do padeiro.

RESPOSTA: C
COMENTRIO: A penas correto o que se afirma em (C), pois
as demais apresentam as seguintes incorrees:
2

Em (A) o sujeito da orao subordinada adjetiva que se leva


de casa em casa o pronome relativo que, relacionando ao
antecedente carta, por isso, simples;
Em (B) os predicados das frases mencionadas so ambos
nominais, pois apresentam como ncleo um predicativo do
sujeito (:alguma carta e telegrama, respectivamente) e
verbo de ligao ();
Em (D) o predicado da orao Ele a faz rir verbal, pois
apresenta como ncleo um verbo nacional, isto , verbo que
indica ao;
Em (E) o substantivo telegrama exerce a funo de aposto.

RESPOSTA: D
COMENTRIO: As opes (A), (B), (C) e (E) so incorretas, pois:
Em (A) o verbo haver no sentido de tempo decorrido configura
orao sem sujeito;.
Em (B) o termo descupada exerce a funo de predicativo do
objeto direto;
Em (C) o segmento da orao um defunto de nada, resposta
pergunta anterior A quem estais carregando, irmo das almas,
embrulhado nessa rede?, exerce a funo de objetos direto
preposicionado;
Em (E) na expresso Ter um hectare de terra constitui um
adjunto adnuminal.

(IFPE-2012) O Texto serve de base para a questo 7.

8.

Voc ler no texto um trecho do poema Morte e vida


Severina, de Joo Cabral de Melo Neto, importante poeta
pernambucano da Gerao de 45 do Modernismo brasileiro.
No excerto, um retirante chamado Severino, protagonista da
obra, encontra dois homens que esto carregando um defunto
numa rede.

A rua do Caloca
Bendito o bairro em que os meninos ainda podem jogar
futebol pelas caladas. Ipanema, as ruas amenas de Ipanema
esto sempre cheias de meninos a chutar bolas. Hoje de
manh, mesmo, passei por dois garotinhos, um de seis anos,
outro de trs, no mximo: o maior ensinava, pacientemente, o
menor a chutar com o peito do p e, ontem, a turma da Rua
Baro de Jaguaribe enfrentou o time da Rua Redentor.
Quando estou folgado, subo e deso as ruas da vizinhana,
olhando os meninos no futebol: so vinte, trinta de cada lado,
todos garotinhos abaixo de dez anos, ardendo na pelada que
nunca tem hora para acabar; no h muito rigor contra a
violncia e s uma regra respeitada a mo na bola. Tocou o
dedo na bola, falta, quem cobra sou eu e forma-se um
bolinho em volta da bola, parece cobrana de pnalti no
Maracan.
De quando em quando, um automvel interrompe a partida
de futebol, mas invariavelmente os motoristas tm o carinho
de reduzir a marcha a uma velocidade que no ponha em risco
a vida das crianas e que, por outro lado, lhes permita
desfrutar, ainda que como espectadores, das emoes da
pelada.
Das ruas, a mais encantadora, sem desmerecer as outras, a
Redentor, que eu sado como a rua do time do Caloca. Pena
que, numa das esquinas, more uma senhora estranha ao clima
espiritual deste bairro, onde as meninas brincam de cantigas
de roda e os meninos de jogar futebol.
Se eu fosse alguma coisa neste pas, j teria corrido de
Ipanema aquela mulher rabugenta, que, na presena de vinte
meninos, furou a bola deles com uma tesoura e, depois,
cortou em pedacinhos. S porque uma rebatida imprecisa do
Diguinho jogou a bola contra a casa dela.
Era uma bonita bola, rosada, seleo de ouro, que os meninos
tinham comprado em uma vaquinha por novecentos cruzeiros.
(Nogueira, Armando. Na grande rea. Rio de Janeiro: Bloch
Editores, 1966. Adaptado).

Texto
A quem estais carregando, irmos das almas, embrulhado
nessa rede? Dizei que eu saiba.
A um defunto de nada, irmo das almas, que h muitas
horas viaja sua morada.
E sabeis quem era ele, irmos das almas, sabeis como ele se
chama ou se chamava?
Severino Lavrador, irmo das almas, Severino Lavrador, mas
j no lavra.
E de onde que o estais trazendo, irmos das almas, onde foi
que comeou vossa jornada?
Onde a Caatinga mais seca, irmo das almas, onde uma
terra que no d nem planta brava.
E foi morrida essa morte, irmos das almas, essa foi morte
morrida ou foi matada?
(...)
E quem foi que o emboscou, irmos das almas, quem
contra ele soltou essa ave-bala?
Ali difcil dizer, irmo das almas, sempre h uma bala
voando desocupada.
E o que havia ele feito irmos das almas, e o que havia ele
feito contra a tal pssara?
Ter um hectare de terra, irmo das almas, de pedra e areia
lavada que cultivava.
Mas que roas que ele tinha, irmos das almas que podia
ele plantar na pedra avara?
Nos magros lbios de areia, irmo das almas, os intervalos
das pedras, plantava palha.
7.

(G1-CPS-2011) Em 1966, Armando Nogueira publicou o livro


em que aparece a crnica que voc ler a seguir.

(IFPE-2012) Em relao sintaxe dos perodos retirados no


texto, correto afirmar que:

Assinale a alternativa em que as duas expresses em destaque


so adjuntos adverbiais de lugar.
(A) ... as ruas amenas de Ipanema esto sempre cheias de
meninos...
... parece cobrana de pnalti no Maracan.
(B) ... os meninos ainda podem jogar futebol pelas caladas.
... forma-se um bolinho em volta da bola...
(C) Pena que, numa das esquinas, more uma senhora...
... ontem, a turma da rua Baro de Jaguaribe enfrentou...
(D) se eu fosse alguma coisa neste pas...
... que eu sado como a rua do time do Caloca.
(E) Quando estou folgado, subo e deso as ruas da
vizinhana...
... j teria corrido de Ipanema aquela mulher rabugenta...

a) Em h muitas horas viaja sua morada, no segundo verso,


o sujeito do verbo haver muitas horas.
b) Em h uma bala voando desocupada, o termo destacado
funciona como um adjunto adverbial de modo.
c) um defunto de nada sujeito do verbo haver que aparece
no mesmo verso.
d) No quinto verso, o possessivo vossa pode estar-se
referindo apenas ao irmo das almas, condutor do defunto,
ou a ambos, ou seja, ao condutor e ao defunto.
e) Na expresso Ter um hectare de terra, de terra constitui
um objeto indireto do verbo ter, tendo em vista ter sido
exigida uma preposio.

RESPOSTA: B
COMENTRIO: Apenas em b) os termos grifados so adjuntos
adverbiais de lugar, j que designam o espao onde ocorrem
as aes de jogar futebol e formar bolinho. Nas opes a),
c), d) e e), apresentam-se como sujeito/adjunto adverbial de
lugar, adjunto adverbial de lugar/adjunto adnominal, adjunto
adverbial de lugar/predicativo do objeto, objeto direto/
adjunto adverbial de lugar, respectivamente
(UNIFESP-2011) Instruo: A questo de nmero 9 toma por
base o fragmento seguinte.
As provocaes no recreio eram frequentes, oriundas do
enfado; irritadios todos como feridas; os inspetores a cada
passo precisavam intervir em conflitos; as importunaes
andavam em busca das suscetibilidades; as suscetibilidades a
procurar a sarna das importunaes. Viam de joelhos o Franco,
puxavam-lhe os cabelos. Viam Rmulo passar, lanavam-lhe o
apelido:mestre-cuca!
Esta provocao era, alm de tudo, inverdade. Cozinheiro,
Rmulo! S porque lembrava culinria, com a carnosidade
bamba, fofa da dos pasteles, ou porque era gordo das
enxndias enganadoras dos fregistas, dissoluo mrbida de
sardinha e azeite, sob os aspectos de mais volumosa sade?
(...)
Rmulo era antipatizado. Para que o no manifestassem
excessivamente, fazia-se temer pela brutalidade. Ao mais
insignificante gracejo de um pequeno, atirava contra o infeliz
toda a corpulncia das infiltraes de gordura solta,
desmoronava-se em socos. Dos mais fortes vingava-se,
resmungando intrepidamente.
Para desesper-lo, aproveitavam-se os menores do escuro.
Rmulo, no meio, ficava tonto, esbravejando juras de morte,
mostrando o punho. Em geral procurava reconhecer algum dos
impertinentes e o marcava para a vindita. Vindita inexorvel.
No decorrer enfadonho das ltimas semanas, foi Rmulo
escolhido, principalmente, para expiatrio do desfastio.
Mestre-cuca! Via-se apregoado por vozes fantsticas, sadas
da terra; mestre-cuca! Por vozes do espao rouquenhas ou
esganiadas. Sentava-se acabrunhado, vendo se se lembrava
de haver tratado panelas algum dia na vida; a unanimidade
impressionava. Mais frequentemente, entregava-se a acessos
de raiva. Arremetia bufando, espumando, olhos fechados,
punhos para trs, contra os grupos. Os rapazes corriam a rir,
abrindo caminho, deixando rolar adiante aquela ambulncia
danada de elefantase.
(Raul Pompeia. O Ateneu.)
9.

10. (UNIFESP-2011)
De tudo que nego torto
Do mangue e do cais do porto
Ela j foi namorada
O seu corpo dos errantes
Dos cegos, dos retirantes
de quem no tem mais nada
D-se assim desde menina
Na garagem, na cantina
Atrs do tanque, no mato
a rainha dos detentos
Das loucas, dos lazarentos
Dos moleques do internato
E tambm vai amide
Coos velhinhos sem sade
E as vivas sem porvir
Ela um poo de bondade
E por isso que a cidade
Vive sempre a repetir
Joga pedra na Geni
Joga pedra na Geni
Ela feita pra apanhar
Ela boa de cuspir
Ela d pra qualquer um
Maldita Geni
(Chico Buarque. Geni e o zepelim.)
Indique a alternativa que apresenta a funo sinttica do verso
De tudo que nego torto.
(A) Adjunto adverbial de modo.
(B) Objeto indireto.
(C) Predicativo do sujeito.
(D) Adjunto adnominal.
(E) Complemento nominal.
RESPOSTA: E
COMENTRIO: O excerto faz parte do perodo De tudo que
nego torto/Do mangue e do cais do porto/Ela j foi namorada
a que corresponderia, na ordem direta: ela j foi namorada de
tudo que nego torto do mangue e do cais do porto. Assim,
De tudo que nego torto complemento do substantivo
namorada, regido de preposio, portanto, complemento
nominal, como de enuncia em (E).
(IFCE-2011)
NASCE UM ESCRITOR
O primeiro dever passado pelo novo professor de portugus
foi uma descrio tendo o mar como tema. A classe inspirou,
toda ela, nos encapelados mares de Cames, aqueles nunca
dantes navegados. O episdio do Adamastor foi reescrito pela
meninada. Prisioneiro no internato, eu vivia na saudade das
praias do Pontal onde conhecera a liberdade e o sonho. O mar
de Ilhus foi o tema de minha descrio.
Padre Cabral levara os deveres para corrigir em sua cela. Na
aula seguinte, entre risonho e solene, anunciou a existncia de
uma vocao autntica de escritor naquela sala de aula. Pediu
que escutassem com ateno o dever que ia ler. Tinha certeza,
afirmou, que o autor daquela pgina seria no futuro um
escritor conhecido. No regateou elogios. Eu acabara de
completar onze anos.
Passei a ser uma personalidade, segundo os cnones do
colgio, ao lado dos futebolistas, dos campees de
1
matemtica e de religio, dos que obtinham medalhas. Fui
2
admitido numa espcie de Crculo Literrio onde brilhavam
3
alunos mais velhos. Nem assim deixei de me sentir

(UNIFESP-2011) Tendo em vista a funo sinttica da palavra


grifada no fragmento Para que o no manifestassem
excessivamente, fazia-se temer pela brutalidade, assinale a
alternativa em que o termo sublinhado exerce a mesma
funo:
(A) Dos mais fortes vingava-se, resmungando intrepidamente.
(B) Para desesper-lo, aproveitavam-se os menores do escuro.
(C) Via-se apregoado por vozes fantsticas, sadas da terra.
(D) Mais frequentemente, entregava-se a acessos de raiva.
(E) Viam de joelhos o Franco, puxavam-lhe os cabelos.
RESPOSTA: E
COMENTRIO: No trecho Para que o no manifestassem
excessivamente, fazia-se temer pela brutalidade, o termo
sublinhado desempenhar a funo de objeto direto, o que
acontece tambm na frase Viram de joelhos o Franco,
puxavam-lhe os cabelos. Em (A), (C) e (D), so objetos
indireto, agente da passiva e objeto indireto, respectivamente,
e em (B) sujeito.
4

prisioneiro, sensao permanente durante os dois anos em


que estudei no colgio dos jesutas.
4
5
Houve, porm, sensvel mudana na limitada vida do aluno
interno: o padre Cabral tomou-me sob sua proteo e colocou
em minhas mos livros de sua estante. Primeiro "As Viagens
de Gulliver", depois clssicos portugueses, tradues de
ficcionistas ingleses e franceses. Data dessa poca minha
paixo por Charles Dickens. Demoraria ainda a conhecer Mark
Twain: o norte-americano no figurava entre os prediletos do
padre Cabral.
Recordo com carinho a figura do jesuta portugus erudito e
amvel. Menos por me haver anunciado escritor, sobretudo
por me haver dado o amor aos livros, por me haver revelado o
mundo da criao literria. Ajudou-me a suportar aqueles dois
anos de internato, a fazer mais leve a minha priso, minha
primeira priso.

Assinale a alternativa em que o verbo destacado tem como


sujeito aquele apresentado entre colchetes.
(A) Logo depois transferiu-se para o trapiche o depsito dos
objetos... [os objetos]
(B) ... o depsito dos objetos que o trabalho do dia lhes
proporcionava. [o depsito dos objetos]
(C) Estranhas coisas entraram ento para o trapiche.
[estranhas coisas]
(D) ... indiferentes ao vento que circundava o casaro
uivando... [o casaro]
(E) ... com os ouvidos presos s canes que vinham das
embarcaes. . . [as embarcaes]
RESPOSTA: C
COMENTRIO: Estranhas coisas praticam a ao verbal de
entrar para o trapiche, por isso o verbo estar conjugado no
plural, concordando com o seu sujeito.

AMADO, Jorge. O menino Grapina. Rio de Janeiro. Record.


1987. p. 117-20.
11. (IFCE-2011) Pondo-se a expresso ... sensvel mudana...
(ref.5) no plural, a forma verbal Houve (ref.4)
A) deve ser trocada por houveram.
B) no deve sofrer alterao.
C) pode ser trocada por existiu.
D) pode ser substituda por esto havendo.
E) pode ser trocada por vo haver.

14. (FGV) Observe a seguinte frase:


Recorrendo a elas, arrisco-me a usar expresses tcnicas,
desconhecidas do pblico, e a ser tido por pedante.
Das alternativas abaixo, assinale aquela em que a palavra
sublinhada exera a mesma funo sinttica de pedante,
dessa frase.
A) As estaes tinham passado rpido, sem que tivesse sido
possvel v-las direito.
B) Fui julgado culpado, embora no houvesse provas decisivas
a respeito do crime.
C) Ele era difcil de convencer, mas concordou quando a
quantia foi oferecida.
D) Caminhou depressa por entre os coqueiros.
E) Ele passeou demasiado ontem; hoje, doem-lhe as pernas.
Vai ser obrigado a deitar-se mais cedo.
RESPOSTA: B
COMENTRIO: O termo pedante, na orao, tem funo de
predicativo do sujeito. O nico item que traz predicativo do
sujeito o item B.

RESPOSTA: B
COMENTRIO: o verbo haver, no sentido de existir,
impessoal, configurando uma orao sem sujeito. Por isso,
deve manter-se na terceira pessoa do singular, mesmo que
acompanhados por outros verbos, o que contraria as opes
a), d) e e). Poderia ser trocado por existiram, no o que
porpe a opo c). Assim, apenas b) correta.
12. (IFCE-2011) A expresso ... alunos mais velhos. (ref.3) exerce
a funo de
A) sujeito da forma verbal brilhavam (ref.2).
B) objeto indireto.
C) agente da ao expressa pela forma verbal obtinham
(ref.1).
D) agente da passiva.
E) objeto direto.
RESPOSTA: A
COMENTRIO: Na orao onde brilhavam alunos mais
velhos, a expresso alunos mais velhos exerce a funo
sinttica de sujeito (simples).

15. (OBJETIVO-SP) Leia o fragmento de texto:


"Baleia queria dormir. Acordaria feliz, num mundo cheio de
pres. E lamberia as mos de Fabiano, um Fabiano enorme. As
crianas se espojariam com ela, rolariam com ela num ptio
enorme, num chiqueiro enorme. O mundo ficaria cheio de
pres, gordos, enormes. "
("Baleia" em: Ramos, G. "Vidas Secas". Rio de Janeiro: Jos
Olympio, 1947.)
Em Acordaria feliz, num mundo cheio de pres., temos:
(A) Predicado verbal com verbo intransitivo.
(B) Predicado nominal com predicativo do sujeito.
(C) Predicado verbal com verbo transitivo e predicativo do
sujeito.
(D) Predicado verbo-nominal com verbo intransitivo e
predicativo do sujeito.
(E) Predicado verbo-nominal com verbo transitivo e
predicativo do sujeito.

13. (FATEC-2010) Leia o texto seguinte e responda


Logo depois transferiu-se para o trapiche [local destinado
guarda de mercadorias para importao ou exportao] o
depsito dos objetos que o trabalho do dia lhes
proporcionava. Estranhas coisas entraram ento para o
trapiche. No mais estranhas, porm, que aqueles meninos,
moleques de todas as cores e de idades, as mais variadas,
desde os 9 aos 16 anos, que noite se estendiam pelo
assoalho e por debaixo da ponte e dormiam, indiferentes ao
vento que circundava o casaro uivando, indiferentes chuva
que muitas vezes os lavava, mas com os olhos puxados para as
luzes dos navios, com os ouvidos presos s canes que
vinham das embarcaes. . .
(AMADO, Jorge. O trapiche. Capites de Areia. So Paulo:
Livraria Martins Ed., 1937. Adaptado.)

RESPOSTA: D
COMENTRIO: Predicado verbo-nominal, cujos ncleos so o
verbo intransitivo acordaria e o predicativo do sujeito
feliz.
INTERPRETAO DE TEXTO

couve, jantar encerrado por um prato fundo de canjica com


torres de acar, Nh Tom saboreou o caf forte e se
estendeu na rede. A mo direita sob a cabea, guisa de
travesseiro, o indefectvel cigarro de palha entre as pontas do
indicador e do polegar, envernizados pela fumaa, de unhas
encanoadas e longas, ficou-se de pana para o ar, modorrento,
a olhar para as ripas do telhado. Quem come e no deita, a
comida no aproveita, pensava Nh Tom E ps-se a
cochilar. A sua modorra durou pouco: Tia Policena, ao passar
pela sala, bradou assombrada:
h! Sinh! Vai drumi agora? No! Num presta D
pisadra e pde morr de ataque de cabea!
Depois do armoo num far m mais despois da janta?!
Cornlio Pires. Conversas ao p do fogo. So Paulo: Imprensa
Oficial do Estado de So Paulo, 1987.

Compare os textos I e II a seguir, que tratam de aspectos


ligados a variedades da lngua portuguesa no mundo e
no Brasil.
Texto I
Acompanhando os navegadores, colonizadores e comerciantes
portugueses em todas as suas incrveis viagens, a partir do
sculo XV, o portugus se transformou na lngua de um
imprio. Nesse processo, entrou em contato forado, o mais
das vezes; amigvel, em alguns casos com as mais diversas
lnguas, passando por processos de variao e de mudana
lingustica. Assim, contar a histria do portugus do Brasil
mergulhar na sua histria colonial e de pas independente, j
que as lnguas no so mecanismos desgarrados dos povos
que as utilizam. Nesse cenrio, so muitos os aspectos da
estrutura lingustica que no s expressam a diferena entre
Portugal e Brasil como tambm definem, no Brasil, diferenas
regionais e sociais. PAGOTTO, E. P. Lnguas do Brasil.
Disponvel em: http://cienciaecultura.bvs.br Acesso em 5 jul.
2009 (adaptado)
Texto II
Barbarismo vcio que se comete na escritura de cada uma
das partes da construo ou na pronunciao. E em nenhuma
parte da Terra se comete mais essa figura da pronunciao
que nestes reinos, por causa das muitas naes que
trouxemos ao jugo do nosso servio. Porque bem como os
Gregos e Romanos haviam por brbaras todas as outras
naes estranhas a eles, por no poderem formar sua
linguagem, assim ns podemos dizer que as naes de frica,
Guin, Asia, Brasil barbarizam quando querem imitar a nossa.
BARROS, J. Gramtica da lngua portuguesa. Porto: Porto
Editora, 1957 (adaptado).
16. Questo 1: Os textos abordam o contato da lngua portuguesa
com outras lnguas e processos de variao e de mudana
decorridos desse contato. Da comparao entre os textos,
conclui-se que a posio de Joo de Barros (Texto II), em
relao aos usos sociais da linguagem, revela
a) atitude crtica do autor quanto gramtica que as naes a
servio de Portugal possuam e, ao mesmo tempo, de
benevolncia quanto ao conhecimento que os povos tinham
de suas lnguas.
b) atitude preconceituosa relativa a vcios culturais das naes
sob domnio portugus, dado o interesse dos falantes dessa
lnguas em copiar a lngua do imprio, o que implicou a
falncia do idioma falado em Portugal.
c) o desejo de conservar, em Portugal, as estruturas da
variante padro da lngua grega em oposio s
consideradas brbaras , em vista da necessidade de
preservao do padro de correo dessa lngua poca.
d) adeso concepo de lngua como entidade homognea e
invarivel, e negao da ideia de que a lngua portuguesa
pertence a outros povos.
e) atitude crtica, que se estende prpria lngua portuguesa,
por se tratar de sistema que no disporia de elementos
necessrios para a plena insero sociocultural de falantes no
nativos do portugus.
RESPOSTA: D
COMENTRIO: A viso preconceituosa de Joo de Barros,
compreensvel e talvez inevitvel na poca, considera a lngua
portuguesa como propriedade dos portugueses e como
entidade homognea, pois rejeita suas variantes.

17. Questo 2: Nesse trecho, extrado de texto publicado


originalmente em 1921, o narrador
a) apresenta, sem explicitar juzos de valor, costume da poca,
descrevendo os pratos servidos no jantar e a atitude de Nh
Tom e de Tia Policena.
b) desvaloriza a norma culta da lngua porque incorpora
narrativa usos prprios da linguagem regional das
personagens.
c) condena os hbitos descritos, dando voz a Tia Policena, que
tenta impedir Nh Tom de deitar-se aps as refeies.
d) utiliza a diversidade sociocultural e lingstica para
demonstrar seu desrespeito s populaes das zonas rurais do
incio do sculo XX.
e) manifesta preconceito em relao a Tia Policena ao
transcrever a fala dela com os erros prprios da regio.
RESPOSTA: A
COMENTRIO: O narrador do texto de Cornlio Pires
inteiramente isento em relao aos hbitos que descreve e
linguagem que reproduz.

Texto:

18. Questo 3: correto armar que a charge visa


A) apoiar a atitude dos alunos e propor a liberao geral da
frequncia s aulas.
B) enaltecer a escola brasileira e homenagear o trabalho
docente.
C) indicar a deagrao de uma greve e incentivar a adeso a
ela.
D) recriminar os alunos e declarar apoio poltica educacional.
E) criticar a situao atual do ensino e denunciar a evaso
escolar.

Texto: Depois de um bom jantar: feijo com carne-seca, orelha


de porco e couve com angu, arroz-mole engordurado, carne
de vento assada no espeto, torresmo enxuto de toicinho da
barriga, viradinho de milho verde e um prato de caldo de

RESPOSTA: E
COMENTRIO: A denncia situao atual do ensino
depreendida pela gura caricata do professor (olhos
6

arregalados, dentes mostra), com uma postura no


amistosa, e pela prpria aluso evaso escolar, insinuada por
um dos sentidos que a palavra sujeito pode assumir no
contexto humorstico (sujeito como sinnimo de pessoa,
indivduo, aluno).

submetidos cirurgia baritrica


exibiram melhora na
capacidade de armazenar informaes 12 semanas depois da
operao. Estamos acompanhando esses indivduos para
checar se a performance continuar a mesma um ano e dois
anos depois, disse ISTO John Gunstad, lder da pesquisa.
Na sua experincia clnica, o mdico Roberto Rizzi, membro da
Sociedade Brasileira de Cirurgia Baritrica e Metablica, j
havia observado a associao. Percebe-se que aps uma
grande perda de peso h melhoria no poder de se lembrar das
coisas, diz. (Isto, 22.06.2011. Adaptado.)

Texto: Leia os versos de Ceclia Meireles, extrados do poema


Epigrama no 8.
Encostei-me a ti, sabendo bem que eras somente onda.
Sabendo bem que eras nuvem, depus a minha vida em ti.
Como sabia bem tudo isso, e dei-me ao teu destino frgil,
quei sem poder chorar, quando ca.
19. Questo 4: O eu lrico reconhece que a pessoa em quem
deps sua vida representava
A) uma relao incerta, por isso os desenganos vividos seriam
inevitveis.
B) um sentimento intenso, por isso tinha certeza de que no
sofreria.
C) um caso de amor passageiro, por isso se sentia enganado.
D) uma angstia inevitvel, por isso seria melhor aquele amor.
E) uma opo equivocada, por isso sempre teve medo de
amar.
RESPOSTA: A
COMENTRIO: O poema de Ceclia Meireles trata de uma
relao sentimental. A instabilidade dessa relao expressa
por imagens como onda, nuvem e frgil. A conscincia
que o eu lrico possui dessa instabilidade, bem como do
sofrimento advindo dela, evidenciada pelas exes do verbo
saber: sabendo e sabia.

21. Questo 6: Um ttulo adequado ao texto


A) Emagrecer faz bem memria.
B) Mdico brasileiro contesta estudo americano.
C) Gordura em excesso potencializa a memria.
D) Memria ca inalterada at dois anos aps cirurgia.
E) Cirurgia traz perda de peso e de memria.
RESPOSTA: A
COMENTRIO: Os trechos pacientes submetidos cirurgia
baritrica [destinada ao emagrecimento] exibiram melhora na
capacidade de armazenar informaes e aps uma grande
perda de peso h melhoria no poder de se lembrar das coisas
evidenciam que emagrecer faz bem memria ttulo
adequado, j que sintetiza a ideia central do texto.
LITERATURA
22. (ITA-2012) A questo refere-se ao poema abaixo, de Oswald
de Andrade, que integra o romance Memrias sentimentais
de Joo Miramar.
Verbo crackar
Eu empobreo de repente
Tu enriqueces por minha causa
Ele azula para o serto
Ns entramos em concordata
Vs protestais por preferncia
Eles escafedem a massa

Texto:
Todo estudante sabe que atualidade tambm questo de
vestibular. Para garantir um bom desempenho, que atento a
temas que se repetem durante alguns dias em jornais, sites ou
canais de TV. Quando estiver se informando, relacione os
acontecimentos aos contedos aprendidos em sala de aula. E
cuidado especial com meio ambiente, sempre em alta nas
provas. (Veja, 18.05.2011.)

S pirata
Sede trouxa
Abrindo o pala
Pessoal sarado.

20. Questo 5: A inteno explcita do texto, considerada a


interlocuo nele de nida,
A) descrever que o prprio vestibular um tema de
atualidade.
B) orientar os vestibulandos sobre como estudar atualidade.
C) mostrar aos professores que a tv importante para se
ensinar atualidade.
D) enfatizar as divergncias entre informaes da mdia e da
escola.
E) indicar aos estudantes que meio ambiente um tema j
desgastado.

Oxal que eu tivesse sabido que esse verbo era irregular.


Anular: fugir, escapar
Abrir o pala: retirar-se furtivamente, escapar
Sarado: valento, abusado
Com base no poema, a nica opo que NO contempla a
proposta modernista
A) o escape da viso lrico-amorosa.
B) a apresentao de problemas existenciais.
C) a inovao da linguagem literria.
D) a apresentao de problemas existenciais.
E) a ironia ao sistema econmico-social.

RESPOSTA: B
COMENTRIO: A partir da armao de que atualidade
tambm questo de vestibular, o texto orienta o
aluno/vestibulando (interlocutor implcito) sobre como
proceder para adquirir conhecimentos de atualidades e obter
um desempenho satisfatrio nas provas. A orientao ca
explcita por meio das formas imperativas empregadas: que
atento, relacione os acontecimentos e (tome) cuidado.

RESPOSTA: B
COMENTRIO: O poema crackar de Oswald de Andrade
exemplifica a proposta modernista de romper com o
academicismo e adotar recursos acessos a frmula
preestabelecidas pelos cnones das estticas anteriores.
Rejeitando as propostas do Parnasianismo, a poesia revela-se
no verso livre e branco para tematizar o cotidiano vulgar,
como a instabilidade evonmica advinda das variaes na
Bolsa da cotao do caf. O verbo crackar, classificando

Texto:
O que o excesso de gordura tem a ver com problemas de
memria? Tudo, segundo estudos recentes. O ltimo foi feito
na Kent State University (EUA) e mostrou que pacientes
7

pelo eu lrico de irregular, estabelece paralelo com a poesia


moderna, avessa a idealizaes amorosas e a tamticas
metafsicas ou existenciais para transcrever, em linguagem
coloquial e muitas vezes de forma irnica, os problemas do dia
a dia.

I. Os versos evidenciam a ltima atitude tomada pelo poeta


objetivando conquistar a moa.
II. A forma mato ela, embora condenada pela norma culta,
no deve ser considerada errada, pois legitimada pelo
registro coloquial predominante no texto.
III. O humor reside no emprego do termo efeito, que agora
no se refere apenas indiferena da jovem.
(so) cor reta(s)
a) apenas I.
b) apenas I e II.
c) apenas II e III.
d) I, II e III.
e) apenas III.

(INSPER-2011) Utilize o texto abaixo para responder aos testes


2 e 3.
Dois anncios
Rond de Efeito para todas as combinaes possveis
Olhei para ela com toda a fora,
Disse que ela era boa,
Que ela era gostosa,
Que ela era bonita pra burro:
No fez efeito.
Virei pirata:
Dei em cima de todas as maneiras,
Utilizei o bonde, o automvel, o passeio a p,
Falei de macumba, ofereci p...
toa: no fez efeito.
Ento banquei o sentimental:
Fiquei com olheiras,
Ajoelhei,
Chorei,
Me rasguei todo,
Fiz versinhos,
Cantei as modinhas mais tristes do repertrio do Nozinho.
Escrevi cartinhas e pra acertar a mo, li Elvira a Morta Virgem
(Romance primoroso e por tal forma comovente que ningum
pode l-lo sem derramar copiosas lgrimas...)
Perdi meu tempo: no fez efeito.
Meu Deus que mulher durinha!
Foi um buraco na minha vida.
Mas eu mato ela na cabea:
Vou lhe mandar uma caixinha de Minorativas,
Pastilhas purgativas:
impossvel que no faa efeito.
Manuel Bandeira. Estrela da vida inteira.

RESPOSTA: C
COMENTRIO: A primeira inferncia falsa, pois os versos da
ltima estrofe, especialmente em "Vou lhe mandar uma
caixinha de Minorativas", evidenciam que a atitude ainda no
foi tomada.
A inferncia II correta, pois a norma culta prope a forma
"mato-a", embora "mato ela" seja variante que concorda com
o registro predominante no poema. A terceira inferncia
tambm correta, pois, na ltima estrofe, "efeito" refere-se s
pastilhas purgativas, causando humor.
25. A Propsito da exposio de Malfatti
H duas espcies de artistas. Uma composta dos que veem
normalmente as coisas e em consequncia disso fazem arte
pura, guardando os eternos ritmos da vida (...). A outra
espcie formada pelos que veem anormalmente a natureza,
e interpretam-na luz de teorias efmeras, sob a sugesto
estrbica de escolas rebeldes, surgidas c e l como furnculos
da cultura excessiva. So produtos do cansao e do sadismo de
todos os perodos de decadncia: so frutos de fins de
estao, bichados ao nascedouro.
(...) Se vssemos na Sra. Malfatti apenas "uma moa que
pinta", como h centenas por a, sem denunciar centelha de
talento, calar-nos-amos, ou talvez lhe dssemos meia dzia
desses adjetivos "bombons", que a crtica aucarada tem
sempre mo em se tratando de moas.

23. (INSPER-2011) Sobre a primeira e segunda estrofes, s no


correto afirmar que

A artista, criticada por ____________________, em texto


conhecido por _________________________, apresentava
caractersticas _____________________.

a) o grau de coloquialidade demonstrado pelo emprego


de expresses como gostosa, bonita pra burro e dei
em cima.
b) os esforos do eu potico para conquistar a jovem so
marcados pelas expresses Utilizei o bonde, o automvel,
o passeio a p.
c) os termos macumba e p denotam alguns dos recursos
de que o eu potico se utilizou em vo para angariar o afeto
da amada.
d) a expresso no fez efeito, proveniente do vocabulrio
mdico, de uso exclusivo da linguagem tcnica.
e) o eu potico recorre a qualificativos de ordem tica
(boa), sexual (gostosa) e fsica (bonita) para atrair
o interesse da jovem.
RESPOSTA: D
COMENTRIO: Considerando a primeira e a segunda estrofes,
a alternativa "d" a nica incorreta, pois restringe a expresso
"no fez efeito" ao uso da linguagem tcnica.
24. (INSPER-2011) Sobre a ltima estrofe, considere as seguintes
inferncias:

Assinale a alternativa que completa corretamente as lacunas:


(A) Oswald de Andrade Manifesto antropfago- realistas.
(B) Monteiro Lobato Paranoia ou mistificao
expressionistas.
(C) Mrio de Andrade Prefcio interessantssimo
surrealistas.
(D) Graa Aranha O esprito moderno impressionistas.
(E) Mrio de Andrade Mestres do passado cubistas.
RESPOSTA: B
COMENTRIO: O perodo corretamente preenchido : A
artista, criticada por Monteiro Lobato, em texto conhecido por
Paranoia ou mistificao, apresentava caractersticas
expressionistas.
26. (ENEM-2010) Aps estudar na Europa, Anita Malfatti retornou
ao Brasil com uma mostra que abalou a cultura nacional do
incio do sculo XX. Elogiada por seus mestres na Europa, Anita
se considerava pronta para mostrar seu trabalho no Brasil,
mas enfrentou as duras crticas de Monteiro Lobato. Com a
inteno de criar uma arte que valorizasse a cultura brasileira,
Anita Malfatti e outros artistas modernistas
8

A) buscaram libertar a arte brasileira das normas acadmicas


europeias, valorizando as cores, a originalidade e os temas
nacionais.
B) defenderam a liberdade limitada de uso da cor, at ento
utilizada de forma irrestrita, afetando a criao artstica
nacional.
C) representaram a ideia de que a arte deveria copiar
fielmente a natureza, tendo como finalidade a prtica
educativa.
D) mantiveram de forma fiel a realidade nas figuras retratadas,
defendendo uma liberdade artstica ligada tradio
acadmica.
E) buscaram a liberdade na composio de suas figuras,
respeitando limites de temas abordados.

28. (PUC-RS-2010) Para responder questo 7, ler o trecho


Memrias sentimentais de Joo Miramar, de Oswald de
Andrade.
A costa brasileira depois de um pulo de farol sumiu como um
peixe. O mar era um oleado azul. O sol afogado queimava
arranha-cus de nuvens.
Dois pontos sujaram o horizonte faiscando longnquos bons
dias sem fio.
Os olhos hipcritas dos viajantes andavam longe dos livros
agora polichinelos sentados nas cadeiras vazias.
A aproximao do texto literrio prosa cinematogrfica,
caracterizada pela _________, permite afirmar que o
fragmento acima, de autoria de Oswald de Andrade,
enquadra-se na esttica _________.
A) simultaneidade de imagens - modernista
B) exaltao de objetos - romntica
C) presena da ironia - realista
D) idealizao da paisagem - ps-moderna
E) explorao do local - simbolista

RESPOSTA: A
COMENTRIO: A libertao da arte brasileira das normas
acadmicas europeias uma das principais propostas dos
artistas modernistas brasileiros que participaram da Semana
de Arte Moderna de 1922. A busca da originalidade na
abordagem de temas principalmente nacionais e a valorizao
das cores tambm so caractersticas presentes nas obras de
Anita Malfatti (Tropical, O homem amarelo, A mulher de
cabelos verdes) exibidas na exposio realizada em 1917, em
So Paulo, quando a artista foi duramente criticada por
Monteiro Lobato, no artigo Paranoia ou mistificao.

RESPOSTA: A
COMENTRIO: Oswald de Andrade representou muito bem o
movimento modernista ao ousar, fazendo a aproximao do
texto literrio prosa cinematogrfica.
29. (CTFMG-2010)

27. (CTFMG-2010)
Vcio na fala
Os sapos
Para dizerem milho dizem mio
Para melhor dizem mi
Para pior pi
Para telha dizem teia
Para telhado dizem teiado
E vo fazendo telhados
ANDRADE, Oswald de. Pau-Brasil. So Paulo: Nova Fronteira,
1992.

Enfunando os papos,
Saem da penumbra,
Aos pulos, os sapos,
A luz os deslumbra.
Em ronco que aterra
Berra o sapo-boi
- "Meu pai foi guerra!"
- "No foi!" - "Foi!" - "No foi!".
O sapo-tanoeiro
Parnasiano aguado,
Diz: - "Meu cancioneiro
bem martelado."
(...)
Vai por cinquenta anos
Que lhes dei a norma:
Reduzi sem danos
A frmas a forma.

O aspecto do modernismo brasileiro destacado no poema


refere-se
a) crtica social.
b) temtica urbana.
c) abordagem poltica.
d) problemtica lingustica.
RESPOSTA: D
COMENTRIO: O poema revela, por meio da oposio entre a
linguagem considerada culta e a linguagem coloquial e oral, a
problemtica da nossa tradio cultural que rejeita a
existncia de determinadas variantes lingusticas. Isso acaba
por rejeitar algumas manifestaes da lngua, utilizando o
domnio da linguagem culta como forma de exercer o poder.

BANDEIRA, Manuel. Estrela da vida inteira. Rio de Janeiro:


Nova Fronteira, 1993.
O poema acima revela, em relao ao parnasianismo, um(a)
a) tributo.
b) pardia.
c) epgrafe.
d) parfrase.
RESPOSTA: B
COMENTRIO: O poema da primeira gerao modernista,
caracteriza, entre outros aspectos, pelo repdio ao formalismo
dos poetas parnasianos. O autor utilizar uma estutura formal
para criticar e ironizar o estilo parnasiano.

30. (UEL) Na primeira fase do Modernismo no Brasil,


A) impuseram-se os nomes pioneiros de Monteiro Lobato,
Euclides da Cunha e Coelho Neto.
B) firmou-se uma nova tendncia do romance regionalista,
com Graciliano Ramos e Jos Lins do Rego.
C) propuseram-se ideias e obras revolucionrias, como
MACUNAMA e MEMRIAS SENTIMENTAIS DE JOO
MIRAMAR.
D) promoveu-se o surgimento de ficcionistas renovadores,
mas em nada foi afetada a linguagem dos poetas.
E) propiciou-se a renovao da literatura, no ocorrendo o
mesmo com a msica e a pintura.
9

happy fingers (verb) email titled I LOVE YOU only to be bitten


by the love bug. Ouch. Lets take a look at some of the worst
network attacks of all time.
- I LOVE YOU (2000) The ILOVEYOU is one of the first viruses
to trick users into opening a file. The virus used mailing lists as
its source for targets, so the messages appeared to come from
someone known to the user, such as a friend or relative,
making it seem safe to open. In reality, the file was a VBS
script with an avalanche of junk mail that deleted thousands of
files, including multimedia, music, and personal files. The
author is not known, (conjuction) it started in the Philippines.
-Nimda (2001)
Built on Code Reds attack system of finding multiple
avenues into machines such as email, websites, and network
connections, Nimda (admin backwards) computer worm
infected Web servers and user machines. It was so effective it
managed to leave a wide trail of casualties in its wake a mere
22 minutes after it was released. Nimda heavily impacted
users of Windows programs. Some groups linked Nimda to AlQaeda because of the massive and quick destruction it caused,
but this theory (passive voice). The author of Nimda remains
unknown but left $635 million in destructive damage.
-Slammer (2003)
The Slammer computer worm spread rapidly, infecting
most of its 75,000 victims (preposition) ten minutes. When
routers started to crash, it slowed the Internet to a crawl and
shut down thousands of websites. The Slammer was named by
Christopher J. Rouland because of its sudden effect. Although
named SQL Slammer, it did not use SQL language. 22,000
systems were affected worldwide and South Koreans reported
th
Internet shut downs for hours nationwide on January 25 ,
2003.
-MyDoom ? 2004
When MyDoom hit the scene it became the fastestspreading computer worm of all time. Once it succeeded to
infect computers, it would in turn send even more junk mail
through infected computers with the subject andy; Im just
doing my job, nothing personal, sorry. There was speculation
that MyDoom was used to attack the website of SCO Group, in
retaliation of legal actions against a Linux or open source
supporter. This claim since then has been dismissed and
denied. The Author of this worm is not known.
-Storm ? 2007
Storm, a backdoor Trojan horse, swooped in and infected
thousands of computers using an email with a subject line
about a weather disaster. A Trojan (adverb) appears to
perform a specific action, but destructs and infects through a
virus attack by performing something completely different.
Trojans install backdoor programs. It mutated and spread via
email spam with a fake attachment, and infected 10 million
computers.
So is it over? Attackers continue to probe and sniff out
weaknesses in hardware or software to gain access to
networks, so take sensible precautions. In 2006, Malware
resulted in $13.3 billion in damages across corporate America.
Remember to avoid opening attachments or download
programs that seem questionable or odd because it can take
one click to bring down your (genitive case).
Adpated
by
Jefferson
Celestino
from
http://www.palaestratraining.com/

RESPOSTA: C
COMENTRIO: Na primeira fase do modernismo brasileiro, os
escritores propuseram a criao de um novo modelo potico
no qual havia uma ruptura com os modelos do sculo XIX. Tais
ideias, consideradas revolucionrias e inovadoras, podem ser
observadas na criao de obras como Macunama, de Mrio
de Andrade, e Memrias sentimentais de Joo Miramar, de
Oswald de Andrade.
31. (UFF)
PERO VAZ CAMINHA
a descoberta
Seguimos nosso caminho por este mar de longo
At a oitava da Pscoa
Topamos aves
E houvemos vista de terra
os selvagens
Mostraram-lhes uma galinha
Quase haviam medo dela
E no queriam pr a mo
E depois a tomaram como espantados
primeiro ch
Depois de danarem
Diogo Dias
Fez o salto real
as meninas da gare
Eram trs ou quatro moas bem moas e bem gentis
Com cabelos mui pretos pelas espduas
E suas vergonhas to altas e to saradinhas
Que de ns as muito olharmos
No tnhamos nenhuma vergonha
ANDRADE, Oswald. Poesias reunidas. Rio de
Janeiro: Civilizao Brasileira,1978. p. 80.
O procedimento potico empregado por Oswald de Andrade
no texto II :
(A) reconhecer e adotar a mtrica parnasiana, criando estrofes
simtricas e com ttulos;
(B) recortar e recriar em versos trechos da carta de Caminha,
dando-lhes novos ttulos;
(C) imitar e refazer em prosa a carta de Caminha, criando
ttulos para as vrias sees;
(D) reconhecer e retomar a prtica romntica, dando ttulos
nacionalistas s estrofes;
(E) identificar e recusar os processos de colagem modernistas,
dando-lhes ttulos novos.
RESPOSTA: B
COMENTRIO: A leitura dos poemas destacados de Oswald de
Andrade apresenta, como recurso potico, o recorte e a
recriao, em versos, de trechos da carta de Caminha, dandolhes novos ttulos que sugerem uma reviso histrica da
famosa carta do navegante portugus, bem como o uso da
pardia, que fora apregoado pelos modernistas.
INGLS
THE 5 WORST COMPUTER VIRUS ATTACKS OF ALL TIME
Its just another day as you slide into your seat and hit the
start button on your computer. You guzzle coffee as you wait
for your system to start up, and bam! Your stomach drops as
you stare at the deep indigo colored screen. Its not a folk tale,
its not an urban legendits the blue screen of death. (BSOD)
Yikes. There goes your day.
We all know that its common sense to stay away from funny
looking email. If you dont know who sent it, dont open it. Its
a great concept, in theory. In fact, lots of us with itchy, trigger-

32. 1. Match the correct sequence of translations:


a) to guzzle
b) to stare
c) itchy
d) to trick
e) backwards
f) to dismiss
10

g) to spread
h) crawl
i) claim
j) to swoop
k) attachment
l) to probe
( ) de trs pra frente
( ) velocidade muito baixa
( ) atacar rapidamente
( ) beber avidamente
( ) anexo
( ) alegao
( ) irrequieto
( ) sondar, investigar
( ) rejeitar, por de lado, descartar
( ) enganar
( ) propagar, espalhar, difundir
( ) fitar
RESPOSTA: E, H, J, A, K, I, C, L, F, D, G, B
COMENTRIO: 1. A sequencia correta :
a) to guzzle = beber avidamente
b) to stare = fitar
c) itchy = irrequieto
d) to trick = enganar
e) backwards = de trs pra frente
f) to dismiss = rejeitar, por de lado, descartar
g) to spread = propagar, espalhar, difundir
h) crawl = velocidade muito baixa
i) claim = alegao
j) to swoop = atacar rapidamente
k) attachment = anexo
l) to probe = sondar, investigar
33. The expression in fact (2nd paragraph) can be replaced by:
a) Although
b) Also
c) Therefore
d) Indeed
e) Unless
RESPOSTA: D
COMENTRIO:A expresso in fact tem o mesmo sentido de
indeed: de fato, na verdade.
34. According to the text, fill in the gaps of the following sentence
meaningfully:
Up till now, _________________ of the authors of the
________________
viruses
ever
created
were
_______________________.
a) all worse caught
b) none worst arrested
c) some most deadly taken
d) nobody most badly incarcerated
e) not a soul most dangerous released.

c) shows an unpleasant situation that could happen to anyone.


d) explains how to avoid worms and viruses in your computer.
e) reports the day the authors computer was attacked by five
viruses.
RESPOSTA: C
COMENTRIO: A introduo do texto descreve uma situao
que pode acontecer a qualquer pessoa: voc chega ao
trabalho, toma um caf enquanto espera seu computador ligar
e, de repente, percebe que h algo errado com sua maquina,
l se vai o seu dia.
36. Which of these questions cannot be answered by the text?
a) How many people were directly affected by Slammer?
b) Why was Al-Qaeda linked to Nimda?
c) How many machines were damaged by Storm?
d) What can be done to avoid computers are infected by
viruses?
e) Which are the worlds most efficient anti-virus programs?
RESPOSTA: E
COMENTRIO: A pergunta Which are the worlds most
efficient anti-virus programs? (Quais so os programas
antivirus mais eficientes do mundo?) no pode ser respondida
pelo texto.
37. Which of the following words cannot receive the suffix ness
as weakness (last paragraph)?
a) Wild.
b) Good.
c) Alone.
d) Selfish.
e) Nervous.
RESPOSTA: A
COMENTRIO: Apenas o vocbulo wild (selvagem) no recebe
o sufixo ness. Os demais: goodness (bondade), aloneness
(solido), selfisheness (egosmo) e nervousness (nervosismo).
38. Fill in the gap (verb) in the text.
a) have not opened
b) has opened
c) have opened
d) has not opened
e) have open
RESPOSTA: C
COMENTRIO: Apenas o Present Perfect Tense (to have + past
participle) completa a lacuna, j que a ao descrita ocorre em
um tempo no definido no passado: have opened (abriu).
39. Fill in the gap (conjunction) in the text:
a) also
b) but
c) besides
d) furthermore
e) hence

RESPOSRA: B
COMENTRIO: Consoante o texto, at agora, nenhum dos
autores dos piores vrus j criados foi preso (Up till now, none
of the authors of the worst viroses ever created were
arrested).
35. The introduction of the text:
a) advises that you should be careful with some kinds of Web
viruses.
b) tells a story impossible to happen depite being an
apparently common situation.

RESPOSTA: B
COMENTRIO: necessria a utilizao de uma conjuno
adversativa porque o trecho em questo mostra que apesar
do fato do autor do vrus ILOVEYOU ser desconhecido, sabe-se
que o vrus partiu das Filipinas: But (mas).
40. Fill in the gap (passice voice) in the text:
a) is dismissed
b) would be dismissed
c) will be dismissed
11

d) were be dismissed
e) was dismissed

d) dont stop to diet


e) stop and go on a diet

RESPOSTA: E
COMENTRIO: A estrutura passiva a ser utilizada precisa estar
no passado, pois a sentena indica que a teoria que ligava o
vrus Nimda rede terrorista Al-Qaeda foi descartada (was
dismissed).

RESPOSTA: A
COMENTARIO: O verbo stop no sentido de deixar de fazer
algo seguido de um verbo na forma ING.

41. Fill in the gap (preposition) in the text:


a) amid
b) among
c) between
d) within
e) into

46. In the excerpt below:


President Barack Obama has approved a significant troop
increase for Afghanistan, Pentagon officials said Tuesday. The
new troop deployment is expected to include 8.000 Marines
from Camp Lejeune, North Carolina, as well as 4.000 additional
Army troops from Fort Lewis, Washington.
Adapted by Jefferson Celestino from
http://www.cnn.com/POLITICS/02/17obama.troops/indexhtl.

RESPOSTA: D
COMENTRIO: A preposio a ser utilizada deve indicar o
perodo de tempo em que uma determinada ao ocorreu.
Apenas within (dentro de) possui tal funo dentre as
apresentadas,

a) there are two different verb tenses.


b) there is no auxiliary verb.
c) there are only regular verbs.
d) there is only one verb tense
e) there are only irregular verb.

42. Fill in the gap (adverb) in the text


a) likely
b) nearly
c) highly
d) widely
e) wholly

RESPOSTA: A
COMENTRIO: O excerto em questo apresenta dois tempos
verbais distintos: Present Perfect (has approved aprovou) e
Simple Past (said disseram). A construo is expected
(espera-se) voz (passiva) e no tempo.
Texto para as questes de 16 e 18.

RESPOSTA: A
COMENTRIO: Apenas likely completa a lacuna corretamente:
A trojan likely appears to perform a specific action (Um cavalo
de tria provavelmente aparece para executar uma ao
especifica).

Working women in Japan are more likely to be married


than not these days, a sharp reversal of the traditional
pattern. But for most of them, continuing to work after the
wedding is an easier choice than having children. Despite
some tentative attempts by government and business to make
the working world and parenthood compatible, mothers say
Japans business culture remains unfriendly to them. Business
meetings often begin at 6 p.m. or later, long hours of unpaid
overtime are expected, and companies, routinely transfer
employees to different cities for years. As a result, many
women are choosing work over babies, causing the Japanese
birthrate to fall to a record low in 1990 an average 1.34
babies per woman an added woe for this aging nation.
THE WASHINGTON POST NATIONAL WEEKLY
EDITION August 21, 2000

43. Fill in the gap (genitive case) in the text:


a) company entires network
b) companys entire network
c) companys entires network
d) companys entire network
e) company entire networks
RESPOSTA: B
COMENTRIO: A estrutura do Genitive Case (s ou ) deve ser
acrescentada ao possuidor, logo, teremos a estrutura
companys entire network (a rede inteira da companhia) como
resposta correta.
44. (UFRJ) Choose the best option:
Before _______________ sent to prison, the thief will be given
the right to defend himself.
a) being
b) be
c) to be
d) he will be
e) he has been
RESPOSTA: A
COMENTRIO:Aps a preposio before usamos o verbo na
forma ING
45. (Unesp-SP) Assinale a alternativa que completa corretamente
a lacuna.
Unfortunately, many people _________________ after the
first week or second week.
a) stop dieting
b) stop not diet
c) dont stop dieting

47. Fuvest According to the passage, the majority of working


women in Japan
a) expect to stop working after getting married.
b) do not like the idea of having children.
c) are choosing to remain single in order to keep their jobs.
d) have been afraid to fight against traditional roles.
e) would rather keep their jobs than have children.
RESPOSTA: E
COMENTRIO: Segundo o texto, a maneira das mulheres
trabalhadoras do Japo preferem manter o emprego do que
ter filhos.
48. Fuvest attempts (...) to make the working world and
parenthood compatible (lines 5-6) means that:
a) married couples are expected to delay having children.
b) efforts have been made to improve the working conditions
of workers with children.
c) working women have to fight hard in order to have children.
d) the government has proved that work and children are
incompatible.

12

e) companies tend to think that people with children make


better workers.
RESPOSTA: B
COMENTRIO: O trecho do texto quer dizer que esforos tem
sido feitos para melhorar as condies de trabalho das
mulheres com filhos.
49. Fuvest Which of these statements is true according to the
passage?
a) The Japanese birthrate used to be much lower.
b) The percentage of marriages in Japan has fallen lately.
c) The Japanese population is getting older.
d) Japans population has stopped growing since 1999.
e) Working women do not have more than one child.

COMENTRIO: Podemos afirmar que os adolescentes e idosos


apresentam o maior ndice de acidentes de carro.
51. One can argue that middle-aged drivers drive more:
a) recklessly
b) aggressively
c) dangerously
d) cautiously
e) impatiently.
RESPOSTA: D
COMENTRIO: A gente pode afirmar, segundo o texto, que os
motoristas de meia-idade dirigem com mais cautela.

ESPANHOL
RESPOSTA: C
COMENTRIO: Segundo o texto, a populao Japonesa est
ficando mais velha.
Texto para as questes 19 e 20.
The risks of distraction
Cell phones use while driving and alcohol causes most of the
car accidents.
Drive distractions are the major causes of car accidents. They
include cell phones and other kinds of gadgets (laptops or
electric razors, for example) used by drivers as they drive. In
response to this problem some states in the US have banned
the use of hand-held cell phones while people are driving.
Alcoholic drinks may be also considered a distraction. Thats
why anyone with 01 of blood alcohol is considered unfit to
drive. Other potentially dangerous habits are rubbernecking
and tailgating. The first one occurs when drivers slow down to
look at another accident on the road or anything that looks
unusual. Thats dangerous because the driver in the car behind
often doesnt have enough time to slow down. The same
applies for tailgating in that a driver who is following a car too
closely may not have enough distance to stop suddenly.
(http://wzinearticles.com/?top-causes-of-caraccidents&id=302163,adaptado)
Who causes car acidents?
Percentage of at-fault accidents in all accidents with injuries
(2006)

52. A inteno do autor com o texto anterior nos parece ter sido:
a) dizer que, para se reeleger, Obama lanar mo de um
Retweet.
b) dizer que Obama um dos poucos polticos que usa redes
sociais.
c) mostrar que Obama est atualizado quanto s redes sociais.
d) criticar o uso do Twitter por Obama.
e) dar a entender que Obama inteligente.
RESPOSTA: A
COMENTRIO: Para essa questo , levemos em considerao
que dizem que Obama se elegeu graas ao uso das redes
sociais e desta vez usar o Retweet.
Es mentira, de verdad...

50. It is true to assert that:


a) Both the teens and the erderly have the lowest accident
rates.
b) Both the teens and the elderly have the highst accident
rates.
c) Middle-aged drivers cause more accidents than teengagers.
d) Middle-aged drivers cause more accidents than the elderly.
e) The erderly cause many more accidents than teenagers.
RESPOSTA: B

53. Com a tira, podemos enteder que:


13

a) a sra. Cospedal um passarinho.


b) a sra. Cospedal tem um salrio bastante duvidoso.
c) a sra. Cospedal quebrou o p direto.
d) a sra. Cospedal ganha 8 mil pesos.
e) a sra. Cospedal passar 40 dias presa.

IV. fogon
V. dormitrios
a) Son acentuadas I, IV, V.
b) Son acentuadas I, II, IV.
c) No son acentuadas I, III, V.
d) Todas son acentudas.
e) Ninguna es acentuda.
RESPOSTA: B
COMENTRIO: Veja a acentuao correta das palavras:
Policia (hiato)
Pjaros (esdrjula).
Ptio.
Fogn (aguda terminada em N).
Dormitrios.
Portanto, esto corretos os nmeros I, II, IV.

RESPOSTA: C
COMENTRIO: Escayolar Enyesar, inmovilizar por mdio de
un vendaje de yeso o de escayola un miembro roto, deslocado
etc. Ej.: Tuvieron que escayolarle el brazo porque se habia
fracturado el codo.
54. En la frase ... solo de momentos, lo destacado es expresin
sinnima de:
a) solamente.
b) piso.
c) suelo.
d) soledad.
e) msica cantada o tocada por una persona apenas.
RESPOSTA: A
COMENTRI: O adverbio slo (somente) tem como sinnimo o
vocbulo solamente ou unicamente. No confundi-lo com
solo, que pode ser adjetivo (sozinho, sem companhia....) ou
substantivo (solo musical, passo de dana que se executa sem
casal ou um tipo de jogo no baralho).
55. Solo y Solo se clasifican, respectivamente, como:
a) substantivo y adjetivo.
b) adverbio y adjetivo.
c) adjetivo y adverbio.
d) substantivo y adverbio.
RESPOSTA: C
COMENTRIO: O adverbio slo (somente) tem como sinnimo
o vocbulo solamente ou unicamente. No confundi-lo com
solo, que pode ser adjetivo (sozinho, sem companhia....) ou
substantivo (solo musical, passo de dana que se executa sem
casal ou um tipo de jogo no baralho).
56. La palabra sabindole est acentuada porque es:
a) aguda acabada en vocal.
b) grave acabada en vocal.
c) verbo acentuada que recibi pronombre.
d) sobresdrjula.
e) todas las esdrjulas son acentuadas.
RESPOSTA: D
COMENTRIO: Sabindole uma forma proparoxtona
(esdrjula) y toda forma proparoxtona acentuada.
57. La palabra sonrea est acentuada:
a) por ser una grave acabada en vocal.
b) por ser un verbo derivado de una palabra acentuada.
c) para deshacer el diptongo.
d) porque cuando en un encuentro voclico la tonacidad cae
en la vocal dbil, esta es acentuada.
e) porque es una palabra esdrjula.
RESPOSTA : D
COMENTRIO: Para resolver a questo, basta recordar a regra
da silaba tonica, quando o acento (tonicidade) recai na vogal
dbil (i/u), esta deve ser acentuada, desfazendo o ditongo.
Sonreia.
58. Acenta correctamente las palabras:
I. policia
II. pajaros
III. ptio

59. El acento de la expresin verbal repugna en ... el prestigio de


lo que repugna a la inmensa mayora., reace en la slaba que
la clasifica como una palabra:
a) aguda
b) grave
c) esdrjula
d) sobresdrjula
RESPOSTA: B
COMENTRIO: A palavra repugna, classificada como uma
palavra grave ou llana, ou seja, o acento recai sobre a
penltima silaba.
60. Marca la alternativa en que todas las palabras se acentuam de
acuerdo con la misma regla:
a) republicacin, elabor, adems.
b) pblicas, acadmicos, Cortzar.
c) utiliz, Andaluci, Isaas.
d) fillogos, tambin, jvenes.
e) Pennsula, Pars, fantasa.
RESPOSTA: A
COMENTRIO: As palavras publicacin, elabor e ademas
esto acentuadas pela mesma regra, que diz: acentuamos
todas as palavras agudas (silaba tnica a ultima) terminada
em vogal, N, S.
61. ? Puedes decirme__________ son aquellas personas que estn
hablando con Antonio?
a) quines
b) quin
c) quien
d) quienes
e) que

RESPOSTA: A
COMENTRIO: ? Puedes decime quines son aquellas
personas...? Utilizamos quines para concordar com aquellas
personas, que plural. Acentuamos porque, nesse caso, tratase de um pronome interrogativo, assim, obrigatrio o
acento.
62. Dnde(la seora pregunta dnde est el andn...) lleva
acento:
a) Porque es una palabra grave terminada en e.
b) Porque es una palabra llana terminada en e.
c) Porque es una palabra esdrjula terminada en vocal.
d) Porque es una palabra aguda.
e) Porque es una partcula interrogativa indirecta.

14

RESPOSTA : E
COMENTRIO: Nesse caso, a palabra donde est acentuada
porque figura um caso de interrogativa indireta; que assim
como os interrogativos diretos tambm recebe acento grfico.

a) para evitar una cacofonia, mejor dicho, un sonido


desagradable al odo.
b) delante de palabras empezadas, respectivamente, por Y y
U.
c) por un motivo que no es de eufonia.
d) a causa de un equvoco gramatical que se admite
excepcionalmente.

63. El solo hecho de pagar impuestos.... ..... no slo a sus


rganos de gobierno .... En las frases arriba transcritas, el
vocablo solo aparece sin acento (en el sentido de nico) y con
acento (en el sentido de solamente). La tilde, diacrtica,
tambien est presenta en:
a) poesa.
b) cancin.
c) an.
d) despus.
RESPOSTA: C
COMENTRIO: O acento diacrtico ou diferencial para
diferenciar um vocbulo com a mesma grafia s que com
classe gramatical diferente. An um dos casos.
Veja ainda:
Aun todavia ainda (advrbio de tempo)
Aun incluso inclusive (advrbio de quantidade)

RESPOSTA: A
COMENTRIO: A conjuo Y substituda por E quando vier
antes de vocbulos comeados por I ou HI seguidos de
consoantes. Tudo isso ocorre para evitar uma cacofonia, ou
seja, um som desagradvel.
68. ... as como el agua puede quitar la sed...
El vocablo agua es precedido del articulo definido el, visto
que:
a) pertenece al gnero masculino.
b) se incluye entre los substantivos ambguos en cuanto al
gnero.
c) se busca evitar un sonido desagradable al odo.
d) admite los dos gneros, con variacin de significado.

64. El rbol lleva articulo masculino pues:


a) es masculina.
b) es femenina.
c) usamos regla de eufonia.
d) est equivocado.
RESPOSTA: A
COMENTRIO: rbol recebe artigo masculino no pela regra
de eufonia, mas sim porque e uma palavra masculina.
65. Est correcta la regla de eufonia en:
a) Clara y yema.
b) Espaol y ingls.
c) Tu y yo.
d) Historia y portugus.
RESPOSTA: A
COMENTRIO: A conjuno Y substituda por E quando vier
antes de vocbulos comeados por l ou HI seguidos de
consoantes.
Assm teramos:
Clara y yema.
Espaol y ingls
T y yo.
Historia y portugus.
66. Estaria correcto el uso de el delante de todas las palabras
siguientes, menos:
a) arbol
b) auge
c) hache
d) viaje
RESPOSTA: C
COMENTRIO: Para essa questo, devemos levar em
considerao as regras de euforia, mais precisamente a 1, que
diz: sempre que a palavra for feminina e comear por A ou H
tonica, devemos substituir os artigos LA/UMA pelos
masculinos EL/UN para evitar um som desagradvel. Vale
lembrar que temos 3 excees: o nome das mulheres, o nome
das letras A e H (hache) e a cidade Haya, com elas sempre
usamos os artigos femininos, mesmo que causem cacofonia.

RESPOSTA: C
COMENTRIO: O vacabulo agua um substantivo feminino
comeado por A tnico, por isso, no se usa artigo feminino
LA, e sim EL para evitar cacofonia.
69. Indica la opcin en que el empleo de los artculos est
debidamente observado:
a) El habla, el Haya, la guila.
b) El arena, el habla, el yarda, la hacha.
c) La Ana, el haba, la gueda.
d) La Ana, el alta cumbre, un rida tierra.
RESPOSTA: C
COMENTRIO: Para essa questo, devemos levar em
considerao as regras de eufonia, mais precisamente a que
diz: sempre que a palavra for feminina e comear por A ou HA
tnica, devemos substituir os artigos LA/UNA pelos masculinos
EL/UN para evitar um som desagradvel. Vale lembrar que
temos 3 excees: o nome das mulheres, o nome das letras A
e H (hache) e a cidade HAYA, com elas sempre usamos os
artigos femininos, mesmo que causem cacafonia.
70. _________alma melanclica puede curarse con el cario, que
es ________ hada milagrosa.
a) El, el
b) El, la
c) La, la
d) La, el
RESPOSTA: A
COMENTRIO: Ver regras da questo anterior. Alma comea
por A tnica, recebe EL; hada comea por HA tnico, tambm
recebe EL.
71. ________ linguaje de ________ prensa es muy pesado.
a) El, la
b) La, la
c) El, el
d) Lo, la
RESPOSTA: A
COMENTRIO: As palavras terminadas em AJE e OLOR so
todas masculinas! Portanto, temos: el linguaje e la prensa
(prensa significa imprensa).

67. Las conjunciones y (y recibe al mismo tiempo una caricia) y o


(... hermoso o feo) se convierten en e y , respectivamente:
15

HISTRIA I (DO BRASIL)


72. (G1 - ifsp 2011) Pai dos Pobres e Me dos Ricos foram
atributos concedidos ao presidente
a) Juscelino Kubitscheck, que com seu ambicioso Plano de
Metas construiu Braslia, rodovias, abriu as portas do pais ao
capital estrangeiro, assegurando trabalho para os pobres e
vida farta e confortvel para os ricos.
b) Eurico Gaspar Dutra, que abriu o Brasil s importaes
estrangeiras, pois nossas reservas estavam altas, devido s
exportaes poca da Segunda Guerra Mundial. Foi
chamado de me dos ricos pelas facilidades de acesso aos
produtos estrangeiros que estes teriam e pai dos pobres
porque trouxe milhares de empregos na construo civil.
c) Getlio Vargas, pois foi o responsvel pela primeira
legislao trabalhista no Brasil. Transformou a questo
operria caso de polcia (no dizer de Washington Luis) em
questo operria caso de poltica. Essa legislao garantia
inmeros direitos aos trabalhadores e assegurava aos ricos um
cenrio sem greves e manifestaes, em que os lucros fluam
naturalmente para os cofres de suas empresas, trazendo a paz
social ao Brasil.
d) Fernando Collor de Mello, que fez sua campanha
presidencial em 1989 apresentando-se como o caador de
marajs. Dispunha-se a eliminar os altos salrios de
funcionrios pblicos (era assim, o pai dos pobres); como
sofreu o impeachment em 1992, no realizou suas promessas
e tornou-se ento a me dos ricos.
e) Tancredo Neves, o primeiro presidente civil aps 21 anos de
governos militares. A expresso pai dos pobres e me dos
ricos, significa, na verdade, o presidente de uma nao que
em 1985 nascia" (da as expresses pai e me) aps os anos de
chumbo.
RESPOSTA: C
COMENTRIO: Tradicionalmente chamado de pai dos
pobres, Vargas compreendido pela maioria como
responsvel por uma poltica social inovadora, com a
concesso de direitos trabalhistas aos trabalhadores urbanos e
preocupado com as questes sociais. Seus crticos o condenam
como manipulador, responsvel por alienar o operariado e,
dessa forma, beneficiar as elites, que dirigiam a economia sem
grandes contestaes e com o apoio do Estado.
73. (G1 - col.naval 2011) "Na madrugada de 11 de maio de 1938,
o jovem tenente e seus homens invadiram o Palcio
Guanabara, onde ento morava Getlio, tentaram metralhar
toda a famlia presidencial, mas logo eram rechaados sem d.
[... ] Mas logo pipocou a primeira bala, no Palcio Guanabara,
e j Filinto [Mller] abandonava os camisas-verdes e se punha,
leal como sempre', ao lado de Vargas. [...] sufocada a
rebelio, Filinto Mller se ps testa da dura represso que
iria ter comeo contra os adeptos, em todo o pas, da verso
brasileira do nazismo de Hitler e do fascismo de Mussolini."
Joel Silveira. Revista Nossa Histria. Abril 2005. Ano 2, n. 18,
p.59. Adaptado
O texto acima faz referncia ao:
a) da ANL, Aliana Nacional Libertadora, grupo poltico de
direita que pretendia implantar um regime fascista no Brasil.
b) do PCB, Partido Comunista do Brasil, que no movimento
denominado Intentona Comunista pretendia implantar um
regime stalinista no Brasil.
c) da AIB, Ao Integralista Brasileira, organizao nacionalista
que pretendia implantar um Estado autoritrio no Brasil, de
inspirao fascista.

d) do BOC, Bloco Operrio Campons, organizao que tinha o


objetivo de lutar contra as oligarquias e acabar com as
desigualdades sociais.
e) da Aliana Liberal, formada por grupos oligrquicos de
inspirao fascista, descontentes com a implantao do Estado
Novo.
RESPOSTA: C
COMENTRIO: Durante a Era Vargas desenvolveu-se no
Brasil o integralismo, movimento defensor da ideologia
fascista. Liderado pelo jornalista Plnio Salgado, os integralistas
apoiaram o golpe que instalou o Estado Novo ditadura
comandada por Vargas e consideravam que representariam
politicamente o novo Estado. No entanto, a poltica
personalista do presidente levou-a a decretar a ilegalidade de
todos os partidos polticos, incluindo a AIB (Ao Integralista
Brasileira), fato que motivou o levante descrito no enunciado
da questo.
74. (G1 - cftmg 2011) Ao longo da dcada de 1930, definiu-se um
novo papel econmico do Estado, porque foram
a) introduzidas diretrizes estatais para controlar os fatores
produtivos de industrializao.
b) estabelecidas regras para priorizar o financiamento e o
crdito das atividades agroexportadoras.
c) criadas estratgias para descentralizar a concesso de
incentivos fiscais aos produtores regionais.
d) implantadas condies institucionais para expandir as
atividades empresariais ligadas ao mercado externo.
RESPOSTA: A
COMENTRIO: Em 1930 iniciou-se a Era Vargas,
caracterizada por transformaes na economia do pas e pela
preocupao do Estado em promover o desenvolvimento
nacional. A poltica intervencionista, nacionalista e
industrialista amparada no trabalhismo formou a base do
que se convencionou chamar de populismo.
75. (G1 - cftmg 2011) Em 1940, Getulio Vargas pronunciou o
seguinte discurso a bordo do encouraado Minas Gerais em
comemorao ao Dia da Marinha de Guerra.
Atravessamos ns, a humanidade inteira transpe, um
momento histrico de graves repercusses, resultante da
rpida e violenta mutao de valores. Marchamos para um
futuro diverso do quanto conhecamos em matria de
organizao econmica, social ou poltica, e sentimos que os
velhos sistemas e frmulas antiquadas entram em declnio.
No , porem, como pretendem os pessimistas e os
conservadores empedernidos, o fim da civilizao, mas o
inicio, tumultuoso e fecundo, de uma nova era. Os povos
vigorosos, aptos vida, necessitam seguir o rumo de suas
aspiraes, em vez de se deterem na contemplao do que
desmorona e tomba em runa. preciso, portanto,
compreender a nossa poca e remover o entulho das ideias
mortas e dos ideais estreis... passou a poca dos liberalismos
imprevidentes, das democracias estreis, dos personalismos
inteis e semeadores de desordem.
GV 40.06.11, Fundao Getulio Vargas. Apud CORSI, Francisco
Luiz. Estado Novo: poltica
externa e projeto nacional. So Paulo: UNESP/FAPESP, 2000.
p.158.
Pode-se avaliar que o discurso sugere o alinhamento ao(s)
a) pases defensores de uma poltica de neutralidade.
b) contedo ideolgico veiculado pelos pases aliados.
c) contedo ideolgico veiculado pelos pases do Eixo.
d) pases defensores de regimes democrticos e liberais.
RESPOSTA: C

16

COMENTRIO: O texto destaca a valorizao daquilo que


novo, de novas ideias e, ao final, critica claramente o
liberalismo e a democracia, demonstrando a inclinao do
governo brasileiro pelas ideias fascistas, novas na Europa, que
se propunham a superar a paralisia das sociedades liberais.
Alguns podem ser induzidos ao erro, pois sabemos que o
governo Vargas acabou mudando de postura, apoiou os
Aliados e levou o pas a ingressar na Guerra, combatendo o
fascismo.

78. (G1 - cftsc 2010) Em 1929, ocorreu a queda da bolsa de


valores de Nova York que desencadeou a Crise de 1929, que se
prolongaria por boa parte da dcada de 1930. Nos Estados
Unidos, surgiu o New Deal para lutar contra a situao vigente.
No Brasil, vivemos nesse perodo a famosa Era Vargas (1930 a
1945), com destaque para o Estado Novo.
Sobre o Estado Novo, correto afirmar que:
a) pregava a interveno do Estado na economia contra o
liberalismo econmico.
b) pregava a lei da oferta e procura sem ao tutelar do
Estado.
c) pregava a democracia como nica soluo para os
momentos de crises.
d) pregava a liberdade social dos sindicatos como meio de
dilogo entre as classes trabalhadoras.
e) pregava a liberdade de imprensa como meio de
comunicao entre o Estado e a populao.

76. (Ufpr 2011) Com relao ao Estado Novo, de 1937 a 1945,


correto afirmar:
a) Foi um perodo de desenvolvimento do liberalismo
democrtico no pas, permitindo com isso a consolidao da
liderana poltica de Getlio Vargas.
b) Ampliou os conflitos oligrquicos e a presso do capital
internacional, culminando com o suicdio de Vargas.
c) A poltica desenvolvimentista de abertura ao capital
estrangeiro permitiu o crescimento das alianas polticas e
comerciais entre Brasil e Estados Unidos.
d) A proximidade poltica de Vargas com os regimes
totalitrios nazi-fascistas levou o Brasil a apoiar militarmente
os pases do Eixo na Segunda Guerra Mundial.
e) Foi marcado pela crtica democracia liberal e pela
organizao de um estado autoritrio, encarregado de
promover o progresso dentro da ordem.
RESPOSTA: E
COMENTRIO: O Estado Novo foi caracterizado pela ditadura
sob a liderana de Getlio Vargas, marcada por autoritarismo
e represso, ao mesmo tempo em que preservou uma poltica
econmica nacionalista e industrialista. Refletiu um momento
de influencia das ideias fascistas sobre o Estado nacional.

RESPOSTA: A
COMENTRIO: O Golpe de Getlio Vargas foi articulado junto
aos militares e contou com o apoio de grande parcela da
sociedade, pois, desde o final de 1935, o governo havia
reforado sua propaganda anticomunista, amedrontando a
classe mdia, quer dizer, preparando-a para apoiar a
centralizao poltica que desde ento se desencadeava. A
partir de novembro de 1937, Vargas imps a censura aos
meios de comunicao, reprimiu a atividade poltica,
perseguiu e prendeu inimigos polticos, adotou medidas
econmicas nacionalizantes.
79. (Uerj 2012)

77. (G1 - ifce 2011) O perodo Constitucional da Era Vargas, que


se estende de 1934 a 1937, foi marcado por tempos difceis e
conturbados. Melhor identifica esse perodo:
a) teve lugar um movimento de rebeldia conhecido como
Revoluo Constitucionalista, que exigia que se fizesse uma
nova constituio para o pas.
b) foi instituda uma nova moeda para o pas o cruzado
como forma de estabilizar o pas que sofria os efeitos da crise
de 1929.
c) pela primeira vez na histria do pas, um governante era
processado por crime poltico (impeachment), perdendo seu
mandato.
d) tem lugar a luta entre duas foras: nacionalistas e
entreguistas, ocasio em que acontece a nacionalizao do
petrleo o petrleo nosso.
e) foram firmados dois movimentos antagnicos, que
refletiam, aqui, o crescimento das ditaduras na Europa: a Ao
Integralista Brasileira (fascista) e a Aliana Nacional
Libertadora (comunista).
RESPOSTA: E
COMENTRIO: Foi o nico momento da Era Vargas marcado
pelo liberalismo e pelo respeito ordem constitucional;
contraditoriamente,
abriu
possibilidades
para
que
movimentos de oposio se organizassem e manifestassem,
em confronto direto com a poltica varguista. De um lado, os
comunistas conseguiram criar um amplo movimento de
massas, com um projeto de esquerda, atravs da Aliana
Nacional Libertadora (ANL) e, de outro, percebe-se a
organizao do fascismo na Ao Integralista Brasileira (AIB).
Tal situao entendida como reflexo da situao
internacional, muitas vezes denominada de polarizao
ideolgica.

No governo Vargas, foi criado o Servio Especial de


Mobilizao de Trabalhadores para a Amaznia S.E.M.T.A.,
uma medida direcionada para a participao do Brasil na
Segunda Guerra Mundial (1939-1945).
Com base no cartaz, as aes programadas por esse servio
tiveram como principal objetivo:
a) ocupao militar relacionada redefinio das fronteiras
nacionais
b) proteo dos trabalhadores rurais em resposta depresso
econmica
c) estmulo migrao para explorao de recursos naturais
estratgicos
d) demarcao de reservas florestais associada poltica de
defesa ambiental

17

RESPOSTA: C
COMENTRIO: Ao observar as informaes dadas pelo
enunciado, o estudante se depara com a origem do cartaz:
Mais borracha para a vitria do Brasil. O enunciado refora a
ideia de que o pas vivia um esforo de guerra e de que
necessitava de trabalhadores para a regio da Amaznia.
Portanto, a observao cuidadosa das informaes oferecidas
permite a concluso.
80. (Uerj 2011) A avenida Presidente Vargas, inaugurada em 7 de
setembro de 1944, existe graas demolio de mais de 500
edificaes nas quadras compreendidas entre a rua General
Cmara e a rua de So Pedro; estas duas ruas passaram a
constituir as pistas laterais (lados mpar e par,
respectivamente), enquanto a pista central ocupa o lugar das
quadras demolidas.
Adaptado de www.skyscrapercity.com

A abertura da avenida Presidente Vargas foi uma das


principais obras realizadas na cidade do Rio de Janeiro durante
a gesto do prefeito Henrique Dodsworth, de 1937 a 1945.
No contexto da poca, essa obra tinha como principal
objetivo:
a) valorizao imobiliria de regies favelizadas
b) integrao das redes rodoviria e ferroviria urbanas
c) ampliao dos acessos entre as zonas central e norte
d) modernizao da infraestrutura habitacional da rea
porturia
RESPOSTA: C
COMENTRIO: Durante a gesto de Henrique Dodsworth, no
governo da ento capital federal, no Estado Novo, a cidade do
Rio de Janeiro foi alvo de reformas urbanas voltadas, entre
outros aspectos, para a ampliao e modernizao de vias de
circulao rodoviria, com o objetivo de facilitar comunicaes
entre as zonas centrais e perifricas do municpio. Entre as
obras que se enquadraram nessa proposta, incluram-se a
construo da avenida Presidente Vargas, conectando a atual
avenida Rio Branco e rea do porto Praa Onze, e a
construo da avenida Brasil.
81. (Uece 2010) Acerca da Constituio outorgada em 1937 por
Getlio Vargas, podemos afirmar corretamente que
a) aboliu o cargo de vice-presidente da Repblica.
b) estabeleceu a diviso do pas em provncias.
c) instituiu a separao entre a Igreja e o Estado.
d) propiciou o voto feminino.
RESPOSTA: A
COMENTRIO: A Carta de 1937, tambm conhecida como
Polaca, reflete o carter centralizador e autoritrio do
perodo do Estado Novo, a ditadura varguista, extinguindo o
cargo de vice-presidente da Repblica. Outra possibilidade de

resoluo seria por excluso de alternativas. O termo


provncia designa a diviso do pas na poca imperial. A
separao entre Estado e Igreja marcou a organizao do pas
desde a primeira constituio republicana em 1891; e o voto
feminino apareceu pela primeira vez na Constituio de 1934.
82. (G1 - cftsc 2010) Sobre a Era Vargas, podemos dizer que nesse
perodo para sair da Crise de 1929, foi implementada uma
poltica de substituio de importaes que buscou integrar o
territrio nacional.
Nesse aspecto, podemos afirmar que a substituio das
importaes ligada integrao do territrio nacional teve
como objetivos:
a) fortalecer o modelo agrrio exportador e oferecer preos
mais baixos no mercado externo, tendo em vista a ampliao
das vendas para os EUA, gerando o capital necessrio aos
projetos de integrao do territrio.
b) ampliar a produo industrial no Brasil e criar um mercado
interno para consumo dessa produo, integrando boa parte
do territrio nacional, especialmente o eixo Centro-Sul, onde
se concentraram as atividades econmicas mais importantes
desse perodo.
c) expandir o setor de servios e fortalecer os sindicatos, de
modo que fosse possvel a integrao do territrio nacional
pela unio dos trabalhadores em grandes centrais sindicais.
d) reforar o setor agrcola e diminuir os investimentos em
indstrias para que houvesse menos importaes de mquinas
e equipamentos dos EUA e Europa, necessrios s empresas
do setor industrial que estavam plenamente integradas no
territrio nacional.
e) fortalecer as exportaes de mquinas e equipamentos e
gerar investimentos externos no pas, para que o Brasil se
desenvolvesse com maior rapidez, permitindo a construo de
uma infraestrutura homognea de integrao do territrio
nacional.
RESPOSTA: B
COMENTRIO: A questo apresenta duas caractersticas
fundamentais do populismo, o industrialismo e o
nacionalismo. A industrializao do pas ganhou impulso ao se
aproveitar da crise internacional que, se por um lado reduziu
os espaos internacionais para o caf, por outro abriu
possibilidades de crescimento industrial, na medida em que a
capacidade das potncias se reduziu.
83. (Enem 2 aplicao 2010) Os generais abaixo-assinados, de
pleno acordo com o Ministro da Guerra, declaram-se dispostos
a promover uma ao enrgica junto ao governo no sentido de
contrapor medidas decisivas aos planos comunistas e seus
pregadores e adeptos, independentemente da esfera social a
que pertenam. Assim procedem no exclusivo propsito de
salvarem o Brasil e suas instituies polticas e sociais da
hecatombe que se mostra prestes a explodir.
Ata de reunio no Ministrio da Guerra, 28/09/1937.
BONAVIDES, P.; AMARAL. R.Textos polticos da histria do
Brasil, v. 5. Braslia: Senado Federal, 2002 (adaptado).
Levando em conta o contexto poltico-institucional dos anos
1930 no Brasil, pode-se considerar o texto como uma tentativa
de justificar a ao militar que iria
a) debelar a chamada Intentona Comunista, acabando com a
possibilidade da tomada do poder pelo PCB.
b) reprimir a Aliana Nacional Libertadora, fechando todos os
seus ncleos e prendendo os seus lderes.
c) desafiar a Ao Integralista Brasileira, afastando o perigo de
uma guinada autoritria para o fascismo.
d) instituir a ditadura do Estado Novo, cancelando as eleies
de 1938 e reescrevendo a Constituio do pas.

18

e) combater a Revoluo Constitucionalista, evitando que os


fazendeiros paulistas retomassem o poder perdido em 1930.

Sem alma porque a tinham vendido


Ao estrangeiro de vestes vermelhas....
Eram os filhos malditos de Caim.
MAUL, Carlos. Toque de Silncio.

RESPOSTA: D
COMENTRIO: O discurso dos militares contido no documento
apresentado claramente anticomunista e representa o apoio
do exrcito a articulao golpista liderada por Getlio Vargas
que instituiu a ditadura do Estado Novo no Brasil. Desde 1934,
com a organizao da ANL, passando pela tentativa comunista
de 1935, o discurso anticomunista tornara-se o principal
elemento de propaganda dos setores mais conservadores,
refletindo, em parte, a situao de polarizao existente na
Europa, que culminou com a ascenso de grupos fascistas ao
poder em diversos pases.

CORRETO afirmar que, nesses versos, o autor faz referncia


a) insurreio de novembro de 1935.
b) Revoluo Constitucionalista de 1932.
c) Revoluo de Outubro de 1930.
d) ao golpe civil-militar de 1964.
RESPOSTA: A
COMENTRIO: A o emprego da expresso vestes vermelhas,
alude Intentona Comunista, uma tentativa de golpe contra o
governo de Getlio Vargas realizado em novembro de 1935
pelo Partido Comunista Brasileiro em nome da Aliana
Nacional Libertadora. A Revolta Vermelha de 1935 foi uma
conspirao de natureza poltico-militar contra um governo
considerado autoritrio. Por suas reivindicaes polticas
imediatas, caracterizou-se como um protesto polticoinstitucional dentro no quadro dos movimentos tenentistas
realizados no Brasil desde a dcada de 1920.

84. (G1 - cftsc 2010)

No prdio acima, a Escola de Aprendizes Artifcies iniciou suas


atividades, no dia 1 de setembro de 1910, na rua Almirante
Alvim, n 17, Florianpolis, em prdio cedido pelo Sr.
Governador do Estado de Santa Catarina, Cel. Gustavo
Richard.
Fonte: ALMEIDA, Alcides Vieira. Dos aprendizes artfices ao
CEFET/SC. Florianpolis: Agnus, 2002. p. 11.

86. (G1 - cftmg 2010) Em 1930, Getlio Vargas j era um lder


conhecido da poltica do Rio Grande do Sul e experimentara
cargos a nvel federal (...). Embora para os gachos fosse uma
importante figura poltica, no conjunto do movimento
revolucionrio de 1930 era apenas uma das lideranas.
Assumiu a chefia do Governo Provisrio apenas por ter sido o
candidato derrotado da eleio que motivara o movimento.
Ou seja, em incios da dcada de 1930, Vargas era um dentre
outros nomes que poderiam ascender na poltica nacional. Se
seu prestgio cresceu ao longo dessa dcada, isso se deveu a
um conjunto de fatores.
(GOMES, ngela de Castro. A ltima cartada. In: Nossa
Histria, ago. 2004, p. 16)
No constitui um fator que favoreceu o crescimento do
prestgio poltico de Vargas ao longo das dcadas de 1930 e
1940, a
a) relao parcial da propaganda do governo com suas
iniciativas.
b) exigncia de lideranas personalistas na prtica poltica
moderna.
c) autonomia das elites poltico-econmicas regionais,
consagrando o pacto federativo.
d) censura dos meios de comunicao de massa exercidos
pelos rgos estatais.

O Decreto 7566 de 23 de setembro de 1909 cria, nas capitais


dos Estados da Repblica, as Escolas de Aprendizes Artfices
destinadas ao ensino profissional primrio e gratuito. A
histria brasileira foi marcada por diversos fatos no perodo
entre a criao das Escolas de Aprendizes Artfices at o atual
Instituto Federal de Educao, Cincia e Tecnologia de Santa
Catarina (IF-SC).
Assinale a alternativa que apresentar corretamente um fato
ocorrido, ao longo do sculo XX, na histria brasileira.
a) Repblica Juliana
b) Estado Novo
c) Guerra de Canudos
d) Guerra do Paraguai
e) Lei urea
RESPOSTA: B
COMENTRIO: O Estado Novo no foi um fato; porm a
nica alternativa relativa ao sculo XX.
85. (Ufmg 2010) Leia estes versos:
Mataram-nos traio quando dormiam,
E foram companheiros que os mataram
No foi a guerra, foi o crime que os matou
Dormiam no quartel, de madrugada,
Mas a seu lado,
Em sinistra viglia,
Companheiros sem alma conspiravam,

RESPOSTA: C
COMENTRIO: O exerccio valoriza o entendimento do papel
de Getlio Vargas na histria do pas a partir de sua ascenso
Presidncia da Repblica. Ao contrrio do que se propaga,
Vargas teve sua imagem construda gradualmente, apoiado e
favorecido por um conjunto de fatores externos e internos,
incluindo o elevado grau de dependncia das elites regionais
frente ao poder central - estrutura criada desde a proclamao
da Repblica, diferentemente do que afirma a alternativa
correta.
87. G1 - cftmg 2010) A "revoluo de 30" inaugurou uma etapa
decisiva do processo de constituio do Estado brasileiro
como um Estado nacional, capitalista e burgus. A quebra das
autonomias estaduais suporte das tradicionais oligarquias
regionalizadas - resultou na crescente centralizao do poder
que alocava no Poder Executivo federal os comandos sobre as
polticas econmica e social e o aparelho coercitivo-repressivo.
Constitua-se o poder do Estado como poder unificado e
19

genrico, representativo do "interesse geral" em sua


concretude. Apesar de iniciado no imediato ps-30, o marco
na acelerao desse processo foi a instaurao do Estado
Novo, em 1937.
FONTE: MENDONA, Snia Regina de. As bases do
desenvolvimento capitalista dependente: da industrializao
restringida internacionalizao. In: LINHARES, Maria Yedda
(org.). Histria Geral do Brasil. Rio de Janeiro: Campus, 1996.
p. 282 (adaptado).
De acordo com o trecho acima, a Revoluo de 30 representou
a(o)
a) poltica intervencionista do Estado associada ao
fortalecimento da estrutura produtiva.
b) dissoluo das disputas em nvel regional com o advento de
maior liberdade poltica.
c) retomada das prticas centralizadoras caractersticas do
perodo inaugural da Repblica no Brasil.
d) descentralizao poltica progressiva, relacionada ao
suposto interesse de todos os trabalhadores.

obrigatria pelas emissoras de rdio. Alm disso, a msica se


destacou na programao das emissoras de rdio, parte delas
com forte contedo ideolgico, de valorizao do pas em seus
aspectos naturais e culturais, destacando inclusive os valores
vinculados ao trabalho.
89. (G1 - cftmg 2010) A relao de Getlio Vargas com os
trabalhadores pode ser considerada
a) populista, porque houve uma manipulao das massas
atravs de uma promessa de reforma agrria.
b) contraditria, na medida em que os trabalhadores
conquistaram direitos, mas eram impedidos de lutar por eles.
c) desptica, devido criao do Departamento de Imprensa e
Propaganda, responsvel pela censura s opinies dos
trabalhadores.
d) amistosa, uma vez que nesse perodo se deu a criao das
leis trabalhistas, comemorada em inmeras manifestaes de
apoio ao governo.
RESPOSTA: B
COMENTRIO: Apesar de caracterizado como populista, a
poltica trabalhista de Vargas se desenvolveu para as massas
urbanas e nunca houve um projeto de reforma agrria.
Marcado, na maior parte do tempo, pela centralizao do
poder e pela dependncia dos sindicatos em relao ao
Estado, os trabalhadores estavam subordinados poltica
oficial e a prtica do peleguismo de suas lideranas reforava
essa dependncia.

RESPOSTA: A
COMENTRIO: O texto destaca a transio de um Brasil
agrrio para um Brasil urbano, industrial e moderno. A
Revoluo de 30 marca essa alterao. Na medida em que o
novo governo representa os setores sociais emergentes, numa
poca em que havia uma crise internacional, que abriu
espaos para a ao do empresariado nacional, os elementos
agrrios ficam em segundo plano, mas sobrevivem.
88. (G1 - cftmg 2010) Leia o trecho da letra de Cantores do Rdio,
composta por Braguinha (Joo de Barro),
Lamartine Babo e Alberto Ribeiro:
Ns somos os cantores do rdio
Levamos a vida a cantar
De noite embalamos teu sono
De manh nos vamos te acordar
Ns somos os cantores do rdio
Nossas canes, cruzando um espao azul,
Vo reunindo
Num grande abrao
Coraes de norte a sul [...]

90. (Pucrj 2010) (...) Preciso de vs, trabalhadores do Brasil,


meus amigos, meus companheiros de uma longa jornada (...).
Preciso de vossa unio; preciso que vos organizeis solidamente
em sindicatos, preciso que formeis um bloco forte e coeso ao
lado do governo (...). Preciso de vossa unio para lutar contra
os sabotadores, para que eu no fique prisioneiro dos
interesses dos especuladores e dos gananciosos, em prejuzo
dos interesses do povo.
Getlio Vargas, no Estdio Vasco da Gama, 01/05/1951.
Considere o segundo governo de Getlio Vargas (1951-1954),
o trecho acima e EXAMINE as afirmativas:
I Vargas se dirige aos trabalhadores do Brasil, urbanos e
rurais, beneficirios da legislao trabalhista implantada
durante o seu primeiro governo.
II O tom de apelo para que os trabalhadores se unissem ao
lado do governo evidencia a busca pelo apoio popular frente
oposio de setores militares e do empresariado brasileiro
ligado ao capital internacional.
III Sobre a unio dos trabalhadores para lutar contra os
sabotadores, Vargas est fazendo aluso aos comunistas, que
pretendiam assumir o poder no Brasil naquela poca.
IV Ainda que se apresente como garantidor dos interesses
do povo, defendendo a ampliao da legislao trabalhista,
Vargas enfrenta reivindicaes dos trabalhadores, ento
atingidos pela alta do custo de vida.

Essa msica, gravada por Carmen e Aurora Miranda, apareceu


no filme Al Al Carnaval de 1936. Sobre o papel do rdio nas
dcadas de 1930-40, incorreto afirmar que
a) o programa Hora do Brasil mostrou o poder do rdio como
instrumento de mobilizao poltica.
b) a difuso do rdio no Brasil decaiu no fim do Estado Novo
com o estmulo do governo para compra de televisores.
c) o alto ndice de analfabetismo, associado a baixa renda da
populao, contribuiu para a massificao do rdio como meio
de comunicao.
d) a popularizao do rdio foi intensa ao ponto de programas
de auditrio, musicais, radionovelas e campeonatos esportivos
terem alcance nacional.
RESPOSTA:[B]
COMENTRIO: A radiodifuso, iniciada no Brasil no incio dos
anos 20, ganhou importante papel como meio de
comunicao, massificando-se nas dcadas seguintes, quando
passou a ser utilizado como instrumento poltico,
principalmente na poca do Estado Novo. O programa Voz do
Brasil iniciou-se em 1935 com o objetivo de propagandear as
realizaes do governo federal. Em 1939, quando Vargas j
estabelecera no pas a ditadura do Estado Novo, o programa,
rebatizado como A hora do Brasil, tornou-se transmisso

Assinale a alternativa correta:


a) Somente as afirmativas I e III esto corretas.
b) Somente as afirmativas I, II e III esto corretas.
c) Somente as afirmativas II e IV esto corretas.
d) Somente as afirmativas I, III e IV esto corretas.
e) Todas as afirmativas esto corretas.
RESPOSTA: C
COMENTRIO:A afirmativa I est incorreta, pois, durante o
governo de Getlio Vargas, a legislao trabalhista
20

contemplou apenas os trabalhadores urbanos, no vigorando


entre os rurais. A afirmativa III est incorreta porque a
passagem do documento Preciso de vossa unio para lutar
contra os sabotadores, para que eu no fique prisioneiro dos
interesses dos especuladores e dos gananciosos-, faz aluso
aos interesses dos setores ligados ao capital internacional que
faziam crticas ao projeto nacionalista do governo de Getlio
Vargas, e no a uma possvel ameaa comunista.
91. (Enem 2010) De maro de 1931 a fevereiro de 1940, foram
decretadas mais de 150 leis novas de proteo social e de
regulamentao do trabalho em todos os seus setores.
Todas elas tm sido simplesmente uma ddiva do governo.
Desde a, o trabalhador brasileiro encontra nos quadros gerais
do regime o seu verdadeiro lugar.
DANTAS, M. A fora nacionalizadora do Estado Novo. Rio de
Janeiro: DIP, 1942. Apud BERCITO, S. R. Nos Tempos de
Getulio: da revoluo de 30 ao fim do Estado Novo. So
Paulo: Atual, 1990.
A adoo de novas polticas pblicas e as mudanas jurdicoinstitucionais ocorridas no Brasil, com a ascenso de Getlio
Vargas ao poder, evidenciam o papel histrico de certas
lideranas e a importncia das lutas sociais na conquista da
cidadania. Desse processo resultou a
a) criao do Ministrio do Trabalho, Indstria e Comrcio,
que garantiu ao operariado autonomia para o exerccio de
atividades sindicais.
b) legislao previdenciria, que proibiu migrantes de
ocuparem cargos de direo nos sindicatos
c) criao da Justia do Trabalho, para coibir ideologias
consideradas perturbadoras da "harmonia social".
d) legislao trabalhista que atendeu reivindicaes dos
operrios, garantido-lhes vrios direitos e formas de proteo.
e) decretao da Consolidao das Leis do Trabalho (CLT), que
impediu o controle estatal sobre as atividades polticas da
classe operria.

RESPOSTA: A
COMENTRIO: Desde o governo provisrio iniciado em 1930
j se podia perceber o carter autoritrio e centralizador do
governo Vargas. A Constituio de 1937, apelidada de
polaca, outorgada por Vargas, deu incio ao Estado Novo,
perodo ditatorial da Histria.
93. (G1 - cftsc 2010) A chamada Era Vargas foi o longo perodo
em que o chefe da Revoluo de 1930, o gacho Getlio
Vargas, manteve-se no poder. Getlio governou por 15 anos
ininterruptos em diferentes situaes: como chefe do Governo
Provisrio (1930 a 1934), como presidente eleito pela
Assembleia Constituinte (de 1934 a 1937) e como ditador
(1937 a 1945). Mas suas linhas bsicas de atuao pouco
mudaram nesse perodo: centralismo poltico-administrativo,
industrialismo, nacionalismo. O ltimo mandato de Getlio
Vargas (1950 a 1954) foi interrompido com sua trgica morte
pelo suicdio. O iderio nacionalista, o estatismo e o
trabalhismo autoritrio e paternalista deixaram marcas
profundas na vida brasileira.
Adaptado de: TEIXEIRA,Francisco M. P. Brasil, Histria e
Sociedade. So Paulo: tica, 2000. p. 260
Sobre a Era Vargas, correto afirmar que:
a) a Rebelio Constitucionalista, ocorrida em So Paulo no ano
de 1932, apoiou e garantiu a posse e a permanncia de
Getlio Vargas na presidncia do Brasil.
b) foi durante a era Vargas que aconteceu a Primeira Guerra
Mundial e o Brasil aliou-se Alemanha para combater contra
os Estados Unidos da Amrica do Norte.
c) o governo de Getlio, no perodo de 1934 a 1945, foi
chamado de populista por ser um regime democrtico com
eleies diretas para todos os cargos do executivo, presidente,
governadores e prefeitos.
d) durante seu governo, Vargas ignorou o desenvolvimento da
indstria e a participao dos operrios na vida do pas, dando
ateno s para o setor agrrio.
e) o golpe de estado de 1937, realizado por Getlio Vargas,
instituiu no Brasil o Estado Novo, um governo autoritrio e
centralizador com o poder nas mos do presidente.

RESPOSTA: D
COMENTRIO: A questo apresenta uma delimitao
cronolgica, at 1940, que exclui a CLT, elaborada em 1943 e
ampliou a ingerncia do Estado nas questes trabalhistas,
prtica adotada pelo Governo Vargas desde seu incio.
A poltica trabalhista de Vargas um dos aspectos mais
lembrados desse perodo e, se de um lado garantiu direitos
aos trabalhadores, de outro teve como objetivo manter o
controle sobre a organizao operria. A represso as
ideologias perturbadoras ficou a cargo da represso policial
e no do Ministrio do Trabalho.
92. (Upe 2010) Viver a democracia era o desejo de muitos grupos
polticos existentes no Brasil dos anos 1930.
No entanto, o governo de Getlio Vargas seguia outros
caminhos, enfrentando as oposies.
Com a Constituio de 1937, Getlio Vargas
a) centralizou mais ainda o poder poltico, firmando o
autoritarismo.
b) procurou modernizar a sociedade, multiplicando os partidos
polticos.
c) refez a legislao sindical, garantindo as reivindicaes
operrias.
d) fortaleceu normas liberais, sem, contudo, deixar seu poder
de centralizador.
e) trouxe ideias sociais mais avanadas, imitando modelos
europeus.

RESPOSTA: E
COMENTRIO: Apesar de Getlio no ter sido o chefe da
Revoluo, o texto faz uma sntese do perodo em que
governou dividido em etapas com caractersticas peculiares.
Em 1937, Vargas e seus aliados, com apoio dos integralistas,
deram um golpe de Estado, impondo um novo modelo de
governo denominado de Estado Novo. Desde 1937 at 1945, o
pas viveu sob uma ditadura de tendncia fascista, que
preservou diversas caractersticas do populismo.
94. (Unesp 2010) Sobre o movimento constitucionalista de 1932,
possvel afirmar que
a) foi resultado da poltica federal, que impedia a exportao
do caf de So Paulo para o Ocidente europeu.
b) atrasou o processo de democratizao brasileira
empreendido por Getlio Vargas a partir de 1930.
c) tinha, como principal objetivo, a separao do estado de
So Paulo do restante da federao.
d) levou o governo federal a negociar com a oligarquia paulista
e a fazer concesses a seus interesses.
e) obteve sucesso, derrotando as tropas de Vargas e
devolvendo a presidncia aos cafeicultores.
RESPOSTA: D
COMENTRIO: A alternativa D esta correta, porm no
destaca as principais caractersticas ou consequncias do
movimento. A revoluo paulista no envolveu apenas
interesses das oligarquias e, de forma geral, representou uma
21

derrota para a elite de So Paulo, apesar de algumas


concesses polticas aos representantes do Estado. Note que o
modelo que havia se iniciado a partir de 30 foi preservado e,
nos anos seguintes, consolidou-se o populismo como forma
poltica predominante.

Constri um edifcio e depois no pode entrar


O samba Pedreiro Waldemar, composto por Wilson Batista e
Roberto Martins, em 1946,
a) livre da censura do Estado Novo, aponta para a luta de
classes, denunciando o intenso grau de explorao a que
estavam submetidos os trabalhadores brasileiros.
b) devido s imposies do Estado Novo, realiza uma
exaltao ao trabalho e ressalta o valor do trabalhador
brasileiro.
c) faz parte da propaganda ufanista desenvolvida ao longo da
ditadura militar, que exaltava as realizaes do povo
brasileiro.
d) ia na contramo do discurso oficial da poca e exaltava o
trabalho, visto pelos compositores como o nico meio de
ascenso social para as pessoas de classe baixa.
e) denunciava as formas de trabalho escravo que persistiam
no Brasil logo aps a aprovao da Lei urea.

95. (Pucrj 2010) O voto direto e universal, smbolo da democracia


e da igualdade poltica, foi institudo na Frana em 1848, fruto
de experincias e reflexes anteriores que propiciaram sua
conquista e sua posterior afirmao, no decorrer do sculo XX.
Sobre as experincias modernas e contemporneas que
contriburam para a conquista e afirmao do sufrgio
universal, CORRETO afirmar que:
a) na Revoluo Gloriosa, inaugurou-se a Era dos Direitos,
abrindo caminho para a eleio do legislativo, com o voto
censitrio, masculino e feminino, para a Cmara dos Comuns.
b) na Revoluo Americana, o ideal democrtico foi aplicado
na Constituio de 1787, exemplificado pela abolio da
escravido e pela adoo do sufrgio universal e da eleio
direta.
c) na Revoluo Francesa, a doutrina da soberania popular foi
aplicada pelo governo jacobino a partir de 1793, garantindo o
sufrgio universal masculino e a defesa da liberdade de
expresso e de oposio.
d) na Amrica Latina, aps as revolues de independncia, a
elite criolla preocupou-se em constituir estados nacionais e
em integrar as classes populares na participao poltica.
e) no Brasil republicano, em 1932, foi institudo o Cdigo
Eleitoral Brasileiro, considerando eleitor o cidado maior de
21 anos, implantando-se, assim, o sufrgio universal sem
distino de sexo.
RESPOSTA: E
COMENTRIO: A afirmativa (A) est incorreta, pois o voto
feminino s foi obtido parcialmente em 1918 (para mulheres
acima de 30 anos) e a paridade de voto com os homens, 21
anos, foi uma conquista de 1928.
A afirmativa (B) est incorreta, pois a abolio da escravatura
nos Estados Unidos se deu atravs da chamada Emancipation
Proclamation, promulgada em 1 de janeiro de 1863, pelo
presidente Abraham Lincoln.
A afirmativa (C) est incorreta, pois O Terror foi um perodo da
Revoluo Francesa compreendido entre maio de 1793 (queda
dos girondinos) e julho de 1794 (priso de Robespierre, exlder dos Jacobinos). Durante esse perodo, as garantias civis
foram suspensas, e o governo revolucionrio perseguiu e
eliminou seus adversrios.
A afirmativa (D) est incorreta, pois, na primeira metade do
sculo XIX, o direito ao voto tendia a obedecer ao critrio
censitrio, garantindo a participao poltica para as elites
fundirias e para os grandes comerciantes exportadores.
Apenas na primeira metade do sculo XX, o voto masculino
universal comeou a ser adotado: em 1930, na Argentina;
democracia com voto restrito aps 1932 no Chile; ampliao
do direito ao voto em 1936, na Colmbia, e direito masculino
ao sufrgio em 1913, na Costa Rica.
96. (G1 - ifsp 2012) Pedreiro Valdemar
Voc conhece o pedreiro Valdemar?
No conhece
Mas eu vou lhe apresentar
De madrugada toma o trem da circular
Faz tanta casa e no tem casa pra morar
Leva a marmita embrulhada no jornal
Se tem almoo, nem sempre tem jantar
O Valdemar, que mestre no ofcio

RESPOSTA: A
COMENTRIO: A letra da msica faz uma crtica social
condio do trabalhador, no sentido de ser o elemento
produtivo e de no poder usufruir do fruto de seu trabalho. O
momento em que a composio foi escrita, 1946,
caracterizado pela redemocratizao do pas, um ano aps o
final do Estado Novo, num clima de liberdade de expresso
oposto ao que vigorou durante a ditadura varguista.
97. 28. (Unesp 2011) A construo de Braslia durante o governo
Juscelino Kubitschek (1956-1961) teve, entre suas motivaes
oficiais,
a) afastar de So Paulo a sede do governo federal, impedindo
que a elite cafeicultora continuasse a control-lo.
b) estimular a ocupao do interior do pas, evitando a
concentrao das atividades econmicas em reas litorneas.
c) deslocar o funcionalismo pblico do Rio de Janeiro,
permitindo que a cidade tivesse mais espaos para acolher os
turistas.
d) tornar a nova capital um importante centro fabril, reunindo
a futura indstria de base do Brasil.
e) reordenar o aparato militar brasileiro, expandindo suas
reas de atuao at as fronteiras dos pases vizinhos.
RESPOSTA: B
COMENTRIO: A construo de uma nova capital para o pas
foi integrada ao Plano de Metas e contribuiu para reforar o
discurso desenvolvimentista da campanha presidencial.
Oficialmente, a nova capital, no interior do pas, contribuiria
para reduzir a concentrao econmica nas reas litorneas e,
portanto, criaria uma dinmica de desenvolvimento do
interior.
98. (Ufrgs 2011) A denominada "Campanha da Legalidade",
ocorrida no Rio Grande do Sul no final de agosto de 1961, foi
uma consequncia da
a) renncia do presidente Jnio Quadros, que provocou a
mobilizao poltica para garantir a posse do vice-presidente
Joo Goulart.
b) vitria eleitoral do PTB, que supostamente ameaava os
setores conservadores da sociedade brasileira.
c) renncia do presidente Juscelino Kubitschek, fato que
provocou uma extensa mobilizao militar visando garantir a
posse de Joo Goulart.
d) vitria eleitoral do PSD, partido que tinha em seus quadros
diversos elementos supostamente golpistas.
e) poltica promovida por Leonel Brizola, que queria impedir a
tomada do poder pelos grupos ligados luta armada.

22

RESPOSTA: A
COMENTRIO: A campanha da Legalidade foi deflagrada por
Leonel Brizola, um dos lderes do PTB, logo aps a renncia de
Jnio Quadros. Tal campanha visava garantir a posse do Vicepresidente Joo Goulart que estava na China no cargo,
garantida pela Constituio, uma vez que os grupos
conservadores liderados pela UDN iniciaram um movimento
contra a posse.
99. (Uff 2011) Visto que, de fato, a Constituio de 1946
estabeleceu normas e medidas para a instalao de uma
estrutura democrtica no pas, dando ensejo a uma abertura
do processo poltico nos dezoito anos subsequentes, ao
observador mais descuidado a redemocratizao pode parecer
mais radical do que na realidade o foi.
SOUZA, Maria do Carmo Campello de. Estado e Partidos
Polticos no Brasil (1930-1964). So Paulo: Alfa-Omega, 1976,
p. 105.
Com base nas afirmaes contidas no texto, possvel afirmar
que
a) a redemocratizao iniciada em 1945 perdeu sua
radicalidade por ter sido apenas um ritual poltico, vazio de
efetivos partidos.
b) a redemocratizao de 1945 s pde existir em funo da
criao de trs novos grandes partidos polticos, totalmente
independentes de vnculos com o Estado Novo: o PSD, a UDN
e o PTB.
c) o retorno do pluripartidarismo e de eleies diretas foram
superpostos estrutura herdada do Estado Novo, marcada
pelo sindicalismo corporativista e pelo sistema de
interventorias.
d) a redemocratizao no foi radical devido preponderncia
que teve, junto a ela, a Unio Democrtica Nacional (UDN),
partido formado com o beneplcito de Vargas.
e) a hipertrofia do Poder Legislativo foi uma das
consequncias da redemocratizao.
RESPOSTA: C
COMENTRIO: Com a redemocratizao se formaram novos
partidos, que tiveram atuao destacada no perodo
subsequente, como por exemplo, o PTB e PSD (ligados ao
varguismo) e a UDN e PSP, (antivarguistas). Ao final da
ditadura, restabeleceu-se uma Constituio e organizao
democrtica do Estado, enfraquecendo o sindicalismo
corporativista e que eliminou o sistema de interventorias.
100. (Unicamp 2011) Em 30 de maro de 1964, o Presidente Joo
Goulart fez um discurso, no qual declarou: Acabo de enviar
uma mensagem ao Congresso Nacional propondo claramente
as reformas que o povo brasileiro deseja. O meu mandato ser
exercido em toda a sua plenitude, em nome do povo e na
defesa dos interesses populares.
(Adaptado de Paulo Bonavides e Roberto Amaral, Textos
polticos da histria do Brasil. Braslia: Senado Federal, 2002,
vol. 7, p. 884.)
Sobre o contexto em que esse discurso foi pronunciado,
possvel afirmar o seguinte:
a) Enfrentando a oposio de setores conservadores, Jango
tentou usar as reformas de base, que deveriam abranger a
reforma agrria, a eleitoral, a educacional e a financeira, para
garantir apoio popular ao seu mandato.
b) Quando Jango apresentou ao Congresso Nacional as
reformas de base, elas j haviam sido alteradas, abrindo mo
da reforma agrria, para agradar aos setores conservadores, e
no apenas s classes populares.
c) Com as reformas de base, Jango buscou afastar a fama de
esquerdista, colocando na ilegalidade os partidos comunistas,

mas motivou a oposio de militares e polticos nacionalistas,


ao abrir o pas ao capital externo.
d) Jango desenvolveu um plano de reformas que deveriam
alterar essencialmente as carreiras dos militares, o que
desagradava muitos deles, mas tambm reprimiu vrias greves
do perodo, irritando as classes populares.
RESPOSTA: A
COMENTRIO: O contexto do discurso de Joo Goulart de
forte polarizao, marca de uma grave crise poltica que
atingiu seu pice em 1964 e, nesse quadro, o presidente busca
reforar os apoios das camadas populares a seu governo a
partir das reformas de base, conjunto de medidas que se
caracterizavam por maior interveno do Estado na vida
econmica e poltica do pas e que reforou o discurso
oposicionista de esquerdismo de Jango, contribuindo para o
golpe militar.
101. (Ufu 2011) Sobre o governo de Joo Goulart (1963-1964),
correto afirmar:
a) Goulart procurou implementar todas as reformas de base,
como a reforma agrria, a reforma urbana e a maior
interveno do Estado na economia, sendo impedido pelo
golpe militar de 1964.
b) Goulart realizou acordos multilaterais com pases europeus
e os Estados Unidos para a criao de filiais das principais
empresas automobilsticas do mundo.
c) Goulart tinha amplo apoio do empresariado nacional, pois
possua ideias arrojadas para a poca, como fazer as reformas
de base, que aumentariam os lucros das empresas sediadas no
Brasil.
d) A Marcha da Famlia com Deus pela Liberdade, realizada em
1964, foi uma manifestao de homenagem a Joo Goulart em
defesa de seu governo e contra as ameaas dos militares.
RESPOSTA: A
COMENTRIO: As Reformas de Base formam o principal eixo
de desenvolvimento do governo Joo Goulart. Era um projeto
de forte contedo social e populista, que procurava atrair os
setores populares para o lado do governo, mas contou com
forte oposio das elites, urbana e agrria, da Igreja catlica e
da classe mdia.
102. (G1 - col.naval 2011) "Por j ter acumulado experincias
parciais de politica econmica planejada, pde o governo, na
gesto Kubitschek, executar sua mais sria e ambiciosa
experincia de planejamento integrado: o Programa de Metas
[...]. O Plano de Metas foi um marco na reelaborao
qualitativa das relaes entre Estado e economia no pas.
Integravam-se pela primeira vez, sob o controle do governo, as
atividades do capital pblico e do privado,atravs de um
planejamento que definia as prioridades de investimento, bem
como as estratgias para dirigir os recursos privados para
essas reas.
(MENDONA, Snia Regina de. Estado e Economia no Brasil:
Opes de Desenvolvimento. 3. ed. Rio de Janeiro, Graal, s/d,
p.59.)
Sobre o Programa de Metas, no governo Juscelino Kubitschek,
correto afirmar que:
a) foram estabelecidas trinta e uma metas, sendo priorizados
os setores de energia, transportes e indstrias de base.
b) o resultado do Programa de Metas foi desastroso,
sobretudo a produo industrial que teve uma acentuada
queda.
c) foram criados vrios mecanismos para impedir a entrada de
capitais estrangeiros, como a utilizao da Instruo 113 da
SUMOC.

23

d) o principal objetivo do Programa de Metas era a prioridade


no setor agrrio-exportador, privilegiando o caf e a soja.
e) com a criao da Superintendncia de Desenvolvimento do
Nordeste (SUDENE), em 1959, o foco da industrializao
transferiu-se para o Nordeste.
RESPOSTA: A
COMENTRIO: O Programa considerado um marco na
industrializao do Brasil, j que o perodo caracterizado
como sendo de progresso. Os investimentos externos forma
favorecidos pela instruo 113 da SUMOC, que permitia a
entrada de mquinas e equipamentos sem cobertura cambial.
Apesar da preocupao com o desenvolvimento do Nordeste,
foi na regio sudeste do pas que se concentraram os
principais polos de desenvolvimento.
No contexto dos cem anos do IF-SC (1909 a 2009), a economia
e a sociedade brasileira tambm sofreram diversas
transformaes. Sobre as transformaes scio-econmicas
ocorridas, ao longo do sculo XX no Brasil, CORRETO afirmar
que:
a) o extrativismo mineral ainda constitui a base econmica dos
estados que pertencem ao Complexo Regional do Centro-Sul.
b) at o incio do sculo XX, predominavam no Brasil as
chamadas indstrias de base (siderrgica, metal-mecnica e
de alimentos).
c) a partir da dcada de 1950, intensifica-se a
internacionalizao da economia brasileira sobretudo nas
atividades industriais.
d) a expectativa de vida, bem como, as taxas de mortalidade
infantil diminuram em todo territrio brasileiro.
e) a desnutrio e o analfabetismo foram erradicados em todo
territrio brasileiro.

103. (G1 - cftmg 2011) Durante as festividades da inaugurao de


Braslia, em 1960, ocorreu a apresentao de uma pea teatral
ao ar livre, que foi assim descrita por um jornalista da poca:
A luta pela interiorizao da capital e a construo de Braslia
foram retratadas [...]. Enxadas, ps, picaretas e outras
ferramentas foram distribudas entre os figurantes, que
representavam os operrios de Braslia [...]. Cada um dos
episdios era contrastado pela oposio de um velho [...]. Mas
quando por fim ele manifesta sua averso fundao de
Braslia, oito tratores barulhentos invadiram a cena e apareceu
no cu um helicptero. Dele desceu um homem, de porte
semelhante ao de Juscelino, a acenar para a multido, que
aplaudia, enquanto o velho pessimista foi metido no
helicptero que o levou para longe [...].
(apud DUARTE, Luiz Srgio. 21 de abril de 1960: inaugurao
de Braslia. IN: BITTENCOURT,
Circe (Org.). Dicionrio de datas da Histria do Brasil. So
Paulo: Contexto, 2007. p. 107-108).
Considerando o contexto histrico no qual a pea foi
representada, a melhor interpretao para sua mensagem :
a) O pas vivia um perodo de otimismo e a adeso a grandes
obras significava apoiar a modernidade.
b) Os construtores da cidade foram afastados de sua obra pela
discriminao social e as cidades satlites nasceram para
abrig-los.
c) A ocupao do interior do territrio nacional deslocou o
centro econmico do pas e possibilitou o nascimento do novo
sindicalismo.
d) As tradies rurais dos goianos foram simbolicamente
incorporadas pelo projeto modernista do pas e a cultura
nacional saiu enriquecida.

RESPOSTA: A
COMENTRIO: O governo Juscelino esteve sempre apoiado na
ideia de modernizar o pas. Desde a eleio presidencial,
quando da utilizao do slogan 50 anos de progresso em 5
anos de governo, JK vinculou seu nome e imagem do
progresso e modernizao do pas e Braslia tornou-se o
principal smbolo dessa modernidade que, estruturalmente,
foi implementada com o incentivo industrializao.

RESPOSTA: C
COMENTRIO: Refere-se ao perodo JK, considerado como de
progresso e de desenvolvimento para o pas. Marcado pela
abertura do mercado aos capitais e empresas estrangeiras e,
desse forma, considera-se que o modelo adotado foi o
capitalismo associado, ou capitalismo dependente, pois os
governantes da poca justificavam tal poltica como necessria
ao desenvolvimento nacional.
105. (G1 - cftmg 2010) "A partir da desapropriao do Engenho
Galilia, as ligas expandiram- se por todo o estado de
Pernambuco, chegando a ter, em 1961, 10.000 associados, e
no mnimo, 40 sedes municipais. (...)
As ligas tambm iriam se estender por vrios estados do
Nordeste e outras regies do pas, tornando-se
particularmente fortes nos estados da Paraba, Rio de Janeiro
e Gois".
FONTE: AZEVEDO, Fernando Antnio. As Ligas Camponesas.
So Paulo: Paz e Terra,
1982, p. 73.
A atuao das Ligas Camponesas est relacionada ao
a) apoio dos sindicatos rurais empresa agromercantil,
fundamental para o crescimento da economia.
b) alinhamento poltico entre os grandes produtores rurais do
nordeste e lideranas vinculadas ao campesinato.
c) compromisso dos trabalhadores rurais na defesa de uma
reforma agrria mantenedora da estrutura latifundiria.
d) modelo fundirio brasileiro, caracterizado por mecanismos
excludentes do campesinato da vida social e poltica.

104. (G1 - cftsc 2010) A histria centenria do Instituto Federal de


Educao, Cincia, e Tecnologia de Santa Catarina (IF-SC)
acima de tudo sinnimo de Escola: Escola de Aprendizes
Artfices, Escola Industrial, Escola Tcnica Federal de Santa
Catarina e Centro Federal de Educao Tecnolgica de Santa
Catarina. O Decreto N. 7.566 de 23 de setembro de 1909 cria
nas capitais dos Estados da Repblica as Escolas de Aprendizes
Artfices destinadas ao ensino profissional primrio e gratuito
e aos desprovidos de fortuna.

RESPOSTA: D
COMENTRIO: As ligas camponesas sindicatos rurais foram
um fenmeno social do final dos anos 50 e incio dos 60, que
congregaram camponeses, principalmente na regio nordeste,
24

trabalhadores que no possuam quaisquer direitos e eram


superexplorados, pois toda legislao trabalhista originria do
varguismo destinava-se apenas aos trabalhadores urbanos.

clebre frase. Denominada de monrosmo, foi considerada


como uma ideologia pan-americana, porm com uma viso
diferente daquela propugnada por Simon Bolvar.

106. (G1 - cftmg 2010) Com o Plano de Metas, lanado pelo


governo brasileiro na segunda metade dos anos 1950, houve
uma ruptura com a poltica econmica vigente no pas, desde
a dcada de 1930, porque se
a) favoreceu o pequeno capitalista nacional por meio de
crdito barato das agncias estatais.
b) incentivou a criao de mercados consumidores e a
produo de bens de consumo durveis.
c) garantiu a estabilizao da economia graas aos
mecanismos de distribuio de renda.
d) reservou o monoplio do processo de industrializao e de
avano tecnolgico as empresas
estatais.

108. (Unesp 2012) O caudilhismo um fenmeno poltico hispanoamericano do sculo XIX, que se associa
a) resistncia contra o intervencionismo norte-americano,
sobretudo nas reas do Caribe e Amrica Central.
b) s guerras civis entre unitrios e federalistas durante o
processo de formao dos Estados nacionais.
c) aos pensadores liberais que lutaram pela emancipao
poltica e econmica do continente.
d) s lideranas militares que atuaram nas guerras de
independncia e defenderam a unificao do continente.
e) ao temor, manifesto sobretudo na regio do Prata, de que o
Imprio brasileiro avanasse militarmente para o sul.

RESPOSTA: B
COMENTRIO: Apesar de considerado como populista, o
governo de JK desenvolveu a economia com algumas
caractersticas diferentes daquelas adotadas por Vargas desde
1930. Se num primeiro momento houve a preocupao com a
indstria de bens de produo, para estabelecer as bases do
desenvolvimento nacional, posteriormente o foco foi a
indstria de bens de consumo que caracterizou a maior
presena de empresas e capitais estrangeiros no Brasil, assim
como a solidificao de uma sociedade de consumo,
reforando a dependncia do pas frente ao capital e
tecnologia externas.

RESPOSTA: B
COMENTRIO: O caudilhismo foi um fenmeno poltico
tipicamente latino americano, principalmente nos pases de
lngua espanhola, aps a independncia e num primeiro
momento reflete a luta pela organizao do Estado, com
maior ou menor centralizao poltica, expressa em maior ou
menos autonomia s provncias. A luta pelo poder nas naes
recm-formadas envolveu grandes proprietrios, que
comandaram exrcitos pessoais formados nas guerras de
independncia e que buscavam o controle da nao atravs da
fora e de laos de fidelidade. A concentrao de poderes nas
mos de um lder local representava o reforo do poder
pessoal e do papel de sua regio na economia do pas.

HISTRIA II (GERAL)
107. (Unicamp 2012) Ningum mais do que eu partidrio de
uma poltica exterior baseada na amizade ntima com os
Estados Unidos. A Doutrina Monroe impe aos Estados Unidos
uma poltica externa que se comea a desenhar. () Em tais
condies a nossa diplomacia deve ser principalmente feita
em Washington (...). Para mim a Doutrina Monroe (...) significa
que politicamente ns nos desprendemos da Europa to
completamente e definitivamente como a lua da terra.

109. (Cesgranrio 2011) Se de fato a Gr-Bretanha tivesse sido a


maior fora por detrs da Guerra da Trplice Aliana contra o
Paraguai, ela estaria adotando poltica e comportamento
totalmente incompatveis com as polticas e os
comportamentos que regiam as suas relaes com a Amrica
Latina, como um todo, naquela poca.
BERTHEL, Leslie. In: NARLOCH, Leandro. Guia Politicamente
Incorreto da Histria do Brasil. So Paulo: Leya, 2009, p. 171172.

(Adaptado de Joaquim Nabuco, citado por Jos Maria de


Oliveira Silva, Manoel Bonfim e a ideologia do imperialismo
na Amrica Latina, em Revista de Histria, n. 138. So Paulo,
jul. 1988, p.88.)

Ao contradizer a afirmao feita por muitos historiadores


acerca do interesse ingls na Guerra do Paraguai, o autor se
baseia na principal caracterstica das relaes entre Amrica
Latina e Gr-Bretanha durante todo o sculo XIX, que foi o(a)
a) controle poltico e econmico das regies insulares da
Amrica como forma de garantir o acesso ingls ao Pacfico
Sul.
b) desinteresse pelo futuro das novas naes independentes,
j que os investimentos ingleses na frica e na Amrica do
Norte eram muito mais lucrativos.
c) defesa do domnio poltico ingls sobre as colnias ibricas
que se tornavam independentes.
d) manuteno das colnias inglesas no Caribe como forma de
garantir a permanncia do comrcio triangular.
e) manuteno da liberdade comercial nos novos pases
independentes, como forma de garantir mercados e vantagens
para os britnicos.

Sobre o contexto ao qual o poltico e diplomata brasileiro


Joaquim Nabuco se refere, possvel afirmar que:
a) A Doutrina Monroe a que Nabuco se refere, estabelecida
em 1823, tinha por base a ideia de a Amrica para os
americanos.
b) Joaquim Nabuco, em sua atuao como embaixador,
antecipou a poltica imperialista americana de tornar o Brasil o
quintal dos Estados Unidos.
c) Ao declarar que a Amrica estava to distante da Europa
como a lua da terra, Nabuco reforava a necessidade
imediata de o Brasil romper suas relaes diplomticas com
Portugal.
d) O pensamento americano considerava legtimas as
intenes norte-americanas na Amrica Central, bem como o
apoio s ditaduras na Amrica do Sul, desde o sculo XIX.

RESPOSTA: E
COMENTRIO: A Inglaterra foi a grande defensora da
liberdade comercial na Amrica Latina, dado seu interesse em
eliminar o monoplio praticado pelas metrpoles Ibricas.
Mercados livres significavam a possibilidade da indstria
inglesa obter vantagens e, na prtica, controlar os mercados
latino-americanos.

RESPOSTA: A
COMENTRIO: A questo aborda uma ideia bsica acerca da
poltica externa dos Estados Unidos na poca do Presidente
James Monroe, momento das independncias na Amrica
Latina, sob cobia do imperialismo ingls, sintetizada na
25

COMENTRIO: A luta de independncia do Haiti foi


influenciada pelas ideias iluministas, pelas propostas dos
jacobinos franceses e foi marcada por grande violncia da
populao escrava contra os proprietrios rurais,
desorganizando a economia nas dcadas seguintes. A
Independncia de Cuba contou com apoio dos Estados Unidos
na luta contra a metrpole espanhola, devido aos interesses
na produo aucareira da Ilha, que passou a abastecer
diretamente o pas, sem o antigo intermedirio. A
independncia cubana no alterou suas estruturas
latifundirias, fato que pode ser visto apenas aps a Revoluo
Cubana de 1959, que abriu caminho para o socialismo.

110. (Ufsm 2011) Simn Bolvar (1783-1830) era "partidrio da


formao de grandes Estados e confederaes, porque sabia
que naes pequenas e fracas nada mais seriam do que
dependncias econmicas da Inglaterra e dos Estados
Unidos".
(CCERES, Florival. Histria da Amrica. So Paulo: Moderna,
1992. p.98.)
O pensamento e a figura do libertador Simn Bolvar se
tornaram representativos de um projeto de Amrica Latina, a
respeito do qual possvel afirmar:
I. Devido sua origem mestia, Simn Bolvar expressava a
ideia de uma sociedade americana liberta do domnio
espanhol e organizada politicamente, de modo a integrar os
povos nativos da Amrica e tambm os negros trazidos da
frica.
II. A formao de empresas estatais e a limitao do tamanho
das propriedades rurais faziam parte do iderio de Simn
Bolvar e foram combatidas pela aristocracia criolla, satisfeita
com a estrutura econmica e social colonial.
III. A Confederao Pan-Americana esboada por Bolvar
enfrentou a oposio de grupos oligrquicos, dispostos tanto a
se afirmarem como foras polticas dominantes em suas
regies quanto a negociarem com as potncias imperialistas.
IV. O projeto de unidade poltica de Simn Bolvar chocou-se
com os interesses das oligarquias rurais da Colmbia,
Venezuela e Equador e tambm da Inglaterra e Estados
Unidos.

112. (Ufv 2010) A expresso Risorgimento designa o conjunto de


movimentos heterogneos que desejaram a unificao da
Itlia no sculo XIX. A vertente vitoriosa que promoveu a
unificao da Itlia foi:
a) o projeto republicano de Giuseppe Mazzini, que criou o
movimento Jovem Itlia.
b) o movimento popular e secreto dos Carbonrios, que
defendeu a instituio de um Estado unitrio e laico, contra a
influncia da Igreja e do Imprio Austraco.
c) o Papado, que defendeu a instituio de uma monarquia
teocrtica com sede no Vaticano.
d) o movimento liderado pelo reino do Piemonte-Sardenha,
que adotou uma monarquia constitucional laica e favoreceu a
industrializao.

RESPOSTA: D
COMENTRIO: A primeira luta do movimento de unificao da
Itlia teve incio depois da deciso do Congresso de Viena
ceder territrios italianos ustria. As primeiras tentativas de
libertao do territrio italiano, foram conduzidas por uma
organizao revolucionria chamada de Jovem Itlia liderada
por Giuseppe Mazzini, que defendia a independncia e a
transformao da Itlia numa repblica democrtica.
Em 1848, os seguidores de Mazzini promoveram uma
manifestao contra a dominao austraca em territrios
italianos, mas foram vencidos pelo poderoso exrcito
austraco. Apesar da derrota, o ideal nacionalista permaneceu
e a partir dessa poca, a luta pela unificao passou a ser
liderada pelo Reino do Piemoste-Sardenha na figura de Camilo
Benso (Conde de Cavour), um dos lderes do Risorgimento,
movimento que pretendia fazer a Itlia reviver seus tempos de
glria. Para alcanar tal objetivo, Cavour teve o apoio da
burguesia e dos proprietrios rurais e colocou em prtica um
plano de modernizao da economia e do exrcito do
Piemonte. Aproximou-se da Frana e conseguiu ajuda militar
para enfrentar a ustria.

Est(o) correta(s)
a) apenas I e II.
b) apenas I e III.
c) apenas I, II e III.
d) apenas III e IV.
e) apenas IV.
RESPOSTA: D
COMENTRIO: Simon Bolvar no era mestio. Considerado
um criollo, era um homem branco, nascido na Amrica,
descendente de espanhis. Latifundirio, tinha uma viso
poltica liberal para a poca, anticolonialista e antiimperialista, porm, ligado ao latifndio e contrrio reforma
agrria. O pan-americanismo foi seu grande ideal, porm
sofreu a oposio de elites de diversas regies, que
pretendiam manter divises polticas como forma de
preservar o poder local.
111. (Unesp 2010) O Haiti se tornou livre da Frana em 1804. Cuba
libertou-se da Espanha apenas em 1898, quase um sculo
depois. Sobre os dois processos de independncia, possvel
afirmar que
a) a ao autonomista dos senhores de escravos determinou a
precocidade da independncia do Haiti e a demora na de
Cuba.
b) as lutas emancipacionistas nos dois pases receberam ajuda
militar e financeira de pases do Ocidente europeu.
c) a libertao do Haiti nasceu de uma rebelio escrava e a de
Cuba contou com participao norte-americana.
d) as lavouras canavieiras das duas colnias foram totalmente
destrudas durante os conflitos de independncia.
e) a independncia de Cuba permitiu a instalao do
socialismo na ilha e a do Haiti gerou o cenrio miservel da
atualidade.

113. (Unicamp 2011) A histria de todas as sociedades tem sido a


histria das lutas de classe. Classe oprimida pelo despotismo
feudal, a burguesia conquistou a soberania poltica no Estado
moderno, no qual uma explorao aberta e direta substituiu a
explorao velada por iluses religiosas.
A estrutura econmica da sociedade condiciona as suas
formas jurdicas, polticas, religiosas, artsticas ou filosficas.
No a conscincia do homem que determina o seu ser, mas,
ao contrrio, so as relaes de produo que ele contrai que
determinam a sua conscincia.
(Adaptado de K. Marx e F. Engels, Obras escolhidas. So Paulo:
Alfamega, s./d., vol 1, p. 21-23, 301-302.0

RESPOSTA: C
26

As proposies dos enunciados acima podem ser associadas


ao pensamento conhecido como
a) materialismo histrico, que compreende as sociedades
humanas a partir de ideias universais independentes da
realidade histrica e social.
b) materialismo histrico, que concebe a histria a partir da
luta de classes e da determinao das formas ideolgicas pelas
relaes de produo.
c) socialismo utpico, que prope a destruio do capitalismo
por meio de uma revoluo e a implantao de uma ditadura
do proletariado.
d) socialismo utpico, que defende a reforma do capitalismo,
com o fim da explorao econmica e a abolio do Estado
por meio da ao direta.

A necessidade de esporte para a juventude um fato


incontestvel.
A burguesia se aproveita desse fato para canalizar todos os
jovens das fbricas para seus clubes.
Que fazem os jovens nos clubes burgueses?
Defendem as cores desses clubes. Se o clube de uma fbrica,
o nome e a cor da fbrica que defendem; a burguesia cultiva
neles a paixo e a luta contra a juventude de outras empresas
[]
Todo operrio footballer deve ingressar nos clubes proletrios.

RESPOSTA: B
COMENTRIO: O Marxismo ou materialismo histrico
compreende a histria da humanidade como a histria da luta
de classes, definidas pela propriedade dos meios de produo
e pela explorao de uma classe sobre a outra. Baseia-se
numa anlise das condies materiais das sociedades
humanas como determinantes para a compreenso de suas
formas polticas e religiosas.

O fragmento do jornal conclama a uma prtica organizativa


prpria do movimento anarquista brasileiro, segundo a qual
a) o exerccio fsico seria o meio para o fortalecimento do
esprito dos militantes.
b) a militncia poltica deveria ser exercida em todas as
dimenses da vida do trabalhador.
c) a participao dos cidados nos clubes de futebol das
fbricas reforaria a harmonia social.
d) a aliana proletrio-burguesa deveria ser buscada por
intermdio das prticas desportivas.
e) os militantes deveriam conscientizar os operrios de que o
futebol um esporte alienante.

O trabalhador grfico. 25 jun. 1928. Apud DECCA, Maria


Auxiliadora Guzzo de. Indstria, trabalho e cotidiano. Brasil
1889 a 1930. So Paulo: Atual, 1991. p. 71. (Adaptado).

114. (Uerj 2010) Socialista surgiu como descrio filosfica em


princpios do sculo XIX. Sua raiz lingustica era o sentido
desenvolvido de social.
A distino decisiva entre socialista e comunista, como em
certo sentido esses termos so hoje comumente utilizados,
veio com a mudana de nome, em 1918, do Partido Operrio
Socialdemocrata Russo para Partido Comunista Panrusso.
Dessa poca em diante, uma distino entre socialista e
comunista tornou-se amplamente vigente.
RAYMOND WILLIAMS Adaptado de Socialista. In: Palavraschave: um vocabulrio de cultura e sociedade. So Paulo:
Boitempo, 2007.
Na histria europeia, durante o sculo XX, estabeleceu-se uma
diferena entre socialismo e comunismo relacionada ao
seguinte aspecto:
a) crtica dos valores liberais
b) controle da indstria pelo Estado
c) defesa da ditadura do proletariado
d) importncia do sentimento patritico

RESPOSTA: B
COMENTRIO: O anarquismo uma teoria de origem
socialista, que pressupe a diviso da sociedade em classes
sociais antagnicas e nega as paixes nacionalistas. Para o
autor, mesmo no esporte existem os interesses de classe e os
jovens esportistas proletrios devem perceber os interesses da
burguesia em atra-los e iludi-los atravs do futebol; portanto,
devem participar efetivamente dos clubes proletrios, fiis
sua classe social.

116. (Uece 2010) Leia com ateno o texto a seguir.


Os homens fazem sua prpria histria, mas no a fazem
como querem; no a fazem sob circunstncias de sua escolha
e sim sob aquelas com que se defrontam diretamente, legadas
e transmitidas pelo passado.
MARX, Karl. O Dezoito Brumrio de Louis Bonaparte. So
Paulo: Centauro, 2006.

RESPOSTA: C
COMENTRIO: Socialismo refere-se a qualquer uma das vrias
teorias de organizao econmica que estabelece a
propriedade e a administrao coletiva dos meios de produo
e distribuio dos bens e de uma sociedade caracterizada pela
igualdade de oportunidade e meios para todos os indivduos.
O socialismo moderno surgiu no final do sculo XVIII tendo
origem na classe intelectual e nos movimentos polticos da
classe trabalhadora que criticavam os efeitos da
industrializao e da propriedade privada sobre a sociedade.
Karl Marx afirmava que o socialismo seria alcanado atravs
da luta de classes e de uma revoluo do proletariado,
tornando-se a fase de transio do capitalismo para o
comunismo, entendido como a plenitude da sociedade
igualitria.

Baseado no texto, assinale a afirmao verdadeira.


a) A histria no construda pelos homens porque ela prdefinida pelo destino.
b) A histria permite perceber que a realidade depende
unicamente das escolhas dos homens.
c) A histria feita pelos homens dentro de condicionamentos
herdados do passado.
d) A histria no feita pelo passado e sim pelas
circunstncias das escolhas.
RESPOSTA: C
COMENTRIO: A resposta exige a interpretao correta do
trecho escolhido e a associao com o pensamento marxista,
segundo o qual a compreenso das circunstncias transmitidas
pelo passado fundamental para a sua transformao atravs
de uma ao revolucionria.

115. (Ufg 2010) Leia o texto a seguir.


Viva o Esporte Proletrio!

117. (Uerj 2012) O presidente Roosevelt, que governou os E.U.A.


entre 1933 e 1945, solicitou a incluso de Walt Disney na lista
27

de visitas de celebridades hollywoodianas aos pases sulamericanos. Aps a visita,


Disney retornou aos Estados Unidos e produziu os desenhos
animados Al, amigos (1942) e Os trs cavaleiros (1945),
mais conhecido no Brasil como Voc j foi Bahia?. Essas
criaes de Disney pretendiam resumir, no plano simblico, os
laos de afeto e de cooperao que uniam os E.U.A. ao Brasil.

As artes so frequentemente utilizadas como instrumento de


propaganda poltica e ideolgica. Os desenhos da Disney, por
exemplo, foram pea importante para a estratgia geopoltica
dos E.U.A. para a Amrica Latina, como se observa no texto
acima.
Essa estratgia geopoltica norte-americana foi concretizada
na dcada de 1940 por meio de um conjunto de aes que
fiou conhecido como:
a) Aliana para o Progresso
b) Poltica da Boa Vizinhana
c) Amrica para os Americanos
d) Doutrina do Destino Manifesto

a) aplicao do livre comrcio


b) qualificao da mo de obra
c) padronizao da estrutura produtiva
d) modernizao dos sistemas de circulao
RESPOSTA: D
COMENTRIO: A ao imperialista europeia na frica e na
sia, na segunda metade do sculo XIX, correspondeu, em
linhas gerais, expanso dos interesses capitalistas por meio
da explorao de novas fontes de matria-prima e de novos
mercados e praas para investimentos diversos. As
justificativas para tais interesses e aes foram buscadas,
entre outros, no argumento do valor da modernizao de
setores estratgicos, com destaque para as comunicaes
telegrficas, transportes ferrovirios e navegao a vapor.
Assim, em nome de uma misso civilizatria, o imperialismo
europeu garantiu a ampliao da circulao de mercadorias e
capitais em escala mundial.

119. (G1 - ifce 2011) A supremacia britnica, na Europa e no


mundo, foi indiscutvel no sculo XIX, atingindo seu apogeu
entre 1850 e 1875. O desenvolvimento econmico, o
progresso social e a estabilidade poltica geraram as condies
necessrias para a formao de um vasto imprio colonial na
Amrica, na frica e na sia, no qual se dizia que o sol nunca
se punha.
O texto acima se refere ao perodo da histria inglesa,
conhecido como
a) Revoluo Comercial. .
b) Revoluo Gloriosa.
c) Era Puritana.
d) Era Vitoriana.
e) Repblica Britnica.

RESPOSTA: B
COMENTRIO: O presidente Franklin Roosevelt substituiu o
"Big Stick" pela "Poltica da Boa Vizinhana" que, embora
preservando a hegemonia norte-americana sobre a Amrica
Latina, assumiu uma feio aparentemente mais cordial. A
poltica de boa vizinhana convinha aos esforos dos Estados
Unidos para se recuperar dos efeitos da crise de 1929 sobre
sua economia. A retrica da solidariedade e os mtodos
cooperativos no relacionamento com os pases latinoamericanos facilitavam a formao de mercados externos para
os produtos e investimentos norte-americanos, alm de
garantir o suprimento de matrias-primas para suas indstrias.

RESPOSTA: D
COMENTRIO: Durante grande parte do sculo XIX a
monarquia inglesa foi representada pela Rainha Vitria que,
apesar de no governar efetivamente, tornou-se smbolo da
nao, do ponto de vista das tradies e da moral, mas,
tambm, do ponto de vista material, numa poca marcada
pelo progresso advindo da Segunda Revoluo Industrial e da
conquista e explorao de diversos mercados, tanto como
investimentos na Amrica, como pela dominao e
colonizao, e em regies de frica e sia.

TEXTO PARA AS PRXIMAS 2 QUESTES:


No sculo XIX, surgiu um novo modo de explicar as diferenas
entre os povos: o racismo. No entanto, os argumentos raciais
encontravam muitas dificuldades: se os arianos originaram
tanto os povos da ndia quanto os da Europa, o que poderia
justificar o domnio dos ingleses sobre a ndia, ou a sua
superioridade em relao aos indianos? A nica resposta
possvel parecia ser a miscigenao. Em algum momento de
sua histria, os arianos da ndia teriam se enfraquecido ao se
misturarem s raas aborgenes consideradas inferiores. Mas
ningum podia explicar realmente por que essa ideia no foi
aplicada nos dois sentidos, ou seja, por que os arianos da ndia
no aperfeioaram aquelas raas em vez de se
enfraquecerem?

118. (Uerj 2011) A palavra imperialismo, no sentido moderno,


desenvolveu-se primordialmente na lngua inglesa, sobretudo
depois de 1870. Seu significado sempre foi objeto de
discusso, medida que se propunham diferentes
justificativas para formas de comrcio e de governo
organizados. Havia, por exemplo, uma campanha poltica
sistemtica para equiparar imperialismo e misso
civilizatria.
Adaptado de WILLIAMS, Raymond. Um vocabulrio de cultura
e sociedade.
So Paulo: Boitempo, 2007.
No final do sculo XIX, os europeus defendiam seus interesses
imperialistas nas regies africanas e asiticas, justificando-os
como misso civilizatria.
Uma das aes empreendidas pelos europeus como misso
civilizatria nessas regies foi:

(Adaptado de Anthony Pagden, Povos e imprios. Rio de


Janeiro: Objetiva, 2002, p. 188-94.)

28

120. (Unicamp simulado 2011) O contexto no qual se deu o


domnio dos ingleses sobre a ndia no sculo XIX pode ser
caracterizado
a) pelo Neo-Colonialismo, no qual as metrpoles europeias
passavam a buscar colnias na frica e na sia a fim de
explor-las como mercado consumidor de seus produtos e
fonte de mo de obra escrava.
b) pelo Imperialismo, cuja poltica colonialista visava
enriquecer a metrpole pela busca de metais preciosos e uma
balana comercial favorvel, mantida por meio do
protecionismo exercido por um Estado interventor.
c) pelo Neo-Colonialismo, que se caracterizava por uma nova
forma de dominao sobre a frica e a sia, baseada na
explorao econmica e no controle ideolgico, sem
necessidade de dominao poltica e territorial.
d) pelo Imperialismo, caracterizado por uma poltica
colonialista, motivada pela busca de matria-prima e
mercados consumidores para a indstria europeia, que levou
ao domnio da frica e da sia.

Sobre o tema assinale a alternativa correta.


a) Nesse perodo, a industrializao se concentrou na
Inglaterra, que era conhecida como a Oficina do Mundo.
b) Esta fase da industrializao ficou limitada ao uso do ferro,
do carvo e do vapor.
c) Nesta poca, devido ao progresso econmico que a
industrializao proporcionou, o movimento operrio
praticamente desapareceu na Europa e nos Estados Unidos.
d) Entre os anos de 1873 e 1896, ocorreu uma grave crise
econmica na Europa, cuja maior consequncia foi o fim dos
trustes e cartis.
e) A deflagrao da I Guerra Mundial (1914-1918) teve como
uma de suas motivaes a rivalidade entre as naes
industrializadas, resultante do acelerado desenvolvimento
econmico ocorrido a partir do final do sculo XIX.
RESPOSTA: E
COMENTRIO: A Segunda Revoluo Industrial, ocorrida na
segunda metade do sculo XIX, caracterizou-se pelo advento
da eletricidade e do petrleo como fontes de energia e do ao
na confeco de mquinas e produtos. Essas inovaes
tecnolgicas empregadas no processo produtivo, promoveram
um aumento espetacular da produo e geraram demandas
por fontes de matrias-primas e mercados, motivando
disputas entre as potncias industriais, que resultaram na
ecloso da Primeira Guerra Mundial no incio do sculo XX.

RESPOSTA: D
COMENTRIO: A alternativa correta da questo j define com
preciso a colonizao da ndia pelos britnicos como
pertencente ao contexto do imperialismo do sculo XIX e suas
motivaes.
121. (Unicamp simulado 2011) Segundo o texto, podemos concluir
que o pensamento racista do sculo XIX
a) era incoerente, pois os britnicos se consideravam
superiores aos indianos, porm ambos possuam a mesma
origem racial; alm disso, o racismo no explicava por que a
miscigenao enfraqueceu as raas superiores e no
fortaleceu as inferiores.
b) era um modo de explicar as diferenas entre os povos a
partir de sua origem racial e da miscigenao, a qual poderia
levar tanto ao fortalecimento dos povos considerados
inferiores quanto ao enfraquecimento dos considerados
superiores.
c) era incoerente porque explicava a superioridade e o
domnio dos ingleses sobre os indianos pelo fato de ambos
terem a mesma origem em povos arianos; porm no
explicava por que a miscigenao no fortaleceu as raas
consideradas superiores.
d) era uma forma de legitimar o domnio dos ingleses sobre os
indianos a partir de suas diferentes origens raciais; porm no
explicava por que a miscigenao entre ingleses e indianos
no levara ao aperfeioamento das raas consideradas
inferiores.

123. (Ufmg 2010)


Considerando-se as intervenes do
Imperialismo europeu na sia durante os sculos XIX e XX,
CORRETO afirmar que
a) a Austrlia se lanou com voracidade conquista de
territrios vizinhos Oceania, com vistas a fortalecer sua
posio de colnia recm-liberta (1905).
b) a conquista da Coria foi uma das expresses do
Imperialismo japons, ocupao que, iniciada em 1910, abriu
caminho posterior invaso da China.
c) o Imprio Chins foi extinto e, em seu territrio, surgiram
vrias colnias europeias, como desdobramentos da
interveno estrangeira na Revolta dos Boxer de 1911.
d) os ingleses ocuparam e colonizaram a Indochina, na ltima
dcada do sculo XIX, para garantir a segurana de suas
possesses na Indonsia.
RESPOSTA: B
COMENTRIO: O imperialismo do sculo XIX (neocolonialismo)
caracterizou-se, sobretudo pelas intervenes das potncias
europeias sobre territrios africanos e asiticos. Nessa mesma
poca o Japo passava por um acelerado processo de
modernizao e industrializao e por isso, tambm desejoso
de reas fornecedoras de matrias-primas e mercados
consumidores, passou a ocupar territrios vizinhos como foi o
caso da Coria e posteriormente a Manchria (China).

RESPOSTA: A
COMENTRIO: A questo caracteriza-se mais como uma
interpretao de texto e remete s teorias racistas
desenvolvidas no sculo XIX conhecidas como Darwinismo
Social, cuja finalidade era tentar explicar a pobreza psrevoluo industrial, sugerindo que os que estavam pobres
eram os menos aptos (segundo a teoria de Darwin) e os mais
ricos que evoluram economicamente seriam os mais aptos a
sobreviver por isso os mais evoludos. As potncias europeias
tambm usaram o Darwinismo Social como justificativa para
o Imperialismo europeu.

124. (Pucmg 2010) Enquanto isso, o capito Nemo, encarnao da


revolta contra os donos do mundo, proclama-se indiano das
ndias, por conseguinte, anti-ingls, como so os heris de La
Maison Vapeur (1880), ou os Maoris da Nova Zelndia, esses
homens orgulhosos que resistem pau a pau aos invasores.
(FERRO, Marc. Histria das colonizaes.)

122. (Unemat 2010)


Para muitos historiadores, o perodo
compreendido entre1850 e 1914, do ponto de vista
econmico, conhecido como Segunda Revoluo
Industrial.

O capito Nemo uma personagem criada por Julio Verne em


Vinte Mil Lguas Submarinas, numa crtica explcita do autor
ao sistema ingls de colonizao O colonialismo do sculo XIX,
criticado por Julio Verne, tem suas bases correlacionadas de
forma correta em todas as alternativas a seguir, EXCETO:

29

a) Nacionalismo na tica da poltica internacional da poca,


a posse de colnias era fonte de prestgio e prova de fora do
pas que as conquistava.
b) Racismo impregnado de vises a respeito do nativo como
inferior e do europeu como orgulho racial e superior, detentor
do direito de dominao.
c) Civilizao os europeus comparavam todas as formas de
governo, numa tentativa de apreender uma forma de vida
mais natural.
d) Alteridade na qualidade de outro, o nativo no existe.
Eles so caados como animais selvagens ou considerados
foras-da-lei.

Mesmo no front, matar que importante [...] Morrer no


nada, isso no existe. Ningum pode imaginar sua prpria
morte. Matar o importante.
Essa a fronteira a ser cruzada. Sim, esse um ato concreto
de vontade. Porque a voc torna sua vontade viva na de outro
homem.
Da carta de um jovem voluntrio da Repblica Social Fascista,
de 1943
A respeito do contexto em que se inserem as Grandes Guerras
Mundiais do sculo XX, considere I, II e III a seguir.
I. Os conflitos econmicos, sociais e ideolgicos entre as
principais potncias capitalistas, tanto no perodo anterior a
1914, quanto naquele que antecede Segunda Guerra,
levaram disputa imperialista e corrida armamentista.
II. Nas origens dos dois grandes conflitos mundiais, podemos
identificar a intensificao da propaganda nacionalista e a
formao de um sistema de alianas poltico-militares entre as
naes imperialistas.
III. Nas duas guerras, o conflito armado entre as potncias
imperialistas, apesar do pesado custo em termos de vtimas,
conseguiu solucionar os problemas econmicos, as
divergncias e os ressentimentos entre as naes beligerantes.

RESPOSTA: C
COMENTRIO: A ocupao colonialista da frica e da sia
pelas potncias europeias no final do sculo XIX, foi justificada
moralmente por teorias que preconizavam a superioridade
racial e a condio de civilizao dos europeus em relao a
outros povos, o que inviabilizava qualquer aceitao de
valores culturais, polticos e econmicos dos colonizados.

125. (Ufu 2011) Tirinha publicada no Dia dos Veteranos norteamericano

Desse modo,
a) somente I est correta.
b) somente II est correta.
c) somente III est correta.
d) somente II e III esto corretas.
e) somente I e II esto corretas.
RESPOSTA: E
COMENTRIO: A impreciso da afirmativa III est no fato de
entre as razes da Segunda Guerra Mundial, identificarem-se
os problemas econmicos, as divergncias e os
ressentimentos entre as naes beligerantes, remanescentes
da Primeira. Por exemplo, as clusulas do Tratado de
Versalhes impostas Alemanha, associadas aos problemas
econmicos desta, foram exploradas pelo governo nazista,
principal responsvel pela ecloso da Segunda Guerra
Mundial.

No quadrinho, uma aranha diz para o Garfield: Se voc me


esmagar, eu ficarei famoso. A aranha continua: Eles iro
criar um dia anual de celebrao em minha honra. No
terceiro quadrinho, uma aranha professora pergunta, diante
da sala de aula: Algum aqui sabe por que ns celebramos o
dia nacional da estupidez?
A celebrao do Dia dos Veteranos ocorre em 11 de novembro
e marca o aniversrio do armistcio que ps fim Primeira
Guerra Mundial (1914-1918). Trata-se de feriado nacional nos
EUA.
Disponvel em: <www.entretenimento.uol.com.br>. Acesso
em 13/11/2010.
Assinale a alternativa incorreta.
a) A tirinha mostra a existncia do consenso nacional em torno
da defesa das minorias sociais.
b) O autor deixa a entender que considera estpida a
celebrao do Dia dos Veteranos.
c) A tirinha satiriza a onda do politicamente correto na
sociedade norte-americana.
d) O autor concede escola o lugar de transmisso dos valores
nacionais norte-americanos.

127. (G1 - cftsc 2010) A Revoluo Russa, ocorrida em 1917, deixou


o mundo abalado. Pela primeira vez, tentava-se estabelecer
um tipo de governo no qual os trabalhadores teriam
participao ativa. Os lderes do novo Estado tinham plena
convico de que estava nascendo a sociedade socialista, em
que as diferenas entre as classes sociais deveriam
desaparecer. Era o aparecimento de um novo modo de
organizar a produo, que substituiria o capitalismo.
Adaptado de: PEDRO, Antonio. Histria do Mundo Ocidental.
So Paulo, 2005. p.379
Assinale a alternativa correta.
a) Quando os operrios das fbricas e das fazendas da Rssia
se mobilizaram para tomar o pode no encontraram
resistncia por parte do grupo que estava no poder.
b) Aps a Revoluo Russa, o Estado desapareceu juntamente
com o fim das classes sociais e os operrios passaram a ser os
donos das fbricas.
c) A Revoluo Russa causou uma Guerra Mundial envolvendo
de um lado os pases capitalistas e de outro lado, os
socialistas.

RESPOSTA: A
COMENTRIO: Questo de interpretao: a tirinha trata de
uma minoria, a dos veteranos, porm entendida pela maioria
da sociedade como setor a ser valorizado, aqueles que
combateram pela ptria em uma guerra.

126. (Mackenzie 2010) Morrer pela Ptria, pela Ideia! [...] No,
isso fugir da verdade.

30

d) O sistema descrito no texto acima, baseou-se na teoria do


filosofo Karl Marx que tinha como uma de suas caractersticas
a propriedade coletiva dos bens de produo.
e) O exemplo da Revoluo Russa espalhou-se fazendo com
que o sistema capitalista desaparecesse do mundo no sculo
XX, dando lugar ao sistema comunista.

federal. Esta caracterstica explicada pelo fato de a


integrao decorrente das RIDE estar associada a:
a) unidades estaduais diferentes
b) reas de fronteira internacional
c) espaos de preservao ambiental
d) complexos industriais estratgicos

RESPOSTA: D
COMENTRIO: O socialismo foi fundamentado por Karl Marx,
considerado ainda hoje como o principal terico de um novo
modelo de organizar a produo e a economia. A teoria
marxista, tambm denominada de socialismo cientfico ou
de materialismo histrico, pressupe a eliminao da
propriedade privada e consequentemente das classes sociais.
A Revoluo Russa foi o primeiro movimento operrio
vitorioso, no sentido de implementar uma sociedade
igualitria e, durante muitos anos, manteve-se como o nico
pas socialista no mundo.

RESPOSTA: A
COMENTRIO: A criao das RIDEs uma ao integrada dos
entes federativos Unio, Estados e Municpios cujo
objetivo a promoo do desenvolvimento econmico e
reduo das desigualdades sociais de regies marcadas por
uma estagnao socioeconmica. Essas regies passam a ter
prioridades nos investimentos dos recursos pblicos com
atuao conjunta dos entes da federao. Atualmente existem
trs RIDEs: do Distrito Federal (Distrito Federal, Gois e Minas
Gerais), Petrolina-Juazeiro (Bahia e Pernambuco) e da Grande
Teresina (Piau e Maranho).

128. (Uff 2012) No sculo XIX, um dos eventos mais importantes


foi a unificao alem. A partir dela o mundo europeu e
colonial caminhou na direo da hegemonia da Alemanha.

130. (G1 - cftmg 2012) Belo Horizonte, RRMBH* e RMBH


Populao e Taxa geomtrica de crescimento (1.940 - 2.000)

Assinale a alternativa que melhor identifica o perodo.


a) A Conferncia Colonial de Berlim, em 1884/1885, simboliza
a fora do Imprio alemo nas conquistas das regies
africanas, como o Egito.
b) A poltica alem no processo de expanso colonial do sculo
XIX reacendeu as rivalidades entre as potncias, como a crise
franco-italiana ocasionada pela conquista da Tunsia pela
Frana.
c) O prussiano Bismark foi o responsvel nico pela unificao
alem e pela vitoriosa expanso colonial na regio da sia
Menor.
d) A poltica socialista de Bismark permitiu o fortalecimento
econmico da Alemanha, pautado no apoio s pequenas
empresas de origem familiar.
e) O perodo marcado, no somente pelo isolamento da
Frana, mas tambm pelo enfraquecimento das relaes
internacionais entre a Alemanha e a Itlia.

* Todos os municpios da RMBH, com exceo de Belo


Horizonte.
Fonte: FERNANDES, Joseane de Souza. A Expanso Urbana de
Belo Horizonte e da Regio Metropolitana de Belo Horizonte: o
caso especfico do municpio de Ribeiro das Neves. Tese de
doutorado. Cedeplar. UFMG, 2008.
A partir da evoluo do processo de metropolizao
apresentado, correto inferir que a
a) variao das taxas de crescimento da RMBH reduziu o
quantitativo de habitantes nesse espao.
b) mudana da metrpole na hierarquia urbana brasileira
influenciou no decrscimo das taxas de crescimento
demogrfico da capital.
c) diminuio nas taxas de crescimento demogrfico em Belo
Horizonte impactou negativamente no tamanho absoluto de
sua populao.
d) expanso da mancha urbana em Belo Horizonte interferiu
no incremento da participao relativa dos outros municpios
no total de habitantes.

RESPOSTA: B
COMENTRIO: A afirmao discutvel, pois, se a ideia de
hegemonia alem no mundo europeu pode ser considerada
devido ao seu rpido crescimento econmico, a hegemonia
colonial ficou longe de se concretizar. Necessitada de ampliar
seus mercados coloniais a grande maioria de territrios
coloniais na frica e sia estavam sob controle ingls e
francs. As pretenses expansionistas alems determinaram
intenso processo de militarizao dos pases europeus e foi
uma das causas para a Grande Guerra.

RESPOSTA: D
COMENTRIO: Naturalmente, o crescimento de uma
metrpole interfere no desenvolvimento das cidades
circunvizinhas, confirmando a alternativa [D], no caso de Belo
Horizonte.

GEOGRAFIA I
129. (Uerj 2012) Artigo 25, pargrafo 3 - Os Estados podero,
mediante lei complementar, instituir regies metropolitanas,
aglomeraes urbanas e microrregies, constitudas por
agrupamentos de municpios limtrofes, para integrar a
organizao, o planejamento e a execuo de funes pblicas
de interesse comum.
Constituio da Repblica Federativa do Brasil
www.planalto.gov.br
O Brasil possui atualmente trs Regies Integradas de
Desenvolvimento RIDE, um tipo especial de regio
metropolitana que s pode ser instituda por legislao

131. (Unicamp 2012) O Brasil experimentou, na segunda metade


do sculo 20, uma das mais rpidas transies urbanas da
histria mundial. Ela transformou rapidamente um pas rural e
agrcola em um pas urbano e metropolitano, no qual grande
parte da populao passou a morar em cidades grandes. Hoje,
quase dois quintos da populao total residem em uma cidade
de pelo menos um milho de habitantes.
(Adaptado de George Martine e Gordon McGranahan, A
transio urbana brasileira: trajetria, dificuldades e lies
aprendidas, em Rosana Baeninger (org.), Populao e
31

cidades: subsdios para o planejamento e para as polticas


sociais. Campinas: Nepo / Braslia: UNFPA, 2010, p. 11.)
Considerando o trecho acima, assinale a alternativa correta.
a) A partir de 1930, a ocupao das fronteiras agrcolas (na
Amaznia, no Centro-Oeste, no Paran) foi o fator gerador de
deslocamentos de populao no Brasil.
b) Uma das caractersticas mais marcantes da urbanizao no
perodo 1930-1980 foi a distribuio da populao urbana em
cidades de diferentes tamanhos, em especial nas cidades
mdias.
c) Os ltimos censos tm mostrado que as grandes cidades
(mais de 500 mil habitantes) tm tido crescimento relativo
mais acelerado em comparao com as mdias e as pequenas.
d) Com a crise de 1929, o Brasil voltou-se para o
desenvolvimento do mercado interno atravs de uma
industrializao por substituio de importaes, o que
demandou mo de obra urbana numerosa.

133. (Uerj 2011) Hoje, a interao espacial entre comunidades,


no que tange ao deslocamento de pessoas moradoras em uma
delas para visitarem amigos ou parentes ou estabelecerem
contatos associativos com pessoas residentes em outras,
tornou-se um tanto difcil, devido aos mecanismos de controle
impostos pelos traficantes e rivalidade e aos choques entre
quadrilhas baseadas em favelas diferentes (...).
SOUZA, Marcelo Lopes de. O desafio metropolitano: um
estudo sobre a problemtica scio-espacial nas metrpoles
brasileiras. Rio de Janeiro: Bertrand Brasil, 2000.
O fenmeno descrito no texto, que vem ocorrendo nas ltimas
dcadas, corresponde mais diretamente ao seguinte processo
socioespacial:
a) hierarquizao
b) regionalizao
c) metropolizao
d) territorializao

RESPOSTA: D
COMENTRIO: A economia brasileira at a dcada de 1930 foi
marcada por sua caracterstica agroexportadora. A crise de
1929 enfraqueceu a economia exportadora de caf e
encareceu os produtos importados, consolidando o processo,
em curso desde o incio do sculo XX, de industrializao
substitutiva das importaes. Foi a partir do processo de
industrializao, estimulado pelo Estado ditatorial varguista,
que se surgiram grandes cidades (So Paulo e Rio de Janeiro).
Tais centros urbanos converteram-se em mercado
consumidor, gerando melhores condies para a acelerao
do processo de urbanizao. A alternativa [C] apresenta uma
caracterstica atual da urbanizao brasileira.

RESPOSTA: D
COMENTRIO: As grandes cidades brasileiras passaram por
processos de crescimento urbano e populacional
desordenado. A falta de planejamento e de investimentos em
moradia, educao, sade e transporte pblico acabaram
produzindo uma cidade com acesso desigual ao territrio. A
cidade includa tem equipamentos urbanos em quantidade
suficiente e as pessoas conseguem ter acesso a moradias. Para
a populao mais pobre restaram as reas de risco e na cidade
do Rio de Janeiro, os morros acabaram sendo ocupados por
essa razo. A ocupao dos morros pelo trfico proporcional
ausncia do Estado na regio. A posse do territrio por esses
grupos acaba impondo sua maneira de ocupar e se locomover
nesse territrio. O trfico impe suas leis aos moradores,
dificultando o livre deslocamento, uma vez que a ausncia do
Estado no garante a liberdade de ir e vir.

132. (Uerj 2011) Deste Planalto Central, desta solido que em


breve se transformar em crebro das altas decises
nacionais, lano os olhos mais uma vez sobre o amanh do
meu pas e antevejo esta alvorada com f inquebrantvel e
uma confiana sem limites no seu grande destino.

A alternativa [A] falsa, hierarquizao diz respeito ao


ordenamento de cidades segundo sua importncia ou grau de
polarizao.
A alternativa [B] falsa, a regionalizao diz respeito a grandes
reas, mais complexas que acabam englobando reas de
morros como no Rio de Janeiro.
A alternativa [C] falsa, metropolizao o processo de
expanso das reas urbanas de uma rede urbana de modo que
as cidades acabam se conurbando em torno de uma cidade
central ou principal.

Juscelino Kubitschek, 02/10/1956


O Globo, 21/04/2010
A realizao mais conhecida do governo de Juscelino
Kubitschek foi a construo de Braslia. No entanto, essa obra
contemplava objetivos mais abrangentes desse governante.
Dentre esses objetivos, destaca-se o de promover a integrao
nacional por meio da seguinte ao:
a) modernizao do setor tercirio
b) ampliao da infraestrutura de transportes
c) interligao das redes de telecomunicaes
d) explorao das regies Nordeste e Centro-Oeste

134. (Ufu 2011)


conurbao.
RESPOSTA: B
COMENTRIO: A ideia fundamental da construo de Braslia,
em termos geopolticos, foi afastar a sede de decises polticas
para longe das reas de poder econmico, alm de promover
uma maior integrao nacional a partir de uma sede de
governo federal no centro do pas, aspecto que s seria
possvel com a ampliao das redes de transporte.
A alternativa [A] falsa: o setor tercirio viria a ser
efetivamente modernizado a partir da dcada de 1990;
A alternativa [C] falsa: as telecomunicaes iro se
modernizar a partir da dcada de 1970 com a criao da
Embratel;
A alternativa [D] falsa: Braslia serviria de base para a
expanso das regies Centro-Oeste e Norte.

32

A figura abaixo representa o processo de

Analise as afirmaes abaixo.


I - A conurbao ocorre quando h a superposio ou o
encontro de duas ou mais cidades prximas devido ao seu
crescimento. Para ocorrer esse processo, as duas cidades
devem ter, necessariamente, o mesmo tamanho e a mesma
densidade populacional.
II - O xodo rural pode ser considerado um dos fatores que
contribuem para o surgimento do processo de conurbao,
pois provoca a expanso dos grandes centros urbanos.
III - O processo de conurbao, em geral, d origem
formao de regies metropolitanas, como por exemplo, a
Regio Metropolitana de So Paulo e Rio de Janeiro.
IV - Conurbao o nome dado para o crescimento de duas ou
mais cidades vizinhas, que acabam por formar um nico
aglomerado urbano, no qual, em geral, h uma cidade
principal e uma (ou mais de uma) cidade-satlite.
a) o municpio de Nova Iguau sempre foi mais rico do que o
de Duque de Caxias.
b) o municpio de Duque de Caxias rivaliza, desde o sculo XIX,
com o de Nova Iguau.
c) as emancipaes distritais de Nova Iguau criaram outros
municpios fluminenses.
d) a Baixada fluminense muito violenta e por isso se
fragmenta municipalmente.
e) as unidades municipais apresentadas existem devido fora
poltica de Nova Iguau.

Assinale a alternativa que apresenta as afirmativas corretas.


a) Apenas I, II e IV.
b) Apenas II, III e IV.
c) Apenas II e III.
d) Apenas I e IV.
RESPOSTA: B
COMENTRIO: O rpido crescimento urbano observado no
Brasil, a partir da dcada de 1960, mostrou um acentuado
padro de adensamento nas cidades dentro de uma rede
urbana influenciada por uma cidade central, favorecendo o
processo de conurbao e, em alguns casos, posterior
metropolizao.
A frase I falsa: a conurbao no depende do tamanho ou da
densidade populacional das cidades envolvidas.
135. (Eewb 2011) O espao regional polarizado menor e
representa uma posio hierrquica intermediaria entre a
metrpole regional e nacional. So exemplos: Manaus, Porto
Velho, Rio Branco, So Luis, Campina Grande, Natal, Ribeiro
Preto, Campo Grande, Londrina e Vitria. Entre as
classificaes das cidades dentro de uma hierarquia urbana, o
trecho acima se refere:
a) Centros regionais.
b) Metrpole global.
c) Centro sub-regionais.
d) Nenhuma das alternativas, pois cada cidade possui uma
hierarquia.

RESPOSTA: C
COMENTRIO: A alternativa [A] falsa, no h dados na figura
que permitam inferir sobre a riqueza dos municpios.
A alternativa [B] falsa, Duque de Caxias emancipa-se em
1943, sculo XX.
A alternativa [D] falsa, no h relao de causa e efeito entre
violncia e fragmentao municipal.
A alternativa [E] falsa, o alto nvel de fragmentao indica
pouca fora poltica de Nova Iguau.
137. (Ufpr 2011) Uma reportagem publicada na Revista Veja (ed.
2180, ano 43, n. 35, de 1 set. 2010, p. 7677) e intitulada A
fora das cidades mdias afirma que a cidade paranaense de
Londrina um exemplo do sucesso do interior do pas.
Segundo a revista, Londrina rompeu a barreira dos 500.000
habitantes em 2009. Deixou de ser, portanto, uma cidade
mdia, para se tornar a irm caula das quarenta metrpoles
nacionais aquelas com mais de meio milho de moradores.

RESPOSTA: A
COMENTRIO: Os espaos regionais polarizados, isto ,
cidades que sofrem ao de influncia ou que influenciam
cidades vizinhas, so centros regionais como os exemplos
citados.
A alternativa [B] falsa, metrpoles globais constituem uma
rede de cidades mundiais caracterizadas por sua estrutura
econmica e financeira, ligadas por redes de bancos, bolsas de
valores e mercantis.
A alternativa [C] falsa, centros sub-regionais so cidades
polarizadas por centros regionais.
A alternativa [D] falsa, as cidades estruturam-se em redes
urbanas polarizadas e rigorosamente hierarquizadas.

Considerando o contedo tratado na reportagem sob a


perspectiva geogrfica, considere as seguintes afirmativas:
1. O crescimento populacional de Londrina na atualidade
explicado pelo elevado fluxo migratrio de origem rural e
destino urbano, tambm denominado de xodo rural.
2. A definio de uma metrpole se faz por meio do critrio
populacional. Toda cidade que atinge 500 mil habitantes
automaticamente elevada categoria de metrpole.
3. Embora Londrina aparea no texto da reportagem
identificada como uma metrpole nacional, ela posicionada
pelo IBGE (2008) no mbito do REGIC (Regio de Influncia das
Cidades) como uma Capital Regional.
4. A classificao dos centros urbanos de uma determinada
rede urbana em diferentes nveis denominada de hierarquia
urbana.

136. (Pucrj 2011) A arquitetura poltico-territorial do cartograma


mostra que:

Assinale a alternativa correta.


a) Somente a afirmativa 2 verdadeira.
b) Somente as afirmativas 3 e 4 so verdadeiras.
c) Somente as afirmativas 1, 3 e 4 so verdadeiras.
33

d) Somente as afirmativas 1 e 2 so verdadeiras.


e) As afirmativas 1, 2, 3 e 4 so verdadeiras.
RESPOSTA: B
COMENTRIO: A partir doas anos 1970, o processo de
urbanizao foi seguido por intenso dinamismo e
descentralizao de atividades. Segmentos industriais
comearam a se deslocar de grandes centros metropolitanos
devido a problemas de saturao em direo a localidades
relativamente menores, beneficiadas por polticas municipais
de incentivo indstria e com base na constituio de 1988
que permite legislao tributria estadual desvinculada do
poder federal, desde que no fira a Lei de Responsabilidade
Fiscal.

A alternativa [B] falsa, as obras da foto so anteriores ao


desmanche do Morro do Castelo.
A alternativa [C] falsa, o Rio, na poca da foto, era a capital
do Brasil independente de Portugal.
A alternativa [D] falsa, a foto anterior Segunda Guerra
mundial.
A alternativa [E] falsa, a via em construo a Rio Branco.
139. (Ufsm 2011) Sobre o papel da luz artificial na construo das
paisagens urbanas noturnas no Brasil, correto afirmar:
I. A iluminao artificial, alm de funes como a segurana e
a visibilidade, tem papel cada vez mais importante no lazer e
na apropriao noturna do territrio.
II. A utilizao de diferentes tipos de iluminao, para projetar
diferentes tipos de uso do espao, auxilia a identificar a
estrutura urbana e aumenta a capacidade de planejar a
segurana dos citadinos.
III. A luz artificial o instrumento bsico utilizado para
implementar os corredores de urbanizao, com reas
conurbadas ou em processo de conurbao.

A frase 1 falsa, o crescimento atual de Londrina resulta da


expanso industrial que fez a cidade surgir no mapa de
empregos industriais a partir do final dos anos 1990.
A frase 2 falsa, reas metropolitanas so definidas por
inmeros critrios. Alm da populao, so levados em conta
aspectos como a produo econmica, infraestrutura,
produo social, sistema financeiro, condies de segurana,
projetos sociais como educao e sade, sistema tributrio e
eficincia administrativa.

Est (o) correta(s)


a) apenas I.
b) apenas I e II.
c) apenas III.
d) apenas II e III.
e) I, II e III.

138. (Pucrj 2011) A fotografia selecionada ao lado mostra o


resultado de algumas obras de modernizao da cidade do Rio
de Janeiro no sculo XX.

RESPOSTA: B
COMENTRIO: As frases I e II esto corretas. A iluminao
artificial noturna exerce um papel fundamental na apropriao
do territrio urbano por seus cidados, uma vez que ela
garante mobilidade e aumenta a segurana das pessoas. Por
sua vez, a frase III est errada, pois os corredores urbanos se
formam a partir dos diferentes usos do solo que ocorrerem
simultaneamente nas cidades: uso industrial, residencial, de
lazer. A luz noturna importante, mas no implementa o
processo de formao desses corredores.
e
Essas obras ocorreram:
a) como reflexo das polticas modernizadoras que a cidade
sofreu desde o final do sculo XIX e que
culminaram na remodelao do centro histrico carioca no
sculo XX.
b) somente com o desmonte total do morro do Castelo (1923),
fato que possibilitou a ligao do
centro histrico carioca s reas de expanso urbana nortesul.
c) com o intuito de que a capital do Imprio portugus se
adaptasse aos padres urbansticos dos
grandes imprios europeus, notadamente o francs.
d) aps a 2 Guerra mundial, durante o governo de Getlio
Vargas, como estratgia de presso
norte-americana para a modernizao brasileira.
e) para a construo da Avenida Central no centro histrico da
cidade, via que interligaria os eixos
leste-oeste do Rio de Janeiro.

140. (Uerj 2011) Na imagem, visualiza-se a regio da Baixada


Santista, com as diversas cidades que compem esse espao
do litoral paulista.

RESPOSTA: A
COMENTRIO: A foto mostra a construo da avenida Rio
Branco. direita e ao fundo pode-se ver o Teatro Municipal
em construo, obras do sculo XIX. Existem poucas pessoas
na rua e no h veculos automotores. V-se uma carroa no
primeiro plano da foto.

A anlise da imagem permite reconhecer a ocorrncia do


seguinte processo socioespacial comum em cidades de reas
metropolitanas:
a) favelizao
b) conurbao
34

c) gentrificao
d) verticalizao
RESPOSTA: B
COMENTRIO: A urbanizao acentuada com rpido
crescimento das cidades de uma rede urbana favorece a
conurbao de seus espaos em torno da cidade principal.
Esse processo facilita o surgimento de uma regio
metropolitana. A conurbao o processo de unio espacial
entre duas ou mais cidades sem zona rural entre elas em
urbanizao contnua.
A alternativa [A] falsa, a favelizao o processo de
expanso de moradias precrias em reas de risco ou
abandonadas ocupadas por populaes de baixa renda sem
acesso a moradias regularizadas.
A alternativa [C] falsa, gentrificao um neologismo que
trata do processo de recuperao de um centro urbano que
mostre algum aspecto decadente, provocando uma melhoria e
(re)valorizao imobiliria. Tanto pode ser organizado pelo
capital estatal como pelo capital privado, no havendo regra
estipulada.
A alternativa [D] falsa, a verticalizao a reutilizao do
espao urbano substituindo casas e outras plantas baixas por
prdios, em geral, de apartamentos ou comerciais.

loteamento e aproveitamento, para especulao imobiliria,


porm, foi invadido por moradores e, hoje, representa um
bairro de invaso, porm, dotado de infraestrutura.
142. (G1 - ccampos 2011) Eu amo a rua. Esse sentimento de
natureza toda ntima no vos seria revelado por mim se no
julgasse, e razes no tivesse para julgar, que este amor assim
absoluto e assim exagerado partilhado por todos vs. Ns
somos irmos, ns nos sentimos parecidos e iguais; nas
cidades, nas aldeias, nos povoados, no porque soframos, com
a dor e os desprazeres, a lei e a polcia, mas porque nos une,
nivela e agremia o amor da rua. este mesmo o sentimento
imperturbvel e indissolvel, o nico que, como a prpria vida,
resiste s idades e s pocas. Tudo se transforma, tudo varia
o amor, o dio, o egosmo. Hoje mais amargo o riso, mais
dolorosa a ironia, os sculos passam, deslizam, levando as
coisas fteis e os acontecimentos notveis. S persiste e fica,
legado das geraes cada vez maior, o amor da rua.
RIO, Joo do. Fundao Biblioteca Nacional do Livro. A alma
encantadora das ruas Departamento Nacional do Livro
Ministrio da Cultura. In: Coleo Cadernos de EJA: Tempo
Livre e Trabalho. Ministrio da Educao, 2007, p. 49.
A imagem retratada das ruas cariocas na transio do sculo
XIX para o sculo XX, presente no texto acima, evidencia um
espao onde os encontros e os desencontros entre indivduos
iguais e diferentes era uma realidade da paisagem urbana.
Contudo, nessa primeira dcada do sculo XXI, observa-se
uma ntida modificao dessa paisagem e dos mecanismos
que alteram o significado das ruas, processo expresso pelo (a):
a) avano do processo de privatizao do espao em razo da
difuso de diversos produtos imobilirios privados como os
condomnios fechados e as praas pblicas.
b) aumento do sentimento que associa a rua como espao
inseguro em oposio aos investimentos do setor privado de
valorizao dos espaos pblicos.
c) menor afastamento entre indivduos das diferentes classes
sociais devido difuso de empreendimentos privados, como
os shopping centers, que permitem uma maior integrao
entre as pessoas.
d) aprofundamento do processo de segregao scio-espacial
em decorrncia da mercantilizao cada vez maior dos
espaos de moradia e de lazer.

141. (Uft 2011) Palmas, por ser uma cidade planejada para ser a
capital do estado do Tocantins, apresenta-se como um espao
urbano diferenciado, no que diz respeito s suas funes:
admistrativas, econmicas e polticas; e suas formas de
planejamento arquitetnico e geoespaciais.
No plano administrativo considerada como um espao
organizador de polticas pblicas para o Estado; do ponto de
vista econmico o lcus de decises que visam
descentralizar o poder econmico dos outros centros urbanos
e de regies geoeconmicas mais ricas, j com dinmicas
prprias. Em relao poltica Palmas o centro de poder
onde se constituiu como um novo territrio de aes de
Partidos, Sindicatos, Movimentos Sociais, Federaes e
Associaes. No entanto, sobre as relaes geoespaciais
imbricadas nas lutas pela ocupao dos espaos habitacionais
da cidade e nas contradies de seu planejamento.
correto afirmar que:
a) Os movimentos sociais urbanos em Palmas no tiveram
fora para interferir em seu plano diretor pelo motivo de que
as leis de posturas territoriais urbanas do municpio no
permitiram tais intervenes.
b) Os arquitetos que planejaram a cidade foram contrrios a
qualquer tipo de modificao em seu projeto original.
c) Os movimentos sociais conseguiram organizar-se e
estabelecer bandeiras de lutas pela moradia que resultou na
transformao da feio urbana, de algumas reas de uso
especfico, no plano diretor da cidade, interferindo assim de
maneira significativa em seu desenho inicial.
d) Palmas ostenta at os dias atuais, o ttulo de capital
ecolgica do pas, pelo fato de que no momento de sua
construo foram preservados os biomas do cerrado
tocantinense.
e) Palmas, como novo centro de poder, domina todas as
relaes scio-culturais do estado chegando a anular as
identidades tradicionais de cunho locais.
RESPOSTA: C
COMENTRIO: Para justificar esta resposta podemos pegar,
como exemplo, Vila Unio, bairro prximo ao lago, vista e
localizao privilegiada, que recebe a brisa do lago; uma rea
bastante ventilada, que estava reservada para posterior

RESPOSTA: D
COMENTRIO: O crescimento urbano vertiginoso ocorrido a
partir da dcada de 1930 desdobrou em inmeros aspectos
concomitantes: crescimento populacional vegetativo e por
migraes; presso sobre a infraestrutura, transporte,
saneamento, moradia, segurana; valorizao da terra;
especulao imobiliria. Um dos desdobramentos mais
importantes de todo o processo foi a segregao espacial
urbana.
A alternativa [A] falsa, praas pblicas so espaos pblicos.
A alternativa [B] falsa, o espao pblico mais seguro na
proporo direta do investimento pblico em paisagismo e
segurana.
A alternativa [C] falsa, shopping centers so enclaves
fortificados, portanto capazes de segregar populaes de
classes sociais diferentes.

143. (Ufrgs 2011) A tabela abaixo fornece dados de alguns dos


municpios gachos que, em 2008, tinham mais de 100 mil
habitantes.

35

Municpio rea

Populao (hab.) PIB per capita em R$ (2007)

1
1.643,9
2
5.715,8
3
70,8
4
1.608,8
Fonte: FEE RS

402.163
125.695
208.812
343.925

km2

gua
3. relevo com pores convexas na divergncia dos fluxos de
gua

24.589
17.050
4.551
9.349

Quais esto relacionados aos deslizamentos das encostas?


a) Apenas 1.
b) Apenas 2.
c) Apenas 3.
d) Apenas 1 e 2.
e) Apenas 1 e 3.

Assinale a alternativa que apresenta os nomes dos municpios


gachos correspondentes aos nmeros 1, 2, 3 e 4,
respectivamente.
a) Uruguaiana Alvorada Pelotas Caxias do Sul
b) Caxias do Sul Uruguaiana Alvorada - Pelotas
c) Caxias do Sul Uruguaiana Pelotas - Alvorada
d) Pelotas Alvorada Uruguaiana Caxias do Sul
e) Pelotas Uruguaiana Alvorada Caxias do Sul

RESPOSTA: D
COMENTRIO: A condio tropical do Brasil, a expanso
urbana desordenada com ocupao de reas de risco em
encostas, o desmatamento, so aspectos que formam um
conjunto favorvel a tragdias urbanas. A alta precipitao no
vero com infiltrao de gua, o desmatamento que precede a
ocupao deixando as encostas desprotegidas e os
adensamentos habitacionais com moradias construdas sem
apoio tcnico e infraestrutura, acabam resultando em
deslizamentos com perda de vidas e prejuzos materiais.
O fator geomorfolgico descrito em 3 falso, pores
convexas na divergncia de fluxos de gua provoca disperso
hdrica e no concentrao.

RESPOSTA: B
COMENTRIO: A questo requer conhecimentos especficos
do estado do RS.
144. (Enem 2011)
O Centro-Oeste apresentou-se como
extremamente receptivo aos novos fenmenos da
urbanizao, j que era praticamente virgem, no possuindo
infraestrutura de monta, nem outros investimentos fixos
vindos do passado. Pde, assim, receber uma infraestrutura
nova, totalmente a servio de uma economia moderna.
SANTOS, M. A Urbanizao Brasileira. So Paulo: EdUSP, 2005
(adaptado).

146. (Uece 2010) Dentre as escalas a seguir, a mais adequada para


a representao de plantas urbanas
a) 1 : 100 000.
b) 1 : 1 000 000.
c) 1 : 1 000.
d) 1 : 500 000.
RESPOSTA: C
COMENTRIO: Escalas so relaes de proporo expressas
em fraes onde o numerador representa o mapa e o
denominador s reas de terreno cartografadas. As fraes
menores implicam em escalas como 1 : 1.000.000, alternativa
[B], onde os mapas so ideais para representar grandes
extenses porm so menos detalhados. As fraes maiores
como na escala 1 : 1.000, alternativa [C], so ideais em
mapeamentos detalhados, porm cartografando reas
menores.

O texto trata da ocupao de uma parcela do territrio


brasileiro. O processo econmico diretamente associado a
essa ocupao foi o avano da
a) industrializao voltada para o setor de base.
b) economia da borracha no sul da Amaznia.
c) fronteira agropecuria que degradou parte do cerrado.
d) explorao mineral na Chapada dos Guimares.
e) extrativismo na regio pantaneira.
RESPOSTA: C
COMENTRIO: A regio Centro-Oeste constituiu-se em rea de
expanso da fronteira agropecuria, com rpido crescimento a
partir dos anos 1980 em diante. Suas cidades, em geral, so
novas e se valeram do boom do agronegcio, tornando-se
prsperas, construdas e reformadas a pouco tempo.
A alternativa [A] falsa: a indstria de base praticamente
inexiste na regio, cuja vocao agropecuria gerou
prosperidade e atraiu a indstria de transformao da
produo agropecuria e de produo de bens de consumo;
A alternativa [B] falsa: o ciclo da borracha teve seu auge
entre o final do sculo XIX e incio do sculo XX;
A alternativa [D] falsa: a Chapada dos Guimares no
apresenta reas de minerao significativas;
A alternativa [E] falsa: a regio pantaneira no se constitui
em rea extrativista.

147. (Unemat 2010)


O processo de urbanizao e de
industrializao gerou a concentrao das atividades
comerciais e dos servios mais importantes em algumas
cidades, acumulando capital e poder poltico nesses espaos
urbanos, chamados de:
a) conurbao.
b) metrpoles.
c) megalpoles.
d) cidades globais.
e) rede urbana.
RESPOSTA: B
COMENTRIO: A urbanizao um fenmeno mundial
atingindo mais da metade da populao da Terra. Com o
desenvolvimento das atividades urbanas como indstria,
comrcio e outros tipos de prestao de servios as cidades
passam por processos de crescimento que podem resultar em
grandes reas com vrias cidades conurbadas, ou seja, em
reas de urbanizao contnua em torno de uma cidade
principal, que formam as regies metropolitanas.

145. (Ufrgs 2010) A combinao de chuvas fortes com moradias


inseguras j tornou rotineiras as tragdias nas grandes cidades
brasileiras. Os deslizamentos nas encostas, muitas vezes
responsveis por tais tragdias, so condicionados por fatores
geomorfolgicos, entre outros.

A alternativa [A] falsa, a conurbao o processo de juno


espacial entre duas ou mais cidades com urbanizao contnua
sem rea rural entre elas.

Considere os seguintes fatores geomorfolgicos.


1. declividade e forma da encosta
2. relevo com pores cncavas na convergncia dos fluxos de
36

A alternativa [C] falsa, megalpoles resultam da conurbao


de metrpoles.
A alternativa [D] falsa, cidades globais so redes (nodais) de
cidades ligadas atravs de sistemas financeiros.
A alternativa [E] falsa, rede urbana um conjunto de cidades
relativamente prximas fortemente hierarquizadas em torno
de uma cidade mais importante.

Com base na imagem e nos conhecimentos sobre urbanizao,


correto afirmar:
a) A existncia de favelas nos entornos dos condomnios
fechados comprova que ricos e pobres convivem
harmonicamente em grandes centros urbanos como So
Paulo.
b) Paisagens onde pobreza e riqueza convivem lado a lado
esto circunscritas s regies metropolitanas no mundo todo.
c) A imagem revela a justia social caracterstica dos grandes
centros, onde aqueles que possuem maiores rendimentos tm
mais direito a moradia com qualidade de vida.
d) A paisagem indica a solidariedade entre ricos e pobres, pois
os primeiros acolhem os mais necessitados no entorno de suas
moradias.
e) A imagem testemunha a desigualdade social presente nas
paisagens urbanas, nas quais coabitam riqueza e pobreza.

148. (Pucrs 2010) Responder questo com base nas afirmativas


que tratam das redes de transporte, associando-as aos
desenhos que representam trs tipos de rede de transporte.

RESPOSTA: E
COMENTRIO: O crescimento urbano por que passa o Brasil
caracterizado pela rapidez e desordenamento. As cidades
acabem se tornando centros de grande dinamismo econmico
e financeiro e como o acesso a escolas de boa qualidade,
qualificao e empregos de melhor salrio em conjunto com a
concentrao de renda, so foras capazes de gerar grandes
desigualdades sociais.

I. As redes reticulares so homogneas e garantem alta


conectividade. O seu traado pode ser observado no exemplo
do desenho A.
II. As redes polares so mais frequentes nas reas com
movimentao pendular entre os grandes centros urbanos e
os ncleos urbanos perifricos. O seu traado pode ser
observado no exemplo do desenho B.
III. As redes em rvores estruturam-se a partir de um eixo
central com ramais que se conectam ao longo de um tronco
principal, como observado nos exemplos dos desenhos A e
C.
IV. As redes ortogonais so heterogneas e se destinam a
reas de alta concentrao de transporte, como nas regies
metropolitanas brasileiras, estando exemplificadas nos
desenhos B e C.
Esto corretas apenas as afirmativas
a) I e II.
b) I, II e III.
c) I e IV.
d) II e III.
e) III e IV.
RESPOSTA: A
COMENTRIO: A frase III falsa, as redes em rvores so
representadas na letra C apenas.
A frase IV falsa, as redes ortogonais esto representadas
pela letra A.

A alternativa [A] falsa, no existe harmonia entre ricos e


pobres em cidades brasileiras e sim reas includas e reas
excludas. Como a disputa pelo espao imobilirio constante
ocorrem proximidades entre reas includas e excludas, estas,
em geral fruto, de invases.
A alternativa [B] falsa, essas paisagens aparecem em centros
urbanos, em geral em pases mais pobres.
A alternativa [C] falsa, a imagem revela que no h justia
social no parcelamento da terra.
A alternativa [D] falsa, no solidariedade entre ricos e
pobres quanto ao fracionamento dos terrenos.
150. (Ufu 2010) O ndice de urbanizao no Brasil muito elevado,
cerca de 80% de toda a populao reside em ambientes
urbanos. A cidade tornou-se palco das diferenas sociais, onde
uma parte das reas perifricas (aquelas que no so
ocupadas pelos condomnios horizontais fechados, por
exemplo) sofre com a falta de infraestrutura e servios
bsicos. No bastando isso, a ocupao de reas irregulares
coloca a populao de baixo poder aquisitivo em uma efetiva
situao de risco, tornando-a vulnervel a situaes de
desastres, como a que aconteceu no morro do Bumba, em
Niteri, no Rio de Janeiro, no incio do ms de abril de 2010.
Sobre esse assunto, analise as alternativas a seguir e assinale a
INCORRETA.
a) A expanso urbana baseia-se em dois tipos principais de
ocupao habitacional: os loteamentos regulares, com projeto
aprovado pelas administraes municipais, e as ocupaes
irregulares (invases) de terrenos privados e pblicos. As
ocupaes irregulares tm ocorrido especialmente nas
encostas de grande declividade, com a implantao de
arruamento precrio, sem proteo e moradias precrias.
b) A segunda metade do sculo XX marcou a acelerao do
processo de urbanizao no Brasil e, entre as consequncias
deste processo, destacam-se a formao de regies
metropolitanas, a verticalizao e adensamento das reas j
urbanizadas e a expanso urbana para as reas perifricas.
c) Os processos de expanso urbana, periferizao e periurbanizao tm fortes impactos socioambientais, dentre eles:
o aumento das jornadas entre o centro e as reas perifricas,
ocasionando o aumento do trnsito e da poluio do ar; a

149. (Ufpr 2010) Analise atentamente a imagem a seguir:

37

ausncia de saneamento bsico e um forte processo de


desmatamento e degradao ambiental.
d) A ocupao e a expanso das periferias urbanas so
estimuladas pela reteno especulativa de terrenos em reas
mais bem localizadas, cujo acesso para todos, devido o alto
valor a ser pago pelas infraestruturas instaladas. No processo
de segregao espacial, o solo urbano torna-se uma
mercadoria disputada por diferentes agentes sociais e
econmicos urbanos, que utilizam de estratgias mercantis
para valorizar todas as reas do espao urbano.
RESPOSTA: D
COMENTRIO: O processo de urbanizao no Brasil recente
e deu-se de forma rpida e desordenada ao sabor, entre
outros aspectos, da especulao imobiliria que valoriza os
terrenos localizados em reas bem situadas e com melhor
infraestrutura, em detrimento de reas de risco em vrzeas e
encostas de morros que sobram para as populaes de baixa
renda.
A alternativa [D] falsa, os terrenos em reas bem localizadas
so os de maior valor cujo acesso s permitido a faixas da
populao que possuem mais renda.

A frase III falsa, a previso de urbanizao se realiza. Alm


disso, os investimentos e financiamentos tem sido aplicados
majoritariamente nas reas urbanas.
A frase IV falsa, no existe esse grande retorno da populao
de reas urbanas inseguras para a zona rural e sim algumas
pessoas buscam cidades menores na busca de melhores
condies de vida.
152. (Unemat 2010) A Regio Metropolitana uma regio
estabelecida por legislao estadual e constituda por
agrupamentos de municpios limtrofes no raramente
conurbados e integrados socioeconomicamente a uma
cidade central, com servios pblicos e de infraestrutura
comuns, ou necessidades de seu estabelecimento, em funo
de um sistema de conexo existente entre as unidades que a
compem.
(ADAS, 2005).
A partir desta definio, assinale a alternativa que aponta a
regio metropolitana do Brasil que possui a maior e a menor
participao em relao populao total do pas.
a) A maior So Paulo e a menor Rio de Janeiro.
b) A maior Rio de Janeiro e a menor Belm.
c) A maior So Paulo e a menor Curitiba.
d) A maior Rio de Janeiro e a menor Salvador.
e) A maior So Paulo e a menor Belm.

151. (Cesgranrio 2010)

RESPOSTA: E
COMENTRIO: A urbanizao acentuada promoveu forte
crescimento de cidades e sua conurbao, formando
complexos urbanos denominados regies metropolitanas. A
cidade de So Paulo foi a 1 regio metropolitana do Brasil,
estabelecida em 1972, constituindo-se na maior regio em
rea. Belm, capital do Par, embora exera influncia em
extensas reas da Amaznia te uma rea urbana conurbada
menor que So Paulo.

Considerando o grfico acima e o contexto social, poltico e


econmico e suas repercusses na organizao do espao
brasileiro, a partir de 1950, analise as afirmaes a seguir.
I As transformaes ocorridas na estrutura urbana brasileira
foram resultado de um rpido crescimento da industrializao
que caracterizou o pas na segunda metade do sculo XX.
II Os problemas decorrentes da urbanizao tendem a se
agravar de acordo com a previso do grfico, e se tornam
urgentes polticas de planejamento urbano e investimentos
em infraestrutura urbana.
III A tendncia, no caso brasileiro, de que essa previso no
se realize, j que os investimentos e o financiamento de
melhorias na rea rural tm sido ao comum nos ltimos
governos.
IV A estimativa apresentada no considerou o retorno de
grande parte da populao urbana para o campo, em virtude
de problemas decorrentes da urbanizao, tais como violncia
e desemprego.

A alternativa [A] falsa, o Rio de Janeiro maior que Belm.


A alternativa [B] falsa, o Rio de Janeiro menor que So
Paulo.
A alternativa [C] falsa, Curitiba maior que Belm.
A alternativa [B] falsa, Salvador maior que Belm.

153. (Ueg 2010) Invadindo espaos


As cidades que antes serviam para abrigar os cidados, hoje
so o ambiente tpico dos automveis.
Nos pases em desenvolvimento, a ao do poder pblico em
favor do automvel foi e tem sido to eficaz que fica cada vez
mais difcil para os moradores das cidades viver com um
mnimo de conforto sem um automvel particular. S os que,
em razo do seu padro de renda, no podem almejar ter um
carro sujeitam-se ao ineficiente sistema de transporte pblico.
Neles perdem vrias horas do dia, muitos dias por ano, alguns
anos de vida.
Se as condies fossem outras, se o transporte pblico fosse
mais eficiente, menor seria a parcela de renda que boa parte
da populao precisa reservar para compra e manuteno de
um carro particular, menores seriam as demandas por
investimentos pblicos no sistema virio, maiores seriam as
disponibilidades da renda pessoal para outras atividades,
incluindo lazer, e maiores seriam os recursos que o poder
pblico poderia destinar para melhorar a qualidade de vida de
uma populao.

Esto corretas APENAS as afirmaes


a) I e II.
b) I e IV.
c) II e III.
d) II e IV.
e) III e IV.
RESPOSTA: A
COMENTRIO: O processo de urbanizao no Brasil, como no
mundo em geral, irreversvel. Apesar do rpido crescimento
e da forma irregular como se mostra a realidade urbana
parte da sociedade brasileira.

38

OKUBARO, Jorge J. O automvel, um condenado? So Paulo:


Senac, 2001. p. 52-53. (Adaptado).
De acordo com a anlise do texto acima, CORRETO afirmar:
a) o elevado custo, os problemas de congestionamento das
grandes cidades (nibus, automveis, caminhes) so os
maiores responsveis pela poluio atmosfrica nos centros
urbanos, ocasionando a reduo na qualidade de vida da
populao.
b) a baixa tarifa do transporte urbano um incentivo ao
trabalhador, independentemente do tempo gasto para o
deslocamento entre a casa e o trabalho, o que resulta em
ganho no oramento no final do ms.
c) a qualidade do transporte coletivo urbano, fruto de
estratgias de planejamento, acaba por estimular a utilizao
do transporte coletivo, diminuindo o nmero de veculos nos
grandes centros urbanos.
d) a crescente preocupao com o planejamento urbano pelos
rgos oficiais do governo tem trazido melhorias na conduo
do trfego e a diminuio dos custos na infraestrutura viria.

RESPOSTA: A
COMENTRIO: O transporte rodovirio tem altos custos, alm
de se caracterizar como emissor de gases estufa. O uso
intensivo e crescente do carro particular dificulta muito o
planejamento de sistemas mais eficientes e menos poluentes.

e) Todos os enunciados.
RESPOSTA: D
COMENTRIO: A construo de cidades planejadas beneficia
muito a sociedade, sendo locais espacialmente ordenados
com baixos custos, maior eficincia e melhor circulao. Por
outro lado, as cidades planejadas arcam com elevados custos
em sua implantao e construo.
A frase II falsa, antes de Braslia havia sido construda de
modo planejado a cidade de Belo Horizonte para se tornar
capital de Minas Gerais em lugar de So Joo DEl Rei.
155. (Ufg 2010) A reordenao produtiva do territrio brasileiro
alterou o padro da rede urbana, impactando a estrutura e as
funes de diferentes tipos de cidades. Em relao dinmica
das cidades mdias,
a) as leis de responsabilidade fiscal contriburam para redefinir
as atividades produtivas.
b) a adoo do marketing de consumo urbano introduziu
novos objetos que reordenaram a morfologia urbana.
c) a ligao rpida com as metrpoles possibilitou a funo de
gerar novas tecnologias.
d) a integrao econmica com o campo por meio da
agroindstria aumentou as funes tercirias.
e) os eventos acadmicos e as atividades tursticas
transformaram os modelos de suas gestes.
RESPOSTA: D
COMENTRIO: A modernizao da economia brasileira vem
atingindo todos os seus segmentos produtivos. As cidades
mdias, em sua maior parte, localizam-se em zonas rurais
produtivas e com o advento do agronegcio, passam por
mudanas em sua estrutura urbana para atender as demandas
por servios dessa forma de atividade.

A alternativa [B] falsa, o transporte urbano nas principais


cidades brasileiras caro.
A alternativa [C] falsa, o transporte coletivo , em geral, mal
planejado e de m qualidade em termos de racionalidade de
linhas fazendo passageiros de ceras linhas perderem muitas
horas de viagem ao dia.
A alternativa [D] falsa, o governo tem um tempo
relativamente lento de resposta a demandas sociais, em
geral..

A alternativa [A] falsa, leis de responsabilidade fiscal tem por


finalidade ajudar a administrao em suas instncias,
municipal, estadual e federal.
A alternativa [B] falsa, em cidades mdias o apelo ao
consumo tem um regionalismo peculiar, onde o marketing de
consumo urbano em grande escala, tem poucos reflexos
locais.
A alternativa [C] falsa, as novas tecnologias tem variadas
maneiras de serem acessadas e adquiridas no dependendo
necessariamente de ligao de proximidade com reas
metropolitanas.
A alternativa [E] falsa, as atividades tursticas so especficas
de localidades mais vocacionadas, no necessariamente em
qualquer cidade mdia.

154. (Pucpr 2010) Em 21 de abril de 2010, Braslia completou 50


anos.
Sobre isso, pode-se considerar que:
I. As cidades satlites deveriam ter sido criadas de forma
planejada e somente aps o Plano Piloto terem sido ocupados,
o que foi desvirtuado j na primeira dcada. Atualmente as
cidades satlites comportam aproximadamente 80% da
populao do Distrito Federal, e apresentam ndices de renda
e educao menores que os de Braslia.
II. Braslia um marco na organizao do espao regional e
urbano, pois a primeira cidade planejada do Brasil. O projeto
modernista de Lcio Costa divide a cidade em setores
ocupacionais. Desde o seu planejamento, constatou-se que o
Distrito Federal no poderia ter vocao industrial ou turstica.
Seria somente a capital administrativa do pas.

156. (Uece 2010) No que se refere cidade, ela analisada


tambm em sua dimenso histrica, produto da diviso do
trabalho. Isso significa que a cada momento a cidade vai
assumir dimenses e contedos diferentes.

III. A construo de Braslia representa um marco importante


na ocupao do centro-oeste brasileiro. A partir dos anos de
1960, construram-se grandes rodovias as quais passaram a
ligar a regio ao restante do pas, o que impulsionou a
ocupao (da regio) e a transformao do cerrado em rea
de grande produo agropecuria.

CARLOS, Ana Fani Alessandri. Repensando a Geografia Urbana:


Uma nova perspectiva se abre. pp. 157-198. apud. Os
Caminhos da Reflexo sobre a Cidade e o Urbano. Ana Fani
Alessandri Carlos. (org.). So Paulo: EDUSP. 1994.
A anlise do urbano no Brasil desafiadora e envolve tambm
a (re)organizao espacial pelo capital na dimenso escalar do
tempo. A partir do excerto anterior e considerando a questo
urbana no Brasil, analise as afirmaes a seguir.

Est(o) correta(s):
a) Somente o enunciado I.
b) Somente os enunciados I e II.
c) Somente os enunciados II e III.
d) Somente os enunciados I e III.

I H uma sutil relao entre o capital e o processo de


urbanizao de tal forma que a cidade pode ser entendida
39

como um dos principais ambientes para a produo do capital.


II A hierarquia urbana brasileira comandada por 12
metrpoles que estabelecem redes urbanas e estas so
diferentes no tamanho, na complexidade e na organizao.
III A morfologia do tecido urbano no Brasil no
homognea, ao contrrio, desigual. Os impactos ambientais
ao longo do tempo sobre as formas de uso e ocupao do
solo, a violncia e o trabalho, so alguns dos temas mais
comuns Cidade no Brasil.
verdadeiro o que se afirma
a) somente em I e II.
b) somente em I e III.
c) somente em II e III.
d) em I, II e III.
RESPOSTA: D
COMENTRIO: A urbanizao um processo recente no Brasil
e ocorreu de modo rpido e desordenado o que resulta em
matriz
causadora
de
diversos
problemas
como
congestionamentos, poluio, insegurana. Mas ao mesmo
tempo, no espao urbano que se exerce a profisso, a
poltica e suas disputas emprestando aos centros urbanos,
grande dinamismo, complexidade e diversidade.

A alternativa [A] falsa, apesar do crescimento das atividades


econmicas ainda no aparecem capitais nordestinas na
listagem.
A alternativa [B] falsa, Manaus tem seu principal vetor de
desenvolvimento econmico na Zona Franca que produz bens
de consumo.
A alternativa [D] falsa, as cidades do entorno das capitais
avanaram com base em atividades industriais e de servios.
158. (Enem 2 aplicao 2010) O crescimento rpido das cidades
nem sempre acompanhado, no mesmo ritmo, pelo
atendimento de infraestrutura para a melhoria da qualidade
de vida. A deficincia de redes de gua tratada, de coleta e
tratamento de esgoto, de pavimentao de ruas, de galerias
de guas pluviais, de reas de lazer, de reas verdes, de
ncleos de formao educacional e profissional, de ncleos de
atendimento mdico-sanitrio comum nessas cidades.
ROSS, J. L .S. (Org.) Geografia do Brasil. So Paulo: EDUSP,
2009 (adaptado)
Sabendo que o acelerado crescimento populacional urbano
est articulado com a escassez de recursos financeiros e a
dificuldade de implementao de leis de proteo ao meio
ambiente, pode-se estabelecer o estmulo a uma relao
sustentvel entre conservao e produo a partir
a) do aumento do consumo, pela populao mais pobre, de
produtos industrializados para o equilbrio da capacidade de
consumo entre as classes.
b) da seleo e recuperao do lixo urbano, que j uma
prtica rotineira nos grandes centros urbanos dos pases em
desenvolvimento.
c) da diminuio acelerada do uso de recursos naturais, ainda
que isso represente perda da qualidade de vida de milhes de
pessoas.
d) da fabricao de produtos reutilizveis e biodegradveis,
evitando-se substituies e descartes, como medidas para a
reduo da degradao ambiental.
e) da transferncia dos aterros sanitrios para as partes mais
perifricas das grandes cidades,visando-se preservao dos
ambientes naturais.

157. (Ufu 2010) Municpios que representam juntos 25% do PIB


nacional
1999
1 So Paulo (SP)
2 Rio de Janeiro (RJ)
3 Braslia (DF)
4 Belo Horizonte (MG)
5 Manaus (AM)
6 Curitiba (PR)
7 Porto Alegre (RS)

2002
1 So Paulo (SP)
2 Rio de Janeiro (RJ)
3 Braslia (DF)
4 Belo Horizonte (MG)
5 Manaus (AM)
6 Duque de Caxias (RJ)
7 Curitiba (PR)
8 Guarulhos (SP)
9 So Jos dos Campos (SP)

Fonte: IBGE. Produto Interno Bruto dos Municpios 1999-2002.


Disponvel em:
http://www.ibge.gov.br/home/presidencia/noticias/noticia_vi
sualiza.php?id_noticia=354

RESPOSTA: D
COMENTRIO: As cidades so grandes concentraes capazes
de consumir e produzir enormes quantidades de produtos e
resduos causando variados graus de poluio no ar, nas guas
e no solo. O uso de produtos reciclveis ou biodegradveis
pode ser uma forma de reduo da degradao ambiental.

Observe o quadro acima e assinale a alternativa correta.


a) O crescimento das atividades econmicas na regio
nordeste justifica o aparecimento das capitais nordestinas na
listagem apresentada.
b) A mudana na posio de Manaus na listagem apresentada
decorrente das dificuldades de comercializao de produtos
de origem florestal.
c) Devido expanso das atividades econmicas nas regies
metropolitanas, as cidades do entorno das capitais passam a
ter representatividade significativa no PIB nacional.
d) A entrada das cidades do entorno das capitais na listagem
apresentada est basicamente associada atividade
econmica representada pelo cinturo verde das regies
metropolitanas.
RESPOSTA: C
COMENTRIO: A metropolizao a partir da dcada de 1970
desenvolveu-se com base nas atividades industriais e
econmico-financeiras e com o passar do tempo algumas
cidades das reas metropolitanas passaram a se destacar na
onda de expanso econmica a partir da cidade central.

A alternativa [A] falsa, o aumento de consumo de produtos


industrializados em qualquer classe social gera resduos
poluentes na forma de lixo.
A alternativa [B] falsa, seleo e recuperao de lixo urbano
ainda no prtica rotineira mundo afora.
A alternativa [C] falsa, o desenvolvimento sustentvel prev
o uso racional de recursos naturais e no sua diminuio
acelerada.
A alternativa [E] falsa, a tendncia que nas partes
perifricas da cidade apaream reas de preservao
ambiental.

159. (Ufg 2010) Leia o texto a seguir.


No fundo do vale o lenol fretico aflora para formar os rios.
Estes tm seus ciclos regulados pelos perodos de cheia e
vazante, e pelos espaos representados pelas plancies de
40

inundao. Este termo encerra em si sua funo: abrigar as


guas do rio quando do seu natural extravasamento nas
pocas de cheias.

favelas , quando se referem a seus locais de moradia.


Empregado pela mdia, pelo governo, pelas associaes locais,
pelas ONGs, o termo comunidade muitas vezes explicita a
dificuldade dessa operao de levar em conta o que pensam
os que se veem nomeados de uma forma negativa.
Se este uso eufemstico recorrente, vale observar que, em
muitas circunstncias, do ponto de vista dos moradores, o que
mais reivindicado a no identificao, ou seja,
preferencialmente, a anulao de qualquer referncia
identidade territorial em trocas sociais diversas.
O termo comunidade em seus usos eufemsticos no capaz
de impedir a associao da pessoa com os traos negativos
provenientes dessa identificao; somente indica a sua penso
destes pelo uso momentneo de aspas que podem ser
retiradas quando for preciso.

LOPES, Luciana Maria. Tragdia ou descaso. Disponvel em:


<www.opopular.com.br/anteriores/03out2009/opiniao>.
Acesso em: 3 out. 2009.
Este texto analisa as recorrentes tragdias na regio Sul do
Brasil, com desmoronamentos, desabamentos de casas,
mortes e centenas de pessoas desabrigadas.
A explicao geogrfica para essas tragdias pode ser
encontrada no seguinte fato:
a) desvios dos leitos dos rios que direcionam o fluxo das guas
em um mesmo sentido, tornando as enchentes inevitveis.
b) ausncia de planejamento do uso do solo causando
especulao imobiliria e possibilitando a ocupao de novos
espaos sem fiscalizao.
c) encostas ngremes que impedem a absoro de quantidade
volumosa de gua vertida em direo aos vales.
d) altas precipitaes pluviomtricas anuais que dificilmente
so previstas devido ao uso de equipamentos meteorolgicos
obsoletos.
e) presena de solos profundos porosos que retm gua,
provocando desabamentos de construes.

BIRMAN, Patrcia. Favela comunidade? In SILVA, L.A.(org.)


Vida sob cerco. Violncia e rotina nas favelas do Rio de
Janeiro. Rio de Janeiro: Nova Fronteira, 2008, pp.106-7.
Adaptao.

160. (Uff 2010) De acordo com o texto, destaca-se o seguinte


trecho:
Observamos que este uso (do eufemismo) generalizado
entre diferentes grupos sociais a mesma preocupao pode
levar a substituir o termo comunidade por outro equivalente,
como morro ou bairro. (ref. 1)
A substituio apontada no trecho acima pode ser encontrada
em letras de algumas canes, como no exemplo a seguir.

RESPOSTA: B
COMENTRIO: A urbanizao desenfreada, a especulao
imobiliria e mecanismos de fiscalizao ineficientes, esto
entre as principais causas dos trgicos processos de
deslizamentos de encosta e desabamentos.

Endereo dos Bailes


A alternativa [A] falsa, as retificaes de traados de rios
normalmente dispersam a gua para evitar concentraes
hdricas.
A alternativa [C] falsa, encostas ngremes vegetadas absorve
gua e no causam problemas de escorregamentos em curto
prazo.
A alternativa [D] falsa, as previses meteorolgicas
atualmente tem um elevado grau de preciso, municiando as
autoridades com informaes fundamentais para a tomada de
decises sobre movimentao de pessoas para fora das reas
de risco.
A alternativa [E] falsa, as reas de encostas, em geral, tem
solos rasos, pouco profundos, mais fceis de encharcar, a
curto prazo sem cobertura vegetal.

(...)
! Se liga que eu quero ver
O endereo dos bailes eu vou
falar pra voc
que de sexta a domingo na
Rocinha o morro
enche de gatinha
Que vem pro baile curtir
Ouvindo charme, rap, melody ou
montagem,
funk em cima, funk embaixo,
Que eu no sei pra onde ir

TEXTO PARA A PRXIMA QUESTO:


UM LUGAR COMUM, O EUFEMISMO E A FAVELA

(...)
Tem outro baile que a galera
toda treme
l no baile do Leme l no
Morro do Chapu
Tem na Tijuca um baile que
sem baguna
A galera fica maluca l no
Morro do Borel
(...)

MC Jnior e MC Leonardo
Essa associao entre favela e morro pode ser explicada pela
combinao dos seguintes aspectos:
a) autossegregao / interferncia do planejamento estatal.
b) segregao social / especificidade do stio urbano.
c) periferizao / espao urbano como mercadoria.
d) metropolizao / busca pela legalizao da posse.
e) verticalizao / poltica demogrfica natalista.

Uma valorizao do eufemismo parece importante na


dinmica das relaes sociais. Seu emprego permitiria, em
parte, contornar o valor negativo que certas expresses
espelham. O eufemismo, no entanto, no afronta o estigma.
Seu uso indica uma relao de cortesia, necessria, no curso
das trocas sociais que se passam com aqueles que no podem
se desfazer de suas marcas.
1
Observamos que este uso generalizado entre diferentes
grupos sociais a mesma preocupao pode levar a substituir
o termo comunidade por outro equivalente, como morro ou
bairro. Sabemos todos que nas trocas sociais o mais
importante o sentido que se elabora no interior das suas
dinmicas. O esforo continuado para no ferir as pessoas que
acompanham as trocas sociais correntes motiva o uso do
termo comunidade em muitos momentos, inclusive por
aqueles diretamente concernidos as pessoas que moram em

RESPOSTA: B
COMENTRIO: As favelas, os bairros populares, as moradias
em reas de risco, so o resultado da ocupao irregular do
espao urbano, mal ou sem nenhum planejamento, ao sabor
dos interesses da especulao imobiliria. So reas
socialmente segregadas no espao urbano e a referncia de
localizao geogrfica. A comunidade fica no morro, ou na
vrzea do rio.
41

A alternativa [A] falsa, a segregao resultado de um


processo referencial externo rea e o planejamento estatal
nessas reas pouco ou nenhum.
A alternativa [C] falsa, periferia um termo referenciado a
um local afastado de um centro sem implicar necessariamente
na sua qualidade urbana.
A alternativa [D] falsa, o termo metropolizao amplo e
abarca inmeros conceitos urbanos onde a segregao um
dos componentes desse conjunto.
A alternativa [E] falsa, a verticalizao diz respeito a um
processo em larga escala, que conta com planejamento estatal
e aplicado em reas urbanas valorizadas, no ficando restrito a
um morro.

GEOGRAFIA II
162. (Unesp 2012) Analise o grfico.

161. (Ueg 2012) Leia o trecho abaixo.


Quando a cidade sepulta no anonimato os seus tipos de rua,
sinal que o humanismo desapareceu das esquinas e o
tempo/relgio passou a controlar a vida de seus habitantes.
[...]. Goinia cresceu. Sepultou seus tipos de rua, atropelados
na pressa dos automveis.

A partir da anlise dos dados apresentados no grfico e de


seus conhecimentos, correto afirmar que:
a) a curva populacional da regio Nordeste apresenta
crescimento acentuado a partir da dcada de 1970, superando
a da regio Sudeste.
b) a regio Sul manteve constante seus ndices de crescimento
populacional em todo o perodo analisado, espelhando um
forte fluxo migratrio para a regio.
c) a curva populacional da regio Sudeste, a partir da dcada
de 1980, apresenta um crescimento mais acelerado do que a
curva populacional do Brasil.
d) apesar de as regies Nordeste e Sudeste, na dcada de
1940, possurem nmeros populacionais semelhantes, a curva
da regio Nordeste supera a da regio Sudeste a partir da
dcada de 1970.
e) as regies Norte e Centro-Oeste, em todo o perodo
analisado, apresentaram comportamentos prximos em seus
nmeros absolutos de populao.

TELES, Jos Mendona. Crnicas de Goinia. Goinia: Kelps,


2005. p. 65-66.
O trecho acima tematiza o crescimento de Goinia e relaciona
desaparecimento do humanismo e velocidade dos automveis
ao tempo controlado da populao. Considerando as leituras
de base sociolgica e geogrfica acerca do espao urbano,
correto afirmar:
a) o crescimento de Goinia significou a formao de uma
moderna cidade capitalista, marcada pelo predomnio do
controle do tempo e pela primazia dos automveis nas ruas.
b) o crescimento de Goinia significou uma mudana cultural
que mescla rural e urbano, mantendo o predomnio de valores
humanos.
c) Goinia cresceu e tornou-se uma cidade moderna, por isso
mudou o nome de suas ruas para o nome de figuras folclricas
e populares, com o intuito de preservar a memria da cidade.
d) Goinia cresceu por causa do processo de expanso da
indstria automobilstica, da construo civil e da interveno
estatal.

RESPOSTA: E
COMENTRIO: O grfico no aponta a ultrapassagem da
populao do Nordeste sobre a regio Sudeste. A alternativa
[A] est, portanto, errada.
A regio Sul no teve sua evoluo constante. Alternativa [B]
errada, portanto.
O Brasil tem um comportamento de curva mais acentuado em
direo ao crescimento populacional que o Sudeste, pois se
trata de uma mdia nacional, que inclui regies com ndices de
crescimento maiores. Errada a alternativa [C].
A alternativa [D] est errada pelo mesmo motivo que a
alternativa [A], j comentada.
O grfico mostra que Centro-Oeste e Norte possuem nmeros
absolutos prximos de populao.

RESPOSTA: A
COMENTRIO: A questo trata do processo de modernizao
da cidade de Goinia. Acompanhando a fluidez da sociedade
contempornea, a cidade tornou-se um local de circulao de
automveis, desumanizando suas ruas e sendo guiada pelo
tempo da lgica de produo capitalista.
Anlise das alternativas:
a) CORRETA O texto indica o crescimento da cidade de
Goinia imprimindo a dinmica do espao produtivo, onde o
trfego se traduz como o conceito da circulao do setor
produtivo e o anonimato das ruas, como a perda da
importncia dos valores humanos.
b) INCORRETA A afirmativa se contrape ao texto do
enunciado, que demonstra com o conceito de anonimato das
ruas a perda dos valores humanos.
c) INCORRETA O texto no faz referncia mudana dos
nomes das ruas.
d) INCORRETA O texto no faz referncia s causas do
crescimento da cidade.

163. (Udesc 2012) Sobre a populao brasileira, pode-se afirmar:


I. No Censo de 2000, a populao brasileira era de quase 170
milhes de habitantes. Em 2010, esse nmero aumentou para
mais de 190 milhes.
II. Na populao brasileira, h aproximadamente 97 milhes
de mulheres e 93 milhes de homens.
III. A maioria das famlias brasileiras (91%) formada por
pessoas com laos de parentesco. Mas existem tambm as
famlias sem parentesco, que podem ser duas ou mais pessoas
ou unipessoais (apenas uma pessoa). O nmero de famlias
42

unipessoais foi o que mais aumentou: de 2,4 milhes em 1991


para 4,1 milhes em 2000.
IV. A taxa de fecundidade vem reduzindo, no Brasil, desde a
dcada de 1960.

Assinale a alternativa correta.


a) Somente a afirmativa 3 verdadeira.
b) Somente as afirmativas 1 e 4 so verdadeiras.
c) Somente as afirmativas 3 e 4 so verdadeiras.
d) Somente as afirmativas 2, 3 e 4 so verdadeiras.
e) As afirmativas 1, 2, 3 e 4 so verdadeiras.

Assinale a alternativa correta.


a) Somente as afirmativas I e IV so verdadeiras.
b) Somente as afirmativas II, III e IV so verdadeiras.
c) Somente as afirmativas I e IV so verdadeiras.
d) Somente as afirmativas II e III so verdadeiras.
e) Todas as afirmativas so verdadeiras.
RESPOSTA: E
COMENTRIO: Segundo os dados do Censo 2010, o Brasil
possui uma populao total de 190.732.694 de pessoas. Em
2000, esse total era de 169.799.170. O nmero de mulheres
na populao superior quantidade de homens, em boa
parte porque na populao masculina o nmero de mortes por
causas no naturais maior (acidentes de trnsito, violncia
urbana etc.). A populao feminina corresponde a 97.342.162
de pessoas, enquanto a populao masculina de 93.390.532.
A taxa de fecundidade tem cado desde a dcada de 1960,
devido ao processo de urbanizao e de insero da mulher no
mercado de trabalho. Em 1960, a taxa de fecundidade era de
6,3 filhos por mulher e em 2010 passou a ser de 1,89 filhos por
mulher.
Na assertiva III h imprecises cronolgicas e de dados. A
porcentagem das famlias compostas por laos de parentesco
de 88,2%, ou seja, menor do que o valor apontado de 91%.
O nmero de famlias unipessoais realmente vem aumentando
nas ltimas dcadas, porm a questo deixa de apontar que o
nmero total de 2010 de famlias com essa caracterstica de
aproximadamente 7,1 milhes.

RESPOSTA: C
COMENTRIO: 1. Incorreta. H uma inverso dos eixos da
pirmide. O enunciado ainda poderia apresentar o significado
de uma pirmide etria, como um grfico que representa a
populao por quantidade e distribuda por faixas etrias.
Seria mais fcil para o aluno relembrar e desenvolver
raciocnio.
2. Incorreta. A pirmide etria no revela em todo o perodo
analisado que o Brasil um pas de jovens, pelo contrrio ela
demonstra que o Brasil um pas em transio demogrfica.
165. (Fatec 2012) Sistematicamente, o IBGE (Instituto Brasileiro de
Geografia e Estatstica) tem apresentado resultados do Censo
2010. Dentre esses resultados pode-se destacar
a) o expressivo crescimento demogrfico das regies Norte e
Centro-Oeste.
b) a manuteno do predomnio de populao rural nos
estados do Amap e do Piau.
c) a reduo do nmero de idosos nas grandes capitais do
Sudeste e do Nordeste.
d) a sensvel reduo da populao indgena nos estados
amaznicos.
e) o crescimento expressivo da taxa de fecundidade em todos
os estados.
RESPOSTA: A
COMENTRIO: Levando em considerao a anlise dos dados
preliminares do Censo de 2010, a alternativa que est correta
a [A], uma vez que as outras alternativas apresentam
afirmativas que no so sustentadas pelos dados divulgados.

164. (Ufpr 2012) Os grficos abaixo representam as pirmides


etrias da populao brasileira das dcadas de 1980 e 2000 e
projees para 2020 e 2040.

166. (G1 - cftmg 2012) Belo Horizonte, RRMBH* e RMBH


Populao e Taxa geomtrica de crescimento (1.940 - 2.000)

Com base nessas pirmides etrias, considere as seguintes


afirmativas:
* Todos os municpios da RMBH, com exceo de Belo
Horizonte.

1. Nas ordenadas esto o contingente populacional e nas


abscissas os grupos de idade.
2. A base larga da pirmide em todo o perodo analisado
revela que o Brasil continuar a ser um pas de jovens e
refora a necessidade do incremento de polticas pblicas de
ateno a tais camadas da populao brasileira.
3. A estrutura etria da populao representada nos grficos
tem relao com a economia e mostra a transformao da
populao economicamente ativa, definida como aquela que
compreende o potencial de mo de obra com que pode contar
o setor produtivo, isto , a populao ocupada e a populao
desocupada.
4. As transformaes nas pirmides no Brasil ao longo do
tempo revelam a transio demogrfica, explicada pela
combinao de fatores como baixas taxas de natalidade,
reduo das taxas de mortalidade, elevao na expectativa de
vida, reduo na taxa de fecundidade e maior acesso e
assistncia sade.

Fonte: FERNANDES, Joseane de Souza. A Expanso Urbana de


Belo Horizonte e da Regio Metropolitana de Belo Horizonte: o
caso especfico do municpio de Ribeiro das Neves. Tese de
doutorado. Cedeplar. UFMG, 2008.
A partir da evoluo do processo de metropolizao
apresentado, correto inferir que a
a) variao das taxas de crescimento da RMBH reduziu o
quantitativo de habitantes nesse espao.
b) mudana da metrpole na hierarquia urbana brasileira
influenciou no decrscimo das taxas de crescimento
demogrfico da capital.
c) diminuio nas taxas de crescimento demogrfico em Belo
Horizonte impactou negativamente no tamanho absoluto de
sua populao.

43

d) expanso da mancha urbana em Belo Horizonte interferiu


no incremento da participao relativa dos outros municpios
no total de habitantes.

III. A biodiversidade da floresta assegura alta renda per capita


aos habitantes da Amaznia, enquanto moradores da caatinga
nordestina padecem em bolses de pobreza.
IV. Embora Braslia detenha alguns dos melhores indicadores
socioeconmicos do pas, o prprio Distrito Federal e
arredores abrigam um bolso de pobreza.

RESPOSTA: D
COMENTRIO: Naturalmente, o crescimento de uma
metrpole interfere no desenvolvimento das cidades
circunvizinhas, confirmando a alternativa [D], no caso de Belo
Horizonte.

Est correto o que se afirma em


a) I, II e III, apenas.
b) I, II e IV, apenas.
c) II e III, apenas.
d) III e IV, apenas.
e) I, II, III e IV.

167. (Fuvest 2012) Ainda no comeo do sculo 20, Euclides da


Cunha, em pequeno estudo, discorria sobre os meios de
sujeio dos trabalhadores nos seringais da Amaznia, no
chamado regime de peonagem, a escravido por dvida. Algo
prximo do que foi constatado em So Paulo nestes dias
[agosto de 2011] envolvendo duas oficinas terceirizadas de
produo de vesturio.

RESPOSTA: B
COMENTRIO: I. CORRETA: nos bolses de pobreza da regio
Sul predomina grande propriedade com criao extensiva de
gado (Pampas) e cultivo de gros (oeste do Paran e de Santa
Catarina);
II. CORRETA: a regio Sudeste apresenta elevada renda per
capita graas diversidade de suas atividades econmicas e
elevada produo industrial; todavia, microrregies de
pobreza so evidentes, como, por exemplo, o Vale do Ribeira
no estado de So Paulo e vale do Jequitinhonha em Minas
Gerais;
III. INCORRETA: os mapas evidenciam a existncia de bolses
de pobreza tanto no litoral quanto na caatinga nordestina.
Alm disso, a Amaznia em seu conjunto, no apresenta renda
per capita superior do Nordeste em geral.
IV. CORRETA: a renda per capita do Distrito Federal elevada,
devido s atividades administrativas ali concentradas; todavia,
as cidades-satlites abrigam populao extremamente carente
de renda e com baixa disponibilidade de servios pblicos.

Jos de Souza Martins, 2011. Adaptado.


No texto acima, o autor faz meno presena de regime de
trabalho anlogo escravido, na indstria de bens
a) de consumo no durveis, com a contratao de imigrantes
asiticos, destacando-se coreanos e chineses.
b) de consumo durveis, com a superexplorao, por meio de
empresas de pequeno porte, de imigrantes chilenos e
bolivianos.
c) intermedirios, com a contratao prioritria de imigrantes
asiticos, destacando-se coreanos e chineses.
d) de consumo no durveis, com a superexplorao,
principalmente, de imigrantes bolivianos e peruanos.
e) de produo, com a contratao majoritria, por meio de
empresas de mdio porte, de imigrantes peruanos e
colombianos.
RESPOSTA: D
COMENTRIO: Atualmente, as oficinas de costura, localizadas
na regio central da cidade de So Paulo, tm recorrido mo
de obra que imigrou clandestinamente, constituda por
trabalhadores bolivianos e peruanos; devido ilegalidade e
no terem direitos trabalhistas, so sujeitos a condies de
superexplorao (baixos salrios e ausncia de benefcios
sociais).

168. (Fuvest 2012) Observe os mapas do Brasil.

169. (Ueg 2012) Quando se analisa a populao economicamente


ativa (PEA) de pases desenvolvidos, verifica-se um elevado
porcentual de ativos com baixos ndices de desemprego. Por
outro lado, a situao dos pases subdesenvolvidos apresenta
uma realidade oposta, com uma considervel parcela da
populao dedicada ao subemprego e, portanto, ligada
economia informal. A esse respeito, correto afirmar:
a) o crescimento da economia informal nos pases
desenvolvidos est diretamente ligado ao processo de
globalizao que gerou o desemprego estrutural.
b) o Estatuto da Criana e do Adolescente probe, no Brasil, o
trabalho de menores de 18 anos, mesmo na condio de
aprendizes.
c) os vendedores ambulantes, guardadores de carros,
diaristas, entre outros, fazem parte da populao
economicamente ativa, pois no tm vnculos empregatcios.
d) na economia informal, os trabalhadores no participam do
sistema tributrio, no tm carteira assinada e nem acesso aos
direitos trabalhistas.
RESPOSTA: D
COMENTRIO: a) INCORRETA O crescimento da economia
informal ocorre nos pases subdesenvolvidos, como citado no
texto.
b) INCORRETA A lei 10.097 de 19/12/2000 probe o trabalho
dos menores de 16 anos de idade, salvo na condio de
aprendiz, a partir dos 14 anos.
c) INCORRETA A ausncia do vnculo empregatcio no
caracteriza a PEA, contudo, a principal caracterstica do
trabalho informal.
d) CORRETA A economia informal define-se como a prtica
de atividades econmicas do setor primrio, secundrio ou

Considere as afirmativas relacionadas aos mapas.


I. Alta concentrao fundiria e pouca diversificao da
atividade econmica so caractersticas de um bolso de
pobreza existente no extremo sul do Brasil.
II. A despeito de seus excelentes indicadores econmicos bem
como de seu elevado grau de industrializao, a Regio
Sudeste abriga bolses de pobreza.

44

tercirio sem o conhecimento do governo, ou seja, alheia a


qualquer institucionalidade ou legalidade.

b) Acordos e tratados internacionais, dos quais o Brasil


signatrio, tratam da questo do trabalho escravo e probem a
escravido por dvida, razo pela qual esse tipo de trabalho
forado no registrado no pas desde 1888.
c) Considerando a oferta de trabalho no Brasil, observa-se
uma mudana de tendncia, com a diminuio de oferta de
emprego no setor primrio e tercirio, e efetivo aumento da
oferta de emprego no setor secundrio da economia.
d) Uma caracterstica marcante das relaes de trabalho na
etapa atual do modo de produo a maior organizao
sindical.

170. (Ueg 2012) O grfico abaixo indica a evoluo e distribuio


da populao economicamente ativa (PEA) no Brasil, entre
1940 e 2006.

Com base na anlise do grfico, correto afirmar:


a) atualmente, a parcela da PEA engajada no comrcio e nos
servios supera em muito os trabalhadores da agropecuria e
da indstria.
b) com a urbanizao do pas, h o decrscimo constante da
populao ligada agropecuria, enquanto o setor secundrio
se sobrepe ao setor dos servios e do comrcio.
c) entre 1940 e 1970, o crescimento do setor primrio
acompanha o do setor secundrio.
d) o nmero de empregados na indstria cresce
gradativamente a partir de 1950, acelerando o crescimento
industrial a partir de 1980.

RESPOSTA: A
COMENTRIO: A alternativa [B] est errada, pois no so raras
as denncias de casos de uso de trabalho semiescravo e
escravo no Brasil. Alm disso, ao contrrio do que afirma a
alternativa [C], o Brasil segue a tendncia mundial de
terceirizao, isto , a mo de obra se concentra
principalmente no setor tercirio, enquanto que o setor
primrio e secundrio, pela mecanizao e uso de tecnologia,
vem reduzindo o emprego de mo de obra. tambm o
desemprego gerado pela globalizao mecanizao e uso da
tecnologia que vem enfraquecendo o movimento sindical: h
uma grande disponibilidade de mo de obra o que rebaixa os
salrios e torna precrias as condies de trabalho.
172. (Ufpb 2012) Observe a tabela a seguir.

RESPOSTA: A
COMENTRIO: a) CORRETA Como o grfico evidencia,
ocorreu um decrscimo constante da participao da PEA no
setor primrio responsvel pelas atividades agropecurias,
dentre outros desde os anos 1940, e em contrapartida, sua
elevao no setor secundrio at os anos 1980, e no setor
tercirio at 2006, absorvendo a maior porcentagem da
populao economicamente ativa.
b) INCORRETA Embora a urbanizao tenha sido uma das
causas do decrscimo da PEA no setor primrio, o setor
tercirio servios e comrcio o que se sobrepe aos
outros dois.
c) INCORRETA No perodo indicado ocorre o decrscimo do
setor primrio, e o crescimento se faz entre o setor secundrio
e tercirio.
d) INCORRETA O crescimento da PEA na indstria setor
secundrio ocorre dos anos 1940 aos anos 1980, quando
comea a declinar em razo da crise da economia brasileira
(A dcada perdida) e nos anos 2000, esse declnio continua
em razo da automao da indstria brasileira. Atualmente, a
menor absoro nesse setor se d em funo do processo de
desindustrializao.

Com base na leitura da tabela e considerando o tema


dinmica demogrfica brasileira, correto afirmar:
a) A populao brasileira aumentou de forma significativa no
perodo 1960-2010. No entanto, na regio Nordeste,
identifica-se um inexpressivo aumento demogrfico, devido ao
contingente populacional que migrou para o Sudeste do pas.
b) A populao brasileira aumentou mais do que o dobro no
perodo 1960-2010. Porm, no se pode constatar o mesmo
fenmeno em relao populao do Cear, Alagoas e
Sergipe, estados com graves problemas sociais que
impulsionaram um xodo de seus habitantes.
c) O crescimento populacional brasileiro identificado em
todos os estados nordestinos. Contudo, h uma estabilizao
nesse crescimento no perodo 1980-1991, devido a polticas
pblicas de controle de natalidade nessa regio.
d) O Estado de Sergipe, menor unidade territorial do pas,
apresentou um aumento da sua populao, devido reduo
das taxas de mortalidade e aos avanos da medicina, ocorridos
em todo o pas.
e) A Bahia o estado nordestino com o maior nmero de
habitantes, seguido por Pernambuco e Maranho. Essa ordem
explicada pelo processo de industrializao de suas regies
metropolitanas.

171. (Unicamp 2012) Importantes transformaes produtivas e na


forma de organizao do trabalho tm ocorrido nas ltimas
dcadas em todo o mundo e tambm no Brasil. Assinale a
alternativa correta.
a) Em todo o mundo vm sendo observadas mudanas em
relao ao assalariamento e ao desemprego, como a
precarizao das relaes de trabalho para desonerao da
produo, e o crescimento da informalidade.

RESPOSTA: D
COMENTRIO:
a) INCORRETO O aumento populacional na regio
nordeste foi elevado.
b)
45

INCORRETO Em todos os estados citados na


alternativa ocorreu aumento populacional.

d) diminuio da taxa de analfabetismo


c)

INCORRETO No perodo 19801991 h crescimento


populacional, ainda que moderado, em alguns estados,
alm de no existir controle pblico sobre a natalidade.

d)

CORRETO O estado de Sergipe apresenta a menor rea


dentre as unidades de federao do Brasil, e seu aumento
populacional, assim como do resto do pas, est associado
reduo de taxa de mortalidade e aumento da
expectativa de vida.

e)

INCORRETO O maior nmero de habitantes pertence


respectivamente Bahia, Pernambuco e Cear.

RESPOSTA: A
COMENTRIO: O perodo mostrado no grfico corresponde a
um conjunto de aspectos internos e externos que possibilitou
ao Brasil, uma importante mudana em relao ao acesso ao
emprego e a renda, deslocando contingentes populacionais
significativos das classes E e D para a classe C e mesmo para a
classe B. A alta nos preos das commodities favoreceu o
balano de pagamentos; a estabilidade monetria interna
possibilitou o acesso ao crdito com taxas baixas e garantia de
cumprir com os pagamentos; a acelerao da economia gerou
mais empregos; a falta de mo de obra mais capacitada
acabou aumentando os salrios de certas categorias pela
oferta e procura. So aspectos que acabam levando a um
aumento do poder aquisitivo da populao. Por outro lado,
vale salientar que o sistema financeiro ainda privilegia o
rentismo, com remuneraes maiores para a especulao em
relao produo; o sistema tributrio incide muito sobre a
renda do trabalho e do consumo; os investimentos em
educao ainda no geraram mo de obra mais qualificada na
quantidade que se necessita atualmente, importante agente
de aumento da renda e a sade ainda precria em
atendimentos afetando a qualidade de vida da populao.

173. (Uerj 2012)

A proporo de homens e mulheres nesta pirmide etria


explicada pelo comportamento do indicador demogrfico
denominado:
a) taxa de migrao
b) expectativa de vida
c) crescimento vegetativo
d) sobremortalidade feminina

A alternativa [B] falsa, a expectativa de vida decorre de


polticas pblicas aplicadas a toda a populao.
A alternativa [C] falsa, apesar de historicamente oscilar
muito, oferta de emprego apresentou vis de alta no perodo.
A alternativa [D] falsa, a queda do analfabetismo apenas no
resgata uma populao de uma faixa de renda para outra
superior.

175. (Ueg 2011) Considere o quadro a seguir:

RESPOSTA: A
COMENTRIO: A expanso do agronegcio no centro oeste do
pas transformou a regio numa rea de migrao positiva,
isto , numa rea de atrao populacional. O maior
contingente de populao masculina se deve a necessidade de
mo de obra para o trabalho na fronteira agrcola.

174. (Uerj 2011) No grfico abaixo, esto representadas mudanas


no perfil socioeconmico da populao brasileira entre 2002 e
2009.

Um dos principais fatores que possibilitaram as mudanas


representadas no grfico :
a) elevao do poder aquisitivo
b) ampliao da expectativa de vida
c) estabilizao da oferta de emprego

Parte da queda da taxa de mortalidade infantil observada no


quadro resultado
a) da adoo de polticas pblicas de saneamento bsico e de
um conjunto de programas sociais, visando sade da
populao, como as campanhas de vacinao e aleitamento
materno, alm da melhoria na qualidade de vida das famlias.
b) de altos investimentos na sade pblica atravs da
construo de creches e hospitais, os quais passaram a
atender toda a populao, alm de inserir a mulher no
mercado de trabalho.
c) do processo de migrao da populao do campo para a
cidade, o que possibilitou a esta populao acesso a mais
emprego, melhoria das condies de vida e aumento salarial.
d) do aumento da produo de alimentos, sobretudo da soja,
que foi incorporada dieta das populaes de baixa renda,
eliminando assim a fome e a desnutrio.

RESPOSTA: A
COMENTRIO:

46

a)

b)
c)

d)

CORRETA A reduo das taxas de mortalidade infantil


resultante de polticas sociais que tm garantido melhorias na
sade pblica e na medicina preventiva.
INCORRETA A sade publica no a causa do ingresso da
mulher no mercado de trabalho.
INCORRETA O xodo rural no a causa direta da reduo
da taxa de mortalidade infantil, alm de no poder ser
associado diretamente elevao dos salrios.
INCORRETA A composio da dieta pela soja no a causa
da reduo da mortalidade infantil, e sim o conjunto de
investimentos em polticas sociais e de sade pblica.

176. (Fgv 2011) Em 01 de agosto de 2010, teve incio o 12 Censo


Demogrfico brasileiro. O Censo 2010 envolve o trabalho
direto de aproximadamente 230 mil pessoas, e seus resultados
vo subsidiar o planejamento de polticas pblicas e privadas
pelos prximos dez anos. A alternativa que descreve uma
mudana introduzida nesta edio do Censo :
a) Investigao sobre arranjos familiares formados por
cnjuges do mesmo sexo.
b) Investigao sobre os grupos tnicos e sua distribuio pelo
territrio nacional.
c) Investigao sobre as comunidades religiosas e sua
distribuio pelo territrio nacional.
d) Investigao sobre os padres de mortalidade e
fecundidade vigentes no pas.
e) Investigao sobre os nveis de renda e de consumo das
famlias brasileiras.

b) da populao residente para a elaborao de polticas


pblicas a partir de 2015.
c) exclusivamente estatsticos da populao residente.
d) da populao residente, incluindo os emigrados.
e) da populao relativa do pas, pois em todos os domiclios
foram aplicados os dois tipos de questionrios.

RESPOSTA: D
COMENTRIO: A cada novo censo, o IBGE incorpora novas
tcnicas e processos estatsticos que aprimoram os resultados
e diminuem as margens de erro, tornando a qualidade das
informaes cada vez melhor, mais precisa e confivel.
A alternativa [A] falsa: muito difcil que todos os domiclios
respondam ao censo. A atividade por si uma amostragem;
A alternativa [B] falsa: o censo fornece informaes para uso
a partir de 2010;
A alternativa [C] falsa: o censo considerou a populao de
emigrados;
A alternativa [E] falsa: o censo considera a populao
absoluta.

178. (Unesp 2011) Cndido Portinari conseguiu retratar em suas


obras o dia a dia do brasileiro comum, procurando denunciar
os problemas sociais do nosso pas. No quadro Os Retirantes,
produzido em 1944, Portinari expe o sofrimento dos
migrantes, representados por pessoas magrrimas e com
expresses que transmitem sentimentos de fome e misria.

RESPOSTA: A
COMENTRIO: O fator que justifica a alternativa [A] centra-se
no fato de que a composio familiar passa por mudanas,
requerendo, portanto, dos rgos de planejamento inquirir
sobre esses novos comportamentos.

177. (Uftm 2011) No ano de 2010, realizou-se, a partir de 1. de


agosto, o 12. Censo Demogrfico do Brasil. Sobre ele, leia o
seguinte texto do IBGE:
O Censo 2010 vai-nos dizer quem somos, onde estamos,
quantos somos e como vivemos. Para conseguir todas essas
informaes, o IBGE vai utilizar dois tipos de questionrio na
coleta de dados: o da amostra e o bsico. Apenas uma parte
dos domiclios ir responder s questes exclusivas dos
questionrios da amostra. Mas as perguntas do questionrio
bsico sero respondidas por todos, inclusive aqueles
domiclios que fazem parte da amostra. Mas que perguntas
bsicas so essas? E por que todo mundo precisa respondlas?
As perguntas do questionrio bsico so subdivididas em
temas. Algumas questes so referentes ao domiclio como
um todo e outras investigam caractersticas individuais de
cada morador. Conhea quais os assuntos que compem o
questionrio bsico:
domiclio;
emigrao internacional;
arranjos familiares;
caractersticas dos moradores;
registro de nascimento;
educao;
rendimento.

Sobre o tema desta obra, afirma-se:


I. Essa migrao foi provocada pelo baixo ndice de
mortalidade infantil do Nordeste, associado boa distribuio
de renda na regio.
II. Contriburam para essa migrao os problemas de cunho
social da regio Sul, com altas taxas de mortalidade infantil.
III. Os retirantes fugiram dos problemas provocados pela seca,
pela desnutrio e pelos altos ndices de mortalidade infantil
no Nordeste.
IV. Contriburam para essa migrao a desigualdade social, no
Nordeste.
correto apenas o que se afirma em
a) I.
b) I e II.
c) II, III e IV.
d) III e IV.
e) IV.

RESPOSTA: D
COMENTRIO: As obras de arte como expresses culturais
podem exercer vrios papis, inclusive em transversalidades,
como poltica e psicanlise. Obras como o quadro Os

O censo demogrfico de 2010 possibilitou conhecer os dados


a) detalhados da populao brasileira, pois todos os domiclios
responderam ao censo de amostra.
47

Retirantes podem ser interpretadas pelo seu vis poltico


como expresso ou forma de protesto contra a situao de
injustia social.
A afirmativa I falsa: alguns aspectos que provocaram fortes
migraes de nordestinos do Serto para outras reas foram,
entre outras, a mortalidade infantil e a m distribuio de
renda;
A afirmativa II falsa: na regio Sul, as taxas de mortalidade
infantil esto entre as menores do Brasil.

Com base nos grficos, analise as seguintes afirmativas a


respeito desse assunto.
I. O crescimento vegetativo da populao tender a aumentar,
na medida em que as condies de vida da populao tornamse adequadas.
II. O crescimento vegetativo tender a diminuir, na medida em
que vo melhorando as condies de educao, sade e a
expectativa de vida.
III. Os investimentos pblicos em educao continuada
precisaro aumentar para garantir trabalho qualificado s
camadas populacionais mais maduras e envelhecidas.
IV. Os programas de previdncia social e sade pblica para os
idosos tero sua demanda um pouco diminuda e aliviada,
permitindo que todos tenham uma aposentadoria garantida.

179. (Unesp 2011) Analise o mapa.

So afirmativas vlidas apenas:


a) I, III e IV.
b) II, III e IV.
c) I e IV.
d) II e III.
e) II e IV.

A partir das informaes do mapa, pode-se afirmar que a


expanso geoeconmica do territrio brasileiro, no perodo
assinalado, anos 1890, mostrou que nesse sculo
a) havia uma importante corrente migratria para o norte, o
que impulsionou o seu desenvolvimento. Os vrios focos
econmicos, embora distantes entre si, tinham o centro de
maior influncia no estado de Mato Grosso.
b) havia vrios focos econmicos distantes entre si, mas que o
centro de maior influncia econmica estava centrado na
atual regio Norte.
c) havia vrios focos econmicos interligados por malhas
virias, o que facilitava o desenvolvimento do pas.
d) o foco econmico de maior importncia era localizado na
regio Nordeste.
e) havia vrios focos econmicos distantes entre si, mas o
maior centro estava localizado na atual regio Sudeste.

RESPOSTA: D
COMENTRIO: O Brasil est em pleno processo de transio
demogrfica, deixando as faixas de altas taxas de crescimento
populacional para ndices menores, aproximando-se de
indicadores de pases desenvolvidos. O pas mostrou um
estreitamento da base da pirmide etria, com a diminuio
de jovens vem se tornando um pas de adultos e idosos com o
aumento da parte central da pirmide.
A frase I falsa, em condies adequadas de vida os ndices de
crescimento populacional tendem a diminuir.
A frase IV falsa, os programas de sade pblica para idosos
tero demanda aumentada.

181. (Uerj 2011) Deste Planalto Central, desta solido que em


breve se transformar em crebro das altas decises
nacionais, lano os olhos mais uma vez sobre o amanh do
meu pas e antevejo esta alvorada com f inquebrantvel e
uma confiana sem limites no seu grande destino.
Juscelino Kubitschek, 02/10/1956
O Globo, 21/04/2010
A realizao mais conhecida do governo de Juscelino
Kubitschek foi a construo de Braslia. No entanto, essa obra
contemplava objetivos mais abrangentes desse governante.
Dentre esses objetivos, destaca-se o de promover a integrao
nacional por meio da seguinte ao:
a) modernizao do setor tercirio
b) ampliao da infraestrutura de transportes
c) interligao das redes de telecomunicaes
d) explorao das regies Nordeste e Centro-Oeste

RESPOSTA: E
COMENTRIO: O final do sculo XIX mostra um pas mais
ocupado, mas ainda com forte concentrao populacional e
econmica na linha costeira.
A alternativa [A] falsa: para o Norte, Belm e Manaus, eram
os principais centros atrativos;
A alternativa [B] falsa: o centro mais influente estava
localizado no Sudeste;
A alternativa [C] falsa: os focos econmicos eram ligados por
rota martima ou fluvial;
A alternativa [D] falsa: o foco de maior importncia era o
Sudeste.

RESPOSTA: B
COMENTRIO: A ideia fundamental da construo de Braslia,
em termos geopolticos, foi afastar a sede de decises polticas
para longe das reas de poder econmico, alm de promover
uma maior integrao nacional a partir de uma sede de

180. (Ifsp 2011) Analise as pirmides etrias do Brasil de 2000 e a


sua projeo para 2050.
48

governo federal no centro do pas, aspecto que s seria


possvel com a ampliao das redes de transporte.
A alternativa [A] falsa: o setor tercirio viria a ser
efetivamente modernizado a partir da dcada de 1990;
A alternativa [C] falsa: as telecomunicaes iro se
modernizar a partir da dcada de 1970 com a criao da
Embratel;
A alternativa [D] falsa: Braslia serviria de base para a
expanso das regies Centro-Oeste e Norte.

182. (Uff 2011) Os versos abaixo, do compositor Assis Valente,


procuram retratar o encontro de uma dona de casa com um
recenseador do IBGE.
Recenseamento
Em 1940
L no morro comearam o
recenseamento
E o agente recenseador
esmiuou a minha vida
foi um horror

E quando viu a minha mo


sem aliana
encarou a criana
que no cho dormia
E perguntou se meu moreno
era decente
E se era do batente ou era da
folia

Os versos da cano permitem pensar em dois indicadores


demogrficos passveis de serem obtidos a partir das
informaes buscadas pelo recenseador. Esses indicadores
referem-se especificamente
a) taxa de urbanizao e esperana mdia de vida.
b) taxa de mortalidade infantil e taxa de matrimnios
estveis.
c) ao ndice de Gini e taxa de alfabetizao de adultos.
d) ao saldo migratrio e renda per capita urbana.
e) taxa de fecundidade e populao economicamente
ativa.
RESPOSTA: E
COMENTRIO: A dcada de 1930 marca um aumento
gradativo nas taxas de urbanizao do Brasil, sendo os dados
expressos no censo de 1940. A urbanizao acaba modificando
os indicadores demogrficos com resultados como: queda na
taxa de fecundidade (nmero mdio de filhos por mulher) e
no nmero e formas de atividades econmicas da populao
economicamente ativa, com diminuio progressiva de
trabalhadores rurais e aumento de trabalhadores na indstria,
no comrcio e na prestao de servios.
A alternativa [A] falsa: no h como inferir a esperana
mdia de vida.
A alternativa [B] falsa: os versos da cano tratam
basicamente do trabalho e da situao civil das pessoas.
A alternativa [C] falsa: o ndice de Gini, medidor do IDH da
populao, foi introduzido ao longo da dcada de 1990 e no
h como inferir sobre taxa de alfabetizao na pesquisa.
A alternativa [D] falsa: no h como inferir sobre saldo
migratrio ou renda per capta urbana nos versos da cano.

183.

A taxa de mortalidade infantil um dos indicadores


demogrficos que permite avaliar as condies de vida das
populaes.
Um dos principais fatores que explicam os diferentes nveis
das taxas de mortalidade infantil observados no grfico est
relacionado :
a) primazia da atividade agrcola
b) predominncia do analfabetismo
c) permanncia da concentrao de renda
d) recorrncia de problemas geoclimticos
RESPOSTA: C
COMENTRIO: A partir da dcada de 1970, com a urbanizao,
o Brasil entrou em processo de transio demogrfica.
Caractersticas de populao urbana relacionadas ao acesso
medicina, medicamentos e macias campanhas de vacinao
acabaram promovendo queda nas taxas de mortalidade
infantil em todo o pas. A diferena apontada no grfico ainda
resqucio de desequilbrios regionais em questes como o
desenvolvimento socioeconmico e a m distribuio de
renda, que no Brasil ainda esto marcantes.
A alternativa [A] falsa: ocorre primazia das atividades para o
setor urbano;
A alternativa [B] falsa: o analfabetismo compromete as taxas
de natalidade;
A alternativa [D] falsa: problemas geoclimticos, como secas
pronunciadas ou grandes enchentes, podem afetar a taxa de
mortalidade em geral.

184. (G1 - cps 2011) Entre 1880 e 1920, milhes de imigrantes


europeus e japoneses chegaram Amrica, fugindo da fome,
do desemprego e das guerras. Atualmente, as migraes
internacionais se inverteram. Elas incluem principalmente
fluxos de trabalhadores dos pases subdesenvolvidos para o
centro do capitalismo nos pases da Europa, nos Estados
Unidos e no Japo, que hoje adotam em geral polticas de
conteno da imigrao. Ao mesmo tempo, tais pases buscam
atrair trabalhadores qualificados do mundo todo, tais como
cientistas, professores, executivos e tambm esportistas
reconhecidos. Em 2007, por exemplo, mais de mil futebolistas
brasileiros emigraram do pas para jogar em clubes da Europa,
do Japo e at da Indonsia e do Vietn. O principal atrativo
so os salrios e as garantias sociais bem como a possibilidade
de subir mais rpido os degraus da glria e do prestgio.
(POMPEU, Renato. Os emigrantes da bola. Guia do
estudante. So Paulo: Abril Cultural, 2008. Adaptado)

(Uerj 2011)

Considere as afirmaes.
I. No incio do sculo XX, milhes de habitantes de pases
perifricos saram em busca de melhores salrios e de uma
vida mais digna nos EUA e no Japo.
II. Hoje, os pases desenvolvidos selecionam os trabalhadores
que lhes interessam, adotando polticas de filtragem e de

49

barreira contra a imigrao estrangeira, principalmente a de


trabalhadores desqualificados.
III. A Indonsia e o Vietn tm atrado jogadores, pois so
casos tpicos de pases asiticos em rpida industrializao e
desenvolvimento econmico.

A partir desses grficos, podemos concluir que a diferena


verificada na expectativa de vida entre os gneros, na segunda
metade do sculo XX,
a) foi uma caracterstica dos pases mais industrializados,
como a Frana.
b) diminuiu quando os pases se industrializaram, uma vez que
as mulheres passaram a ter mais direitos e oportunidades.
c) ocorreu apenas em pases com altas taxas de criminalidade
entre jovens adultos do sexo masculino, como o Brasil.
d) aumentou quando a expectativa de vida alcanou nveis
mais altos.

vlido o que se afirma em


a) I, apenas.
b) III, apenas.
c) I e II, apenas.
d) II e III, apenas.
e) I, II e III.
RESPOSTA: D
COMENTRIO: O Brasil, apesar de seu atual desenvolvimento
ainda possui demandas sociais reprimidas como educao de
qualidade e sade mais universalizada e de melhor
atendimento. A tradicional concentrao de renda e demais
recursos produo dificultam o acesso da populao a bens
materiais e de produo, colocando o pas num crculo vicioso
de emprego informal que no recolhe impostos, que
dificultam os investimentos sociais deixando a mo de obra
pouco qualificada. Nesse universo, a prtica profissional do
futebol capaz de inserir parte dos jovens num mercado
competitivo e possvel de gerar ganhos elevados. Porm, os
grandes clubes do Brasil sofrem com ms administraes e
acabam sendo atrados por propostas milionrias de clubes
estrangeiros, principalmente europeus, que melhor
organizados e localizados em pases de condies
socioeconmicas mais avanadas, levam vantagens, podendo
pagar por um jogador um valor difcil de atingir por clubes
brasileiros.
A frase I falsa, as grandes migraes em busca de melhores
condies de vida ocorreram principalmente a partir dos anos
1970.

RESPOSTA: D
COMENTRIO: Os grficos mostram as diferentes expectativas
de vida onde podemos comparar o Brasil e Frana e
verificarmos as diferenas em qualidade social a partir da
expectativa de vida. Outro aspecto que pode ser notado a
evoluo da expectativa de vida em funo de melhorias
mdico-bioqumicas e acessibilidade a servios de sade.
A alternativa [A] falsa, a melhor expectativa de vida atingiu
tambm o Brasil.
A alternativa [B] falsa, aumenta com a industrializao, pois
h aumento da massa salarial e maiores investimentos em
sade.
A alternativa [C] falsa, no h subsdios que permitam inferir
taxas de criminalidade no contexto.
186. (Ufpb 2011) A insero da mulher no mercado de trabalho
um fenmeno mundial, sendo que a tendncia que essa
participao aumente cada vez mais. Essa realidade permite
garantir e consolidar a independncia da condio feminina
junto ao conjunto total da sociedade. No Brasil, observa-se
que, de forma geral, essa dinmica se repete. No entanto,
verifica-se que a participao da mulher no mercado de
trabalho nacional e desigual, quando comparada s diferentes
unidades da federao, conforme mapa a seguir.

185. (Unicamp simulado 2011) Os grficos abaixo apresentam as


expectativas de vida de homens e de mulheres nascidos nos
anos de 1920 a 2000 no Brasil e de 1830 a 1990, na Frana.

Com base no exposto e na literatura sobre o tema, correto


afirmar:
a) So Paulo o mais rico, industrializado, e mais importante
estado da Federao, o que lhe favorece apresentar as mais
baixas taxas de participao feminina no mercado de trabalho.
b) O Amap apresenta elevada participao feminina no
mercado de trabalho, por possuir um territrio muito
populoso e bastante urbanizado.
c) O Rio de Janeiro apresenta altas taxas de industrializao e
de urbanizao, o que determina baixa participao de
mulheres em seu mercado de trabalho.
d) O Rio Grande do Sul apresenta alto ndice de participao
de mulheres no conjunto do mercado de trabalho, pelo fato
de esse estado ser o mais industrializado e urbanizado do
Brasil.
50

e) O Distrito Federal apresenta territrio de dimenso


limitada, porm denso e fortemente urbanizado, onde a
elevada taxa de mo de obra feminina, no conjunto da
populao economicamente ativa, absorvida especialmente
no ramo dos servios.
RESPOSTA: E
COMENTRIO: A participao feminina nas diversas
possibilidades econmicas e sociais recente no mundo. No
Brasil a participao feminina segue no tempo e no espao de
acordo com os estgios de desenvolvimento. Desta maneira
normal observarmos maior concentrao de trabalho feminino
onde as condies socioeconmicas so mais desenvolvidas.
So alguns estados do Sul, Sudeste, Nordeste e no Distrito
Federal.
A alternativa [A] falsa, So Paulo, pelo seu desenvolvimento
favorece altas taxas de participao feminina no mercado de
trabalho.
A alternativa [B] falsa, o Amap tem baixa participao
feminina no trabalho, pouco populoso e relativamente
pouco urbanizado.
A alternativa [C] falsa, o Rio de Janeiro tem alta taxa de
participao feminina no mercado de trabalho.
A alternativa [D] falsa, o Rio Grande do Sul no o estado
mais industrializado do Brasil ( So Paulo), nem o mais
urbanizado ( o Rio de Janeiro).

RESPOSTA: B
COMENTRI: Os trs critrios principais para se medir o IDH
so: renda per capita, grau de escolaridade e expectativa de
vida.
Segundo o mapa, as reas de que compreendem os melhores
ndices esto em torno do eixo RJ-SP e tambm no eixo
Anhanguera at Ribeiro Preto.
interessante expor ainda que a afirmativa contida na
alternativa [D] est correta, contudo o que foi solicitado na
questo os explicativos da territorializao do IDH sudestino,
o que acaba por validar a alternativa [B] como a assertiva a ser
assinalada.
188. (Ufpr 2011) A tabela a seguir apresenta os dados de migrao
no Brasil entre os anos de 2003 e 2008.

(Fonte:
http://www.ipea.gov.br/portal/images/stories/PDFs/100817_
grafscomuniipea61.pdf)

187. (Fgv 2011) O IDH (ndice de Desenvolvimento Humano) foi


criado para indicar as condies gerais de vida das populaes
nas mais diversas regies. Observe o IDH da Regio Sudeste e
assinale a alternativa que melhor explique a territorializao
deste indicador.

Com base nessas informaes, assinale a alternativa correta.


a) A diferena existente entre o nmero de imigrantes e
emigrantes no Sudeste caracteriza essa regio como de baixa
mobilidade populacional.
b) Uma das caractersticas da dinmica apresentada na tabela
que a maioria das regies apresenta maior ndice de
migrantes dentro da prpria regio.
c) O maior deslocamento de pessoas ocorre das regies com
maior densidade demogrfica em direo quelas de menor
densidade.
d) Os dados mostram que o Nordeste, regio que
tradicionalmente deslocava elevado nmero de migrantes,
sobretudo para o Sudeste, agora apresenta o fenmeno
inverso, ou seja, o nmero de migrantes favorvel quela
regio.
e) Regies ainda consideradas como fronteira agrcola tm
como caracterstica atrair migrantes cujas atividades esto
associadas agricultura, haja vista a disponibilidade de terras
ainda existentes.
RESPOSTA: B
COMENTRIO: A partir dos anos 1990, a economia brasileira
tornou-se mais dinmica e urbana, modificando a utopia dos
cidados em relao a migraes internas em busca de
melhores condies de vida. Praticamente inexistem grandes
fluxos inter-regionais substitudos por fluxos locais. Os
empregos agora podem ser criados em todas as regies.

a) Observa-se que o IDH melhor nas reas rurais, como o


interior de Minas Gerais, onde a ausncia de poluio
contribui para a qualidade de vida.
b) Nota-se que o IDH determinado pela combinao de
desenvolvimento industrial e alto poder aquisitivo,
caractersticas das regies do Sudeste, em que este ndice
mais elevado.
c) Pode-se concluir que o IDH sempre maior nas reas mais
populosas, j que nas reas mais despovoadas, como o Vale
do Jequitinhonha (MG) e Vale do Ribeira (SP), no h muitos
indicadores a serem medidos.
d) O IDH resulta da combinao de fatores como renda,
expectativa de vida e educao e, quanto mais prximo a 1,
melhor.
e) O alto IDH observado no interior paulista e Tringulo
Mineiro explica-se pela modernizao da agropecuria e do
sistema virio que agiliza o fluxo de pessoas, informaes e
mercadorias.

A alternativa [A] falsa, a regio tem alta mobilidade


populacional.
A alternativa [C] falsa, os maiores fluxos so das reas de
menor densidade para as reas de maior densidade.
A alternativa [D] falsa, apesar de ter diminudo o fluxo de
nordestinos para o Sudeste, ainda maior do que a taxa de
retorno.
A alternativa [E] falsa, as fronteiras agrcolas no sinais de
esgotamento.

51

FILOSOFIA
189. Uma moral racional se posiciona criticamente em relao a
todas as orientaes da ao, sejam elas naturais, autoevidentes, institucionalizadas ou ancoradas em motivos
atravs de padres de socializao. No momento em que uma
alternativa de ao e seu pano de fundo normativo so
expostos ao olhar crtico dessa moral, entra em cena a
problematizao. A moral da razo especializada em
questes de justia e aborda em princpio tudo luz forte e
restrita da universalidade. (HABERMAS, Jrgen. Direito e
democracia: entre facticidade e validade. v. I. Trad. Flvio
Beno Siebeneichler. Rio de Janeiro: Tempo Brasileiro, 1997. p.
149.)
Com base no texto e nos conhecimentos sobre a moral em
Habermas, correto afirmar:
A) A formao racional de normas de ao ocorre
independentemente da efetivao de discursos e da
autonomia pblica.
B) O discurso moral se estende a todas as normas de aes
passveis de serem justificadas sob o ponto
de vista da razo.
C) A validade universal das normas pauta-se no contedo dos
valores, costumes e tradies praticados no interior das
comunidades locais.
D) A positivao da lei contida nos cdigos, mesmo sem o
consentimento da participao popular, garante a soluo
moral de conflitos de ao.
E) Os parmetros de justia para a avaliao crtica de normas
pautam-se no princpio do direito divino.
RESPOSTA: B
COMENTRIO: Jrgen Habermas prope um novo conceito de
razo, a razo comunicativa, como forma de retornar o
projeto emancipatria da humanidade em novas bases. E sua
validade ser tanto maior quanto melhores forem as
condies nas quais se d o dilogo, o que se consegue com o
aperfeioamento moral da democracia.
190. Leia o texto a seguir e responda questo.
Texto
A proposta tica de Habermas no comporta contedos. Ela
formal. Ela apresenta um procedimento, fundamentado na
racionalidade comunicativa, de resoluo de pretenses
normativas de validade.
(DUTRA, D. J. V. Razo e consenso em Habermas. A teoria
discursiva da verdade, da moral, do direito e da biotecnologia.
Florianpolis: Editora da UFSC, p. 158.)
Com base no texto e nos conhecimentos sobre a obra de
Habermas, correto afirmar que, na tica do Discurso,
A) o processo de justificao das normas morais e o
procedimento de deliberao das pretenses de validade de
correo normativa so falveis.
B) o formalismo da tica habermasiana idntico ao
formalismo presente nas ticas de Kant e Benthan, pois
desconsidera o que resulta concretamente das normas morais.
C) o modelo monolgico da tica kantiana reformulado na
perspectiva de uma comunidade discursiva na qual os
participantes analisam as pretenses de validade tendo como
critrio a fora do melhor argumento.
D) o puro respeito lei considerado por Habermas como o
critrio fundamental para conferi moralidade ao, restando
excludos do debate da tica discursiva os desejos e as
necessidades manifestados pelos indivduos.
E) o princpio U possibilita que sejam acatadas normas que
no estejam sintonizadas com uma vontade universal,
coadunando, dessa forma, particularismo e universalismo
tico.

RESPOSTA: C
COMENTRIO: Habermas, portanto, prope uma nova
perspectiva sobre a razo: a razo dilogo, que brota do
dilogo e da argumentao entre os agentes interessados
numa determinada situao. a razo que surge da chamada
ao comunicativa, do uso da linguagem como meio de
conseguir o consenso. Para tanto, necessrio uma ao
social que fortalea as estruturas capazes de promover as
condies de liberdade e de no constrangimento
imprescindveis ao dilogo.
Leia o texto a seguir e responda s questes 191 e 192.
Texto
A ao poltica pressupe a possibilidade de decidir, atravs da
palavra, sobre o bem comum. Esta acepo do termo
poltica, somente vlida enquanto ideal aceito, guarda uma
estreita relao com a concepo de poltica defendida por
Habermas. Em particular, com o modelo normativo de
democracia que este desenvolveu no incio dos anos de 1990 e
que inclui um procedimento ideal de deliberao e tomada de
decises: a chamada poltica deliberativa.
(VALASCO ARROYO, J. C. Para leer a Habermas. Madrid:
Alianza, 2003, p.93.)
191. Com base no texto e nos conhecimentos sobre a democracia
no pensamento de Habermas, considere as afirmativas a
seguir.
I Aa normas se tornam legtimas pelo fato de terem sido
submetidas ao crivo participativo de todos os concernidos.
II O princpio da regra da maioria est subordinado
possibilidade prvia de que todos os concernidos tenham tido
a oportunidade de apresentar seus posicionamentos de forma
argumentativa e sem coero.
III A deliberao visa formalizar posies cristalizadas pelos
membros da sociedade poltica, limitando-se ao endosso das
opinies prvias de cada um.
IV As prticas polticas democrticas restringem-se escolha,
mediante sufrgio universal, dos lderes que governam as
cidades.
Assinale a alternativa correta.
A) somente as afirmativas I e II so corretas.
B) somente as afirmativas II e IV so corretas.
C) somente as afirmativas III e IV so corretas.
D) somente as afirmativas I, II e III so corretas.
E) somente as afirmativas I, III e IV so corretas.
RESPOSTA: A
COMENTRIO: Razo e verdade deixam de constituir
contedos ou valores absolutos e passam a ser definidos
consensualmente. A nfase dada por Habermas razo
comunicativa pode ser entendida como uma maneira de
tentar salvar a razo, que teria chegado a um beco sem
sada. Assim, se o mundo contemporneo regido pela razo
instrumental, conforme denunciaram os filsofos que o
antecederam na Escola de Frankfurt, para Habermas caberia
razo comunicativa, enfim, o papel de resistir a e reorientar
essa razo instrumental para o aperfeioamento da
democracia.
192. Sobre o pensamento de Habermas, correto afirmar que, no
modelo da democracia deliberativa, a noo de cidadania
enfatiza
A) os direitos e as liberdades metafsicas.
B) as liberdades individuais e a heteronomia.
C) os direitos objetivos e o cerceamento da sociedade civil.
D) os direitos subjetivos e as liberdades cidads.
E) os direitos naturais originrios e a submisso autoridade.

52

RESPOSTA: D
COMENTRIO: Verdade intersubjetiva. Habermas prope o
entendimento da verdade no mais como uma adequao do
pensamento realidade, mas como fruto da ao
comunicativa; no como verdade subjetiva, mas como verdade
intersubjetiva (entre sujeitos diversos), que surge do dilogo
entre indivduos/cidados. Nesse dilogo aplicam-se algumas
regras, como a no contradio, a clareza de argumentao e
a falta de constrangimento de ordem social.
Leia o texto seguinte e responda s questes 193 e 194.
Texto
O debate nascido nos anos 80 sobre a crise da modernidade
tem como pano de fundo a conscincia do esgotamento da
razo, no que se refere a sua incapacidade de encontrar
perspectivas para o prometido progresso humano. O
pensamento de Habermas situa-se no contexto dessa crtica. A
racionalidade ocidental, desde Descartes, pretendeu a
autonomia da razo, baseada no sujeito que solitariamente
representa o mundo. [...] A racionalidade prevalente na
modernidade a instrumental [...].
(HARMANN, N. O pensamento de Habermas. In: Filosofia,
Sociedade e Educao. Ano I, n. I. Marlia: UNESP, 1997. P.
122-123.)
193. Com base no texto e nos conhecimentos sobre a Teoria Crtica
de Adorno e Horkheimer e sobre o pensamento de Jrgen
Habermas, correto afirmar que a racionalidade instrumental
constitui
I um conhecimento que se processa a partir das condies
especficas da objetividade emprica do fato em si.
II o processo de entendimento entre os sujeitos acerca do
uso racional dos instrumentos tcnicos para o controle da
natureza.
III uma forma de uso amplo da razo, que torna o homem
livre para compreender a si mesmo a partir do domnio do
conhecimento cientfico.
IV um saber orientado para a dominao e o controle
tcnico sobre a natureza e sobre o prprio ser humano.
Assinale a alternativa correta.
A) somente as afirmativas I e II so corretas.
B) somente as afirmativas I e IV so corretas.
C) somente as afirmativas III e IV so corretas.
D) somente as afirmativas I, II e III so corretas.
E) somente as afirmativas II, II e IV so corretas.
RESPOSTA: B
COMENTRIO: De acordo com Max Horkheimer e Theodor
Adorno, a razo iluminista, que visava a emancipao dos
indivduos e o progresso social, terminou por levar a uma
crescente dominao das pessoas, em virtude justamente do
desenvolvimento
tecnolgico-industrial.
Horkheimer
acreditava que o problema estava na prpria razo
controladora e instrumental, que busca sempre a dominao,
tanto da natureza quanto do prprio ser humano.
194. Sobre a crtica frankfurtiana concepo positivista da cincia
e tcnica, correto afirmar que a racionalidade tcnica
I dissocia meios e fins e redunda na adorao fetichista de
seus prprios meios.
II constitui um saber instrumental cujo critrio de verdade
o seu valor operativo na dominao do homem e da natureza.
III aprimora a ao do ser humano sobre a natureza e
resgata o sentido da destinao humana.
V - incorpora a reflexo sobre o significado e sobre os fins da
cincia no contexto social.
Assinale a alternativa correta.
A) I e II.
B) I e IV.

C) III e IV.
D) I, II e III.
E) II, III e IV.
RESPOSTA: A
COMENTRIO: A Escola de Frankfurt concentrou seu interesse
na anlise da sociedade de massa, termo que busca
caracterizar a sociedade atual, na qual o avano tecnolgico
colocado a servio da reproduo da lgica capitalista,
enfatizando o consumo e a diverso como formas de garantir
o apaziguamento e a diluio dos problemas sociais. Portanto,
a razo como saber instrumental tem em si o critrio de
verdade o seu valor operativo na dominao do homem e da
natureza.
195. Com base no pensamento esttico de Adorno e Benjamin,
considere as afirmativas a seguir.
I Apesar de terem o mesmo ponto de partida, a saber, a
anlise crtica das tcnicas de reproduo, Adorno e Benjamin
chegam a concluses distintas. Adorno entende que a
reprodutibiliidade das obras de arte algo negativo, pois
transforma esta ltima em mercadoria; para Benjamin, apesar
de a reprodutibilidade ter aspectos negativos, uma forma de
arte como o cinema pode ser usado potencialmente em favor
da classe operria.
II Para Adorno, o discurso revolucionrio na arte torna esta
forma de expresso humana instrumentalista, e isto significa
abolir a prpria arte. Por seu turno, Benjamin considerava que
os novos meios de comunicao no deveriam ser
substitudos, mas sim transformados ou subvertidos segundo
os interesses da comunicao burguesa.
III Para Adorno, a noo de aura na obra de arte preservara a
conscincia de que a realidade poderia ser melhor, mas o
processo de massificao da arte dissolveu tal noo e, com
ela, a dimenso crtica, da arte. Para Benjamin, a perda da
aura destruiu a unicidade e a singularidade da obra de arte,
que perde o seu valor de culto e se torna acessvel.
IV Adorno v positivamente a reprodutibilidade da arte, j
que a obra de arte se transforma em mercadoria padronizada
que possibilita a todos o acesso e o desenvolvimento do gosto
esttico autnomo; para Benjamin, a reproduo tem como
dimenso negativa essencial o fato de impossibilitar s massas
o acesso s obras.
Assinale a alternativa correta.
A) Somente as afirmativas I e II so corretas.
B) Somente as afirmativas I e III so corretas.
C) Somente as afirmativas II e IV so corretas.
D) Somente as afirmativas I, III e IV so corretas.
E) Somente as afirmativas II, III e IV so corretas.
RESPOSTA: B
COMENTRIO: Enquanto, na viso de Adorno e Horkheimer, a
cultura veiculada pelos meios de comunicao de massa no
permite que as classes assalariadas assumam uma posio
crtica em relao realidade, Benjamin acredita que a arte
dirigida s massas pode servir como instrumento de
politizao.
196. [...] no encontramos, j prontos, valores ou ordens que
possam legitimar a nossa conduta. Assim, no teremos nem
atrs de ns, nem na nossa frente, no reino luminoso dos
valores, nenhuma justificativa e nenhuma desculpa. Estamos
ss, se desculpas. o que posso expressar dizendo que o
homem est condenado a ser livre. Condenado, porque no se
criou a si mesmo, e como, no entanto, livre, uma vez que foi
lanado no mundo, responsvel por tudo o que faz.
(SARTRE, Jean-Paul. O existencialismo um humanismo. 3 ed.
So Paulo: Nova Cultura, 1987, p.9.)
53

Tomando o texto acima como referncia, assinale a alternativa


correta.
A) Sartre afirma que o homem est condenado a ser livre e
que, por esta razo, deve ser responsvel por tudo o que
acontece ao seu redor.
B) Sartre considera que o homem no responsvel por seus
atos, porque no se criou a si mesmo, sendo, por esta razo,
totalmente livre.
C) Ao dizer que [...] no encontramos, j prontos, valores ou
ordens que possam legitimar a nossa conduta, Sartre defende
que o existencialismo no admite qualquer valor, nem a
liberdade.
D) O existencialismo de Sartre defende a tese da absoluta
responsabilidade do homem em relao aos atos que pratica,
porque sua moral parte do princpio de uma liberdade
coerente e comprometida com o bem comum.
E) Sartre afirma que o homem um ser condenado ao pecado
e ao erro.

RESPOSTA: A
COMENTRIO: Niilismo = ciclo incompleto. Quando o homem
nega
199. Os pensadores da Escola de Frankfurt, especialmente
Theodor Adorno e Max Horkheimer, so crticos da
mentalidade que identifica o progresso tcnico-cientfico
com o progresso da humanidade. Para eles, a ideologia da
indstria cultural submete as artes servido das regras do
mercado capitalista.
Com base nos conhecimentos sobre as crticas de Adorno e
Horkheimer Indstria Cultural, assinale a afirmativa
correta:
A) A indstria cultural proporcionou a democratizao das
artes eruditas, tornando as obras raras e caras acessveis
maioria das pessoas.
B) Sob os efeitos da massificao pela indstria e consumo
culturais, as artes tendem a ganhar fora simblica e
expressividade.
C) A indstria cultural fomentou os aspectos crticos,
inovadores e polmicos das artes.
D) O progresso tcnico-cientfico pode ser entendido como um
meio que a indstria cultural usa para formar indivduos
crticos.
E) A expresso indstria cultural indica uma cultura baseada
na ideia e na prtica do consumo de produtos culturais
fabricados em srie.

RESPOSTA: D
COMENTRIO: A teoria existencialista, a qual Sartre faz parte,
destaca a liberdade individual, a responsabilidade e a
subjetividade do ser humano. Considerando cada homem com
um ser nico que mestre dos seus atos e do seu destino.
197. Texto
O que proveitoso constitui o valor. O homem o criador de
valores, mas se esquece de sua criao. A moralidade o
instinto gregrio do indivduo. Quem punido quem pratica
os atos. Na sociedade, existem os instintos de rebanho.
Atribuem-se s palavras um sentido fixo e acha-se que ela
espelha a realidade, que tem carter transitrio. O homem
chega, pelos costumes, convico de que preciso
obedecer. No inverso disso, existe o prazer, a
autodeterminao e a liberdade de vontade.
Nietzsche.
A partir do fragmento (texto) acima responda as questes a
seguir.
Segundo Nietzsche, o esprito de rebanho uma crtica (aos):
A) valores ticos de Scrates.
B) tica apolnea.
C) tica crist.
D) tica socialista.
E) valores alemes.

RESPOSTA: E
COMENTRIO: bastante conhecida a expresso Indstria
Cultural formulada por Adorno e Horkheimer e sua denncia
de que, nos tempos do capitalismo contemporneo, a
produo cultural, muitas vezes, submete-se s leis de
mercado. Tal situao provoca a perda da expressividade
transformando a arte apenas em mercadoria de consumo
produzida em srie.
200.

Leia o texto de Adorno a seguir.


Se as duas esferas da msica se movem na unidade da sua
contradio recproca, a linha de demarcao que as separa
varivel. A produo musical avanada se independentizou do
consumo. O resto da msica sria submetido lei do
consumo, pelo preo de seu contedo. Ouve-se tal msica
sria como se consome uma mercadoria adquirida no
mercado. Carecem totalmente de significado real as distines
entre a audio da msica clssica oficial e da msica ligeira.

RESPOSTA: C
COMENTRIO: O niilismo moderno apontado por Nietzsche
assentava-se, entre outras coisas, na afirmao da morte de
Deus, que interpretada como a rejeio crena num ser
absoluto e transcendental, capaz de traar para todos os
humanos o caminho, a verdade e a vida. Portanto, niilismo
seria o sentimento coletivo de que nossos sistemas
tradicionais de valorao, tanto no plano do conhecimento
quanto no tico-religioso, ou sociopoltico, ficaram sem
consistncia e j no podem mais atuar como instncias
doadoras de sentido e fundamento para o conhecimento e a
ao.

(ADORNO, T. W. O fetichismo na msica e a regresso da


audio. In: BENJAMIN, W. et all. Textos escolhidos. 2. ed. So
Paulo: Abril Cultural, 1987. p. 84.)
Com base no texto e nos conhecimentos sobre o pensamento
de Adorno, correto afirmar:
A) A msica sria e a msica ligeira so essencialmente crticas
sociedade de consumo e indstria cultural.
B) Ao se tornarem autnomas e independentes do consumo, a
msica sria e a msica ligeira passam a realar o seu valor de
uso em detrimento do valor de troca.
C) A indstria cultural acabou preparando a sua prpria
autorreflexividade ao transformar a msica ligeira e a sria em
mercadorias.
D) Tanto a msica sria quanto a ligeira foram transformadas
em mercadoria com o avano da produo industrial.
E) As esferas da msica sria e da ligeira so separadas e nada
possuem em comum.

198. A ideia de homem presente no fragmento :


A) um super-homem, determinado, alm do bem e do mal,
com uma vontade de poder.
B) um super-homem, obediente aos instintos e aos valores
morais.
C) um super-homem, livre de regras e de valores, um
verdadeiro cristo.
D) um super-homem, fiel aos instintos.
E) um super-homem, livre e feliz por seus valores culturais.

54

RESPOSTA: D
COMENTRIO: A progressiva fragmentao e padronizao da
msica, bem como a sua transformao em mercadoria que,
uma vez associada aos grandes veculos de comunicao de
massas, imprime em seus ouvintes, contedos estticos
formais pr-fabricados, os quais caracterizam as relaes
entre arte e sociedade em tempos de tecnicizao da
produo artstica e decadncia da apreciao esttica. Uma
vez enquadrados pela cultura de massa, os objetos de arte
estaro perpetuamente condenados cega obedincia aos
ditames de uma sistematizao que, racionalmente, se
organiza em funo do poder do capital que, de forma
subjacente e oculta, orienta o que se pode e se deve produzir,
divulgar e vender. Entram em cena os veculos de
comunicao de massa que, ostensivamente circulam imagens
e sons; que sugerem adeso a determinados valores,
comportamentos e atitudes.

como valor fundamental em detrimento de uma reflexo


sobre os fins. O questionamento principal que os autores
fazem como um suposto esclarecimento e a ideia de
progresso nos conduziu a um mundo, ainda hoje, repleto de
injustias e desigualdades, no qual prevalece a dominao de
uns sobre outros.

202. Observe a tira e leia o texto a seguir:

201. Leia o texto a seguir:


A ideia de progresso manifesta-se inicialmente, poca do
Renascimento, como conscincia de ruptura. [...] No sculo
XVIII tal ideia associa-se conscincia do carter progressivo
da civilizao, e assim que a encontramos em Voltaire. Tal
como para Bacon, no incio do sculo XVII, o progresso
tambm uma espcie de objeto de f para os iluministas. [...]
A certeza do progresso permite encarar o futuro com
otimismo.
(Adaptado de: FALCON, F. J. C. Iluminismo. 2. ed. So Paulo:
tica, 1989, p. 61-62.)
Quando se concebeu a ideia de razo, o que se pretendia
alcanar era mais que a simples regulao da relao entre
meios e fins: pensava-se nela como o instrumento para
compreender os fins, para determin-los.
Segundo a filosofia do intelectual mdio moderno, s existe
uma autoridade, a saber, a cincia, concebida como
classificao de fatos e clculo de probabilidades.

Na primeira metade do sculo XX, a ideia de progresso


tambm se transformou em objeto de anlise do grupo de
pesquisadores do Instituto de Pesquisa Social vinculado
Universidade de Frankfurt.
Tendo como referncia a obra de Adorno e Horkheimer,
correto afirmar:
A) Por serem herdeiros do pensamento hegeliano, os autores
entendem que a superao do modelo de racionalidade
inerente aos conflitos do sculo XX depende do justo
equilbrio entre uso pblico e uso privado da razo.
B) A despeito da Segunda Guerra, a finalidade do iluminismo
de libertar os homens do medo, da magia e do mito e tornlos senhores autnomos e livres mediante o uso da cincia e
da tcnica, foi atingido.
C) Os autores propem como alternativa s catstrofes da
primeira metade do sculo XX um novo entendimento da
noo de progresso tendo como referncia o conceito de
racionalidade comunicativa.
D) Como demonstra a anlise feita pelos autores no texto O
autor como produtor, o ideal de progresso consolidado ao
longo da modernidade foi rompido com as guerras do sculo
XX.
E) Em obras como a Dialtica do esclarecimento, os autores
questionam a compreenso da noo de progresso
consolidada ao longo da trajetria da razo por ela estar
vinculada a um modelo de racionalidade de cunho
instrumental.

(HORKHEIMER, M. Eclipse da Razo. So Paulo: Labor, 1973,


pp.18 e 31-32.)
Com base na tira, no texto e nos conhecimentos sobre o
pensamento de Horkheimer a respeito da relao entre
cincia e razo na modernidade, correto afirmar:
I. Se a razo no reflete sobre os fins, torna-se impossvel
afirmar se um sistema poltico ou econmico, mesmo no
sendo democrtico, mais ou menos racional do que outro.
II. O processo que resulta na transformao de todos os
produtos da ao humana em mercadorias se origina nos
primrdios da sociedade organizada medida que os
instrumentos passam a ser utilizados tecnicamente.
III. A razo subjetivada e formalizada transforma as obras de
arte em mercadorias, das quais resultam emoes eventuais,
desvinculadas das reais expectativas dos indivduos.
IV. As atividades em geral, independentes da utilidade,
constituem formas de construo da existncia humana
desvinculadas de questes como produtividade e
rentabilidade.

RESPOSTA: E
COMENTRIO: A alternativa E est correta, pois Adorno e
Horkheimer fazem uma crtica ao esclarecimento, isto , uma
crtica ao pensamento racional-cientfico tal qual conhecemos
desde o Iluminismo. A crtica se fundamenta sobretudo no
carter instrumental dessa racionalidade ocidental, que
transformou os meios, ou seja, a tcnica (e seu domnio),

Assinale a alternativa correta.


A) Somente as afirmativas I e II so corretas.
B) Somente as afirmativas I e III so corretas.
C) Somente as afirmativas III e IV so corretas.
D) Somente as afirmativas I, II e IV so corretas.
E) Somente as afirmativas II, III e IV so corretas.

55

RESPOSTA: B
COMENTRIO: Cr Horkheimer que a razo desembaraa-se
da reflexo sobre os fins e torna-se incapaz de dizer que um
sistema poltico ou econmico irracional. Por mais que
parea que o filsofo seja cruel e desptico, contanto que
funcione, a razo o aceita e no deixa ao homem outra
escolha a no ser a resignao. A teoria justa, ao contrrio,
escreve Horkheimer, "nasce da considerao dos homens de
tempos em tempos, vivendo sob condies determinadas e que
conservam sua prpria vida com a ajuda dos instrumentos de
trabalho". Ao considerar que a existncia social age como
determinante da conscincia, a teoria crtica no est
anunciando sua viso do mundo, mas diagnosticando uma
situao que deveria ser superada e isto que evidencia a tira.

BIOLOGIA I
204. Considere a tabela abaixo, com informaes sobre o sistema
circulatrio de vertebrados:

I.
II.
III.

IV.

203. Leia o seguinte texto de Adorno e Horkheimer:

V.

O esclarecimento, porm, reconheceu as antigas potncias no


legado platnico e aristotlico da metafsica e instaurou um
processo contra a pretenso de verdade dos universais,
acusando-a de superstio. Na autoridade dos conceitos
universais ele cr enxergar ainda o medo pelos demnios,
cujas imagens eram o meio, de que se serviam os homens, no
ritual mgico, para tentar influenciar a natureza. Doravante, a
matria deve ser dominada sem o recurso ilusrio a foras
soberanas ou imanentes, sem a iluso de qualidades ocultas.
O que no se submete ao critrio da calculabilidade e da
utilidade torna-se suspeito para o esclarecimento.

CIRCULAO
Dupla e completa, com aorta
curvada para a direita.
Simples e completa.

Dois
trios
e
dois
ventrculos, aorta com
Formen de Panizza.
Dois
trios
e
dois
ventrculos.
Dois
trios
e
um
ventrculo.

Dupla e incompleta.

Dupla e completa, com aorta


curvada para a esquerda.
Dupla e incompleta.

Assinale a alternativa com a sequncia correta de animais a


que correspondem as caractersticas indicadas de I a IV.
a) I - bem-te-vi; II - truta; III - crocodilo; IV - homem; V - r;
b) I - foca; II - sardinha; III - jacar; IV - pato; V - sapo.
c) I - sabi; II - salmo; III - r; IV - boi; V - jabuti.
d) I - pardal; II - baleia; III - tartaruga; IV - ona; V - girino.
e) I - gato. II - atum; III - cascavel; IV - quero-quero; V - enguia.
RESPOSTA: A
COMENTRIO: A diferena entre a circulao das aves em
relao aos mamferos o sentido da aorta. Nas aves a artria
aorta aps sair do ventrculo esquerdo do corao curva-se
para a direita e nos mamferos, curva-se para o lado esquerdo.
Dois trios e dois ventrculos com circulao dupla e completa,
com aorta curvada para a direita caracterizam o corao das
aves (bem-te-vi). Um trio e um ventrculo com circulao
fechada e simples caracterizam o corao dos peixes (truta). O
corao dos rpteis possui trs ou quatro cavidades, com
circulao fechada e incompleta. Sendo que o Formen de
Panizza permite a mistura do sangue venoso com o arterial no
corao dos crocodilianos (crocodilo). Dois trios e dois
ventrculos com circulao dupla e completa, com aorta
curvada para a esquerda caracterizam o corao dos
mamferos (homem). Corao com trs cavidades e circulao
fechada, dupla e incompleta tpico dos anfbios (r).

(ADORNO, T.; HORKHEIMER, M. Dialtica do Esclarecimento.


Fragmentos filosficos. Traduo de Guido Antonio de
Almeida. Rio de Janeiro: Jorge Zahar, 1985, p. 21.)
Com base no texto e no conceito de esclarecimento de Adorno
e Horkheimer, correto afirmar:
A) O esclarecimento representa, em oposio ao modelo
matemtico, a base do conhecimento tcnico-cientfico que
sustenta o modo de produo capitalista na viabilizao da
emancipao social.
B) O esclarecimento demonstra o domnio substancial da
razo sobre a natureza interna e externa e a realizao da
emancipao social levada adiante pelo capitalismo.
C) O esclarecimento compreende a realizao romntica da
racionalidade que acentuou, de forma intensa, a interao
harmnica entre homem e natureza.
D) O esclarecimento abrange a racionalizao das diversas
formas e condies da vida humana com o objetivo de tornar
o ser humano mais feliz, quando da realizao de prticas
rituais e religiosas.
E) O esclarecimento concebe o abandono gradual dos
pressupostos metafsicos e a operacionalizao do
conhecimento por meio da calculabilidade e da utilidade,
redundando num modelo prprio de razo instrumental.

CORAO
Dois
trios
e
dois
ventrculos.
Um trio e um ventrculo.

205. (Ueg 2012) A figura abaixo representa a dinmica das presses


que atuam nas trocas capilares. Os capilares arteriais tendem
a levar gua do plasma aos tecidos, e os capilares venosos
tendem a reabsorver lquidos dos tecidos. As presses
onctica (PO) e hidrosttica (PH) atuam contrariamente,
sendo a presso onctica atribuda s protenas plasmticas.

RESPOSTA: E
COMENTRIO: Na Dialtica, os autores ao invs de traar uma
sistematizao geral, em seu desenvolvimento atravs de eras
e perodos, buscam em alguns pontos-chave da histria os
elementos necessrios para criar sua teoria dialtica sobre o
esclarecimento. A anlise dos autores do esclarecimento se faz
por meio de um olhar detalhado em obras literrias que
funcionam como smbolos de diferentes instantes. Em seu
primeiro excurso analisam a epopeia homrica, o surgimento
do processo de luta do esclarecimento contra a mitologia;
enquanto que no segundo excurso, o fazem com as crnicas
escandalosas, as obras dos escritores sombrios da
burguesia, como Sade e Nietzsche.

A respeito da manuteno metablica exercida pelo fluxo


entre os capilares e os tecidos, correto afirmar:
a) no glomrulo renal, o aumento da PO ocorre devido
grande rede de filtrao que retm protenas plasmticas.
56

b) a diminuio da PH na vnula ocorre por causa da menor


concentrao de protenas plasmticas e consequente
aumento da PO.
c) a reduzida concentrao de protenas plasmticas, por
deficincia nutricional, ocasiona o aumento da PH com
consequente formao de edema.
d) a fora exercida por PH e PO determina o fluxo de O 2, CO2 e
de glicose, de maneira dependente, principalmente, da
concentrao plasmtica de albumina.

Existem algumas causas que levam a esta obstruo, sendo a


principal delas a aterosclerose - acmulo de gordura na
parede das artrias, formando verdadeiras placas, as quais
podem vir a obstruir o vaso e impedir o fluxo de sangue a
partir daquele local. Essa obstruo normalmente ocorre
quando a placa se rompe e a ela agregam-se plaquetas,
formando um cogulo (trombo) e obstruindo a artria. Essa
obstruo faz com que o paciente tenha os sintomas
caractersticos do infarto agudo do miocrdio.

RESPOSTA: C
COMENTRIO: A reduzida concentrao de protenas
plasmticas, por deficincia nutricional, ocasiona a diminuio
da presso onctica do sangue (PO). A reduo da PO causa o
acmulo do lquido intersticial no reabsorvido pelas
terminaes venosas dos capilares que irrigam os tecidos do
corpo de um animal dotado de circulao fechada.

O(s) vaso(s) sanguneo(s) obstrudo(s) pelas placas causadoras


do infarto e que irriga(m) o msculo cardaco (so):
a) artria coronria;
b) artria pulmonar;
c) veias cavas;
d) artria aorta;
e) veias pulmonares.

206. (Ufrn 2012) O corao humano tem sido alvo de estudos da


engenharia para a produo de dispositivos alternativos que
ajudem a resolver as dificuldades decorrentes dos
transplantes naturais. Embora existam hoje coraes
artificiais, nenhum deles substituiu o original altura no seu
funcionamento. Alguns detalhes mecnicos so fundamentais
para o seu perfeito funcionamento. Assim, na construo de
um prottipo mais parecido com o corao humano,
necessrio considerar que
a) as vlvulas devem impedir o retorno do sangue dos
ventrculos para os trios.
b) o lado direito deve possuir uma maior capacidade de
bombeamento do sangue.
c) o lado direito da bomba deve ter a capacidade de aspirar e
o esquerdo, de impelir o sangue.
d) os conectores de entrada e sada devem ser 4, um para
cada trio e um para cada ventrculo.

RESPOSTA: A
COMENTRIO: A circulao das coronrias responsvel pela
irrigao dos msculos do corao. O infarto consiste na
interrupo da circulao num vaso pertencente as coronrias.

RESPOSTA: A
COMENTRIO: A construo de um prottipo do corao
humano dever conter estruturas anlogas s valva tricspide
e bicspide (mitral). Essas valvas impedem o refluxo de sangue
dos ventrculos para os trios.

208. (G1 - cps 2011) Durante qualquer atividade competitiva, o


estresse fsico, cardiovascular e emocional pode provocar
alterao da pulsao, taquicardia, a morte do msculo
cardaco ou originar arritmias ventriculares fatais.
Sobre esse assunto, assinale a alternativa correta.
a) As arritmias cardacas so alteraes assintomticas que
no interferem no ritmo normal do corao.
b) No processo de taquicardia, os batimentos tornam-se mais
lentos e o corao bombeia, para a cabea e o corpo, mais
sangue a cada contrao.
c) Nas competies, o prprio corpo dos atletas controla o
estresse por meio da produo de hormnios corticoides
produzidos pela glndula tireoide.
d) O controle da frequncia cardaca e da presso arterial dos
atletas, em situao de estresse, pode ser feito com
adrenalina, pois ela diminui os batimentos cardacos.
e) A verificao da pulsao de um atleta, ao pressionar uma
artria, evidencia o fluxo sanguneo impulsionado pela
contrao do ventrculo esquerdo.

207. (UNICENP) O Ataque Cardaco


O ataque cardaco, denominao popular paro o infarto do
miocrdio, atinge e mata pessoas diariamente em todo o
mundo, acometendo 2 em cada 100 pessoas por ano, sendo a
maior causa de morte sbita em adultos (aproximadamente
1/3 dos casos so fatais).
Em situaes normais, o sangue bombeado pelo corao e
circula, atravs das artrias e veias, irrigando todos os tecidos
do corpo, inclusive o prprio corao. No infarto do miocrdio,
h uma interrupo ou diminuio do fluxo de sangue para o
corao, levando a uma reduo acentuada da quantidade de
oxignio que chega ao msculo cardaco. Quando o corao
no recebe oxignio em quantidade suficiente ocorre leso da
musculatura e, dependendo do tempo de durao deste
bloqueio, uma parte do corao morre e para de funcionar.
Na maioria das vezes o bito ocorre por falta de atendimento
imediato. As intervenes precoces, feitas nos primeiros
minutos aps o incio dos sintomas, podem salvar as vitimas
da morte sbita (parada cardiorrespiratria) e deve ser feita
por quem estiver mais prximo da vitima.

RESPOSTA: E
COMENTRIO: Arritmias interferem no ritmo cardaco e,
geralmente, so sintomticas. A taquicardia corre quando o
ritmo cardaco acelerado. Os hormnios corticoides como o
cortesol e corticosterona so produzidos pelo crtex da
glndula suprarrenal. A adrenalina acelera os batimentos
cardacos.
209. (VUNESP) multo comum ouvirmos a seguinte afirmao: "As
artrias carregam sangue arterial, rico em oxignio, e as velas
carregam sangue venoso, pobre em oxignio". Entretanto,
esta generalizao est incorreta, visto que o sangue venoso
(aquele que corre nas veias) nem sempre pobre em oxignio,
e o sangue arterial (aquele que corre nas artrias) nem sempre
rico em oxignio.
Quais dos vasos abaixo relacionados poderiam exemplificar
que essa generalizao no ocorre nos mamferos?
I - Vasos que saem do ventrculo esquerdo do corao.
II - Vasos que chegam ao trio esquerdo do corao.
III - Vasos que saem do ventrculo direito do corao.
IV - Vasos que chegam ao trio direito do corao.

Por que o infarto ocorre?


O infarto do miocrdio a morte de um segmento do msculo
cardaco, determinada pela interrupo brusca da corrente
sangunea numa das artrias nutridoras do corao.

Das afirmativas acima, esto corretas apenas:


a) I e II
57

b) I e III
c) II e IV
d) II e III
e) II e IV

objetivo principal de Guilherme a partir da seguinte relao:


220 batimentos 100%
155 batimentos x
x = 220 / 155 = 0,704 70%.

RESPOSTA: D
COMENTRIO: No corao chegam as veias e saem as artrias.
Do lado direito do corao passa apenas sangue venoso e do
lado esquerdo apenas sangue arterial. Todo vaso que chega ou
sai dos pulmes recebe o nome de pulmonar. Assim sendo,
vejamos: vaso que sai do ventrculo esquerdo do corao a
artria aorta (vai para o organismo), transportando sangue
arterial. Vasos que chegam ao trio esquerdo do corao so
as veias pulmonares (vem dos pulmes) transportando sangue
arterial. Vaso que sai do ventrculo direito do corao a
artria pulmonar (vai para os pulmes) transportando sangue
venoso. Vasos que chegam ao trio direito do corao so as
veias cavas (vem do organismo), transportando sangue
venoso.

Dessa forma, consultando a tabela, conclui-se que Guilherme


pretende adquirir resistncia.

210. (G1 - cps 2011) Nas academias, geralmente as pessoas do


muita ateno ao desenvolvimento dos msculos, mas
acabam esquecendo o mais importante deles: o corao. O
corao um timo indicador do nvel de condicionamento
fsico de uma pessoa e trein-lo adequadamente pode
melhorar a sade em todos os aspectos.
Especialistas, observando o funcionamento do corpo,
concluram que existem faixas de batimentos cardacos, nas
quais o organismo responde de forma diferente. A essas faixas
de batimentos cardacos deu-se o nome de Zonas de
Batimentos Cardacos Alvo ou Zonas de Treinamento.
Observou-se que existe uma Zona de Batimento Cardaco Alvo
em que a pessoa deve se exercitar para alcanar cada objetivo
que deseja, por exemplo, a perda de gordura ou o aumento de
resistncia fsica.
De acordo com algumas pesquisas, pode-se achar a Zona de
Batimentos Cardacos Alvo com um mtodo simples:
diminuir a idade da pessoa de 220(*) para calcular o seu
Batimento Cardaco Mximo (BCMax) por minuto;
escolher o objetivo de treinamento, na tabela, e calcular a
Zona de Batimento Cardaco Alvo correspondente ao que se
deseja, utilizando as porcentagens do BCMax.

211. (Ueg 2011) A fora gravitacional desempenha um papel


fundamental na estabilidade dos organismos vivos. Dentro de
uma espaonave em rbita os tripulantes experimentam uma
aparente perda de peso, situao denominada de estado de
imponderabilidade. No caso de um astronauta em um
ambiente onde a gravidade praticamente nula, correto
afirmar:
a) o crebro ser menos irrigado por correntes sanguneas.
b) a presso para baixo, sobre a coluna vertical, torna-se
maior.
c) o seu corao bombeia mais facilmente sangue para todas
as regies do seu corpo.
d) a quantidade de movimento linear do fluido sanguneo
passa a ser uma funo da densidade do sangue.
e) a quantidade de movimento linear do fluido sanguneo
passa a ser uma funo da densidade da linfa.
RESPOSTA: C
COMENTRIO: Em situao de imponderabilidade, a presso
arterial aumenta e, consequentemente, o corao bombeia
mais facilmente sangue para todas as regies do corpo.
212. (G1 - cps 2011) Voc j pensou o que acontecer com o seu
corpo sem a realizao de atividades fsicas? Segundo as
informaes do IBGE, 80% da populao brasileira vive nas
cidades e mais de 60% dos adultos que vivem nessas reas no
praticam exerccios fsicos com a frequncia adequada. Por
esse motivo, necessrio combater um problema que vem
assumindo grande importncia em reas urbanas, o
sedentarismo. Pesquisas comprovam que o sedentarismo
afeta aproximadamente 70% da populao brasileira, o que
corresponde a uma porcentagem maior do que a da
obesidade, da hipertenso, do tabagismo, do diabetes e do
colesterol alto; sendo assim, praticar atividades fsicas hoje
uma questo de sade pblica.

(*) 220 nmero de batimentos adotado como medida padro


Objetivo do treinamento
Sade do Corao
Queima de Gorduras
Resistncia
Performance
Esforo Mximo

Pensando nisso, pode-se afirmar que uma consequncia do


sedentarismo a
a) ocorrncia de acidentes vasculares que provocam a mistura
de sangue arterial e venoso nos ventrculos, resultando em
deficincia de oxigenao nos tecidos do corpo.
b) ocorrncia constante de processos inflamatrios que
resultam no aumento de tamanho dos gnglios linfticos, o
que se conhece popularmente como cibras.
c) reduo do volume de ar que cabe nos pulmes e da
capacidade de o corao bombear sangue para todo o corpo,
o que afeta a oxigenao celular.
d) alterao do material gentico ocasionando diminuio da
mineralizao ssea, o que aumenta a osteoporose e os
problemas nas articulaes.
e) incidncia de infarto agudo do miocrdio, devido ao
aumento da irrigao do msculo cardaco pelo sangue
contido no interior do corao.

Porcentagem do BCMax
de 50% a 60%
de 60% a 70%
de 70% a 80%
de 80% a 90%
de 90% a 100%

(http://www.corpoperfeito.com.br/artigo/VisualizacaoArtigo.a
spx?IdArtigo=54 Acesso em: 08.08.2010. Adaptado)
Guilherme, de 20 anos, tem mantido em suas atividades fsicas
de 145 a 155 batimentos cardacos por minuto.
Nessas condies, pode-se afirmar que Guilherme tem como
objetivo principal
a) a resistncia.
b) a performance.
c) o esforo mximo.
d) a sade do corao.
e) a queima de gordura.

RESPOSTA: C
COMENTRIO: Nos mamferos adultos no ocorre mistura de
sangue arterial e venoso. A cimbra resultante do acmulo
de cido ltico nos msculos esquelticos. A osteoporose e
problemas articulares no so, necessariamente, relacionados
a alteraes genticas. O infarto agudo do miocrdio o

RESPOSTA: A
COMENTRIO: Considerando-se 220 batimentos por minuto
como sendo o esforo mximo (90 a 100%), pode-se calcular o
58

resultado da irrigao sangunea deficiente do msculo


cardaco.

Comparando o corao de um sapo com o corao humano,


pode-se afirmar que
a) no h diferenas significativas entre os dois quanto
estrutura das cmaras.
b) enquanto no sapo o sangue chega pelos trios cardacos, no
corao humano o sangue chega pelos ventrculos.
c) ao contrrio do que ocorre no sapo, no corao humano o
sangue chega sempre pelo trio direito.
d) ao contrrio do que ocorre no sapo, nas cmaras do
corao humano por onde passa sangue arterial no passa
sangue venoso.
e) nos dois casos, o sangue venoso chega ao corao por dois
vasos, um que se abre no trio direito e o outro, no trio
esquerdo.

213. (G1 - col.naval 2011) Preencha corretamente as lacunas da


sentena abaixo.
Em uma pessoa jovem e com boa sade, quando ocorre a
sstole (contrao) dos ventrculos, as grandes arterias
_______ e a presso sangunea em seu interior atinge, em
mdia, cerca de ______________ .
Assinale a opo correta.
a) contraem-se; 120 mmHg.
b) contraem-se; 80 mmHg.
c) relaxam-se; 120 mmHg.
d) relaxam-se; 80 mmHg.
e) no se alteram; 120 mmHg.
RESPOSTA: C
COMENTRIO: A sstole ventricular bombeia o sangue do
corao em direo s grandes artrias. Nesse momento, a
musculatura lisa das paredes desses vasos sofrem
relaxamento
para
suportar
uma
presso
de,
aproximadamente, 120 mmHg.

RESPOSTA: D
COMNTRIO: O corao de sapo possui apenas uma cmara
ventricular. Dessa maneira, por essa cmara passam tanto
sangue venoso quanto arterial. De forma diferente, no
corao humano, h duas cmaras ventriculares. O sangue
venoso passa pelo ventrculo direito, enquanto o sangue
arterial passa pelo ventrculo esquerdo.
217. (Upe 2011) A msica Bate corao refere-se aos batimentos
cardacos. Mas, por que o corao bate?

214. (G1 - cftmg 2011) A formao de varizes deve-se, entre outros


fatores, ao acumulo de sangue nas veias, as quais ficam
comprometidas na funo de
a) impedir o refluxo do sangue.
b) reforar as paredes dos vasos.
c) retardar as pulsaes cardacas.
d) acelerar as contraes sistmicas.
e) retardar as contraes sistmicas.

Bate corao (Elba Ramalho)


Bate, bate, bate, corao
Dentro desse velho peito.....
Tum, tum, bate corao
Oi, tum, corao pode bater
Oi, tum, tum, tum, bate, corao
Que eu morro de amor com muito prazer

RESPOSTA: A
COMENTRIO: As varizes so formadas a partir de veias
obstrudas por cogulos anormais.

Assinale a alternativa que explica corretamente o evento


relacionado aos batimentos cardacos.
a) A atividade parassimptica reduz os batimentos cardacos,
contribuindo para o repouso do corao.
b) A atividade simptica, sob ao da noradrenalina, diante de
situaes de defesa ou ataque, diminui a frequncia cardaca.
c) A contrao do corao distole e o seu relaxamento
sstole so controlados por fenmenos miognicos.
d) Apesar de sua contrao voluntria, os batimentos
cardacos tm mecanismos reguladores relacionados com o
sistema nervoso autnomo, e a atuao desses nervos ajusta a
frequncia conforme as necessidades do organismo.
e) O nervo vago, cardiomediador, libera adrenalina como
mediador qumico, e os nervos cardacos, cardioaceleradores,
liberam acetilcolina.

215. (Ufrgs 2011) Um reduzido suprimento de sangue no msculo


cardaco de mamferos acarreta
a) a diminuio do suprimento de oxignio.
b) o aumento do nmero de hemcias.
c) o aumento do nmero de mitocndrias.
d) o aumento do metabolismo oxidativo.
e) a diminuio da concentrao de cido lctico.
RESPOSTA: A
COMENTRIO: A reduo do suprimento sanguneo no
msculo cardaco acarreta a diminuio da oxigenao do
miocrdio e, consequentemente, insuficincia cardaca ou
infarto.
216. (Fuvest 2011)
A figura abaixo representa, em corte
longitudinal, o corao de um sapo.

RESPOSTA: A
COMENTRIO: O nervo vago (parassimptico) libera o
cardiomediador acetilcolina, que reduz os batimentos
cardacos contribuindo para o repouso do miocrdio.
218. (Uece 2010) Certo paciente ao medir sua presso encontrou
130 mm Hg por 110 mm Hg. Relatou o ocorrido para o mdico
que sugeriu que ele colocasse um aparelho para registrar as
medidas ao longo de 24 horas. Associando este resultado aos
dados de outros exames, o mdico julgou importante este
acompanhamento.
Sobre a presso do paciente, assinale o correto.
a) A presso diastlica estava fora da faixa de normalidade.
b) A presso sistlica estava fora da faixa de normalidade.
c) Ambas as presses, sistlica e diastlica, estavam fora da
faixa de normalidade.
59

d) No havia anormalidade. Ambas as presses, sistlica e


diastlica, estavam dentro da faixa de normalidade.
e) A presso diastlica estava dentro da faixa de normalidade.
RESPOSTA: A
COMENTRIO: A presso arterial considerada normal
quando a presso sistlica (mxima) no ultrapassar 130 mm
Hg e a diastlica (mnima) for inferior a 85 mm Hg. Nesse caso,
o paciente apresentava presso sistlica limtrofe, porm
dentro da faixa de normalidade (130 mm Hg) e presso
diastlica alta (110 mm Hg).

COMENTRIO: Aps um abundante almoo ou jantar, o


sistema nervoso parassimptico desloca uma quantidade
maior de sangue para os tecidos do trato digestivo, porm no
h uma alterao significativa no ritmo dos batimentos
cardacos devido a esse fato.
221. (UFV) Leia com ateno o trecho abaixo:
"...e como nesses organismos o sangue no permanece
somente no interior de vasos, sua presso baixa e a
velocidade do fluxo reduzida. Por essa razo, a distribuio de
substncias lenta e no vence longas distncias, e no
entanto, esses organismos podem apresentar movimentos to
rpidos que indicam que a oxigenao dos tecidos muito
eficiente."
(Avancini, E. B. e Favaretto, J. A. "Biologia". So Paulo:
Moderna, 1997. v.2, p. 655.)
Pode-se afirmar que o organismo a que o trecho acima se
refere um exemplo de:
a) inseto, pois a oxigenao dos tecidos independe do sistema
circulatrio.
b) celenterado, pois as brnquias transferem o oxignio
diretamente para os tecidos.
c) nematoide, pois o lquido celomtico se encarrega de levar
o oxignio aos tecidos.
d) aneldeo, pois, embora a respirao seja cuticular, o
oxignio transportado pelos tbulos de Malpighi.
e) platelminto, pois na fase adulta, alm de respirao
branquial, apresenta tambm respirao cutnea.

219. (UEL) Os esquemas a seguir referem-se circulao de trs


grupos de vertebrados.

I. Nos peixes, o sangue passa uma s vez pelo corao a cada


ciclo.
II. No ventrculo dos anfbios, h mistura de sangue arterial
com venoso.
III. Nos mamferos, no h mistura entre sangue arterial e
venoso.

RESPOSTA: A
COMENTRIO: Observe que esse animal tem sistema
circulatrio aberto, pois o seu sangue no circula apenas
dentro de vasos. Porm esse animal tem movimentos rpidos,
o que exige grande consumo de oxignio. Sistema circulatrio
aberto aparece nos artrpodes e moluscos. Apenas os insetos
tm essa caracterstica de receber o oxignio em suas clulas
de forma bem rpida, pois sua respirao no depende do
sistema circulatrio. Possuem respirao traqueal. O ar
penetra pelos estigmas ou espirculos aberturas localizadas no
abdome e vai diretamente para as clulas.

correto o que se afirma em


a) I, somente.
b) III, somente.
c) I e II, somente.
d) II e III, somente.
e) I, II e III.
RESPOSTA: E
COMENTRIO: Fique calminho que o desenho do corao dos
peixes est complicado. Mas voc saberia responder, pois o
corao desses animais tem duas cavidades com circulao
fechada e simples.

222. (Udesc 2010) Associe a primeira coluna com a segunda


quanto circulao nos vertebrados.
( 1 ) circulao completa ( ) relaxamento do corao
( 2 ) distole ( ) anfbios
( 3 ) circulao incompleta ( ) sangue venoso
( 4 ) sangue arterial ( ) contrao do corao
( 5 ) lado direito do corao ( ) peixe
( 6 ) sstole ( ) aorta

220. (Fgv 2010) Em condies normais de sade e repouso, o


nmero de pulsaes de um homem adulto da ordem de 70
por minuto. Aps um abundante almoo ou jantar, em que se
ingerem carnes, conservas, pes e doces, o que se espera em
relao ao nmero de pulsaes por minuto que
a) haja aumento desse nmero, devido atividade cardaca
que se acelera em razo da diminuio da temperatura interna
do corpo.
b) haja aumento desse nmero devido maior necessidade de
irrigao sangunea dos tecidos do trato digestivo.
c) haja reduo desse nmero, uma vez que a temperatura do
corpo sofrer pequena reduo e, com isso, a atividade
cardaca diminui.
d) no haja qualquer alterao, uma vez que os alimentos
ingeridos sofrero digesto no estmago e intestino, sem
qualquer interferncia com a atividade cardaca.
e) no haja qualquer alterao desse nmero, mas que haja
aumento da presso sangunea em decorrncia da quantidade
de sal ingerida.
RESPOSTA: D

Assinale a alternativa que contm a sequncia correta, de


cima para baixo.
a) 2 3 4 6 5 1
b) 6 3 5 4 2 1
c) 2 1 6 5 3 4
d) 2 3 5 6 1 4
e) 6 1 5 2 3 4
RESPOSTA: D
COMENTRIO: A circulao nos vertebrados fechada, ou
seja, o sangue circula dentro de vasos e rgos especializados
e as artrias e veias esto ligadas por meio de vasos capilares.
Em funo do seu grau evolutivo, a circulao pode ser
simples ou dupla, e, nesse ltimo caso, ainda, incompleta ou
completa. Nos peixes ocorre a circulao simples completa. A
circulao dupla incompleta ocorre nos anfbios e rpteis e a
60

circulao dupla completa, nas aves e mamferos. O corao


dos mamferos possui quatro cavidades: duas aurculas e dois
ventrculos. No lado direito do corao circula apenas sangue
venoso e no lado esquerdo apenas sangue arterial. Sstole o
movimento de contrao de cada parte do miocrdio. Distole
o perodo de relaxamento muscular.
223. (UFG) O sistema circulatrio dos vertebrados tem em comum
o seguinte aspecto:
a) As artrias e as veias so interligadas por uma rede de
capilares.
b) O sangue e a linfa fluem pelo corpo por meio de vasos e
lacunas.
c) O corao dividido em quatro cavidades, duas aurculas e
dois ventrculos.
d) O sangue arterial no se mistura com o sangue venoso.
e) As hemcias circulantes so nucleadas e apresentam
carioteca.
RESPOSTA: A
COMENTRIO: A rede de capilares encontrada nos tecidos,
onde o sangue arterial deixa o oxignio e recebe o gs
carbnico, se transformado em sangue venoso.
A afirmativa b est incorreta porque o sistema circulatrio dos
vertebrados fechado no possui lacunas.
A afirmativa c est incorreta porque corao com quatro
cavidades pode ser encontrado apenas entre crocodilianos,
aves e mamferos.
A afirmativa d est incorreta porque mistura de sangue
venoso com o arterial encontrado apenas no corao de
anfbios e rpteis.
A afirmativa e est incorreta porque as hemcias dos
mamferos so anucleadas.

a) A que filo esse animal pertence?


b) Como uma colnia destes hidrozorios se origina? E como
esta colnia d origem a novas colnias?
c) Que estrutura comum aos plipos e medusas encontrada
somente neste filo? Qual sua funo?
Sugesto de resposta:
a) Esse organismo pertence ao filo dos cnidrios (ou
celenterado, filo Cnidria);
b) Obelia sp um animal pertencente ao filo Cnidria e
classe Hidrozoa. A colnia desses animais formada por meio
de reproduo assexuada (geralmente brotamento) do plipo.
Uma colnia d origem a outra colnia por metagnese. Na
metagnese as medusas se formam atravs de reproduo
assexuada (gemiparidade) que ocorre nos plipos. Por
reproduo sexuada, as medusas formam o zigoto e deste
origina-se a larva plnula a qual d origem a um novo plipo.
c) A estrutura o cnidoblasto ou cnidcito, a clula urticante
dos cnidrios. Essas clulas realizam a defesa e a captura dos
alimentos.

BIOLOGIA II
224. (PUC-PR) Em relao ao Phylum Cnidria, foram feitas as
seguintes proposies:
I. Os cnidrios so aquticos, diblsticos e com simetria radial,
sendo encontrados em duas formas: plipos (fixos) e medusa
(livres).
II. A digesto nos cnidrios extra e intracelular e no h
sistemas respiratrio, circulatrio ou excretor e o sistema
nervoso difuso.
III. Nos cnidrios, a reproduo sexuada ocorre por
brotamento ou estrobilizao.
IV. Os corais e a anmona-do-mar so exemplos da classe dos
cifozorios.
Assinale a alternativa correta:
a) Todas esto INCORRETAS.
b) Apenas III e IV esto corretas.
c) Apenas I est correta.
d) Todas esto corretas.
e) Apenas I e II esto corretas.

226. (UFRS) A restrio do tamanho e da forma em planrias est


diretamente relacionada
a) ausncia de um sistema circulatrio.
b) ao sistema nervoso difuso.
c) presena de protonefrdios.
d) ao hermafroditismo.
e) possibilidade de regenerao.
RESPOSTA: A
COMENTRIO: A planria pertence ao filo dos platelmintos.
Animais com sistema nervoso ganglionar, hermafroditas, com
sistema excretor do tipo protonefrediano (unidades clulasflama) e com grande capacidade de regenerao. Esses
animais no possuem sistemas circulatrio e respiratrio. O
fato de serem achatados e com restrio de tamanho facilita a
difuso do alimento de clula para clula e a respirao
(tambm por difuso).

RESPOSTA: E
COMENTRIO: A afirmativa III est errada, pois brotamento e
estrobilizao so formas assexuadas de reproduo. A
afirmativa IV est errada porque os corais e anmonas
pertencem classe anthozoa.
225. (Unicamp) Alguns hidrozorios coloniais, como a Obelia sp,
ocorrem na natureza sob forma de plipos e medusas. O
esquema representa o ciclo de vida de deste invertebrado.

227. (UFF) Os platelmintos pertencem ao primeiro grupo de


animais a possuir um sistema excretor. Este bastante
primitivo, formado por clulas-flama. A principal funo destas
clulas :
a) remover o excesso de gua e os resduos nitrogenados do
sangue e lan-los para o intestino;
61

b) remover o excesso de gua e os resduos nitrogenados do


sistema circulatrio e lan-los para o exterior;
c) remover o excesso de gua e os resduos nitrogenados do
tecido epitelial e lan-los para o intestino;
d) remover o excesso de gua e os resduos nitrogenados do
ectoderma e lan-los para os tbulos de Malpighi;
e) remover o excesso de gua e os resduos nitrogenados do
mesoderma e lan-los para o exterior.
RESPOSTA: E
COMENTRIO: A letra a est errada porque os platelmintos
no possuem sangue e nem intestino. A letra b est errada
porque esses animais no possuem sistema circulatrio, alm
disso, retiram os excretas do mesoderma. A letra c est errada
devido no possurem intestino. A letra d est errada devido
no retirarem excretas do ectoderma e tambm porque no
possuem tbulos de Malpighi.
228. (MACKENZIE) A respeito dos platelmintos INCORRETO
afirmar que:
a) h representantes que possuem tanto reproduo
assexuada como sexuada.
b) h representantes tanto de vida livre como parasitas.
c) so todos triblsticos acelomados.
d) possuem sistema respiratrio e circulatrio.
e) h representantes hermafroditas e de sexos separados.
RESPOSTA: D
COMENTRIO: Os platelmintos no possuem sistemas
circulatrio e respiratrio. As tnias possuem reproduo
assexuada (estrobilizao) e sexuada (autofecundao). A
planria tem vida livre, as tnias e o S. mansoni so parasitas.
As tnias e a planria so hermafroditas (monoicos) e o
S.mansoni dioico (sexos separados). Os platelmintos no
possuem sistemas circulatrio, respiratrio e esqueltico.
229. (UEL) Nematdeos so animais vermiformes de vida livre ou
parasitria, encontrados em plantas e animais, inclusive no
homem. Sobre as caractersticas presentes em nematdeos,
considere as afirmativas a seguir.
I.
II.
III.
IV.

e) digesto extracelular.
RESPOSTA: A
COMENTRIO: Os animais do filo Porferos (Espongirios) so
monoicos ou dioicos. A fecundao interna e o
desenvolvimento ocorre por meio de uma fase larvria,
denominada anfiblstula ou parnquimula.
231. (FUVEST-mod) A caracterstica abaixo que no condiz com os
porferos :
a) respirao e excreo por difuso direta.
b) obteno de alimentos a partir das partculas fagocitadas
pelos coancitos.
c) hbitat aqutico, vivendo presos ao fundo (ssseis).
d) clulas organizadas em tecidos bem definidos.
e) alta capacidade de regenerao.
RESPOSTA: D
COMENTRIO: Os porferos no possuem tecidos verdadeiros
(parazorios).
232. (Ufpr 2012) A esquistossomose uma doena parasitria
considerada grave, por ser a que mais causa morte em
humanos dentre as causadas por organismos multicelulares.
Uma forma de se combater essa doena o controle biolgico
pelo uso de peixes como o tambaqui. De que maneira esse
peixe ajuda a combater a doena em humanos?
a) O peixe serve como o hospedeiro definitivo do verme da
esquistossomose, do gnero Schistosoma, no lugar do
homem.
b) O tambaqui se alimenta da cercaria, forma do parasita que
infecta ativamente o humano.
c) O miracdio, forma que infecta o caramujo (hospedeiro
intermedirio), passa a infectar o peixe e nele no consegue
completar seu ciclo vital.
d) O caramujo (hospedeiro intermedirio) comido pelo
peixe, e o parasita no tem como completar seu ciclo de vida.
e) O peixe e o caramujo (hospedeiro intermedirio) competem
pelos mesmos recursos naturais e o primeiro elimina o
segundo por competio.

Corpo no-segmentado coberto por cutcula.


Trato digestrio completo.
rgos especializados para circulao.
Pseudoceloma.

a) I e III.
b) I e IV.
c) II e III.
d) I, II e IV.
e) II, III e IV.
RESPOSTA: D
COMENTRIO: Nematdeos, nematelminthes, nematoda ou at
asquelmintes so os mesmos animais. Vermes cilndricos, no
segmentados, com tubo digestivo completo (primeiro na escala
evolutiva), pseudocelomados, com respirao por difuso ou
anaerbios, com sistema excretor do tipo tubular e com sistemas
circulatrio e respiratrio ausentes.

RESPOSTA: D
COMENTRIO: O controle biolgico do platelminto
Schistosoma pode ser bem sucedido pela utilizao de peixes
como o Tambaqui. Esse peixe se alimenta de caramujos, que
so hospedeiros intermedirios de esquistossomos.
233. (Ufpr 2012) Considere o quadro abaixo, que apresenta dados
sobre saneamento bsico no ano de 2000 em duas cidades do
Paran:

Proporo de moradores
CURITIBA ADRIANPOLIS
Rede geral de esgoto
75,7%
13,2%
Abastecimento de gua 98,6%
46,4%
Fonte: Ministrio da Sade/DATASUS
A partir dos dados apresentados, correto afirmar que, em
relao populao de Curitiba, a populao do municpio de
Adrianpolis est mais exposta a adquirir:
a) ascaridase e sfilis.
b) clera e cisticercose.
c) dengue e ttano.
d) esquistossomose e malria.
e) febre amarela e doena de Chagas.

230. (Ufpb 2010) Os porferos so considerados os representantes


mais simples entre todos do reino Animalia.
Sobre os representantes desse grupo, correto afirmar que
possuem
a) um estdio larval durante seu desenvolvimento.
b) sistema nervoso simples e difuso pelo corpo.
c) representantes protostmios.
d) representantes diploblsticos.
62

RESPOSTA: B
COMENTRIO: Os dados apresentados na tabela revelam que
os habitantes de Adrianpolis esto mais expostos a adquirir
doenas transmitidas pela gua contaminada com fezes
humanas, tais como o clera e a cisticercose.

epidemia de diarreia. Lemeta tambm verifica se a famlia


possui uma latrina e fala aos moradores sobre as vantagens de
ferver a gua de beber, lavar as mos e banhar-se duas vezes
por semana.
(Adaptado de: ROSENBERG, Tina. O fardo da sede. Revista
National Geographic.
ed.121,
2010.
Disponvel
em:
<http://viajeaqui.abril.com.br/national-geographic/edicao121/busca-agua-propria-542206.shtml?page=3>. Acesso em: 3
ago. 2011.)

234. (Ufrn 2012) Leia o texto que segue:


A esquistossomose mansnica uma endemia mundial,
ocorrendo em 52 pases e territrios, principalmente na
Amrica do Sul, Caribe, frica e Leste do Mediterrneo, onde
atinge as regies do Delta do Nilo, alm de pases como Egito
e Sudo. No Brasil, a transmisso ocorre em 19 estados, numa
faixa contnua ao longo do litoral, desde o Rio Grande do
Norte at a Bahia, na regio Nordeste, alcanando o interior
do Esprito Santo e Minas Gerais, no Sudeste.
Brasil. Ministrio da Sade. Secretaria de Vigilncia em Sade.
Guia de vigilncia epidemiolgica / Ministrio da Sade,
Secretaria de Vigilncia em Sade. 6 ed. Braslia: Ministrio
da Sade, 2005.
O texto ressalta a grande ocorrncia da esquistossomose no
Mundo e no Brasil, permitindo pensar suas formas de
preveno. Sabe-se que as aes de educao em sade e a
mobilizao comunitria so muito importantes no controle
desse mal, e que o saneamento ambiental da maior eficcia
para as modificaes de carter permanente das condies de
transmisso da esquistossomose. Com o objetivo de quebrar o
ciclo de vida do parasita, para prevenir essa doena, deve-se
a) exterminar as populaes de caramujos que infectam os
hospedeiros intermedirios.
b) incentivar o uso de gua potvel e construir aterros para
eliminar colees hdricas que sejam criadouros de mosquitos.
c) impedir que os ovos do parasita presentes nas fezes de uma
pessoa contaminem corpos aquticos.
d) controlar as populaes de nematdeos, hospedeiros
intermedirios do parasita.

235. (Uel 2012) A diarreia, citada no texto, um dos sintomas mais


comuns de parasitoses do trato digestivo humano. A maior
incidncia dessas doenas ocorre em regies sem gua tratada
e sistema de esgoto sanitrio. Assinale a alternativa correta
que contm apenas doenas cuja medida preventiva est
associada a instalaes sanitrias adequadas.
a) Ascaridase, difteria, doena de Chagas e tenase.
b) Amarelo, dengue, esquistossomose e tenase.
c) Amebase, clera, esquistossomose e giardase.
d) Ascaridase, cisticercose, leishmaniose e oxiurose.
e) Ancilostomose, clera, febre tifoide e malria.
RESPOSTA: C
COMENTRIO: A difteria uma infeco bacteriana
transmitida de pessoa a pessoa, diretamente pelo ar. A
dengue, a leishmaniose e a malria so transmitidas pela
picada de fmeas de mosquitos.
236. (Ufpr 2011) Observe as figuras abaixo, extradas de folhetos
distribudos populao por rgos pblicos:

RESPOSTA: C
COMENTRIO: O platelminto causador da esquistossomose
mansnica, denominado Schistosoma mansoni, um
endoparasita, cujas fmeas liberam ovos que so eliminados
do corpo humano, juntamente com as fezes. O saneamento
bsico, redes de esgoto e a educao sanitria so medidas
eficazes para impedir a contaminao aqutica pelos dejetos
humanos e, consequentemente, diminuir a transmisso da
verminose.
TEXTO PARA A PRXIMA QUESTO:
No Konso [Etipia], o homem carrega gua apenas nas duas ou
trs semanas subsequentes ao nascimento de seu beb.
Garotos pequenos pegam gua tambm, mas apenas at os 7
ou 8 anos. Essa regra seguida risca por homens e
mulheres. Se garotos mais velhos carregam gua, as pessoas
comeam a fofocar que a me deles preguiosa, diz Aylito.
A reputao de uma mulher do Konso, diz ela, assenta-se no
trabalho duro. Se eu ficar sentada em casa e no fizer nada,
ningum vai gostar de mim. Mas, se eu correr para cima e para
baixo com 45 litros de gua, eles diro que sou uma mulher
sbia que trabalha duro. Lemeta, tmido, para na casa de
Aylito Binayo e pede permisso ao marido dela, Guyo Jalto,
para checar seus gales. Jalto leva-o at a palhoa onde eles
so guardados. Lemeta abre a tampa de um deles e cheira,
balanando a cabea em aprovao a famlia est usando
WaterGuard, um aditivo base de cloro. Uma tampinha cheia
do produto purifica um galo de gua. O governo passou a
distribuir WaterGuard logo no comeo da mais recente

As figuras A e B representam, respectivamente, medidas de


preveno da:
a) doena de chagas e cisticercose.
b) ancilostomose e clera.
c) malria e dengue.
d) esquistossomose e febre amarela.
e) tenase e ascaridase.
RESPOSTA: D
COMENTRIO: A esquistossomose uma verminose que pode
ser evitada com medidas sanitrias adequadas, porque a
transmisso se d por meio de ovos do platelminto
Schistossoma mansoni, eliminados com as fezes de indivduos
infestados. A febre amarela uma virose transmitida pela
picada do mosquito Aedes aegypti fmea. Pode-se evitar essa
doena impedindo a formao de colees de gua parada,
locais onde se desenvolvem os pernilongos transmissores.
237. (Fuvest 2011) Ao noticiar o desenvolvimento de mecanismos
de preveno contra a esquistossomose, um texto jornalstico
trouxe a seguinte informao:
Protena do parasita da doena ensina organismo a se
defender dele.
Folha de S. Paulo, 06/08/2010.
Traduzindo a notcia em termos biolgicos, correto afirmar
que uma protena, presente
63

a) no platelminto causador da doena, ao ser introduzida no


ser humano, estimula resposta
imunolgica que, depois, permite o reconhecimento do
parasita no caso de uma infeco.
b) no platelminto causador da doena, serve de modelo para a
produo de cpias de si mesma no
corpo do hospedeiro que, ento, passa a produzir defesa
imunolgica contra esse parasita.
c) no molusco causador da doena, estimula a produo de
anticorpos no ser humano, imunizando-o
contra uma possvel infeco pelo parasita.
d) no molusco causador da doena, atua como anticorpo, no
ser humano, favorecendo a resposta
imunolgica contra o parasita.
e) no nematelminto causador da doena, pode ser utilizada na
produo de uma vacina capaz de
imunizar o ser humano contra infeces por esses organismos.

As formas de controle da doena envolvem o tratamento das


pessoas doentes, a implantao de medidas de saneamento
bsico e a eliminao dos moluscos hospedeiros
intermedirios.
Sobre as formas de controle da doena mencionadas acima,
leia as seguintes afirmativas:
I. O tratamento das pessoas doentes por meio do uso de
medicao anti-helmntica visa eliminao das formas
adultas do parasito, as quais s esto presentes no hospedeiro
definitivo.
II. O tratamento das pessoas doentes por meio do uso de
medicao anti-helmntica visa eliminao das formas larvais
do parasito, que s esto presentes no hospedeiro definitivo.
III. A implantao de medidas de saneamento bsico impede
que os ovos do parasito, eliminados com as fezes do
hospedeiro definitivo, cheguem aos corpos de gua (rios,
audes, crregos) e liberem os miracdios, que so larvas
infectantes para os hospedeiros intermedirios.
IV. A eliminao dos moluscos visa interrupo do ciclo do
parasito, uma vez que as cercrias, que so as formas
infectantes para os humanos, s se desenvolvem nos
moluscos hospedeiros intermedirios.
V. A eliminao dos moluscos visa interrupo do ciclo do
parasito, uma vez que os ovos do Schistosoma mansoni, que
so as formas infectantes para os humanos, so eliminados
com as fezes dos moluscos.

RESPOSTA: A
COMENTRIO: A esquistossomose causada por platelmintos
do gnero Schistosoma. Suas protenas, ao serem introduzidas
no ser humano, agem como antgenos que estimulam o
organismo humano a produzir, como resposta imunolgica
presena dessas protenas estranhas, anticorpos especficos
que, no caso de uma infeco, reconheceriam e combateriam
o parasita.
238. (G1 - ifsc 2011) Os animais so comumente divididos em dois
grupos: vertebrados e invertebrados. Os invertebrados
representam cerca de 97% de todas as espcies de animais, no
entanto costumam ser menos conhecidos do que os
vertebrados. Sobre os animais vertebrados e invertebrados,
CORRETO afirmar que:
a) com frequncia as guas-vivas causam acidentes aos
banhistas durante o vero. Os tentculos desses animais
possuem clulas urticantes que causam a sensao de
queimaduras e podem desencadear reaes alrgicas graves.
b) os artrpodes apresentam um esqueleto externo que no
acompanha o crescimento do animal. Para crescer o indivduo
troca seu exoesqueleto de tempos em tempos buscando a
casca abandonada por um indivduo maior procarionte.
c) os equinodermos, como as estrelas-do-mar, possuem
esqueleto calcrio e apesar de no possurem vrtebras
podem ser considerados animais vertebrados.
d) mexilhes, ostras e mariscos so muito utilizados como
fonte de alimento em regies litorneas. Estes moluscos
possuem o corpo segmentado com uma concha protetora
espiralada.
e) as aranhas e escorpies esto entre os principais
representantes dos insetos. Estes animais podem produzir
veneno para caar e, eventualmente, causam acidentes ao
homem.

Esto corretas:
a) as afirmativas I, II e III.
b) as afirmativas II, III e IV.
c) as afirmativas I, III e IV.
d) as afirmativas I, IV e V.
e) as afirmativas II, IV e V.
RESPOSTA: C
COMENTRIO: As formas larvais do platielminto Schistosoma
masoni ocorrem no homem (cercrias) e no molusco
hospedeiro intermedirio (miracdio e cercarias). Os ovos do S.
mansoni so eliminados nas fezes humanas.
240. (Uesc 2011) As espcies do gnero Schistosoma que afetam o
homem chegaram s Amricas durante o trfico de escravos
(S. mansoni) e com os imigrantes orientais e asiticos (S.
haematobium e S. japonicum). Entretanto, apenas o S.
mansoni aqui se fixou, seguramente pelo encontro de bons
hospedeiros intermedirios e pelas condies ambientais
semelhantes s da regio de origem.
NEVES, David Pereira. Parasitologia Humana. 10a ed. So
Paulo: Atheneu, 2002. p 175.

RESPOSTA: A
COMENTRIO: As guas-vivas so cnidrios dotados de clulas
urticantes, os cnidcitos. Essas clulas liberam uma
neurotoxina proteica que pode desencadear sensaes de
queimadura e alergia em humanos.
239. (Ufjf 2011) A esquistossomose intestinal uma doena
parasitria
causada
pelo
Schistosoma
mansoni
(Platyhelminthes, Trematoda, Digenea), tambm conhecida
como xistose ou barriga-dgua. O Brasil um foco
endmico da esquistossomose, com mais de seis milhes de
pessoas infectadas.
O ciclo do Schistosoma mansoni envolve dois hospedeiros; o
homem o hospedeiro definitivo e os caramujos aquticos do
gnero Biomphalaria so os hospedeiros intermedirios.

Considerando-se as informaes apresentadas a respeito da


biologia desse parasita, correto afirmar:

64

a) O S. mansoni encontrou, no Brasil, uma nova espcie de


hospedeiro definitivo, que permitiu uma boa adaptao
desses vermes ao novo ambiente.
b) As cercrias maduras penetram ativamente no caramujo
para que possam completar seu estgio de desenvolvimento.
c) A presena de caramujos da famlia dos planorbdeos
essencial para que o miracdio complete seu ciclo de
desenvolvimento por um processo sexuado de reproduo.
d) A ingesto de gua e alimentos contaminados com ovos do
parasita a principal forma de contgio de seres humanos
para esse tipo de verminose.
e) A construo de instalaes sanitrias nas moradias para
evitar que os ovos do esquistossoma contaminem rios e lagos
considerada como uma medida profiltica adequada para
essa endemia.
RESPOSTA: E
COMENTRIO: O caramujo planorbdeo o hospedeiro
intermedirio do Schistosoma mansoni. A larva miracdeo se
desenvolve no interior do molusco formando as larvas
cercrias, por pedognese. A forma de contgio do S. mansoni
ocorre por meio das larvas cercarias que penetram ativamente
na pele humana.

Assinale a alternativa que apresenta a sequncia correta, de


cima para baixo.
a) F F V F.
b) V V F V.
c) F F V V.
d) V V V F.
e) V V V V.
RESPOSTA: E
COMENTRIO: Todas as afirmativas so verdadeiras e
correlacionadas com o texto.
242. (Ufsm 2010) No curso de sua histria, a espcie humana tem
exercido diversos tipos de interaes com o meio natural,
sendo a interao com os animais uma das mais
proeminentes. Apesar de muitas delas serem benficas, outras
tantas so prejudiciais, tais como as doenas causadas por
diversos tipos de animais, particularmente os invertebrados.
Com base nessa informao, assinale a afirmativa correta.
a) A Taenia solium, um verme platelminto, pode provocar uma
doena denominada equinococose.
b) Pode-se contrair a ascaridase atravs da ingesto de gua
ou alimentos contaminados por ovos de um verme
nematdeo.
c) O ciclo da esquistossomose mansnica, ou doena do
caramujo, envolve dois hospedeiros, sendo um deles a espcie
humana, onde uma larva chamada miracdio completa o ciclo,
penetrando atravs da pele e mucosas.
d) A elefantase, causada por um nematdeo conhecido como
filria, transmitida pela picada do mosquito do gnero
Aedes.
e) A febre amarela causada por um vrus transmitido
exclusivamente pela picada do mosquito do gnero Culex.

241. (Ufpr 2010) Leia o texto a seguir sobre os recifes de coral.


Afetados por mudanas climticas, poluio, introduo de
espcies invasoras e pesca predatria, muitos corais esto em
perigo. Um tero dos recifes de corais de todo o planeta est
ameaado de extino. A regio do Caribe a que apresenta a
mais alta concentrao de corais ameaados. o que revela o
primeiro levantamento global para determinar o seu status de
conservao. Os resultados do estudo foram publicados pela
revista Science. Construdos ao longo de milhes de anos, os
recifes de corais so o habitat de mais de 25% das espcies
marinhas, configurando-se como o ecossistema marinho com
maior diversidade biolgica. Os corais constroem recifes em
guas rasas tropicais e subtropicais e tm-se mostrado
altamente sensveis a mudanas em seus ambientes. Milhes
de pessoas em todo o planeta dependem desses ecossistemas
para o seu sustento, seja atravs da pesca ou do turismo. Os
pesquisadores apontaram como principais ameaas aos corais
o aquecimento global e alteraes locais decorrentes da pesca
predatria, a introduo de espcies invasoras e o declnio na
qualidade das guas por causa da poluio e da ocupao
desenfreada da zona costeira.
(Marcados para Morrer. Jornal da Cincia n 3552, de 11 de
julho de 2008 Adaptado.)
Sobre o impacto sofrido pelos recifes de corais, identifique as
afirmativas a seguir como verdadeiras (V) ou falsas (F):
( ) As mudanas climticas podem aumentar a temperatura
da gua e a intensidade da radiao solar, o que leva
descolorao dos corais e a doenas que podem acarretar a
sua mortalidade.
(
) Com o aumento da concentrao de na atmosfera, os
oceanos absorvem volumes cada vez maiores desse gs,
levando a um aumento da acidez e diminuio do pH, o que
prejudica gravemente a capacidade dos corais de construrem
seus esqueletos, os quais formam as fundaes dos recifes.
(
) A pesca de arrasto, utilizando bombas ou cianeto para
captura de espcie ornamentais, exemplo de pesca que
prejudica bastante os recifes de corais, uma vez que, alm das
espcies capturadas, destroem o ecossistema.
(
) A introduo de espcies invasoras pode se dar, por
exemplo, pelo despejo de gua de lastro ou soltura de
espcies exticas no ambiente natural.

RESPOSTA: B
COMENTRIO: A Taenia Solium pode causar tenase (solitria)
e cisticercose no homem; a larva cercaria do Schistosoma
mansoni penetra ativamente pela pele e mucosas. A
elefantase transmitida pela picada do mosquito do gnero
Culex. A febre amarela uma virose transmitida pela picada
do mosquito Aedes.
243. (Mackenzie 2010) As verminoses representam um grande
problema
de
sade,
principalmente
nos
pases
subdesenvolvidos. A falta de redes de gua e de esgoto, de
campanhas de esclarecimento pblico, de higiene pessoal e de
programas de combate aos transmissores, leva ao
aparecimento de milhares de novos casos na populao
brasileira.
Dentre as verminoses humanas causadas por nematdeos,
citam-se, corretamente,
a) tenase, ascaridase e ancilostomose.
b) filariose, ancilostomose e ascaridase.
c) esquistossomose, ascaridase e ancilostomose.
d) esquistossomose, filariose e oxiurose.
e) tenase, filariose e esquistossomose.
RESPOSTA: B
COMENTRIO: Filariose, ancilostomose ascaridase e oxiurose
so verminoses causadas por nematdeos. Tenase e
esquistossomose so causadas por platelmintos.

65

BIOLOGIA III

b) acumule na face inferior do caule e, por isso, determine um


crescimento maior dessa parte.
c) concentre na extremidade do caule e, por isso, iniba o
crescimento nessa parte.
d) distribua uniformemente nas faces do caule e, por isso,
iniba o crescimento de todas elas.
e) concentre na face inferior do caule e, por isso, iniba a
atividade das gemas laterais.

244. (Ufpb 2010)


Os seres vivos apresentam diversas
caractersticas que lhes permitem, por exemplo, apresentar
uma grande diversidade morfolgica e sobreviver em
diferentes ambientes.
Uma dessas caractersticas est ilustrada na figura a seguir,
que mostra a Dionaea muscipula, um tipo de planta insetvora,
que captura e digere insetos como forma de obteno de
energia.

O mecanismo pelo qual a D. muscipula captura o inseto pode


ser definido como:
a) reao.
b) catabolismo.
c) metabolismo.
d) canibalismo.
e) fototropismo.

RESPOSTA: A
COMENTRIO:O efeito de crescimento e curvatura do vegetal
em resposta a um estmulo ambiental denominado tropismo
e depende da distribuio desigual de auxinas. Esses
hormnios podem acelerar ou inibir o crescimento de um
rgo vegetal, dependendo da concentrao em que se
encontram no local onde atuam.
246. (Ufpb 2010) Uma prtica bastante conhecida na cultura do
abacaxi (Ananas comosus) no estado da Paraba o controle
qumico da diferenciao floral, realizado por volta do dcimo
segundo ms de cultivo. O mtodo, alm de antecipar e
homogeneizar a florao promove economia de mo de obra
no controle de pragas e na colheita.
Na prtica, consiste na adio de pedras de carbureto de
clcio na base das folhas que contm gua, resultando na
liberao de um gs que se difunde nos espaos entre as
clulas.
Com base no exposto, a funo de induo da florao
promovida pelo gs liberado durante o processo a mesma do
hormnio vegetal denominado:
a) Giberelina
b) Auxina
c) cido abscsico
d) cido Indol Actico
e) Etileno
RESPOSTA: E
COMENTRIO:O etileno um gs voltil que participa do
desenvolvimento da florao e desenvolvimento dos frutos de
bromlias. A florao e frutificao do abacaxi ocorrem ao
mesmo tempo nas plantas cultivadas expostas ao gs etileno,
favorecendo o manejo, a extrao e a preveno de pragas
que atacam os cultivares.

RESPOSTA: A
COMENTRIO: A captura de um inseto por uma planta
angiosperma uma reao denominada nastismo. Nesse tipo
de movimento, as folhas da Dionaea so tocadas por insetos e
reagem fechando os fololos. A armadilha movida por
mecanismos de deslocamento de gua entre as clulas
componentes dos vrtices dos fololos.
245. (Enem 2 aplicao 2010) A produo de hormnios vegetais
(como a auxina, ligada ao crescimento vegetal) e sua
distribuio pelo organismo so fortemente influenciadas por
fatores ambientais. Diversos so os estudos que buscam
compreender melhor essas influncias. O experimento
seguinte integra um desses estudos.

247. (Uemg 2010) A sabedoria popular prdiga em vrios


exemplos de atitudes que apresentam resultado satisfatrio,
mesmo sem o devido conhecimento biolgico que explique
corretamente aquele resultado. Uma dessas atitudes pode ser
observada nas fazendas, onde se costuma pendurar na
cozinha, sobre o fogo lenha, cachos de bananas verdes para
que elas amaduream mais depressa, o que realmente
acontece.
Utilizando seus conhecimentos sobre fisiologia vegetal e
considerando o fenmeno mencionado acima, s est
CORRETO afirmar que
a) o calor do fogo acelera as reaes qumicas necessrias
para o processo de amadurecimento das bananas.
b) a queima da lenha libera muito CO2, que acelera o processo
de fotossntese, levando ao amadurecimento rpido das
bananas.
c) a queima da madeira libera um hormnio gasoso, o etileno,
que provoca o amadurecimento dos frutos.
d) o calor do fogo impede o desenvolvimento de fungos e
outros parasitas que prejudicam o processo de
amadurecimento das bananas.

O fato de a planta do experimento crescer na direo


horizontal, e no na vertical, pode ser explicado pelo
argumento de que o giro faz com que a auxina se
a) distribua uniformemente nas faces do caule, estimulando o
crescimento de todas elas de forma igual.
66

d) o calor do fogo no impede o desenvolvimento de fungos


e outros parasitas que prejudicam o processo de
amadurecimento das bananas.
RESPOSTA: C
COMENTRIO: O amadurecimento dos frutos provocado
pelo etileno (C2H4), uma substncia gasosa produzida em
diversas partes da planta. A queima de madeira libera esse
gs.
248. (Udesc 2010) Os hormnios vegetais so substncias que
estimulam, inibem ou modificam os processos fisiolgicos da
planta. Eles podem agir distncia do seu local de sntese e
so especficos.
Associe a primeira coluna de acordo com a segunda.
( 1 ) Giberelina
( 2 ) Auxina
( 3 ) cido abscsico
( 4 ) Etileno
( 5 ) Citocinina
(
) Envelhecimento vegetal, queda das folhas
amadurecimento de frutos.
(
) Diviso celular e desenvolvimento de gemas laterais.
(
) Inibio da germinao de sementes e das gemas
durante condies desfavorveis.
(
) Alongamento de caule e estmulo formao de razes.
(
) Estmulo germinao de sementes.

a) I-1; II-3; III-5; IV-4; V-2.


b) I-2; II-4; III-3; IV-1; V-5.
c) I-3; II-5; III-2; IV-l; V-4.
d) I-4; II-1; III-5; IV-3; V-2.
e) I-5; II-2; III-4; IV-3; V-l.
RESPOSTA: D
COMENTRIO: A correlao exata entre os problemas
enfrentados pelos vegetais para superar as condies
ambientais adversas e as solues adaptativas esto
relacionadas corretamente na alternativa [D].
250. (Ufla 2010) Qual das estruturas a seguir pode secretar leos
essncias, taninos, compostos fenlicos ou outras substncias
utilizadas para a defesa da planta?
a) Acleos
b) Papilas
c) Espinhos
d) Tricomas
e) Estmatos
RESPOSTA: D
COMENTRIO: Os tricomas so estruturas filamentosas que
formam pelos que secretam diversas substncias que
procuram afugentar animais herbvoros e alguns parasitas,
como bactrias e fungos.

Assinale a alternativa que contm a sequncia correta, de


cima para baixo.
a) 4 3 5 1 2
b) 5 3 2 1 4
c) 5 4 3 2 1
d) 4 5 3 1 2
e) 3 5 4 2 1

251. (Ufla 2010) Tecido de revestimento do corpo vegetal, tecido


de conduo de gua e sais, e tecido de sustentao
constitudo de clulas vivas e paredes celulares espessadas
so, respectivamente:
a) epiderme, xilema e colnquima
b) epiderme, floema e esclernquima
c) periderme, xilema e esclernquima
d) periderme, floema e colnquima
e) periderma, colnquima e floema

RESPOSTA: D
COMENTRIO: O etileno promove o envelhecimento vegetal, a
queda das folhas e o amadurecimento de frutos; a citocinina
estimula a diviso celular e o desenvolvimento de gemas
laterais; o cido abscsico inibe a germinao de sementes e
das gemas em condies desfavorveis; a giberelina promove
o alongamento de caule e estimula a formao de razes; a
auxina estimula a germinao de sementes.

RESPOSTA: A
COMENTRIO: A epiderme um tecido vegetal de
revestimento formado por clulas vivas. O xilema um tecido
condutor da seiva inorgnica (mineral) e formado por clulas
alongadas e mortas. O colnquima um tecido vegetal de
sustentao mecnica constitudo por clulas vivas,
clorofiladas e dotadas de reforos de celulose nos vrtices
internos das paredes celulares.

249. (Ufjf 2010) Para a sua sobrevivncia, as plantas vasculares


precisam superar condies ambientais adversas. Alguns
problemas encontrados pelas plantas e as solues utilizadas
por elas para superar tais limitaes so apresentados a
seguir.

252. (UFRS) Em pesquisas desenvolvidas com eucalipto, 1.


constatou-se que a partir das gemas de um nico ramo podese gerar cerca de 200 000 novas plantas, em
aproximadamente 200 dias, enquanto os mtodos tradicionais
permitem a obteno de apenas cerca de cem mudas a partir
de um mesmo ramo. A cultura de tecidos feita a partir de:
a) clulas da epiderme.
b) clulas meristemticas.
c) clulas do sber.
d) clulas do esclernquima.
e) clulas do colnquima.

Problema:
I- Proteo contra agentes lesivos e contra a perda de gua
II- Sustentao
III- Preenchimento de espaos
IV- Transporte de materiais
V- Execuo de movimentos orientados
Soluo:
1) Esclernquima e colnquima
2) Fitormnios
3) Xilema e floema
4) Epiderme e sber
5) Parnquimas
A associao correta entre o Problema e a Soluo encontrada
pelas plantas :

RESPOSTA: B
COMENTRIO: As clulas meristemticas, por serem de
origem embrionria, permitem o desenvolvimento de mudas
muito mais rpido do que qualquer outra parte do vegetal.
253. (Ufpa) As plantas aquticas possuem um parnquima muito
desenvolvido que permitem a vida nesse tipo de ambiente.
Qual o tipo de parnquima citado?
a) Aqufero.
b) Aerfero.
67

c) Clorofiliano.
d) Amilfero.
e) Assimilador.
RESPOSTA: B
COMENTRIO: As plantas aquticas possuem o parnquima
aerfero bem desenvolvido, que permite a flutuao das razes
areas e at a respirao em ambientes facilmente inundados.

mais se encurvando em direo a luz. A luz no estimula a


produo das auxinas.
257. (FURG) As clulas epidrmicas das razes (pelos absorventes)
absorvem gua do solo, normalmente, quando:
a) a concentrao de sais das clulas for menor que a
concentrao de sais do solo.
b) a concentrao de sais das clulas for maior que a
concentrao de sais do solo.
c) a concentrao de sais das clulas for igual a concentrao
de sais do solo.
d) a concentrao de gua das clulas for maior que a
concentrao de gua do solo.
e) a concentrao de gua das clulas for igual concentrao
de gua do solo.

254. (Utpr) As roseiras possuem estruturas de proteo nos seus


caules. Como se denomina essa estrutura?
a) Espinhos.
b) Acleos.
c) Lenticelas.
d) Estmatos.
e) Ritidomas.
RESPOSTA: B
COMENTRIO: As roseiras possuem acleos que so
estruturas originadas da epiderme.

RESPOSTA: B
COMENTRIO: A absoro de gua do solo pelos vegetais
por osmose. A osmose funo da membrana plasmtica. Por
osmose, a gua passa de um meio hipotnico (menos
concentrado) para outro hipertnico (mais concentrado),
atravs de uma membrana semipermevel (membrana
plasmtica). Portanto, para que a planta consiga absorver
gua do solo a concentrao salina das clulas da raiz tem que
ser maior que a concentrao do solo. O solo deve ser
hipotnico e as clulas da raiz devem ser hipertnicas. (FURG)
As clulas epidrmicas das razes (pelos absorventes)
absorvem gua do solo, normalmente, quando:

255. (PUC-SP) Qual das alternativas abaixo apresenta um


associao INCORRETA entre uma estrutura vegetal e sua
funo?
a) Lenticela eliminao de nctar.
b) pice do caule produo de auxinas.
c) Estmato troca gasosa.
d) Esclernquima sustentao.
e) Xilema conduo da seiva bruta.
RESPOSTA: A
COMENTRIO: As lenticelas so aberturas no crtex do caule
que permitem a troca gasosa.

258. (FUVEST) As substncias orgnicas de que uma planta


necessita para formar os componentes de suas clulas so
a) sintetizadas a partir de substncias orgnicas retiradas do
solo.
b) sintetizadas a partir de substncias orgnicas retiradas do
solo e de substncias inorgnicas retiradas do ar.
c) sintetizadas a partir de substncias inorgnicas retiradas do
solo e do ar.
d) extradas de bactrias e de fungos que vivem em associao
com suas razes.
e) extradas do solo juntamente com a gua e os sais minerais.

256. (UNIRIO) A figura adiante ilustra um fenmeno que ocorre


com vegetais. A esse respeito, analise as seguintes afirmaes.

RESPOSTA: C
COMENTRIO: Os vegetais produzem os seus compostos
orgnicos (alimento) atravs da fotossntese. Usam para isso a
energia luminosa, gs carbnico e gua. Tanto o gs carbnico
como a gua so substncias inorgnicas. A gua retirada do
solo enquanto que o gs carbnico vem do ar atmosfrico
atravs dos estmatos.
I - O fenmeno mostrado decorrente da atividade das
auxinas.
II - A esse fenmeno d-se o nome genrico de fototropismo.
III - A planta cresce voltando-se na direo da luz porque esta
estimula a produo das auxinas.

259. (UFSCAR) O grfico mostra a transpirao e a absoro de uma


planta, ao longo de 24 horas.

A(s) afirmao(es) correta(s) (so):


a) somente a III.
b) somente a I e a II.
c) somente a I e a III.
d) somente a II e a III.
e) a I, a II e a III.

RESPOSTA: B
COMENTRIO: A afirmativa III est incorreta porque a planta
cresce voltando-se na direo da luz porque ocorre acumulo
de auxina na face no iluminada. Comi isso, essa parte cresce

A anlise do grfico permite concluir que:


68

a) quando a transpirao mais intensa, mais rpida a


subida da seiva bruta.
b) quando a transpirao mais intensa, os estmatos
encontram-se totalmente fechados.
c) das 22 s 6 horas, o lenho, sob tenso, dever ficar esticado
como se fosse um elstico, reduzindo o dimetro do caule.
d) no existe qualquer relao entre transpirao e absoro,
e um processo nada tem a ver com o outro.
e) das 12 s 16 horas, quando se observa maior transpirao,
pequena a fora de tenso e coeso das molculas de gua no
interior dos vasos lenhosos.
RESPOSTA: A
COMENTRIO: A ascenso da seiva bruta ou inorgnica
atravs dos vasos lenhosos do xilema depende da
transpirao. Quanto mais o vegetal transpira, mais rpida a
subida da seiva bruta pelos vasos lenhosos do xilema e,
consequentemente maior a absoro dessa seiva pelas
razes.
A afirmativa b est incorreta porque quando a transpirao
mais intensa, os estmatos encontram-se totalmente abertos
e no fechados. A afirmativa c est incorreta porque das 22 as
6 horas a transpirao praticamente nula e o lenho ou
xilema no se encontra cheio de gua sob tenso.
A afirmativa d est incorreta porque existe relao entre
transpirao vegetal e absoro de gua pelas plantas.
A afirmativa e est incorreta porque das 12 s 16 horas
quando se observa maior transpirao, grande a fora de
tenso e coeso das molculas de gua no interior dos vasos
lenhosos.

Assinale a alternativa que apresenta a sequncia numrica


correta, de cima para baixo, na coluna dois.
a) 2 - 3 - 1 - 3 - 2
b) 3 - 2 - 2 - 3 - 1
c) 3 - 1 - 3 - 1 - 2
d) 2 - 1 - 2 - 1 - 3
e) 1 - 2 - 3 - 2 1
RESPOSTA: B
COMENTRIO: Cutcula encontrada na epiderme das folhas
tem a funo de limitar a transpirao. O floema tecido onde
ocorre a translocao (transporte) da seiva elaborada.
Sacarose o principal carboidrato (acar) translocado
(transportado) pelo floema. Os estmatos realizam a
transpirao (perda de gua na forma de vapor). Os vegetais
absorvem atravs de suas razes os sais minerais na forma de
ons inorgnicos.
262. (FATEC) Um pesquisador, a fim de demonstrar a influncia de
hormnios no crescimento vegetal, realizou uma experincia
com plantas de mandioca tratadas com diferentes
concentraes de solues aquosas de auxinas A e B. Os
resultados obtidos esto representados na tabela a seguir.
Legenda:
Crescimento Sinal
acelerado
+
lento

normal
0
Observando os resultados, o pesquisador chegou seguinte
concluso:

260. (Ufpr 2011) Imagine que voc tenha recebido do seu


nutricionista a seguinte recomendao para uma dieta: ingerir
diariamente uma poro de tubrculos, razes tuberosas,
folhas verdes, frutos do tipo baga e sementes do tipo cariopse.
Qual das alternativas abaixo apresenta os vegetais que
atendem a dieta indicada?
a) Batata, cenoura, espinafre, uva e milho.
b) Beterraba, rabanete, couve-flor, abacate e arroz.
c) Mandioca, cebola, couve, pssego e semente de girassol.
d) Nabo, alho, brcolis, tomate e amendoim.
e) Batata-doce, alface, rcula, acerola e ervilha.
RESPOSTA: A
COMENTRIO: Tubrculos so caules subterrneos que
armazenam amido (batata-inglesa). Razes tuberosas como a
cenoura, a mandioca e a beterraba tambm contm reservas
nutritivas. O espinafre formado por folhas comestveis. A
baga um fruto com vrias sementes pequenas (uva, goiaba,
laranja,...) e o gro de milho um fruto seco indeiscente (no
se abre) que contm uma semente denominada cariobse.
261. (UFRS) Associe os processos fisiolgicos, listados na coluna
um, com as estruturas e elementos relacionados ao
movimento da gua e de solutos nas plantas, na coluna dois.

a) O efeito das auxinas A e B depende do rgo em que atuam.


b) A ao da auxina diretamente proporcional
concentrao de auxina usada.
c) A ao da auxina depende da espcie vegetal considerada
na experincia.
d) Os resultados obtidos independem do tipo de auxina
utilizada.
e) Os resultados obtidos com a auxina B so os mesmos que
foram obtidos apenas com gua.
RESPOSTA: A
COMENTRIO: A concentrao ideal para o crescimento da
raiz e do caule diferente. A raiz necessita de menos auxina
que o caule para crescer.
263. (UFV) prtica comum na poda das plantas a eliminao da
parte apical e/ou de suas folhas, conforme exemplificado em
A. Esse procedimento tem o efeito de retirar o local principal
de sntese de auxina, hormnio responsvel pelo sinal
correlativo da dominncia apical. Experimentalmente, se aps
a poda a gema apical ou folhas substitudas por um bloco de
gar, contendo auxina (setas), as gemas laterais inferiores
permanecem inibidas.

Coluna um
1. absoro
2. translocao
3. transpirao
Coluna dois
( ) cutcula
( ) floema
( ) sacarose
( ) estmato
( ) ons inorgnicos
69

Observe as situaes indicadas e assinale a opo que NO se


esperaria que ocorresse neste experimento:
a) V.
b) II.
c) IV.
d) III.
e) I.
RESPOSTA: C
COMENTRIO: Dominncia apical o fenmeno onde a auxina
produzida pela gema apical inibe o crescimento das gemas
laterais. Quando a gema apical retirada, as gemas laterais
crescem normalmente. Isso pode ser observado no
experimento I. Observe que experimentos II, III e V a gema foi
substituda pelo bloco de gua contendo auxina, no ocorreu
crescimento das gemas laterais. O nico experimento que no
poderia ocorrer o IV, porque a gema apical foi substituda
pelo bloco de gua contendo auxina. Portanto, as gemas
laterais no poderiam ter crescido.

265. (Uff 2011) Evitar ou controlar o impacto causado pelas


atividades humanas no meio ambiente uma preocupao
mundial. Como em muitas outras atividades, a fabricao de
produtos qumicos envolve riscos. Mas a indstria qumica,
apontada por muitos anos como vil nas agresses natureza,
tem investido em equipamentos de controle, em novos
sistemas gerenciais e em processos tecnolgicos para reduzir
ao mnimo o risco de acidentes ecolgicos. Quando se utilizam
sabes e detergentes nos processos de lavagem industriais
ou domsticos , os resduos vo para o sistema de esgoto.
Aps algum tempo, os resduos so decompostos por microorganismos existentes na gua. Diz-se, ento, que esses
compostos so biodegradveis. As estruturas apresentadas a
seguir so exemplos dessas substncias:

Com base nas estruturas observadas, pode-se afirmar que


a) os sabes so produtos de hidrlise cida de teres.
b) os detergentes so compostos orgnicos obtidos a partir da
hidrlise de gorduras animais e leos vegetais.
c) os detergentes mais comuns so sais de cidos sulfnicos de
cadeias curtas.
d) tanto os sabes quanto os detergentes derivados de cidos
sulfnicos so denominados catinicos.
e) na estrutura do sabo, a parte apolar interage com a
gordura e a parte polar com a gua.

QUIMICA I
264. (Uerj 2012)
Na indstria de alimentos, a anlise da
composio dos cidos carboxlicos no ramificados presentes
na manteiga composta por trs etapas:
- reao qumica dos cidos com etanol, formando uma
mistura de steres;
- aquecimento gradual dessa mistura, para destilao
fracionada dos steres;
- identificao de cada um dos steres vaporizados, em funo
do seu ponto de ebulio.

RESPOSTA: E
COMENTRIO:
Teremos:

O grfico a seguir indica o percentual de cada um dos steres


formados na primeira etapa da anlise de uma amostra de
manteiga:

Na amostra analisada, est presente em maior quantidade o


cido carboxlico denominado:
a) octanoico
b) decanoico
c) hexanoico
d) dodecanoico

266. (Uerj 2011) As bolas de futebol so feitas, atualmente, de


poliuretano, um polmero sinttico cuja obteno pode ser
representada pela seguinte equao qumica, na qual R e R
so cadeias de hidrocarbonetos:

RESPOSTA: C
COMENTRIO:
Teremos:
ster mais abundante: C8H16O2 (40%).

Pode-se observar, na molcula de poliuretano assim obtida, a


formao de um grupo correspondente seguinte funo
qumica:
a) cido
b) amida
c) lcool
d) aldedo

A reao de hidrlise fornece o cido e o etanol:

C8H16O2 H2O C2H6O C6H12O2


C6H12O2 : cido hexanoico

70

RESPOSTA: B
COMENTRIO:
Teremos:

e) presena de grupos funcionais da classe dos steres,


teres, e alcois de substncias qumicas volteis.

267. (Cesgranrio 2011)


O gnero Plocamium (Rhodophyta)
compreende mais de 40 espcies de algas marinhas, que so
amplamente distribudas em todos os oceanos. A investigao
qumica dessas algas tem sido comentada em artigos
cientficos, resultando no isolamento de um nmero
considervel de substncias orgnicas halogenadas. A figura a
seguir representa uma das substncias isoladas da alga citada.
DADOS: C 12; H 1; Br 80; O 16

RESPOSTA: E
COMENTRIO: Os flavorizantes utilizados para realar os
sabores e os aromas de alimentos, dentre outros fatores,
esto associados presena de grupos funcionais da classe
dos steres, teres, e alcois de substncias qumicas volteis,
as quais permitem esta extrao.
269. (Fuvest 2011) Em 2009, o mundo enfrentou uma epidemia,
causada pelo vrus A(H1N1), que ficou conhecida como gripe
suna. A descoberta do mecanismo de ao desse vrus
permitiu o desenvolvimento de dois medicamentos para
combater a infeco, por ele causada, e que continuam
necessrios, apesar de j existir e estar sendo aplicada a
vacina contra esse vrus. As frmulas estruturais dos princpios
ativos desses medicamentos so:

Analisando a estrutura da substncia, na figura acima, concluise que a massa molecular e as funes orgnicas presentes na
substncia so, respectivamente,
a) 240 u - cido carboxlico, cetona.
b) 310 u - aldedo, haleto.
c) 311 u - haleto, fenol.
d) 316 u - ster, haleto.
e) 320 u - ter, lcool.

Examinando-se as frmulas desses compostos, verifica-se que


dois dos grupos funcionais que esto presentes no oseltamivir
esto presentes tambm no zanamivir.
Esses grupos so caractersticos de
a) amidas e teres.
b) steres e alcois.
c) cidos carboxlicos e teres.
d) steres e cidos carboxlicos.
e) amidas e alcois.

RESPOSTA: B
COMENTRIO: A massa molecular 310 u e as funes
presentes so:

RESPOSTA: A
COMENTRIO: Examinando-se as frmulas desses compostos,
verifica-se que dois dos grupos funcionais que esto presentes
no oseltamivir esto presentes tambm no zanamivir: amidas
e teres.

268. (Uesc 2011) Com novas tcnicas de extrao das essncias


dos alimentos, pesquisadores captaram as molculas de
flavorizantes que podem dar o cheirinho apetitoso e o sabor a
produtos alimentcios industrializados. Basicamente, o
segredo est na extrao de odores de carnes, peixes,
legumes, e na transferncia aos alimentos. Assim, so criados
os sabores de caldos de carne e de galinha, o de salgadinhos, o
de sopas, os de temperos e os de sucos. Os flavorizantes
utilizados para realar os sabores e os aromas de alimentos,
dentre outros fatores, esto associados
a) extrao de flavorizantes apolares por meio de solues
diludas de cloreto de sdio.
b) ao alto ponto de ebulio e a presso de vapor prxima de
zero de determinadas substncias qumicas.
c) presena de

270. (Uesc 2011)

1,0g de flavorizante em 10,0m3 de ar, o

que corresponde a 1,0ppm (m/v).


d) velocidade de difuso de vapores no ar atmosfrico que
diretamente proporcional massa molar do flavorizante.

O composto orgnico, representado pela frmula estrutural,


um amburosdeo B isolado da Amburana cearenis, que possui

71

atividade antimalria. Em relao a essa substncia qumica,


correto afirmar:
a) um monossacardeo.
b) Apresenta cadeia carbnica acclica e saturada.
c) Possui apenas grupos funcionais da classe dos alcois.
d) Apresenta os grupos funcionais das classes dos steres e
dos teres.
e) Reage com cidos em soluo aquosa porque um
composto orgnico de propriedades bsicas.
RESPOSTA: D
COMENTRIO: O composto orgnico, representado pela
frmula estrutural, apresenta os grupos funcionais das classes
dos steres e dos teres:

Alternativa [C] est correta, j que o cido linoleico a forma


cis de um composto e a isomeria cis/trans s ocorre em
compostos insaturados.
Alternativa [D] est correta. Sabes so misturas de sais
alcalinos de cidos graxos.
272. (Fuvest 2011) Um slido branco apresenta as seguintes
propriedades:
I. solvel em gua.
II. Sua soluo aquosa condutora de corrente eltrica.
III. Quando puro, o slido no conduz corrente eltrica.
IV. Quando fundido, o lquido puro resultante no conduz
corrente eltrica.
Considerando essas informaes, o slido em questo pode
ser
a) sulfato de potssio.
b) hidrxido de brio.
c) platina.
d) cido cis-butenodioico.
e) polietileno.
RESPOSTA: D
COMENTRIO: As caractersticas apresentadas descrevem um
slido molecular.
Nos slidos moleculares os pontos do retculo cristalino so
ocupados por molculas.
Sabemos que as ligaes existentes entre os tomos de uma
molcula so ligaes covalentes e estas molculas so
eletricamente neutras.
Quando um slido (soluto) adicionado a um lquido
(solvente) se inicia um processo de destruio de sua estrutura
cristalina.
Lentamente as partculas do solvente atacam a superfcie do
retculo cristalino e comeam a remover as partculas que
formam o slido, cercando-as e arrastando-as para longe, ou
seja, ocorre uma disperso das partculas do slido cristalino.
Como consequncia desse fenmeno temos a destruio do
slido (soluto) e a alterao da estrutura do solvente que
carrega outras partculas deferentes das suas.
Este fenmeno ocorre com maior ou menor intensidade de
acordo com as foras de atrao entre as partculas
formadoras do solvente e do soluto e tambm das interaes
existentes entre as partculas do soluto entre si (soluto-soluto)
e do solvente entre si (solvente-solvente).

271. (Ufu 2011) Diariamente produtos novos so lanados no


mercado e muitos possuem como matria-prima leos ou
gorduras. Tais substncias, classificadas como lipdeos, podem
ser encontradas em tecidos animais ou vegetais e so
constitudas por uma mistura de diversos compostos qumicos,
sendo os mais importantes os cidos graxos e seus derivados.
Os cidos graxos so compostos orgnicos lineares que
diferem no nmero de carbonos que constitui a sua cadeia e,
tambm, pela presena de insaturaes.
Existem diversos cidos graxos conhecidos, sendo alguns
listados na tabela a seguir.
cido graxo

Nome sistemtico

Lurico
Palmtico
Palmito
leico
Esterico
Oleico
Linoleico

Dodecanoico
Hexadecanoico
cis-9 hexadecanoico
Octadecanoico
cis-9 octadecanoico
cis-9,
cis-12
Octadecanoico

Frmula
mnima
C12H24O2
C16H32O2
C16H30O2
C18H36O2
C18H34O2
C18H32O2

P . F.
(C)
44,8
62,9
0,5
70,1
16,0
-5,0

Considerando essas informaes, o slido em questo pode


ser o cido cis-butenodioico:

A partir das informaes acima e de seus conhecimentos de


qumica, assinale a alternativa incorreta.
a) O ponto de fuso do cido lurico menor que o ponto de
fuso do cido esterico, pois possui maior massa molar.
b) As molculas do cido esterico so apolares.
c) O cido linoleico um cido graxo insaturado.
d) O sabo uma mistura de sais alcalinos de cidos graxos.
RESPOSTA: A
COMENTRIO: Alternativa [A] est incorreta, pois o ponto de
fuso do cido lurico menor que o ponto de fuso do cido
esterico pelo fato dele possuir menor massa molar que o
cido lurico, o que pode ser confirmado pelo clculo das
massas molares a partir das frmulas mnimas. As
temperaturas de fuso dos compostos orgnicos so
influenciadas pela massa molar da molcula. Quanto maior a
massa molar, maior a temperatura de fuso.
Alternativa [B] est correta. Molculas de cidos carboxlicos
com mais de 10 carbonos so consideradas apolares.

273. (Ita 2011) Assinale a opo que apresenta o cido mais forte,
considerando que todos se encontram nas mesmas condies
de concentrao, temperatura e presso.
a) CH3COOH
b) CH3CH2COOH
c) (CH3)3CCOOH
d) CCH2COOH
e) C3CCOOH
RESPOSTA: E
COMENTRIO: O cido mais forte :

72

2. Verdadeira. Os ftalatos possuem uma poro de cadeia


cclica aromtica.

Quanto maior a quantidade de tomos de cloro ligados ao


carbono ligado carboxila, mais os eltrons das ligaes
covalentes so atrados na direo deles enfraquecendo o
tomo de oxignio da hidroxila que fica positivado e,
consequentemente, libera o hidrognio com mais facilidade,
ou seja, a fora cida aumenta.

3. Verdadeira. As massas molares do DEP, DBP e DEHP so,


-1
respectivamente, 222, 278 e 390 g. mol .

274. (Ufpr 2011)


Ftalatos correspondem a uma classe de
compostos que so empregados como plastificantes na
confeco de utenslios. A funo do plastificante conferir
flexibilidade, transparncia e durabilidade ao material.
Ftalatos so muito utilizados para conferir tais propriedades
em policloreto de vinila e em policarbonato. Pases da
comunidade europeia, Canad e EUA vm interrompendo o
uso de ftalatos, principalmente em artigos destinados a bebs,
devido a problemas de sade associados exposio a esse
produto. A figura abaixo indica a frmula bsica e os grupos
correspondentes aos ftalatos que recebem as siglas DEP, DBP
-1
e DEHP. (M (g.mol ): H = 1,008; O = 15,999; C = 12,01)

4. Verdadeira. Dentre os ftalatos DEP, DBP e DEHP, esperado


que o DEP seja o que apresente menor temperatura de
ebulio e seja o mais voltil, pois possui menor superfcie de
contato ou menor massa.
TEXTO PARA A PRXIMA QUESTO:
No esquema abaixo, esto representadas as duas etapas finais
do processo fermentativo em clulas musculares quando
submetidas a condies de baixa disponibilidade de oxignio.

Com base nas informaes fornecidas, considere as seguintes


afirmativas:
1. Os ftalatos possuem a funo orgnica ster.
2. Os ftalatos possuem uma poro de cadeia cclica
aromtica.
3. As massas molares do DEP, DBP e DEHP so,
-1
respectivamente, 222, 278 e 390 g. mol .
4. Dentre os ftalatos DEP, DBP e DEHP, esperado que o DEP
seja o que apresente menor temperatura de ebulio e seja o
mais voltil.

275. (Uerj 2011) O grupo funcional encontrado nos trs compostos


que participam das etapas representadas :
a) fosfato
b) hidroxila
c) carbonila
d) carboxilato

Assinale a alternativa correta.


a) Somente a afirmativa 2 verdadeira.
b) Somente as afirmativas 1 e 3 so verdadeiras.
c) Somente as afirmativas 1, 2 e 4 so verdadeiras.
d) Somente as afirmativas 3 e 4 so verdadeiras.
e) As afirmativas 1, 2, 3 e 4 so verdadeiras.

RESPOSTA: D
COMENTRIO:
Teremos:

RESPOSTA: E
COMENTRIO: Anlise das afirmativas:
1. Verdadeira. Os ftalatos possuem a funo orgnica ster.

276. (Ufpr 2010) O oseltamivir, representado na figura abaixo,


um antiviral que pode ser utilizado para o tratamento de
vrias cepas de Influenza, incluindo a H1N1.

73

podem produzir todas as cores sem que haja carotenoides. Na


ausncia de melanina negra, essas cores, ditas estruturais, no
aparecem.
O esquema a seguir mostra o incio da produo de melanina
em alguns organismos vivos:

Assinale a alternativa que especifica os grupos funcionais


presentes no oseltamivir:
a) ter, amina, amida, ster.
b) cido carboxlico, amina, amida, cetona.
c) ster, amina, lcool, aldedo.
d) Amida, ter, cetona, cido carboxlico.
e) ter, amida, amina, fenol.

Segundo os conceitos fundamentais da Qumica e o esquema


acima, pode-se afirmar que:
a) a dopaquinona no reage com NaOH.
b) a tirosina uma substncia aquiral.
c) no possvel realizar reaes de substituio aromtica
eletroflica na tirosina.
d) na tirosina o grupo funcional mais bsico o grupo amino.
e) na tirosina o grupamento mais cido a hidroxila.

RESPOSTA: A
COMENTRIO:
Teremos:

RESPOSTA: D
COMENTRIO: Na tirosina o grupo funcional mais bsico o
grupo amino.

277. (Fatec 2010) A anlise de certo composto orgnico, utilizado


como removedor de esmaltes de unhas, revelou a presena de
um composto de frmula molecular C4H8O2. Tal composto
pode ser
a) um ster.
b) uma cetona.
c) um hidrocarboneto.
d) uma amina.
e) uma nitrila.

279. (Ufrgs 2010) Ao serem descobertos e identificados, muitos


compostos orgnicos isolados de vegetais receberam nomes
em funo da espcie em que foram encontrados. Em alguns
casos, esse batismo do novo composto levou a nomes
peculiares, como, por exemplo, os casos do megaphone e do
clitoriacetal, que esto presentes nas razes da Aniba
megaphylla e da Clitoria macrophylla, respectivamente.
Observe a estrutura dos dois compostos referidos.

RESPOSTA: A
COMENTRIO: A frmula C4H8O2 pertence a um ster de cido
carboxlico, pois
(ster de cido carboxlico)
contm dois tomos de oxignio.

278. (Uff 2010) A melanina um pigmento produzido na pele de


mamferos. Por outro lado, sua funo na colorao da
plumagem das aves fundamental: sem ela o azul e o verde
no existiriam e o pavo seria todo branco.
No pavo, a luz decomposta por uma srie de microlminas
situadas nas brbulas das penas. O mesmo ocorre nos beijaflores. A eumelanina situada na profundidade absorve as
radiaes no refletidas. Igualmente, so as microlminas que
do origem s cores irisadas de numerosas aves: pombo,
corvo etc. Essas cores mudam segundo a orientao da luz que
incide na pena. A presena de carotenoides na pena permite
passar do azul e do violeta prpura, mas as microlminas

As funes orgnicas comuns s estruturas do megaphone e


do clitoriacetal so
a) cido carboxlico, ter e fenol.
b) lcool, cetona e ster.
c) lcool, ster e fenol.
d) lcool, cetona e ter.
e) cetona, ter e fenol.
RESPOSTA: D
74

COMENTRIO:

LOCAIS DE
ARMAZENAMENTO
freezer
ambiente
estufa
Pode-se concluir,
desconhecida a(o)
a) etanol.
b) acetona.
c) cicloexano.
d) cido ltico.

280. (Fgv 2010) A gripe suna, influenza A (H1N1), tem sido assunto
de destaque nos noticirios de TV, em 2009. O fosfato de
oseltamivir, comercializado com o nome de Tamiflu, o
medicamento recomendado pela OMS (Organizao Mundial
da Sade) para combater a gripe suna. A estrutura qumica do
oseltamivir apresentada na figura.

Na estrutura qumica do oseltamivir, so encontrados os


grupos funcionais orgnicos oxigenados:
a) cido carboxlico, aldedo e ter.
b) cido carboxlico, ter e amina.
c) amida, aldedo e ster.
d) amida, ter e ster.
e) amina, ter e ster.

ESTADOS FSICOS
Slido
lquido
lquido
corretamente,

que

substncia

RESPOSTA: D
COMENTRIO: De acordo com a tabela o cicloexano estar no
o
estado gasoso a 90 C.
o
O cido ltico estar no estado slido a 12 C (freezer) e no
o
estado lquido a 90 C (estufa).
282. (Ufg 2010) A conferncia do clima (COP-15) terminou em
18/12/2009 sem um acordo global para a reduo dos gases
do efeito estufa. Segundo o jornal O Popular (20/12/2009),
Gois produz 4% do total de gases do efeito estufa emitidos
pelo Brasil e, desse total, 9% oriundo da pecuria. O gs
emitido por esse setor da economia que tem maior impacto
no efeito estufa o
a) NH3
b) CO2
c) N2O
d) CO
e) CH4
RESPOSTA: E
COMENTRIO: O gs emitido por esse setor da economia que
tem maior impacto no efeito estufa o metano (CH4).
283. (Pucpr 2010) Recentemente, como medida antitabagista, o
estado de So Paulo adotou uma lei proibindo fumar em
diversos estabelecimentos. Um dos compostos mais ativos
presente no tabaco um alcaloide que apresenta dois tomos
de nitrognio em sua estrutura, a nicotina.

RESPOSTA: D
COMENTRIO: Na estrutura qumica do oseltamivir, so
encontrados os grupos funcionais orgnicos amida, ter e
ster:

TEMPERATURAS ( C)
FUSO EBULIO
Etanol
- 117
78
acetona
- 95
56
cicloexano
6
81
cido ltico
18
122
Pode-se afirmar com respeito a essa molcula que:
a) Ambos os tomos de nitrognio fazem parte do anel
aromtico.
b) O nitrognio (1) est num sistema aromtico e o nitrognio
(2) faz parte de um grupo amida.
c) A nicotina uma amina que apresenta um anel aromtico.
d) O nitrognio (2) refere-se a uma amina primria.
e) Por ser um composto aromtico, a nicotina responsvel
pelo odor caracterstico da fumaa do cigarro.
SUBSTNCIAS

281. (G1 - cftmg 2010) Uma substncia desconhecida colocada


0
em um freezer a 12 C.
0
Em seguida, fica exposta temperatura ambiente (25 C) e,
0
por fim, inserida em uma estufa a 90 C.
Os quadros seguintes mostram os estados fsicos nas situaes
consideradas anteriormente, bem como as temperaturas de
mudanas de fase para algumas substncias.

RESPOSTA: C
75

COMENTRIO: Analisando a figura percebemos que a nicotina


uma amina que apresenta um anel aromtico, ou seja, ressonncia.

c) a molcula possui um grupamento benznico que


hidroflico.
d) a cadeia intermediria da molcula separa espacialmente
suas duas extremidades e pode ser classificada como uma
amida.
e) um grupamento amina secundria caracteriza uma das
extremidades da molcula.

284. (Unemat 2010)


Certos vegetais produzem substncias
orgnicas denominadas alcaloides, que apresentam anis
heterocclicos contendo nitrognio. No organismo humano, o
consumo de alguns alcaloides pode causar dependncia
qumica, com perturbaes graves sade, podendo levar at
a morte.
Observe a estrutura da cocana e da nicotina, dois alcaloides,
representados abaixo.

A nicotina e a cocana pertencem funo orgnica:


a) amida.
b) cetona.
c) amina.
d) aldedo.
e) ter.

RESPOSTA: C
COMENTRIO: A nicotina e a cocana pertencem funo
orgnica amina:

RESPOSTA: D
COMENTRIO: Observamos que a cadeia intermediria da
molcula separa espacialmente suas duas extremidades e
pode ser classificada como uma amida:

286. (Pucpr 2010) Em meados da dcada de 1980, uma grande


ateno foi dada chamada Sndrome da Deficincia
Imunolgica Adquirida (AIDS). Apesar de ainda no haver cura,
possvel tratamento resultando na estabilizao da doena.
Das drogas utilizadas as que apresentam maior eficincia,
garantindo esse quadro de estabilidade, so a U75875 e a
azidotimidina (conhecida como AZT).

Entre os diversos grupos funcionais presentes nessa molcula,


podem-se destacar os grupos:
a) Cetona, cido carboxlico e nitrila.
b) ter, lcool e azida.
c) ter, alcino e ster.
d) ster, nitrila e aromtico.
e) Aldedo, cetona e lcool.

285. (Cesgranrio 2010) Um estudo recente concluiu que adicionar


lidocana a 1% injeo de radiocoloide usado em
mapeamento do linfonodo sentinela, em pacientes com
cncer de mama precoce, melhora significativamente o
conforto da paciente durante o procedimento. A frmula
estrutural da lidocana apresentada a seguir.

RESPOSTA: B
COMENTRIO: Observe as funes presentes na estrutura
dada:

Analisando a estrutura molecular da lidocana, quanto s suas


caractersticas e propriedades, conclui-se que
a) a lidocana apresenta frmula molecular C14H25N2O.
b) a molcula possui 2 carbonos tercirios, 4 secundrios e 8
primrios.

76

287. (G1 - cftmg 2010) O tamiflu, um dos medicamentos utilizados


no tratamento da Influenza A/H1N1 e popularmente
conhecida como gripe suna, apresenta a seguinte frmula
estrutural:

AgC (s)

Ag (aq) C (aq)

O equilbrio desloca para a esquerda e ocorre a formao de


precipitado, pois aumentar a concentrao de ons C na
soluo. A quantidade de slido aumentar.
II. Borbulha-se sulfeto de hidrognio gasoso na soluo por
certo perodo de tempo:
Teremos as seguintes reaes:
H2S(g) 2Ag (aq) 2C (aq) Ag2S(s) 2HC (aq)
ou
H2S(g) 2Ag (aq) 2C (aq) Ag2S(s) 2H (aq) 2C (aq)
ou
H2S(g) 2Ag (aq) Ag2S(s) 2H (aq)

.
Com base em sua estrutura, pode-se afirmar, corretamente,
que
a) contm as funes ter, cetona, amida e amina.
b) apresenta tomos de nitrognio de carter cido.
c) realiza ligaes de hidrognio intra e intermoleculares.
d) mais solvel em meios aquosos na presena de bases.

Ocorrer a formao de precipitado e a quantidade de slido


aumentar.
III. Adiciona-se certa quantidade de uma soluo aquosa 1
1
mol.L em nitrato de prata.
O nitrato de prata um sal muito solvel em gua:

AgNO3 Ag (aq) NO3 (aq) .

RESPOSTA: C
COMENTRIO: Teremos:

Com o aumento da concentrao de ctions Ag , o equilbrio

AgC (s)

Ag (aq) C (aq) ser deslocado para a

esquerda e a quantidade de slido aumentar.


289. (Ita 2012) Considere uma amostra aquosa em equilbrio a 60
C, com pH de 6,5, a respeito da qual so feitas as seguintes
afirmaes:
I. A amostra pode ser composta de gua pura.
II. A concentrao molar de H3O

QUMICA II

igual concentrao de

OH .
III. O pH da amostra no varia com a temperatura.
IV. A constante de ionizao da amostra depende da
temperatura.

288. (Ita 2012) So descritos trs experimentos (I, II e III)


utilizando-se em cada um 30 mL de uma soluo aquosa
saturada, com corpo de fundo de cloreto de prata, em um
bquer de 50 mL a 25 C e 1 atm:

V. A amostra pode ser uma soluo aquosa 0,1 mol L em

H2CO3 , considerando que a constante de dissociao do

I. Adiciona-se certa quantidade de uma soluo aquosa 1


1
mol.L em cloreto de sdio.
II. Borbulha-se sulfeto de hidrognio gasoso na soluo por
certo perodo de tempo.
III. Adiciona-se certa quantidade de uma soluo aquosa 1
1
mol.L em nitrato de prata.

H2CO3 da ordem de 1 107 .


Das afirmaes acima est(o) correta(s) apenas
a) I, II e IV.
b) I e III.
c) II e IV.
d) III e V.
e) V.

Em relao aos resultados observados aps atingir o


equilbrio, assinale a opo que apresenta o(s) experimento(s)
no(s) qual(is) houve aumento da quantidade de slido.
a) Apenas I
b) Apenas I e II
c) Apenas I e III
d) Apenas II e III
e) Apenas I, II e III

RESPOSTA: A
COMENTRIO: Anlise das afirmaes:
I. Correta. A amostra pode ser composta de gua pura ou por
uma soluo neutra.
A 60 C, simplificadamente, teremos:

RESPOSTA: E
COMENTRIO: O produto de solubilidade do cloreto de prata
muito baixo ( 1,0 10

10

), ou seja, este sal muito pouco

solvel.
I. Adiciona-se certa quantidade de uma soluo aquosa 1
1
mol.L em cloreto de sdio:

77

H3O OH

H2O H2O

CH3COONa H2O CH3COOH (K menor ) Na OH

K W [H3O ] [OH ] 1013

(meio mais bsico)

10pH 10pOH 1013


pH pOH 13

Cloreto de sdio:

pH pOH 13

NaC H2O H C

pH 6,5 (soluo neutra)

Na OH

(meio neutro)
Nitrito de sdio:

II. Correta. A concentrao molar de H3O

NaNO2 H2O HNO2 Na OH

igual

concentrao de OH .

(meio bsico)

Numa soluo neutra [H3O ] [OH ] 10

13

mol / L .

III. Incorreta. O pH da amostra varia com a temperatura, pois a


-14
25 C o valor de KW (10 ) diferente daquele comparado a 60
-13
C (10 ).
IV. Correta. A constante de ionizao da amostra depende da
temperatura.
V. Incorreta. Teremos:

H2CO3
(incio)

0,1 mol / L
x

(durante)

(equilbrio) 0,1 x

H HCO3
0

Como o cido nitroso possui uma constante de dissociao (K)


maior do que a do actico, conclumos que a soluo de
acetato de sdio mais bsica, ou seja, pHI > pHIII > pHII ou pHII
< pHIII < pHI.
291. (G1 - cftmg 2011) A fotossntese um processo bioqumico
que converte gs carbnico e gua em molculas de glicose.
Diferente do que aparenta, equivale a uma sequncia
complexa de reaes que acontecem nos cloroplastos.
Considere que esse fenmeno ocorra em uma nica etapa,
representada pela equao qumica, no-balanceada, e pela
curva da variao das concentraes em funo do tempo,
mostradas abaixo.

H2O( ) CO2(g)

C6H12O6(aq) O2(g)

( 0,1)
Ka

[H ] [HCO3 ]
[H2CO3 ]

1,0 107

x
x 1,0 10 4
0,1

[H ] 1,0 104 mol / L pH 4


pH 4 pH 6,5
Nessa situao, a constante de equilbrio ( K c ) para a reao

290. (Ita 2011)


A 25C, trs frascos (I, II e III) contm,
1
respectivamente, solues aquosas 0,10 mol L em acetato de
sdio, em cloreto de sdio e em nitrito de sdio.
Assinale a opo que apresenta a ordem crescente correta de
valores de pHx (x = I, II e III) dessas solues sabendo que as
constantes de dissociao (K), a 25C, dos cidos cloridrico
(HC), nitroso (HNO2) e actico (CH3COOH), apresentam a
seguinte relao:

, aproximadamente, igual a
a) 0,1.
b) 1,5.
c) 11.
d) 15.
RESPOSTA: C
COMENTRIO:
Teremos:

KHC > KHNO2 > KCH3COOH


a) pHI < pHII < pHIII
b) pHI < pHIII < pHII
c) pHII < pHI < pHIII
d) pHII < pHIII < pHI
e) pHIII < pHII < pHI

6H2O( ) 6CO2(g)

C6H12O6(aq) 6O2(g)

RESPOSTA: D
COMENTRIO: Como a constante de ionizao do cido
clordrico maior, conclumos que ele mais forte e est mais
ionizado.
Observe a hidrlise dos sais.
Acetato de sdio:

KC

78

[C6H12O6(aq) ]1[O2(g) ]6
[CO2(g) ]6

11 36
26

11,39

Alternativa [C] est correta, pois uma soluo tampo tem a


propriedade de estabilizar o pH.
Alternativa [D] est correta, pois a constante de equilbrio
dada pelo produto das concentraes dos produtos da reao,
divide-se pelo produto das concentraes dos reagentes,
todos elevados a seus coeficientes estequiomtricos.

292. (Ueg 2011) A presena de tampo fundamental para manter


a estabilidade de ecossistemas menores, como lagos, por
exemplo. ons fosfato, originrios da decomposio da matria
orgnica, formam um tampo, sendo um dos equilbrios
expressos pela seguinte equao:

H2PO4 (aq)

HPO24 (aq) H (aq)

Se no equilbrio foram medidas as concentraes molares

[H2PO4 ] 2 mol L1 ,

[HPO24 ] 1 mol L1

[H ] 0,2 mol L , o valor da constante de equilbrio :

294. (Cesgranrio 2010) A maior parte do CO2 produzido no


metabolismo celular, cerca de 80 %, transportada pelo

sangue, dissolvido na forma de on bicarbonato (HCO3 ), e o


restante, sob a forma de carboxiemoglobina (HbCO2). O CO2
reage rapidamente com a gua contida no sangue atravs da
seguinte reao:

a) 2
b) 0,2
c) 0,1
d) 0,01

anidrase
carbnica
H2O(l) + CO2(g)

[HPO24 ][H ]
[H2PO4 ]

1 0,2
0,1
2

293. (Ufu 2011)


Pessoas que passam por tratamento
quimioterpico e radioterpico tm um grande desconforto
causado pela baixa salivao (xerostomia). Uma soluo para
isso encontrada pelo uso da saliva artificial que nada mais
do que um lubrificante oral, cuja finalidade garantir que o
funcionamento da cavidade oral continue estvel. Na saliva o
sistema tampo mais importante o sistema cido
carbnico/bicarbonato. A concentrao do on bicarbonato
depende fortemente do fluxo salivar e a termodinmica desse
sistema complicada pelo fato de envolver o gs carbnico
dissolvido na saliva. O equilbrio completo simplificado (no
qual a enzima anidrase carbnica, que est presente na saliva,
catalisa a reao, formando dixido de carbono do cido
carbnico e vice-versa) pode ser escrito da seguinte forma:

1) CO2 g H2O I

2) H2CO3 aq H2O I

por ke

H2CO3

CO2 H2O

RESPOSTA: C
COMENTRIO: Observando o equilbrio:
H2O(l) + CO2(g)

H2CO3

H (aq) + HCO 3(aq)

A velocidade da reao direta (da esquerda para a direita)


pode ser dada por: v(direta) = K[H2O][CO2].
Conclumos que aumento da concentrao de CO2 no sangue
deslocaria o equilbrio para a direita, aumentando a
+
concentrao de H e diminuindo o pH, pois a velocidade da
reao direta seria maior.

I2CO3 (aq)
HCO3 aq H3 O aq

A partir do texto e de seus conhecimentos de qumica, assinale


a alternativa incorreta.
a) O aumento da concentrao do cido carbnico na reao 1
causar maior sada de dixido de
carbono da saliva.
b) A reduo da quantidade de gua na reao 2 facilita o
aumento da concentrao de on
bicarbonato.
c) A soluo tampo representada pelas reaes mantm o
pH, praticamente, inalterado.
d) O equilbrio qumico da primeira equao pode ser escrito

H (aq) + HCO 3(aq)

Assim, analisando a equao, conclui-se que o(a)


a) equilbrio representado pela equao influenciado
+
diretamente pela concentrao dos ons H , em virtude de
formar um sistema tampo, sofrendo uma grande variao de
pH.
b) H2CO3 um cido fraco, pois se dissocia completamente em
+
meio aquoso, produzindo altas concentraes de H .
c) aumento da concentrao de CO2 no sangue deslocaria o
+
equilbrio para a direita, aumentando a concentrao de H e
diminuindo o pH.
d) aumento da concentrao de ons bicarbonato no sangue
deslocaria o equilbrio para a esquerda, aumentando a
+
concentrao de ons H e diminuindo o pH.
e) anidrase carbnica apresenta ao cataltica por no alterar
os estados de transio na reao qumica.

RESPOSTA: C
COMENTRIO: K e

H2CO3

295. (Fgv 2010) Uma soluo aquosa de cido ciandrico, HCN, a 25


C tem pH = 5. Sabendo-se que a constante de ionizao desse
10
cido, a 25 C, 5 x 10 , ento essa soluo tem
concentrao de HCN, em g/L, igual a:
a) 2,7.
b) 5,4.
c) 8,1.
d) 10,8.
e) 13,5.

RESPOSTA: B
COMENTRIO:
Teremos:

RESPOSTA: B
COMENTRIO: Alternativa [A] est correta, pois de acordo
com o Princpio de Le Chatelier, o equilbrio de uma reao se
desloca de forma a repor uma substncia retirada ou a
consumir uma substncia adicionada, ento a adio de CO2
ir deslocar o equilbrio no sentido de consumi-lo.
Alternativa [B] est incorreta, pois a reduo de gua na
reao 2 ir deslocar o equilbrio para o lado dos reagentes, de
forma a repor o que foi retirado, logo diminuir a
concentrao de on bicarbonato.

-5

pH = 5 [H ] = 10 M
Ento,

79


H CN
HCN

105

[HCN]

KIONIZAO

II. Correta. O aumento da temperatura resulta em um


aumento da concentrao de ons hidroxila, pois o equilbrio
desloca para a direita.
III. Correta. A adio de ons amnio resulta numa reduo do
pH do sistema, pois desloca o equilbrio para a esquerda
diminuindo a concentrao dos ons OH .

105

[H ][CN ]
[HCN]

105 x 105
[HCN]
[HCN] 0,2 M
5 x 1010

1 mol HCN
0,2 mol HCN

27 g
m

m 5,4 g de HCN 5,4 g / L


296. (Uemg 2010) A presena do oxignio gasoso (O2) na gua se
deve, em parte, dissoluo do ar atmosfrico na gua. A
equao, a seguir, representa o equilbrio de dissoluo do O 2
em gua:
O2 (g)

O2 (aq)

Baseando-se nessas informaes e em outros conhecimentos


sobre o assunto, INCORRETO afirmar que
a) a solubilidade do O2 aumenta em temperaturas mais
elevadas.
b) a solubilidade do O2 baixa, devido ao carter polar da
molcula de gua.
c) a disponibilidade de O2 em guas superficiais influenciada
pela altitude.
d) a ligao entre tomos de oxignio ocorre por
compartilhamento de eltrons.

298. (Ufu 2010) Pesquis


adores descobriram que os atletas que
bebem gua somente para saciar a sede, sem considerar a
gua perdida pelo suor nos exerccios, apresentam resistncia
fsica menor quando comparados queles que consomem
gua em quantidade adequada. Por outro lado, atletas que
bebem mais gua tm mais disposio fsica e sua
temperatura corporal permanece perto da normal, pois o
corpo utiliza a gua para manter adequado o pH dos fluidos
biolgicos tambm, o equilbrio qumico das substncias do
organismo.
J o consumo de outras bebidas, como sucos e refrigerantes,
no provoca o mesmo efeito que o consumo de gua. Essas
bebidas, alm de outras substncias, contm acar, o qual
pode retardar a digesto, contribuir para o ganho de peso e
provocar oscilao nos nveis de glicose do organismo. Alm
disso, os refrigerantes do tipo cola possuem, em sua
composio, cido fosfrico (H3PO4) que pode prejudicar o
teor de clcio no corpo, pois os nions oriundos da ionizao
do cido fosfrico reagem com o clcio dos ossos,
enfraquecendo-os. O cido fosfrico um policido e suas
-3
constantes de ionizao so iguais a: K 1 = 7,5 x 10 , K2 = 6,2 x
-8
-13
10 e K3 = 5,0 x 10 . Se no tomarmos gua suficiente, os rins
tm de trabalhar mais arduamente para eliminar substncias
txicas do organismo. A quantidade adequada de gua
ingerida pode ser verificada, grosso modo, pela perda
acentuada da cor amarela caracterstica da urina.
A partir do texto, assinale a alternativa correta.
a) A primeira constante de ionizao do cido fosfrico
inversamente proporcional sua fora cida.
b) A falta de gua no organismo induz a doenas, porque as
espcies
vitais
mudam
suas
concentraes
e,
consequentemente, o equilbrio qumico no interior do corpo.
c) O pH do estmago permanece inalterado quando ingerimos
lquidos em grande quantidade durante nossas refeies.
d) A urina mais clara indica uma maior concentrao de
substncias eliminadas pelo organismo.

RESPOSTA: A
COMENTRIO: A solubilidade dos gases em lquidos diminui
com a elevao da temperatura.
297. (Fgv 2010) A reao da amnia com a gua representada na
equao:
NH3 (aq) + H2O

NH4 (aq) + OH (aq)


5

A constante de equilbrio a 5 C 1,45 x 10 e a 25 C 1,8 x


5
10 .

RESPOSTA: B
COMENTRIO: Anlise das alternativas:

Considere as seguintes afirmaes sobre o sistema em


equilbrio:
I. a reao qumica classificada como reao exotrmica;
II. o aumento da temperatura resulta em um aumento da
concentrao de ons hidroxila;
III. a adio de ons amnio resulta numa reduo do pH do
sistema.

a) Alternativa incorreta. A primeira constante de ionizao do


cido fosfrico maior.
b) Alternativa correta. A falta de gua no organismo induz a
doenas, porque as espcies vitais mudam suas concentraes
e, consequentemente, o equilbrio qumico no interior do
corpo.
c) Alternativa incorreta. O pH do estmago sofre alterao
quando ingerimos lquidos em grande quantidade durante
nossas refeies, pois o volume da soluo analisada aumenta.
d) Alternativa incorreta. A urina mais clara indica uma menor
concentrao de substncias eliminadas pelo organismo.

So corretas as afirmaes
a) I, II e III.
b) I e II, apenas.
c) I e III, apenas.
d) II e III, apenas.
e) III, apenas.
RESPOSTA: D
COMENTRIO: Anlise das afirmaes:
I. Incorreta. A reao qumica classificada como reao
endotrmica, pois a constante de equilbrio aumenta com a
elevao da temperatura.

299. (Ita 2010) Em cinco bqueres foram adicionados 50 mL de


uma soluo de referncia, que consiste de uma soluo
aquosa saturada em cloreto de prata, contendo corpo de
fundo, a 25C e 1 atm. A cada bquer, foram adicionados 50
mL de uma soluo aquosa diluda diferente, dentre as
seguintes:
I. Soluo de cloreto de sdio a 25C.
80

II. Soluo de Glicose a 25C.


III. Soluo de Iodeto de sdio a 25C.
IV. Soluo de Nitrato de prata a 25C.
V. Soluo de Sacarose a 50C.

medida que o gs cloro formado pela perturbao do

seguinte equilbrio na fase aquosa C + CO + H2O


C 2 +

2OH , a fase que contm o hidrocarboneto vai adquirindo a


colorao esverdeada tpica deste halognio. Considerando
que a cada um dos cinco frascos contendo quantidades
idnticas da mesma soluo de hipoclorito de sdio e de
hidrocarboneto lquido, foi adicionada uma das seguintes
solues: cloreto de sdio, hidrxido de sdio, cido actico,
cido clordrico e nitrato de amnio, todas com as mesmas
concentraes molares, haver a maior produo de gs cloro
no tubo ao qual foi adicionado a soluo de
a) Cloreto de sdio.
b) Hidrxido de sdio.
c) cido actico.
d) cido clordrico.
e) Nitrato de amnio.

Considere que o corpo de fundo permanece em contato com


as solues aps rpida homogeneizao das misturas
aquosas e que no ocorre formao de xido de prata slido.
Nestas condies, assinale a opo que indica a(s)
soluo(es), dentre as acima relacionadas, que altera(m) a
constante de equilbrio da soluo de referncia.
a) Apenas I, III e IV
b) Apenas I e IV
c) Apenas II e V
d) Apenas III
e) Apenas V
RESPOSTA: E
COMENTRIO: A equao da velocidade de uma reao
qumica, num processo elementar aA + bB produtos dada
a
b
por v = k[A] [B] .
Como a constante do equilbrio qumico depende das
constantes de velocidade da reao direta e inversa:
aA + bB
cC + dD
No equilbrio v1 = v2.
Podemos dizer, por comparao, que a causa da variao da
velocidade da reao direta ou inversa est relacionada com a
alterao da temperatura.
A relao entre a velocidade da reao e temperatura foi
descoberta em 1887 por Vant Hoff e, depois em 1889 por
Arrhenius.
Arrhenius estudou profundamente esta relao e deduziu a
equao conhecida pelo seu nome (equao de Arrhenius):

RESPOSTA: D
COMENTRIO: Para que ocorra maior produo de gs cloro

no tubo, o equilbrio C + CO + H2O


C 2 + 2OH dever
ser deslocado para a direita. Isto significa que deve ocorrer o

aumento de C , CO (efeito on comum) ou diminuio de

OH .
Dos compostos citados no enunciado, o HC desloca o

equilbrio para a direita, pois aumenta a concentrao de C e

diminui a de OH .
301. (Unemat 2010) O bicarbonato de sdio est presente em
medicamentos usados no combate azia estomacal. Uma vez
ingerido, o bicarbonato sofre as reaes representadas abaixo:

I HCO3 H2O

II H2CO3 CO2 H2O

Eat
Ae RT

Assinale a afirmativa incorreta.


+

a) O excesso de H no estmago neutralizado pelos ons OH


produzidos quando se faz uso do medicamento.
b) A reao I uma reao de hidrlise.
c) Quanto mais OH for consumido no estmago, menos gs
carbnico ser produzido ao final da reao.
d) A soluo aquosa de bicarbonato tem carter bsico
segundo Bronsted-Lowry.
e) O reagente da reao II sofre decomposio.

A = fator de frequncia de colises.


Eat = energia de ativao.
R = constante universal.
T = temperatura.

E
Observe que, pela equao, quando T aumenta, at
RT

diminui e consequentemente o valor de K tambm aumenta.


Ou seja, a temperatura um fator que atua no equilbrio
qumico. Como a soluo de sacarose est com o dobro da
o
temperatura (50 C) da soluo de referncia, conclumos que
a constante de equilbrio da soluo de referncia ser
alterada apenas por V.
300. (Unesp 2010) Uma das etapas finais do tratamento da gua
envolve o borbulhamento de cloro no efluente para
desinfeco. A substncia cloro encontrada como um gs
amarelo-esverdeado a 25 C e 1 atm. Pequenas quantidades
deste gs podem ser geradas em laboratrio de acordo com o
experimento ilustrado:

H2CO3 OH

RESPOSTA: C
COMENTRIO:
Teremos:

I HCO3 H2O

H2CO3 OH

II H2CO3 CO2 H2O


-

Como a diminuio da concentrao de OH no estmago, o


equilbrio I deslocar para a direita, aumentando a
concentrao de H2CO3 e consequentemente mais CO2 ser
liberado.
302. (Ita 2010) A 25C e 1 atm, uma amostra de 1,0 L de gua pura
foi saturada com oxignio gasoso (O2) e o sistema foi mantido
em equilbrio nessas condies. Admitindo-se comportamento
ideal para o O2 e sabendo-se que a constante da Lei de Henry
3
1
para esse gs dissolvido em gua igual a 1,3 10 mo L .
1
atm , nas condies do experimento, assinale a opo
CORRETA que exprime o valor calculado do volume, em L, de
O2 solubilizado nessa amostra.
3
a) 1,3 x 10
3
b) 2,6 x 10
3
c) 3,9 x 10
2
d) 1,6 x 10
81

e) 3,2 x 10

e) Trata-se de uma reao de sntese sem que haja variao


dos nmeros de oxidao dos elementos N e H.

RESPOSTA: E
COMENTRIO: De acordo com a lei de Henry a solubilidade do
oxignio diretamente proporcional sua presso parcial (S =
k.p).
S = k.p(O2)
-3
S = 1,3 x 10 x 1
-3
-1
S = 1,3 x 10 mol.L
-3

RESPOSTA: C
COMENTRIO: O rendimento da reao pode ser melhorado
pela retirada de amnia na medida em que esse produto
formado, pois a velocidade da reao inversa diminui.
305. (Ufg 2010) O grfico a seguir representa uma previso futura
sobre as condies ambientais na Terra.

Conclumos que 1 L de gua dissolve 1,3 x 10 mol de gs


-3
o
oxignio, ou seja, n(O2) = 1,3 x 10 mol; T = 25 C = 298 K; R =
-2
-1
-1
8,21 x 10 atm.L.K .mol , ento:
PV = n(O2)RT
-3
-2
1 x V = 1,3 x 10 x 8,21 x 10 x 298
-2
V = 3,18 x 10 L
303. (Ufrgs 2010)
Observe o grfico a seguir, no qual a
concentrao do reagente e do produto de uma reao
elementar A B foi monitorada em funo do tempo.

Com base nessa figura possvel concluir que, de hoje at os


prximos 1000 anos, a emisso de CO2
a) levar a um aumento do nvel do mar.
b) diminuir como resultado da estabilizao da concentrao
de CO2.
c) apresentar um mximo como consequncia do constante
aumento da temperatura na Terra e do nvel do mar.
d) se estabilizar juntamente com a temperatura.
e) diminuir na mesma proporo que aumenta o nvel do
mar.
Assinale a alternativa correta a respeito dessa reao.
a) A reao ultrapassa o equilbrio, porque a concentrao
final do produto maior do que a do reagente.
b) A velocidade de desaparecimento de A sempre igual
velocidade de formao de B.
c) A velocidade de formao de B torna-se maior que a
velocidade de desaparecimento de A aps o ponto em que as
curvas se cruzam.
d) A velocidade da reao direta igual velocidade da reao
inversa no ponto em que as curvas se cruzam.
e) A lei cintica para essa reao v = k [A] [B].

RESPOSTA: D
COMENTRIO: De acordo com a figura verificamos que a
emisso de CO2 se estabilizar juntamente com a
temperatura:

RESPOSTA: B
COMENTRIO: Como as curvas so simtricas, a velocidade de
desaparecimento de A sempre igual velocidade de
formao de B.
304. (Pucrj 2010) A equao a seguir descreve a reao de
formao de amnia a partir de matria prima abundante na
natureza (gases nitrognio e hidrognio). Essa reao
exotrmica e catalisada por ferro.
N2(g) + 3H2(g)
2NH3(g)
Assinale a afirmativa correta.
a) A presena do catalisador faz com que a reao se torne
endotrmica.
b) A equao da constante de equilbrio da reao K = [N2]
[H2] / [NH3].
c) O rendimento da reao pode ser melhorado pela retirada
de amnia na medida em que esse produto formado.
d) Na presso constante, o volume ocupado pela mistura
reacional tende a aumentar na medida em que o produto se
forma.

306. (Ufrgs 2010) A reao de sntese do iodeto de hidrognio,


representada a seguir, muito utilizada em estudos de
equilbrio qumico.
H 2 + I2

2HI

Essa reao atinge o equilbrio qumico aps um tempo


suficientemente longo. Depois de atingido o equilbrio, no
tempo t1, adicionada uma dada quantidade de H2.
Assinale o grfico que melhor representa a evoluo das
concentraes com o tempo.
82

a)

307. (G1 - cftmg 2010) A reao entre N2(g) e H2(g) para obteno de
NH3(g) conhecida como processo Haber-Bosch. Essa reao
exotrmica e favorecida em altas presses. Se o valor da
constante de equilbrio a 500 C 0,061, ento, o valor de Kc a
750 C
a) igual a 0,061.
b) igual a 0,091.
c) maior que 0,061.
d) menor que 0,061.

RESPOSTA: D
COMENTRIO: Reaes exotrmicas so desfavorecidas em
altas temperaturas, logo KC diminui com a elevao da
temperatura.

b)

308. (G1 - cftmg 2010) O grfico mostra a variao das


concentraes, em funo do tempo, temperatura
constante, para a seguinte reao:
X(g) + Y(g)

Z(g)

H > 0

c)

Sobre o grfico e reao, afirma-se que, no equilbrio, a(o)


a) aumento da concentrao de Z desloca o equilbrio para a
direita.
b) constante Kc permanece inalterada para qualquer valor de
temperatura.
c) uso de um catalisador afeta as concentraes de reagentes
e produtos.
d) mesma quantidade de matria de X e Y gasta para se
obter o produto Z.

d)

RESPOSTA: D
COMENTRIO:
Teremos:
X(g) +
Y(g)

e)

RESPOSTA: C
COMENTRIO:
Teremos:
Essa reao atinge o equilbrio qumico aps um tempo
suficientemente longo. Depois de atingido o equilbrio, no
tempo t1, adicionada uma dada quantidade de H2.

Z(g)
1,8
1,5
0
- 0,5
-0,5
+0,5
1,3
1,0
0,5
A mesma quantidade de matria de X e Y (0,5 M) gasta para
se obter o produto Z.
309.

(Ufrgs 2010) Observe a reao qumica que segue.


NO2 (g) + CO (g) CO2 (g) + NO (g)
Nessa reao, apenas o NO 2 (g) apresenta colorao
vermelho-castanha; os demais reagentes e produtos so
incolores.
Considere as seguintes afirmaes a respeito dessa reao,
que se realiza isotermicamente.
I. Ao se partir de uma mistura equimolar de NO2 e CO, chega-

83

se, aps um tempo suficientemente longo, a uma mistura com


a mesma colorao a que se chegaria caso se partisse de uma
mistura equimolar de CO2 e NO.
II. Ao se partir de uma mistura de dois mols de NO2 e 1 mol de
CO, chega-se a uma mistura com a mesma colorao a que se
chegaria caso se partisse de uma mistura equimolar dos
reagentes.
III. No equilbrio, as velocidades das reaes direta e inversa
so iguais e, portanto, a colorao do sistema no mais se
altera.
Quais esto corretas?
a) Apenas I.
b) Apenas II.
c) Apenas III.
d) Apenas I e II.
e) Apenas I e III.
RESPOSTA: E
COMENTRIO: Quando o equilbrio atingido a velocidade da
reao direta igual inversa, logo I (mistura equimolar) e III
esto corretas.
QUMICA III
310. (Uerj 2012) As curvas que descrevem as velocidades de
reao de muitas enzimas em funo das variaes das
concentraes de seus substratos seguem a equao de
Michaelis. Tal equao representada por uma hiprbole
retangular cuja frmula :

Vmax x S

K m S

10 u
10 u

, teremos:

Vmax 103

9 103 103
Vmax 103
10 103

Vmax 100 u

311. (Unicamp 2012) Glow sticks ou light sticks so pequenos


tubos plsticos utilizados em festas por causa da luz que eles
emitem. Ao serem pressionados, ocorre uma mistura de
perxido de hidrognio com um ster orgnico e um corante.
Com o tempo, o perxido e o ster vo reagindo, liberando
energia que excita o corante, que est em excesso. O corante
excitado, ao voltar para a condio no excitada, emite luz.
Quanto maior a quantidade de molculas excitadas, mais
intensa a luz emitida. Esse processo contnuo, enquanto o
dispositivo funciona. Com base no conhecimento qumico,
possvel afirmar que o funcionamento do dispositivo, numa
temperatura mais baixa, mostrar uma luz
a) mais intensa e de menor durao que numa temperatura
mais alta.
b) mais intensa e de maior durao que numa temperatura
mais alta.
c) menos intensa e de maior durao que numa temperatura
mais alta.
d) menos intensa e de menor durao que numa temperatura
mais alta.
RESPOSTA: C
COMENTRIO: A reao do ster com o perxido de
hidrognio libera a energia que excita o corante. Em uma
temperatura mais baixa, esta liberao de energia ser
favorecida e a luz apresentar maior durao.
Quanto maior o grau de agitao das molculas do corante,
maior a liberao de energia. Numa temperatura mais baixa, o
grau de agitao das molculas do corante menor e,
consequentemente, a luz emitida ser menos intensa.

K m S

V = velocidade de reao
velocidade mxima de reao
Vmax =
constante de Michaelis
Km =
concentrao de substrato
[S] =
A constante de Michaelis corresponde concentrao de

TEXTO PARA A PRXIMA QUESTO:


CONSTANTES
23
-1
Constante de Avogadro = 6,02 10 mol
4
-1
4
-1
Constante de Faraday (F) = 9,65 10 C mol = 9,65 10 J V
-1
mol
Volume molar de gs ideal = 22,4L(CNTP)
-19
Carga elementar = 1,602 10 C

V
substrato na qual v max .
2
Considere um experimento em que uma enzima, cuja
-3
constante de Michaelis igual a 9 10 milimonl/L, foi
incubada em condies ideais, com concentrao de substrato
-3
igual a 10 milimol/L. A velocidade de reao medida
correspondeu a 10 unidades. Em seguida, a concentrao de
substrato foi bastante elevada de modo a manter essa enzima
completamente saturada. Neste caso, a velocidade de reao
medida ser, nas mesmas unidades, equivalente a:
a) 1
b) 10
c) 100
d) 1000

Constante dos gases = 8,21 10


-1

-1

-2

-1

-1

atm L K mol = 62,4 mm hh L

K mol
-2
Constante gravitacional (g) = 9,81 m/s
DEFINIES
-2

Presso de 1 atm = 760 mmHg = 101 325 Nm = 760Torr


2 -2
1 J = 1 Nm = 1 kg m s
Condies normais de temperatura e presso (CNTP): 0C e
760 mmHg
Condies ambientes: 25C e 1 atm
Condies-padro: 25C e 1 atm; concentrao das solues =
-1
1 mol L (rigorosamente: atividade unitria das espcies);
slido com estrutura cristalina mais estvel nas condies de
presso e temperatura em questo.
(s) = slido; ( ) = lquido; (g) = gs; (aq) = aquoso; (CM) =
circuito metlico; (conc) = concentrado; (ua) = unidades
arbitrrias; [A] = concentrao da espcie qumica A em mol L

RESPOSTA: C
v 10 u
Vmax velocidade mxima de reao

Km 9 103 milimol / L

S 103

Vmax x S

milimol / L

Substituindo na equao:

84

MASSAS MOLARES

Elemento
Qumico

Nmero
Atmico

H
Li
C
N
O
F
Na
Mg
Al
Si
P

1
3
6
7
8
9
11
12
13
14
15

Massa
Molar
(g. mol
1
)
1,01
6,94
12,01
14,01
16,00
19,00
22,99
24,30
26,98
28,08
30,97

c) A partir das informaes, teremos:

Elemento
Qumico

Nmero
Atmico

S
Cl
K
Ca
Mn
As
Br
Ag
I
Pt
Hg

16
17
19
20
25
33
35
47
53
78
80

n n X nY
1
1
4
nY n X n n X n X n X
3
3
3
3
3n
nX n [X]
4
4V
1
1 3
1
1n
nY nX nY n nY n [Y]
3
3 4
4
4V

Massa
Molar
(g.mol1)
32,07
35,45
39,10
40,08
54,94
74,92
79,90
107,90
126,90
195,08
200,59

Anlise das afirmaes:


2

I. v K[X][Y]

vb K

1 n 3 n
9 n3

K
b
4 V 4 V
64 V 3

vc K

3 n 1 n
3 n3

K
c
4 V 4 V
64 V 3

Concluso :
v c v a vb
II. v K[X][Y]

Baseando nestas informaes, considere que sejam feitas as


seguintes afirmaes:
2
I. Se a lei de velocidade da reao for v = k[X].[Y] , ento vC <
vA < vB.
II. Se a lei de velocidade da reao for v = k[X].[Y], ento v B = vC
< vA.
III. Se a lei de velocidade da reao for v = k[X], ento t1/2(C) >
t1/2(b) > t1/2(a), em que t1/2 = tempo de meia-vida.
Das afirmaes acima, est(o) correta (s) apenas
a) I.
b) I e II.
c) II.
d) II e III.
e) III.

va K

1 n 1 n 1 n2
4 n2

K
va
K
2 V 2 V 4 V2
16 V 2

vb K

1n 3n
3 n2

K
4 V 4 V 16 V 2

vc K

3n 1n
3 n2

K
4 V 4 V 16 V 2

Concluso :
vb v c v a
III. v K[X] representa uma reao de primeira ordem.

RESPOSTA: B
COMENTRIO: a) A partir das informaes, teremos:

Nas reaes de primeira ordem os tempos de meia-vida so


iguais, pois a velocidade e a concentrao de X so lineares:

n: nmero total de mols; nX = nY .

n nY nY n 2nY nY

n n
1 n3
8 n2

va K
va
K

2V 2V
8 V3
64 V 2
2

312. (Ita 2012) Considere que a reao hipottica representada


pela equao qumica X+ Y Z ocorra em trs condies
diferentes (a, b e c), na mesma temperatura, presso e
composio total (nmero de molculas de X+Y), a saber:
a- O nmero de molculas de X igual ao nmero de
molculas de Y.
b- O nmero de molculas de X 1/3 do nmero de molculas
de Y.
c- O nmero de molculas de Y 1/3 do nmero de molculas
de X.

n nX nX n 2n X n X

va K

Observaes:
Grfico para uma reao de primeira ordem:

n
n
[X]
2
2V
n
n
[Y]
2
2V

b) A partir das informaes, teremos:

n n X nY
1
1
4
n X nY n nY nY nY
3
3
3
3
3n
nY n [y]
4
4V
1
1 3
1
1n
nX nY n X n n X n [X]
3
3 4
4
4V

Como loge = 2,303 log, multiplicando a equao:

85

d)

log[R]
t log[R]0
2,303
por 2,303, teremos:

2,303 log[R] 2,303


t 2,303 log[R]0
2,303
loge = n loge [R] = n[R]
n[R] Kt n[R]0

RESPOSTA: B
COMENTRIO: Com catalisador a velocidade da reao direta
aumenta, consequentemente, a concentrao de butanoato
de etila tambm, at atingir um valor constante.

No tempo de meia-vida a concentrao do reagente cai pela


metade, ou seja, [R] = [R]0/2. Substituindo na equao acima,
vem:

ln[R]0 ln[R] Kt
t() tempo de meia vida
[R]0
Kt()
2
[R]
Kt() n 0 Kt() n2 Kt() 0,693
[R]0
2
0,693
t()
(No var ia)
K
n[R]0 n

314. (Espcex (Aman) 2011) Considere a equao balanceada:


4 NH3 + 5 O2 4 NO + 6 H2O
-1

Admita a variao de concentrao em mol por litro (mol L )


do monxido de nitrognio (NO) em funo do tempo em
segundos (s), conforme os dados, da tabela abaixo:

313. (Uerj 2011) A fim de aumentar a velocidade de formao do


butanoato de etila, um dos componentes do aroma de
abacaxi, emprega-se como catalisador o cido sulfrico.
Observe a equao qumica desse processo:

-1

[NO] (mol L )
Tempo (s)

0
0

0,15
180

0,25
360

0,31
540

0,34
720

A velocidade mdia, em funo do monxido de nitrognio


(NO), e a velocidade mdia da reao acima representada, no
intervalo de tempo de 6 a 9 minutos (min), so,
-1
-1
respectivamente, em mol L min :
-2
-3
a) 2 10 e 5 10
-2
-2
b) 5 10 e 2 10
-2
-2
c) 3 10 e 2 10
-2
-3
d) 2 10 e 2 10
-3
-2
e) 2 10 e 8 10

As curvas de produo de butanoato de etila para as reaes


realizadas com e sem a utilizao do cido sulfrico como
catalisador esto apresentadas no seguinte grfico:
a)

RESPOSTA: A
COMENTRIO:
Teremos:

[NO] mol L1
Tempo (min)

b)

vNO

0,15

0,25

0,31

0,34

12

(0,31 0,25) mol.L1 0,06 mol.L1

0,02 mol.L1.min
(9 6) min
3 min

vNO 2,0 102 mol.L1.min1

4 NH3 5 O2 4 NO 6 H2O

c)

vmdia

vNO 2,0 102

0,5 102 mol.L1.min1


4
4

vmdia 5,0 103 mol.L1.min1


315. (Ita 2011) A figura mostra o perfil reacional da decomposio
de um composto X por dois caminhos reacionais diferentes, I e
II. Baseado nas informaes apresentadas nessa figura,
assinale a opo errada.

86

v O2
vNe

MNe
.
MO2

III. Afirmao correta. A densidade do gs no cilindro C1


maior que a densidade do gs no cilindro C2. A densidade
pode ser obtida pela seguinte relao: d

PM
.
RT

Como a presso e a temperatura so as mesmas nos dois


cilindros, conclumos que a densidade do gs oxignio maior
do que a do gs nenio.
a) O caminho reacional II envolve duas etapas.
b) A quantidade de energia liberada pelo caminho reacional I
igual a do caminho reacional II.
c) O composto K um intermedirio no processo reacional
pelo caminho II.
d) O caminho reacional I mostra que a decomposio de X de
primeira ordem.
e) O caminho reacional II refere-se reao catalisada.

IV. Afirmao incorreta. A distribuio de velocidades das


molculas contidas no cilindro C1 e menor que a das contidas
no cilindro C2.
TEXTO PARA A PRXIMA QUESTO:
A cintica qumica estuda as velocidades das reaes qumicas,
a rapidez com que os reagentes so consumidos e os produtos
so formados, o modo como as velocidades de reao
respondem a mudanas das condies ou presena de um
catalisador e a identificao das etapas pelas quais passa uma
reao. Ao se estudarem processos biologicamente
importantes, nota-se que um processo que parece ser lento
pode ser o resultado de muitas etapas rpidas. Processos
fotobiolgicos, tais como os responsveis pela fotossntese e
pelo lento desenvolvimento de uma planta, podem ocorrer em
cerca de 1 ps. O efeito da ligao de um neurotransmissor
ocorre aps, aproximadamente, 1 ms. Uma vez que o gene
tenha sido ativado, uma protena pode surgir em mais ou
menos 100 s. Em uma viso mais abrangente, algumas das
equaes de cintica qumica so aplicveis ao
comportamento de populaes inteiras de organismos. Essas
7
9
sociedades mudam em escalas de tempo de 10 - 10 s. A
velocidade inicial de uma reao qumica definida de acordo
a
com a seguinte frmula: r0 = K [ X0 ] , em que r0 a velocidade
inicial da reao, X0 a concentrao inicial de uma espcie X
e o valor a, a ordem da reao que tem constante de
velocidade igual a k.
Pode-se obter um grfico linear do logaritmo decimal da
velocidade inicial versus o logaritmo decimal da concentrao
inicial do reagente, por meio da seguinte expresso: log10r0 = K
+ alog10 [X0].

RESPOSTA: D
COMENTRIO: De acordo com a figura o caminho reacional I
ocorre em uma nica etapa e o caminho reacional II ocorre em
duas etapas. Ambos so exotrmicos, pois a entalpia dos
produtos menor do que a dos reagentes.
Como o caminho reacional I envolve uma nica etapa,
podemos concluir que:
2X 2T + Z
Ento,
2
v = k[X] (reao de segunda ordem)
316. (Ita 2011) Considere dois cilindros idnticos (C1 e C2), de
paredes rgidas e indeformveis, inicialmente evacuados. Os
cilindros C1 e C2 so preenchidos, respectivamente, com O 2(g)
e Ne(g) at atingirem a presso de 0,5 atm e temperatura de
50C. Supondo comportamento ideal dos gases, so feitas as
seguintes afirmaes:
I. O cilindro C1 contm maior quantidade de matria que o
cilindro C2.
II. A velocidade mdia das molculas no cilindro C1 e maior
que no cilindro C2 .
III. A densidade do gs no cilindro C1 maior que a densidade
do gs no cilindro C2.
IV. A distribuio de velocidades das molculas contidas no
cilindro C1 e maior que a das contidas no cilindro C2.
Assinale a opo que apresenta a(s) afirmao(es) correta(s).
a) Apenas I e III.
b) Apenas I e IV.
c) Apenas II.
d) Apenas II e IV.
e) Apenas III.

A tabela abaixo mostra dados da concentrao e da velocidade


inicial de reao de uma espcie X.
-1

[X0](mol L )
0,0001
0,001
0,01

-1

-1

r0(mol L s )
0,1
1,0
10,0

317. (Unb 2011) Acerca de reaes qumicas, assinale a opo


correta.
a) As reaes de segunda ordem apresentam

RESPOSTA: E
COMENTRIO: Anlise das afirmaes:

1
da velocidade
4

inicial das reaes de primeira ordem.


b) As velocidades das reaes qumicas aumentam medida
que a temperatura aumenta.
c) Em temperaturas mais altas, as colises entre molculas
diminuem.
d) O estado fsico dos reagentes no altera a velocidade de
uma reao qumica.

I. Afirmao incorreta. De acordo com a hiptese de Avogadro


(gases ideais) como os dois cilindros se encontram na mesma
temperatura e presso e ocupam o mesmo volume, possuem
a mesma quantidade de molculas.
II. Afirmao incorreta. A velocidade das molculas no cilindro
C1 menor do que no cilindro C2, pois a massa molar do O 2
maior do que a do Ne. As velocidades das partculas que
formam a mistura gasosa so inversamente proporcionais
raiz quadrada das respectivas massas molares ou moleculares.

RESPOSTA: B

87

COMENTRIO: As velocidades das reaes qumicas


aumentam medida que a temperatura aumenta, pois
aumenta, tambm, o grau de agitao das molculas e a
quantidade de choques. Esta observao est representada na
equao de Arrhenius, onde T representa a temperatura:

E
a
A e RT

318. (Ufrgs 2010) Considere a reao a seguir, que est ocorrendo


a 556 K.
2HI (g) H2 (g) + I2 (g)
Essa reao tem a sua velocidade monitorada em funo da
concentrao, resultando na seguinte tabela.
-1

[HI] (mol L )
0,01
0,02

A lei da velocidade depende da etapa mais lenta e ser dada


4+
2+
por: v = k[A ][B ], conclumos que a alternativa d est errada.
A reao de segunda ordem, pois a soma dos expoentes da
4+ 1 2+ 1
equao v = k[A ] [B ] (1 + 1) igual a 2.
320. (Ita 2010) Um recipiente contendo gs hidrognio (H2)
mantido temperatura constante de 0C. Assumindo que,
nessa condio, o H2 um gs ideal e sabendo-se que a
velocidade mdia das molculas desse gs, nessa
3
1
temperatura, de 1,85 x 10 m s , assinale a alternativa
CORRETA que apresenta o valor calculado da energia cintica
mdia, em J, de uma nica molcula de H2.
24
a) 3,1 x 10
24
b) 5,7 x 10
21
c) 3,1 x 10
21
d) 5,7 x 10
18
e) 2,8 x 10

-1 -1

Veloc. (mol L s )
-11
3,5 x 10
-11
14 x 10

RESPOSTA: D
COMENTRIO: No ensino mdio devemos considerar apenas a
energia cintica mdia translacional em uma direo, que
dada pela seguinte expresso matemtica:

Nessas condies, o valor da constante cintica da reao, em


-1 -1
L mol s ,
-11
a) 3,5 x 10 .
-11
b) 7,0 x 10 .
-9
c) 3,5 x 10 .
-7
d) 3,5 x 10 .
-7
e) 7,0 x 10 .

Calculando a massa de uma molcula de hidrognio, (H = 1,01;


H2= 2,02), teremos:
23
2,02 g 6,02 x 10 molculas H2
m 1 molcula de H2
-23
m = 0,335548 x 10 g
-24
-27
m = 3,36 x 10 g = 3,36 x 10 kg

RESPOSTA: D
COMENTRIO: Podemos notar que a concentrao de HI
dobra e a velocidade quadruplica, ento:
2
velocidade = k[HI] , a partir da segunda linha da tabela,
teremos:
-11
2
14 x 10 = k(0,02)
k=

14 x 1011
(2 x 102 )2

319. (Ita 2010)


genrica:
4+

2+

3+

Substituindo em (I), vem:


E=

3+

A + B A + B (etapa lenta)
4+
3+
3+
4+
A + B A + B (etapa rpida)
+
4+
3+
2+
C + B C + B (etapa rpida)
Com relao a este mecanismo, assinale a opo ERRADA.
+
4+
a) A reao global representada pela equao C + 2 A
3+
3+
C +2A .
2+
b) B catalisador.
3+
4+
c) B e B so intermedirios da reao.
+
4+
d) A lei de velocidade descrita pela equao v = k[C ][A ].
e) A reao de segunda ordem.

4+

2+

3+

3+

2+

2 -2

E = 5,7498 x 10 x 10 kg.m .s
-21
E = 5,75 x 10 J

321. (Ita 2010) Assinale a opo que apresenta a afirmao


CORRETA sobre uma reao genrica de ordem zero em
relao ao reagente X.
a) A velocidade inicial de X maior que sua velocidade mdia.
b) A velocidade inicial de X varia com a concentrao inicial de
X.
c) A velocidade de consumo de X permanece constante
durante a reao.
d) O grfico do logaritmo natural de X versus o inverso do
tempo representado por uma reta.
e) O grfico da concentrao de X versus tempo
representado por uma curva exponencial decrescente.
RESPOSTA: C
COMENTRIO: Para uma reao de ordem zero em relao a
um reagente X, teremos:

RESPOSTA: D
COMENTRIO: A partir das equaes fornecidas, somando
teremos a equao global:
A +B A +B
4+
3+
3+
4+
A +B A +B
+
4+
3+
2+
C +B C +B
4+
+
3+
3+
2A + C 2A + C

1
-27
3 2
x 3,36 x 10 x(1,85 x 10 )
2
-27

3,5 x 107

Considere o seguinte mecanismo de reao

1
mv2 (I)
2

v = k[X] v = k
Consequentemente, a velocidade de
permanece constante durante a reao.

(lenta)
(rpida)
(rpida)
(global)
4+

Como B entra com o reagente A e sai no final do processo


3+
com o produto C , podemos deduzir que ele um catalisador.
3+
4+
B e B so produtos intermedirios, pois so fabricados em
uma etapa e consumidos na seguinte.

consumo

de

322. (Pucpr 2010) Compostos naturais so muito utilizados na


denominada Medicina Naturalista. Povos indgenas
amaznicos h muito fazem uso da casca da Quina (Coutarea
hexandra) para extrair quinina, princpio ativo no tratamento
da malria. Antigos relatos chineses tambm fazem meno a
uma substncia, a artemisina, encontrada no arbusto Losna

88

(Artemisia absinthium), que tambm est relacionada ao


tratamento da malria.
Em estudos sobre a cintica de degradao da quinina por
cido, foram verificadas as seguintes velocidades em unidades
arbitrrias:
Quinina
-1
(mol L )
-4

1,0 x 10
-4
1,0 x 10
-4
0,5 x 10
-4
2,0 x 10

cido
-1
(mol L )

1
2
3

-3

-3

2,4 x 10
-3
9,6 x 10
-3
4,8 x 10
-3
1,2 x 10

A partir desses dados, pode-se concluir que a lei de velocidade


assume a forma
2
a) V = K [quinina]
b) V = K

quinina2
cido

-4

4 x 10 = K[1,0] [4,0] (1)


-4
x
y
32 x 10 = K[2,0] [4,0] (2)
Dividindo (2) por (1), teremos:
x
3
x
8=2 2 =2 x=3

cido2
quinina

Como x = 3, ento
-4
3
y
4 x 10 = K[1,0] [4,0] (3)
-4
3
y
32 x 10 = K[2,0] [4,0] (4)

RESPOSTA: D
COMENTRIO: A partir da anlise da segunda e da terceira
linha da tabela (de baixo para cima), teremos:
Quinina
-1
(mol L )

-4

410
-4
3210
-4
210

RESPOSTA: C
COMENTRIO: Observe a resoluo algbrica dada a seguir.
De acordo com a tabela e pela equao da velocidade, vem:

c) V = K2 [quinina]
2
d) V = K [quinina] [cido]
e) V = K

4,0
4,0
2,0

A equao de velocidade para essa reao pode ser escrita


x
y
como v = k [A] [O2] , em que x e y so, respectivamente, as
ordens de reao em relao aos componentes A e O2.
Assim, de acordo com os dados empricos obtidos, os valores
de x e y so, respectivamente,
a) 1 e 3.
b) 2 e 3.
c) 3 e 1.
d) 3 e 2.
e) 2 e 1.

Velocidade
(u.a.)

5,0 x 10
-2
1,0 x 10
-2
1,0 x 10
-3
2,5 x 10

1,0
2,0
1,0

cido
-1
(mol L )

Dividindo (3) por (4), teremos:

[4,0]y
(2,0)y
2
2 2.
[2,0]y
(2,0)y

Velocidade
(u.a.)

2 =2 y=1
Concluso, x = 3 e y = 1.

-4

-2

1,0 x 10 (dobrou)
-4
0,5 x 10

-3

1,0 x 10 (constante)
-2
1,0 x 10 (constante)

9,6 x 10 (dobrou)
-3
4,8 x 10

Outra resoluo:
Pela tabela percebemos que:
Quando a [A] fica constante, [O2] dobra e v tambm, logo o
expoente 1, ou seja, y = 1.
Quando [O2] fica constante, [A] dobra e v octuplica, logo o
expoente 3, ou seja, x = 3.

Como a concentrao de quinina dobrou e a velocidade


tambm, conclumos que o expoente da quinina 1.
A partir da anlise da primeira e da segunda linha da tabela
(de cima para baixo), teremos:
Quinina
-1
(mol L )
1,0
x
(constante)
1,0
x
(constante)

cido
-1
(mol L )
10
10

-4

-4

Velocidade
(u.a.)

CH4(g) + 2O2(g) CO2(g) + 2H2O(l)


1

-2

0,5 x 10
1,0
x
(dobrou)

324. (Ufc 2010) Metano (CH4) o gs produzido a partir da


biomassa, e a sua queima na indstria, para obteno de
energia trmica, corresponde seguinte reao:

10

-2

Se a velocidade de consumo do metano 0,01 mol min ,


assinale a alternativa que corretamente expressa o nmero de
moles de CO2 produzido durante uma hora de reao.
a) 0,3
b) 0,4
c) 0,5
d) 0,6
e) 0,7

-3

2,4 x 10
-3
9,6 x 10 (quadruplicou)

Como a concentrao do cido dobrou e a velocidade


quadruplicou, conclumos que o expoente do cido 2.
323. (Mackenzie 2010) Os dados empricos para a velocidade de
reao, v, indicados no quadro a seguir, foram obtidos a partir
dos resultados em diferentes concentraes de reagentes
iniciais para a combusto do gs A, em temperatura
constante.

EXPERIMENTO

-1

[A] (moL )

-1

[O2] (mo )

-1

RESPOSTA: D
COMNTRIO:
Teremos:

vCO2 vCH4 vCO2 0,01 mol.min1


0,01 mol 1 min
n 60 min n = 0,6 mol
TEXTO PARA A PRXIMA QUESTO:

-1

v (moL min )
89

O carbonato de clcio pode ser encontrado na natureza na


forma de rocha sedimentar (calcrio) ou como rocha
metamrfica (mrmore). Ambos encontram importantes
aplicaes industriais e comerciais. Por exemplo, o mrmore
bastante utilizado na construo civil tanto para fins
estruturais como ornamentais.
J o calcrio usado como matria-prima em diversos
processos qumicos, dentre eles, a produo da cal.
325. (Unesp 2010)
Considerando o papel do mrmore na
construo civil, de suma importncia conhecer a resistncia
desse material frente a desgastes provenientes de ataques de
cidos de uso domstico. Em estudos de reatividade qumica
foram realizados testes sobre a dissoluo do mrmore
(carbonato de clcio) utilizando cidos actico e clordrico. As
concentraes e os volumes utilizados dos cidos em todos os
experimentos foram iguais a 6 M e 15 mL, respectivamente,
assim como a massa de mrmore foi sempre igual a 1 g,
variando-se a temperatura de reao e o estado de agregao
do mrmore, conforme a tabela a seguir:

Uma certa fonte sonora, vibrando com frequncia de 480 Hz,


produz uma onda sonora que se desloca no ar, com velocidade
de mdulo 340 m/s, num referencial em que o ar est parado.
Se a mesma fonte vibrar com frequncia de 320 Hz, o mdulo
da velocidade de propagao da onda sonora correspondente,
no ar, em m/s,
a) 113,3.
b) 226,7.
c) 340,0.
d) 510,0.
e) 1020,0.
RESPOSTA: C
COMENTRIO: A velocidade de uma onda sonora em um meio
independe da frequncia. Portanto, mantidas as condies do
meio, a velocidade de propagao 340 m/s para qualquer
frequncia.
327. (IFSP 2011) O eco um fenmeno que consiste em se escutar
um som aps a reflexo da onda sonora emitida. Suponha que
voc e seu amigo encontrem-se separados 60 metros entre si,
e ambos a 40 metros de um obstculo A, perpendicular ao
solo, que pode refletir ondas sonoras. Se seu amigo emitir um
som, voc perceber que o intervalo de tempo entre o som
refletido e o som direto ser aproximadamente, em segundos,
de
Dado: velocidade do som no ar V = 340 m/s

Com relao aos experimentos, pode-se afirmar que


a) os experimentos 5 e 6 apresentam a mesma velocidade de
dissoluo do mrmore porque a superfcie de contato de um
slido no afeta a velocidade de uma reao qumica.
b) o experimento 1 ocorre mais lentamente que o 2, porque
quanto maior for a temperatura, menor ser a velocidade de
uma reao qumica.
c) o experimento 1 ocorre mais rapidamente que o 4, porque a
concentrao de ons H+ em 1 maior que no experimento 4.
d) o experimento 4 ocorre mais lentamente que o 5, porque
quanto maior for a temperatura, menor ser a probabilidade
de ocorrer colises efetivas entre os ons dos reagentes.
e) o experimento 3 ocorre mais lentamente que o 6, porque
quanto maior for a concentrao dos reagentes, maior ser a
velocidade de uma reao qumica.

a) 0,12.
b) 0,20.
c) 0,50.
d) 0,80.
e) 1,80.

RESPOSTA: C
COMENTRIO: Anlise das afirmativas:

RESPOSTA: A
COMENTRIO: A figura a seguir ilustra os dois percursos
realizados pelo som.

(a) Incorreta. O experimento 5 apresenta maior velocidade,


pois a superfcie de contato maior do que no experimento 6.
(b) Incorreta. O experimento 1 ocorre mais rapidamente do
que o 2, pois sua temperatura maior e afeta a velocidade da
reao.
(c) Correta. O experimento 1 ocorre mais rapidamente do que
o 4, pois Ka em 1 maior, ou seja, apresenta maior quantidade
de ons no sistema.
(d) Incorreta. O experimento 4 ocorre mais rapidamente do
que o 5, pois a temperatura da reao maior.
(e) Incorreta. O experimento 3 ocorre mais rapidamente do
que o 6, pois apresenta maior valor de Ka.
FSICA I
326. (Ufsm 2011) O som uma onda mecnica longitudinal
percebida por muitos seres vivos e produzida por vibraes
mecnicas, as quais podem ser induzidas por causas naturais,
como o vento. O objeto que, ao vibrar, produz um som,
chamado de fonte sonora.

Para o som direto a distncia percorrida distncia entre


voc e o seu amigo: Ddir = 60 m.
90

Para o som refletido temos:

d12 302 402

d1 50 m.

Como o ngulo de incidncia igual ao de reflexo:


d1 = d2 = 50 m.
A distncia percorrida , ento, Dreflet = d1 + d2 = 100 m.
A diferena de percurso :
D = 100 60 = 40 m.
O intervalo de tempo entre as recepes dos dois sons :

D 40

V
340

t 0,12 s.

328. (Unesp 2011) Um aluno, com o intuito de produzir um


equipamento para a feira de cincias de sua escola, selecionou
3 tubos de PVC de cores e comprimentos diferentes, para a
confeco de tubos sonoros. Ao bater com a mo espalmada
em uma das extremidades de cada um dos tubos, so
produzidas ondas sonoras de diferentes frequncias. A tabela
a seguir associa a cor do tubo com a frequncia sonora emitida
por ele:
Cor

vermelho

azul

roxo

Frequncia (HZ)

290

440

494

Na tabela de frequncias dadas:


fvermelho < fazul < froxo. Ento:
Lvermelho > Lazul > Lroxo
329. (Unesp 2011) Na gerao da voz humana, a garganta e a
cavidade oral agem como um tubo, com uma extremidade
aproximadamente fechada na base da laringe, onde esto as
cordas vocais, e uma extremidade aberta na boca. Nessas
condies, sons so emitidos com maior intensidade nas
frequncias e comprimentos de ondas para as quais h um n
(N) na extremidade fechada e um ventre (V) na extremidade
aberta, como ilustra a figura. As frequncias geradas so
chamadas harmnicos ou modos normais de vibrao. Em um
adulto, este tubo do trato vocal tem aproximadamente 17 cm.
A voz normal de um adulto ocorre em frequncias situadas
aproximadamente entre o primeiro e o terceiro harmnicos.

Considerando que a velocidade do som no ar 340 m/s, os


valores aproximados, em hertz, das frequncias dos trs
primeiros harmnicos da voz normal de um adulto so
a) 50, 150, 250.
b) 100, 300, 500.
c) 170, 510, 850.
d) 340, 1 020, 1 700.
e) 500, 1 500, 2 500.

Podemos afirmar corretamente que, os comprimentos dos


tubos vermelho (Lvermelho), azul (Lazul) e roxo (Lroxo), guardam a
seguinte relao entre si:
a) Lvermelho < Lazul > Lroxo.
b) Lvermelho = Lazul = Lroxo.
c) Lvermelho > Lazul = Lroxo.
d) Lvermelho > Lazul > Lroxo.
e) Lvermelho < Lazul < Lroxo.

RESPOSTA: E
COMENTRIO: A figura mostra o quinto harmnico.

RESPOSTA: D
COMENTRIO: Consideremos que os trs tubos estejam
emitindo harmnicos de mesma ordem.
A velocidade de propagao do som mesma, pois se trata do
mesmo meio, no caso, o ar.
Da equao fundamental da ondulatria:
v = f

v
. (I)
f

Somente para demonstrao, consideremos o n-simo


harmnico de um tudo aberto:

4L 4x0,17

0,136m

5
5
4
V
340
Como V f f
f5
2500 Hz

0,136
f
2500
f1 5
500Hz
5
5
f3 3f1 3x500 1500Hz
Observe que L 5.

O comprimento de cada fuso, como mostrado, igual a meio


comprimento de onda. Assim, para n fusos:
L= n

f5 2500 Hz

. (II)
2

Substituindo (I) em (II), vem:

Ln f
2

nv
.
2 f

Dessa expresso, conclumos que o comprimento do tubo


inversamente proporcional frequncia do som emitido.

330. (Udesc 2011) Dois tubos sonoros de um rgo tm o mesmo


comprimento, um deles aberto e o outro fechado. O tubo
fechado emite o som fundamental de 500 Hz temperatura
o
de 20 C e presso atmosfrica. Dentre as frequncias abaixo,
indique a que esse tubo no capaz de emitir.
a) 1000 Hz
91

b) 1500 Hz
c) 4500 Hz
d) 2500 Hz
e) 3500 Hz

Consultando a tabela acima, pode-se concluir que o som


produzido pelo violino era o da nota
-3
Dado: 1 ms = 10 s
a) d.
b) mi.
c) sol.
d) l.
e) si.

RESPOSTA: A
COMENTRIO: Os tubos fechados s ressoam para harmnicos
mpares. Se a frequncia fundamental 500Hz, ele ressoar
para: 1500Hz, 2500Hz, 3500Hz, 4500Hz, etc.
331. (UFPB 2011) Sonares so dispositivos frequentemente usados
na indstria naval. Os navios possuem sonares para detectar
obstculos no fundo do mar, detectar cardumes etc. Um
determinado sonar de um navio produz ondas sonoras
progressivas, com comprimento de onda de 2,0 m e
frequncia 200 Hz.

RESPOSTA: C
COMENTRIO: Analisando o grfico, notamos que o perodo
(T) ligeiramente maior que 2,5 ms.

Nesse caso, um obstculo a 80 m do sonar ser detectado pelo


navio em um intervalo de tempo de:
a) 0,4 s
b) 1,0 s
c) 1,2 s
d) 1,6 s
e) 2,0 s

Para o perodo de 2,5 ms, a frequncia seria: f =

1
1

400 Hz.
T 2,5 103

RESPOSTA: A
COMENTRIO:

frequncia

ligeiramente menor que 400 Hz, ou seja, est sendo emitida a


nota sol.

V f 2x200 400m / s
S
160
V
400
t 0,4s .
t
t
332. (UFPB 2011) Uma ambulncia, enquanto resgata um enfermo,
deixa a sirene ligada, a qual emite um sinal sonoro com
frequncia de 500 Hz. Um carro se aproxima da ambulncia
com uma velocidade de 85 m/s.
Nesse contexto, o condutor do carro ir escutar o som da
sirene com uma frequncia de:
a) 570 Hz
b) 625 Hz
c) 710 Hz
d) 735 Hz
e) 792 Hz

334. (Udesc 2010) Na superfcie de um planeta de massa M, um


pndulo simples de comprimento L tem perodo T duas vezes
maior que o perodo na superfcie da Terra. A acelerao,
devido gravidade neste planeta, :
2
a) 20,0 m/s
2
b) 5,0 m/s
2
c) 2,5 m/s
2
d) 15,0 m/s
2
e) 40 m/s
RESPOSTA: C
COMENTRIO: O perodo (T) de um pndulo simples para
oscilaes de pequena abertura ( < 10) dado pela
expresso:

T 2

RESPOSTA: B
COMENTRIO: Usando a expresso do efeito Doppler, vem:

f f0

Logo,

L
, sendo L o comprimento pendular e g a
g

intensidade do campo gravitacional local.

V VO
340 85
f 500
625Hz
V
340

Assim, sendo T e g, respectivamente, o perodo e a


intensidade do campo gravitacional no referido planeta,
temos:

333. (Fuvest 2010) Um estudo de sons emitidos por instrumentos


musicais foi realizado, usando um microfone ligado a um
computador. O grfico a seguir, reproduzido da tela do
monitor, registra o movimento do ar captado pelo microfone,
em funo do tempo, medido em milissegundos, quando se
toca uma nota musical em um violino.

T 2 L g'

T ' 2 g L

T'

g'
g

2 T

g'
.
g

Elevando os dois membros ao quadrado e substituindo o valor


2
o g = 10 m/s , vem:

1 g'
2

g = 2,5 m/s .
4 10

Nota
Frequncia (HZ)

d
262

r
294

mi
330

f
349

sol
388

l
440

335. (PUCRS 2010) O comprimento de uma corda de guitarra


64,0 cm.
Esta corda afinada para produzir uma nota com frequncia
igual a 246 Hz quando estiver vibrando no modo fundamental.
Se o comprimento da corda for reduzido metade, a nova
si
frequncia fundamental do som emitido ser:
494
92

a) 123 Hz
b) 246 Hz
c) 310 Hz
d) 369 Hz
e) 492 Hz

No tubo fechado, a ordem do harmnico dada pelo nmero


de meios fusos formados no seu interior.
Para o 1 harmnico, o comprimento do tubo corresponde a 1
meio fuso, ou seja, do comprimento de onda. Assim:

1
L 1 4 L.
4

RESPOSTA: E
COMENTRIO: Se a intensidade da trao (F) na corda no se
altera, a velocidade de propagao da onda tambm no se
altera, pois, de acordo com a equao de Taylor:
v=

Mas:
v = 1 f1 340 = 4 L (170) L = 0,5 m.

F
, sendo a densidade linear da corda.

Para o 3 harmnico, so formados 3 meios fusos: Ento:

No primeiro harmnico de uma corda, forma-se nela um nico


fuso, ou seja:

Mas:

L = 2 L.
2

v =3 f3 f3 =

Ento:
1 f1 = 2 f2 2 L f1 2

3
4L
L 3
.
4
3

L
f2 f2 = 2 f1 = 2 (246) f2
2

OBS: o gabarito oficial da UDESC d como resposta correta a


opo [A]. Se no houve enganos por parte da banca
examinadora, talvez ela tenha raciocinado da seguinte forma:
como o tubo fechado s emite harmnicos mpares, a
sequncia crescente de frequncias a dada a seguir:

= 492 Hz.
336. (Udesc 2010) Determine a velocidade de propagao da onda
para um fio de ao de 80,0 cm de comprimento e 200,0 g de
massa, que mantido tracionado pelas extremidades fixas.
Nesse fio originam-se ondas mecnicas estacionrias,
formando 5 (cinco) ns, quando excitado por uma fonte de
onda de 80,0 Hz. Assinale a alternativa correta, em relao ao
contexto.
a) 16,0 m/s
b) 25,6 m/s
c) 32,0 m/s
d) 12,8 m/s
e) 8,0 m/s
RESPOSTA: C
COMENTRIO: Observe a onda estacionria com 5 ns.

v
v
3v 3(340)

f3 = 510 Hz.
3 4L 4L 4(0,5)
3

1 harmnico: f1 = 170 Hz;


3 harmnico: f3 = 3 f1 = 510 Hz;
5 harmnico: f5 = 5 f1 = 850 Hz.
Como o 5 harmnico a 3 possibilidade, ela considerou a
resposta como f3 = 850 Hz.
338. (Ufpb 2010) Em um trecho reto de determinada estrada, um
fusca move-se do ponto A para o ponto B com velocidade de
20 m/s. Dois outros carros esto passando pelos pontos A e B,
com velocidade de
20 m/s, porm com sentido contrrio ao do fusca, conforme
ilustrado na figura a seguir. Nesse momento, o motorista do
fusca comea buzinar e o som emitido pela buzina tem
frequncia f.

2 80 40 cm 0,4 m
V f 0,4 80 32 m / s
Denominando as frequncias ouvidas pelos motoristas dos
carros que passam pelos pontos A e B de fA e fB ,
respectivamente, correto afirmar que
a) fA = fB > f
b) fA = fB < f
c) fA > f > fB
d) fA < f < fB
e) fA = fB = f

337. (Udesc 2010) A frequncia fundamental de um tubo de rgo


fechado igual a 170,0 Hz. O comprimento do tubo fechado e
a frequncia do terceiro harmnico so, respectivamente:
a) 0,5 m e 850 Hz
b) 1,0 m e 850 Hz
c) 1,0 m e 510 Hz
d) 0,5 m e 510 Hz
e) 2,0 m e 340 Hz

RESPOSTA: D
COMENTRIO: O efeito Doppler nos garante que, quando
ocorre aproximao relativa entre a fonte e o ouvinte, a
frequncia percebida maior que a real, sendo menor quando
ocorre afastamento relativo entre ambos.

RESPOSTA: D
COMENTRIO: Dados: f1 = 170 Hz; v = 340 m/s.
A figura mostra os dois harmnicos citados.

Assim: fA f e fB f

fA f fB.

339. (Fgvrj 2010) A avaliao audiolgica de uma pessoa que


apresentava dificuldades para escutar foi realizada
determinando-se o limiar de nvel sonoro de sua audio
(mnimo audvel), para vrias frequncias, para os ouvidos
93

direito e esquerdo separadamente. Os resultados esto


apresentados nos grficos abaixo, onde a escala de frequncia
logartmica, e a de nvel sonoro, linear.

A partir desses grficos, pode-se concluir que essa pessoa


a) no escuta um sussurro de 18 dB, independente de sua
frequncia.
b) percebe o som da nota musical l, de 440 Hz, apenas com o
ouvido esquerdo, independente do nvel sonoro.
c) surda do ouvido esquerdo.
d) escuta os sons de frequncias mais altas melhor com o
ouvido direito do que com o esquerdo.
e) escuta alguns sons sussurrados, de frequncia abaixo de
200 Hz, apenas com o ouvido direito.

e) Correta. Interpretando sussurros como sons de nvel sonoro


abaixo de 15 dB, frequncias abaixo de 200 Hz, apenas o
ouvido direito escuta.
340. (PUCRS 2010) Em relao s ondas sonoras, correto afirmar:
a) O fato de uma pessoa ouvir a conversa de seus vizinhos de
apartamento atravs da parede da sala um exemplo de
reflexo de ondas sonoras.
b) A qualidade fisiolgica do som que permite distinguir entre
um piano e um violino, tocando a mesma nota, chamada de
timbre e est relacionada com a forma da onda.
c) Denominam-se infrassom e ultrassom as ondas sonoras
cujas frequncias esto compreendidas entre a mnima e a
mxima percebidas pelo ouvido humano.
d) A grandeza fsica que diferencia o som agudo, emitido por
uma flauta, do som grave, emitido por uma tuba, a
amplitude da onda.
e) A propriedade das ondas sonoras que permite aos
morcegos localizar obstculos e suas presas denominada
refrao.
RESPOSTA: B
COMENTRIO: a) Errada. O fenmeno predominante nesse
caso a difrao do som.
b) Correta. por isso que o timbre conhecido como a cor do
som. Pois, assim como uma cor pode ser ou no agradvel
aos nossos olhos, um timbre pode ser ou no agradvel aos
nossos ouvidos. O timbre uma caracterstica individual de
cada fonte sonora, de cada instrumento.
c) Errada. Infrassom e ultrassom so as ondas sonoras de
frequncia abaixo e acima das frequncias mnima e mxima
percebidas pelo ouvido humano, respectivamente (20 Hz e
20.000 Hz)
d) Errada. A grandeza que diferencia um som agudo (alto) de
um som grave (baixo) a frequncia.
e) Errada. A propriedade em questo e a reflexo.

RESPOSTA: E
COMENTRIO:

341. (Uff 2012) Afigura abaixo representa um modo de vibrao de


uma corda presa nas suas extremidades.

O grfico nos d a menor intensidade sonora que cada ouvido


da pessoa pode perceber, ou seja: somente so escutados
sons com intensidades acima da linha do grfico para cada
ouvido. Por exemplo, para a frequncia de 1.000 Hz, o ouvido
direito comea a ouvir a partir da intensidade de 63 dB e o
esquerdo, a partir de 38 dB. Portanto, para frequncias acima
de 200 Hz, ele ouve melhor com o ouvido esquerdo do que
com o ouvido direito. Para frequncia abaixo de 200 Hz, ele
ouve melhor com o ouvido direito do que com o esquerdo.

Marque a alternativa que quantifica corretamente as


velocidades dos pontos 1, 2 e 3 da corda no instante em que
ela passa pela configurao horizontal.
a) v1 v 2 v3 0
b) v1 v 2 v3 0
c) v1 v 2 v3 0

Assim, analisemos as opes:


a) Errada. Como mostra o grfico, h uma pequena faixa onde
a linha de 18 dB est acima dos dois grficos, portanto os dois
ouvidos podem escutar um sussurro de 18 dB.
b) Errada. Um som de frequncia 440 Hz o ouvido esquerdo
escuta a partir de 28 dB e, o direito, a partir de 41 dB.
c) Errada.
d) Errada.

d) v1 v3 0; v 2 0
e) v1 v3 0; v 2 0
RESPOSTA: D
COMENTRIO: Observamos na figura a formao de uma onda
estacionria com quatro ns e trs ventres, onde os pontos 1 e
3 representam dois ventres consecutivos, e o ponto 2 um n.
94

COMENTRIO: Primeiro analisemos a corda. A velocidade de


propagao das ondas na corda dada pela equao

m 0,001

5 104 kg / m
L
2

(densidade linear de

massa da corda)
Calculando a velocidade de propagao da onda na corda,
temos:
O n de uma onda estacionria no oscila, permanecendo
sempre em repouso, ou seja, V2 0.

80

Como os pontos 1 e 3 representam ventres consecutivos, suas


oscilaes so opostas, ou seja, se o ponto 1 estiver subindo o
ponto 2 estar descendo, e vice-versa.

5 104

400m / s

No modo fundamental de vibrao da corda, temos:

2L 4,0m
2

Por outro lado: V f 400 4f f 100Hz


O

som

produzido

ter

comprimento

de

onda:

V f 330 100 3,3m


Ou seja: V1 V3 ou V3 V1
342. (Ufpr 2012) Uma cerca eltrica foi instalada em um muro
onde existe um buraco de forma cilndrica e fechado na base,
conforme representado na figura. Os fios condutores da cerca
eltrica esto fixos em ambas as extremidades e esticados sob
uma tenso de 80 N. Cada fio tem comprimento igual a 2,0 m
e massa de 0,001 kg. Certo dia, algum tocou no fio da cerca
mais prximo do muro e esse fio ficou oscilando em sua
frequncia fundamental. Essa situao fez com que a coluna
de ar no buraco, por ressonncia, vibrasse na mesma
frequncia do fio condutor. As paredes do buraco tm um
revestimento adequado, de modo que ele age como um tubo
sonoro fechado na base e aberto no topo. Considerando que a
velocidade do som no ar seja de 330 m/s e que o ar no buraco
oscile no modo fundamental, assinale a alternativa que
apresenta corretamente a profundidade do buraco.

O tubo fechado. Portanto, H

3,3

0,825m .
4
4

343. (Uel 2011) Aps ter afinado seu violo utilizando um diapaso
de 440 Hz, um msico notou que o quarto harmnico da corda
L do instrumento emitia um som com a mesma frequncia do
diapaso.
Com base na observao do msico e nos conhecimentos de
ondulatria, considere as afirmativas a seguir.
I. O comprimento de onda da onda estacionria formada na
corda, no quarto harmnico, igual metade do
comprimento da corda.
II. A altura da onda sonora emitida no quarto harmnico da
corda L diferente da altura da onda emitida pelo diapaso.
III. A frequncia do primeiro harmnico da corda L do violo
110 Hz.
IV. O quarto harmnico da corda corresponde a uma onda
estacionria que possui 5 ns.
Assinale a alternativa correta.
a) Somente as afirmativas I e II so corretas.
b) Somente as afirmativas II e IV so corretas.
c) Somente as afirmativas III e IV so corretas.
d) Somente as afirmativas I, II e III so corretas.
e) Somente as afirmativas I, III e IV so corretas.
RESPOSTA: E
COMENTRIO: I. Correta.
Para um harmnico de ordem n, o comprimento de onda em
relao ao comprimento da corda :

a) 0,525 m.
b) 0,650 m.
c) 0,825 m.
d) 1,250 m.
e) 1,500 m.

n
2L
L n
.
2
n

Para o quarto harmnico:

RESPOSTA: C

2L
4

L
.
2

II. Incorreta.
Ondas sonoras de mesma frequncia tm a mesma altura.
95

III. Correta.
Para um harmnico de ordem n, a frequncia, em relao do
primeiro harmnico :

fn nf1.
Para o quarto harmnico:

f4 4f1 440 4f1 f1 110 Hz.


IV. Correta.
Como no violo os extremos so fixos, para um harmnico de
ordem n, a onda estacionria na corda apresenta n ventres e
n+1 ns. Portanto, para o quarto harmnico so 5 ns, como
mostra a figura abaixo.

344. (Udesc 2011) Dois tubos sonoros de um rgo tm o mesmo


comprimento, um deles aberto e o outro fechado. O tubo
fechado emite o som fundamental de 500 Hz temperatura
o
de 20 C e presso atmosfrica. Dentre as frequncias abaixo,
indique a que esse tubo no capaz de emitir.
a) 1500 Hz
b) 4500 Hz
c) 1000 Hz
d) 2500 Hz
e) 3500 Hz

4L 4x0,17

0,136m

5
5
4
V
340
Como V f f
f5
2500 Hz

0,136
f
2500
f1 5
500Hz
5
5
f3 3f1 3x500 1500Hz
Observe que L 5.

f5 2500 Hz
346. (Epcar (Afa) 2011) Um diapaso de frequncia conhecida igual
a 340 Hz posto a vibrar continuamente prximo boca de
um tubo, de 1 m de comprimento, que possui em sua base um
dispositivo que permite a entrada lenta e gradativa de gua
como mostra o desenho abaixo.

RESPOSTA: C
COMENTRIO: Os tubos fechados s ressoam para harmnicos
mpares. Se a frequncia fundamental 500Hz, ele ressoar
para: 1500Hz, 2500Hz, 3500Hz, 4500Hz, etc.
345. (Unesp 2011) Na gerao da voz humana, a garganta e a
cavidade oral agem como um tubo, com uma extremidade
aproximadamente fechada na base da laringe, onde esto as
cordas vocais, e uma extremidade aberta na boca. Nessas
condies, sons so emitidos com maior intensidade nas
frequncias e comprimentos de ondas para as quais h um n
(N) na extremidade fechada e um ventre (V) na extremidade
aberta, como ilustra a figura. As frequncias geradas so
chamadas harmnicos ou modos normais de vibrao. Em um
adulto, este tubo do trato vocal tem aproximadamente 17 cm.
A voz normal de um adulto ocorre em frequncias situadas
aproximadamente entre o primeiro e o terceiro harmnicos.

Quando a gua no interior do tubo atinge uma determinada


altura h a partir da base, o som emitido pelo tubo muito
reforado. Considerando a velocidade do som no local de 340
m/s, a opo que melhor representa as ondas estacionrias
que se formam no interior do tubo no momento do reforo

a)

Considerando que a velocidade do som no ar 340 m/s, os


valores aproximados, em hertz, das frequncias dos trs
primeiros harmnicos da voz normal de um adulto so
a) 50, 150, 250.
b) 100, 300, 500.
c) 170, 510, 850.
d) 340, 1 020, 1 700.
e) 500, 1 500, 2 500.

b)

c)

RESPOSTA: E
COMENTRIO: A figura mostra o quinto harmnico.
96

No primeiro harmnico de uma corda, forma-se nela um nico


fuso, ou seja:

L = 2 L.
2
Ento:
d)

1 f1 = 2 f2 2 L f1 2

RESPOSTA: D
COMENTRIO: Dados: v = 340 m/s; f = 340 Hz

= 492 Hz.

A frequncia da onda sonora emitida pelo diapaso tem a


mesma frequncia que ele.
Calculando o comprimento de onda:

v f

v 340

f 340

L
f2 f2 = 2 f1 = 2 (246) f2
2

348. (Ufg 2010)


Um violo possui seis cordas de mesmo
comprimento L, porm, de massas diferentes. A velocidade de
propagao de uma onda transversal em uma corda dada
por v = T / , onde T a tenso na corda e , sua
densidade linear de massa. A corda vibra no modo
fundamental, no qual o comprimento L corresponde a meio
comprimento de onda . A frequncia de vibrao de uma
corda do violo aumentar se
a) aumentar.
b) v diminuir.
c) L diminuir.
d) aumentar.
e) T diminuir.

1 m.

Trata-se de um tubo fechado. Para os estados de ondas


estacionrias num tudo fechado, o comprimento (L) da coluna
de ar :

Ln .
4
Lembrando que um tubo fechado somente emite harmnicos
mpares, os comprimentos possveis para a coluna de ar so:

RESPOSTA: C

n 1 L 1 4 L 0,25 m.

L 0,75 m.
n 3 L 3
4

n 5 L 5 4 L 1,25 m (no convm)

COMENTRIO: O enunciado fornece a expresso: v

T
e

tambm sugere (densidade linear de massa) que a densidade


linear da corda =
v=

O comprimento mximo para a coluna de ar igual ao


comprimento do tubo, portanto, 1 m. So possveis, ento, os
estados mostrados nas figuras a seguir.

T
v
m
L

m
. Assim a expresso dada fica:
L
TL
.
m

No modo de vibrao fundamental, L =


sugere o enunciado. Assim: = 2 L.
Como f =
f=

, como tambm
2

v
v
1 TL 1 TL
f
f

2L
2L m 2 mL2

1 T
.
2 mL

Analisando essa expresso, conclumos que quando L diminui,


a frequncia aumenta.
347. (Pucrs 2010) O comprimento de uma corda de guitarra 64,0
cm.
Esta corda afinada para produzir uma nota com frequncia
igual a 246 Hz quando estiver vibrando no modo fundamental.
Se o comprimento da corda for reduzido metade, a nova
frequncia fundamental do som emitido ser:
a) 123 Hz
b) 246 Hz
c) 310 Hz
d) 369 Hz
e) 492 Hz

Obs: um estudante que aprecia msica no precisa de toda


essa manipulao matemtica para chegar a essa concluso.
Basta notar que, quando o violonista diminui o comprimento
da corda, o som se torna mais agudo, ou seja, a frequncia
aumenta.
349. (Ita 2010) Considere o modelo de flauta simplificado
mostrado na figura, aberta na sua extremidade D, dispondo de
uma abertura em A (prxima boca), um orifcio em B e outro
em C. Sendo AD 34,00 cm, AB BD,BC CD e a
velocidade do som de 340,0 m/s, as frequncias esperadas nos
casos: (i) somente o orifcio C est fechado, e (ii) os orifcios B
e C esto fechados, devem ser, respectivamente:

RESPOSTA: E
CPOMENTRIO: Se a intensidade da trao (F) na corda no se
altera, a velocidade de propagao da onda tambm no se
altera, pois, de acordo com a equao de Taylor:
v=

F
, sendo a densidade linear da corda.

97

II Incorreta. Quanto menor a frequncia, menor a altura.


III Incorreta. Batimento se refere s flutuaes na
intensidade do som quando h interferncia de duas ondas
sonoras de frequncia muito prximas.
a) 2000 Hz e 1000 Hz.
b) 500 Hz e 1000 Hz.
c) 1000 Hz e 500 Hz.
d) 50 Hz e 100 Hz.
e) 10 Hz e 5 Hz.
RESPOSTA: C
COMENTRIO: Com os dados fornecidos, podemos calcular as
distncias mostradas na Fig 1.

351. (PUC-2011) Patrcia ouve o eco de sua voz direta, refletida por
um grande espelho plano, no exato tempo de uma piscada de
olhos, aps a emisso. Adotando a velocidade do som no ar
como 340 m/s e o tempo mdio de uma piscada igual a 0,4 s,
podemos afirmar que a distncia d entre a menina e o espelho
vale
a) 68 m
b) 136 m
c) 850 m
d) 1.700 m
e) 8 .160 m
A distncia percorrida pelo som at que Patrcia oua o eco de
sua voz direta igual a 2d (ida e volta) --- v=S/t --- v =
2d/t --- 340=2d/0,4 --- d=136/2 --- d=68m --- R- B

A flauta comporta-se como um tubo sonoro aberto. Para o


harmnico fundamental, temos ento:

352. (UFMS) Em um movimento harmnico simples, a elongao (x)


e dada por x 5 A cos(t 1 ). Desejando-se percentualizar a
grandeza elongao, definiu-se %x=100x/xmx, onde xmax
representa o mximo valor que a elongao pode assumir. Os
percentuais (%v) e (%a) das grandezas velocidade e
acelerao,
respectivamente,
receberam
definies
semelhantes. O grfico desses percentuais, em funo do
tempo, t(s), esta representado abaixo, com legenda
posicionada na parte superior.

L 2L .
2
Como v = f v = 2Lf f

v
.
2L

Sendo v = 340 m/s a velocidade do som:


2
Para o orifcio C fechado, L = 17 cm = 1710 m.
f=

340
f = 1.000 Hz.
2 17 102
2

Para os orifcios C e D fechados, L = 34 cm = 3410 m.


f=

340
f = 500 Hz.
2 34 102
Considere as afirmativas:

350. (Uece 2010) Os termos a seguir esto relacionados s ondas


sonoras.

I. O perodo do movimento e de 10 segundos.


II. A frequncia do movimento e de 0,1 hertz.
III. A pulsao do movimento e de /6 rad/s.
IV. O angulo inicial de fase () e de /6 radianos.
V. A velocidade do objeto e nula no instante t = 4 s.

I - Volume se refere intensidade da sensao auditiva


produzida por um som e depende da intensidade e da
frequncia da onda.
II - Altura se refere a uma qualidade da onda que depende
somente da sua frequncia: quanto menor a frequncia maior
a altura.
III - Batimento se refere s flutuaes na intensidade do som
quando h interferncia de duas ondas sonoras de mesma
frequncia.
IV - Timbre uma caracterstica que depende da frequncia e
da intensidade dos tons harmnicos que se superpem para
formar a onda sonora.
Est correto o que se afirma em
a) I e II, apenas.
b) II e III, apenas.
c) III e IV, apenas.
d) I e IV, apenas.

E correto afirmar que:


a) apenas a afirmativa I esta correta.
b) as afirmativas I e II esto corretas.
c) apenas a afirmativa III e correta.
d) as afirmativas III e V esto corretas.
e) todas as afirmativas esto corretas.
RESPOSTA: D
COMENTRIO: I. Errada. O tempo decorrido entre 100% e
100% T = (10 4) s T = 12 s

II. Errada. A frequncia f = 1 1 0,083 Hz.


T 12
III. Correta. 2 2

RESPOSTA: D
COMENTRIO: Justificando as incorretas:

T 12

98

rad / s

IV. Errada. %x =

A intensidade de radiao apreveita para o aquecimento da


gua (ltil) dada por:

100 x
100 x
Emt 0 50
50 xm x 100 x
xmx
xmx

Pot
A
mc
Q
0,5l
0,5l
t.A
t.A
ltil

xmx
50 xmx
xmx
x
x
1
xx
cos
2 mx .
100
2
xmx xmx
2 xmx
cos

Admitindo-se que a massa de gua correspondente a 6,0 seja


igual a 6,0kg (H O 1,0kg / ), vem:

rad
2
3

3
0,5 . 1,0 . 10 = 6,0 . 4,2 . 10 .
60 . 10

V. Correta. Em t = 4 s %v =

100v
100v
0
100v 0 v 0
vmx
vmx

11,9 oC 12o C

353. (FUVEST-SP) Enquanto uma folha de papel puxada com


velocidade constante sobre uma mesa, uma caneta executa
movimento de vaivm perpendicularmente direo de
deslocamento do papel, deixando registrado na folha um trao
em forma de senide. A figura abaixo representa um trecho
AB do trao, bem como as posies de alguns de seus pontos e
os respectivos instantes.

355. (UEL) Onda uma perturbao ou distrbio transmitido


atravs do vcuo ou de um meio gasoso, lquido ou slido. As
ondas podem diferir em muitos aspectos, mas todas podem
transmitir energia de um ponto a outro. Quando no h
dissipao de energia, pode-se dizer que a intensidade I de
uma onda progressiva igual energia E transmitida pela
onda, dividida pela rea S perpendicular direo de
propagao, em um intervalo de tempo t. Essa intensidade
tambm pode ser escrita em termos de potncia transmitida I
= P/S. Considere uma fonte puntiforme de ondas luminosas
com emisso constante em todas as direes. Com base nas
leis da Fsica, considere as afirmativas a seguir.
I. A rea total, atravs da qual a onda se propaga, a rea da
superfcie de uma esfera, tendo a fonte luminosa como seu
centro.
II. A uma distncia d da fonte, a intensidade luminosa dada
2
por P/4d .
III. Sendo a intensidade da radiao solar na Terra igual a
3
2
1,35 . 10 W/m , a intensidade dessa radiao no planeta
Mercrio, cuja distncia do Sol de 0,387 vezes a distncia do
3
2
Sol Terra, igual a 9,01 . 10 W/m .
IV. O Sol no pode ser considerado como fonte luminosa
puntiforme em nenhuma situao de anlise.

Pede-se:
a) a velocidade de deslocamento da folha
b) a razo das freqncias do movimento de vaivm da caneta
entre os instantes 0 a 3s e 5s a 13s.
RESPOSTA:
a) V=DS/Dt ---- V=26/13 ---- V=2cm/s
b) f1=1/2Hz
---- f2=1/8Hz ---- f1/f2=1/2X8/1
f1/f2=4

----

Esto corretas apenas as afirmativas:


354. (ITA-SP) Um painel coletor de energia solar para aque
cimento residencial de gua, com 50% de eficincia, tem su
2
per fcie coletora com rea til de 10m . A gua cir cula em
tubos fixados sob a super fcie coletora. Suponha que a
3
2
intensidade da energia solar incidente de 1,0 . 10 W / m e
que a vazo de suprimento de gua aquecida de 6,0 litros
por mi nuto. Assinale a opo que indica a variao da tem
3
peratura da gua. Dado: cH2O = 4,2 . 10 J/kg. K
a) 12C
b) 10C
c) 1,2C
d) 1,0C
e) 0,10C
RESPOSTA: A
COMENTRIO:

a) I e II
b) II e IV
c) I e III
d) I, II e III
e) III e IV
RESPOSTA: D
COMENTRIO: I. VERDADEIRA.
II. VERDADEIRA. I P

P
4 xd 2

III. VERDADEIRA.

P
IM 4 xdM2
I
4 xdT2
P

M
.
2
P
IT
IT
4 x (0,387dT )
P
2
4 xdT
IM
1

Da qual : IM 9,01 . 103W / m2


1,35 . 103 (0,387)2
IV. FALSA.
356. (UECE) Um sistema oscilante massa-mola possui uma energia
mecnica igual a 1,0 J, uma amplitude de oscilao 0,5 m e
uma velocidade mxima igual a 2 m/s. Portanto, a constante
da mola, a massa e a freqncia so, respectivamente, iguais
a:
a) 8,0 N/m, 1,0 kg e 4/p Hz
99

b) 4,0 N/m, 0,5 kg e 4/p Hz


c) 8,0 N/m, 0,5 kg e 2/pHz
d) 4,0 N/m, 1,0 kg e 2/p Hz

RESPOSTA: C
COMENTRIO:
Em-1J A=0,5m
Vmxima=2m/s
2
2
Em=1/2.kA --- 1=1/2.k.(0,5) --2
2
k=8N/m
Em=1/2.mV mxima --- 1=1/2.m.(2) --- m=0,5kg
T=2pm/k --- T=2p0,5/8 --- T=2p.1/4 --- T=p/2 s --- f=1/T
--- f=1/p/2 --- f=2/pHz

fundamental da dinmica --- FR=ma --- FR=0,5.1,5 --FR=0,75N --359. (PUC-2010) A figura a seguir mostra um corpo de massa m =
0,05 kg, preso a uma mola de constante elstica k = 20 N/m. O
objeto deslocado 20 cm para a direita, a partir da posio de
equilbrio sobre uma superfcie sem atrito, passando a oscilar
entre x = A e x = - A.

357. (MACKENZIE-SP) Um corpo de 250g de massa encontra-se em


equilbrio, preso a uma mola helicoidal de massa desprezvel e
constante elstica k igual a 100N/m, como mostra a figura
abaixo.

O atrito entre as superfcies em contato desprezvel. Esticase a mola, com o corpo at o ponto A, e abandona-se o
conjunto nesse ponto, com velocidade zero. Em um intervalo
de 1,0s, medido a partir desse instante, o corpo retornar ao
ponto A
a) um vez
b) duas vezes
c) trs vezes
d) quatro vezes
e) seis vezes
RESPOSTA: C
COMENTRIO: Vamos calcular o perodo T, que o tempo que
ele demora para efetuar um vai e vem completo.
-2
T=2m/k --- T=2.3.0,25/100 --- T=6.5.10 --- T=0,3s
0,3s ------ 1 vai e vem completo
1,0s ------ n
N=3,3 -----

Assinale a afirmativa CORRETA.


a) Na posio x = -20 cm, a mola tem uma energia cintica de
0,4 J e a energia potencial elstica do corpo nula.
b) Na posio x = -20 cm, toda a energia do sistema vale 0,4 J e
est no objeto sob a forma de energia cintica.
c) Na posio x = 0, toda a energia do sistema est no corpo na
forma de energia cintica e sua velocidade vale 4 m/s.
d) Na posio x = 20 cm, toda a energia do sistema vale 0,8 J
sendo 0,6 J na mola e o restante no objeto.
RESPOSTA: C
COMENTRIO: No MHS (movimento harmnico simples) o sistema
apresenta energia potencial elstica mxima nas extremidades (A e
A) e energia cintica mxima no centro (0). Desta forma a
velocidade da partcula no centro do sistema dada por --2
2
mv /2=kx /2 --2
2
2
0,05v =20.(0,2) --- v =0,8/0,05 --- v=16 --- v=4m/s --360. (ITA-2009) Um cubo de 81,0 kg e 1,00 m de lado flutua na
3
gua cuja massa especfica = 1000 kg/m . O cubo ento
calcado ligeiramente para baixo e, quando liberado, oscila em
um movimento harmnico simples com uma certa frequncia
angular. Desprezando-se as foras de atrito e tomando g = 10
2
m/s , essa frequncia angular igual a:
a) 100/9 rad/s.
b) 1.000/81 rad/s
c) 1/9 rad/s.
d) 9/100 rad/s.
e) 81/1.000 rad/s
RESPOSTA: A
COMENTRIO: Com o cubo em equilbrio, a fora resultante
sobre ele nula --- empuxo = peso --- .g.v = m.g --1.000v=81 --3
2
V=0,081m --- como a rea da base 1 m , isto significa que o
ponto de equilbrio fica a 0,081 m ou 8,1 cm abaixo da linha da
superfcie, pois V=s.h --- 0, 081=1.h --- h=0,081m --- o cubo
forado para baixo, digamos uma profundidade x alm de
0,081 m --- a fora resultante sobre o cubo funcionar como
a fora restauradora do MHS --- Fresultante = Empuxo Peso --Fresultante = .g.(0,081 + x) m.g --- Fresultante = .g.0,081 + .g.x
m.g = .g.x --- se esta fora a restauradora do MHS ento
.g.x = k.x --- k = .g --- frequncia angular de um sistema
oscilante --- w=(k/m)= (.g/m)=(1.000x10/81) --w=(10.000/81) --- w=100/9 rad/s ---

358. (MACKENZIE-2011)
Um
corpo
de
0,50kg
oscila,
periodicamente, sobre uma reta em torno de um ponto, com
sua posio x em funo do tempo, na reta, dada em relao a
esse ponto, pela funo x = 0,30cost. A posio x medida
em metros, em rad/s e t em segundos. Dentre as
alternativas, o valor mais prximo da fora resultante que age
sobre esse corpo, no instante t=1/3s s,
a) 0,74N
b) 0,82N
c) 0,96N
d) 1,20N
e) 1,48N
RESPOSTA: A
2
COMENTRIO: - Relao fundamental do MHS --- a= - w .x --quando t=1/3s
--x=0,3cost=0,3cos.1/3
--x=0,3cos/3=0,3.1/2 --- x=0,15m --- x=0,3c0swt --- w= --2
2
2
2
a=-w .x= - .0,15 --- =10 --- a=1,5m/s --- lei

361. (UFU-MG) Uma massa m executa um MHS. Sua energia


potencial U, em funo de sua posio x, est no grfico
abaixo.

100

c) O perodo do pndulo ser o mesmo, no entanto o perodo


do bloco pendurado na mola ser alterado.
d) Os perodos de ambos os osciladores sofrero modificao
em relao a quando estavam na Terra.
RESPOSTA: B
COMENTRIO: Perodo de oscilao (T) de um pndulo simples
de comprimento L em um local onde o campo gravitacional
tem intensidade g, para oscilaes de pequenas amplitudes --T=2L/g --- observando essa expresso voc conclui que,
quando a intensidade do campo gravitacional diminui o
perodo aumenta, ou seja, o pndulo passa a oscilar mais
vagarosamente --- na ausncia total de gravidade, o pndulo
teria perodo infinito, ou seja, deixaria de oscilar --- para um
sistema massa-mola (m e k) o perodo de oscilao (T) dado
por --- T=2m/k --- observe nessa expresso que o perodo
desse sistema independe da gravidade local --- assim, nesse
ambiente de microgravidade, o perodo do sistema bloco-mola
no sofrer alterao, j o perodo do pndulo simples deixar
de ser o mesmo ---

Se E for sua energia total, teremos:


a) em x1, sua energia cintica ser a
b) em x1, sua energia potencial ser b
c) em x1, sua energia cintica ser +b
d) na posio x2 sua energia cintica ser mxima
e) na posio x2 sua energia potencial ser nula.
RESPOSTA: C
COMENTRIO: Em X2 --- Ep mxima e Ec nula. Em X1 --Ep = a --- E = Ec + Ep --- E = a + b --- Ec + a = a + b --- Ec = +b
362. (FGV-2011) Na Terra, o perodo de oscilao de um pndulo,
isto , o tempo que ele demanda para completar um ciclo
completo, corresponde, com boa aproximao, raiz
quadrada do qudruplo do comprimento do pndulo. O
pndulo de um carrilho, ao oscilar, bate o segundo e
constitudo por uma fina haste de ao de massa desprezvel,
unida a um grande disco de bronze, que guarda em seu centro
o centro de massa do conjunto haste-disco. Suponha que a
20C, o centro de massa do conjunto esteja a 1 metro do eixo
de oscilao, condio que faz o mecanismo funcionar com
exatido na medida do tempo.
Considerando que o coeficiente de dilatao linear do ao
-6
-1
10.10 C e supondo que o centro de massa da haste-disco
se mantenha sempre no centro do disco se a temperatura do
conjunto haste-disco subir 10C, a medida do tempo,
correspondente a meio ciclo de oscilao do pndulo, se
tornar
a)
1,0001
s,
fazendo
com
que
o
relgio
adiante.
b) 2,0002 s, fazendo com que o relgio adiante.
c)
1,0001
s,
fazendo
com
que
o
relgio
atrase.
d) 2,0002 s, fazendo com que o relgio atrase.
e) 3,0003 s, fazendo com que o relgio atrase.
RESPOSTA: C
COMENTRIO: Novo comprimento do pndulo quando a
o
temperatura subir de 10 C --- = o (1 + (t to) --- = 10(1
-6
+ 10.10 .10 --=1,0001m --- clculo do perodo T que pelo enunciado vale
T= (4) --- T= (4.1,0001) --- T= 2.1,0001 s --- para meio
ciclo de oscilao o perodo ser T=1,0001 s --- se, para cada
meia oscilao o perodo aumenta de T=1,0001 s, o relgio
ir atrasar
363. (UFOP-2010) Dois sistemas oscilantes, um bloco pendurado
em uma mola vertical e um pndulo simples, so preparados
na Terra de tal forma que possuam o mesmo perodo. Se os
dois osciladores forem levados para a Estao Espacial
Internacional (ISS), como se comportaro os seus perodos
nesse ambiente de microgravidade?
a) Os perodos de ambos os osciladores se mantero os
mesmos de quando estavam na Terra.
b) O perodo do bloco pendurado na mola no sofrer
alterao, j o perodo do pndulo deixar de ser o mesmo.

364. (UFAL-2010) Um relgio de pndulo construdo tal que o seu


pndulo realize 3600 oscilaes completas a cada hora. O
relgio est descalibrado, de modo que o pndulo oscila em
um movimento harmnico simples de frequncia angular igual
a 5/2 rad/s. Nessa situao, ao final de 3600 oscilaes
completas do pndulo tero se passado:
a) 32 min
b) 45 min
c) 48 min
d) 52 min
e) 56 min
RESPOSTA: C
COMENTRIO: Clculo do perodo T de cada oscilao --w=2/T --- T=2/w=2/5/2 --- T=0,8s --- tempo (t) gasto
em n=3.600 oscilaes --- t=nT=3.600x0,8 --- t=2.880/60
min --- t=48 min
365. (Mackenzie-SP) Comenta-se que o clebre fsico e matemtico
Galileu Galilei, ao observar a oscilao do lampadrio da
catedral de Pisa, na Itlia, concluiu tratar-se de um movimento
peridico, semelhante ao que hoje chamaramos de pndulo
simples. Para tal concluso, teria medido o perodo do
movimento, utilizando, como unidade de medida para o
tempo, seu prprio batimento cardaco. Se considerarmos um
grande pndulo simples, de comprimento 10 m, oscilando
2
num local ondeg=10m/s , e que a freqncia dos batimentos
cardacos de 86 batidas por minuto, o perodo do
movimento desse pndulo ser de aproximadamente:

a) 3 batidas.
b) 6 batidas.
c) 9 batidas.
d) 12 batidas.
e) 15 batidas
RESPOSTA: C
COMENTRIO: T=210/10 --- T=2ps --- p=3 --- T=6s
86 batidas ---- 60s
X batidas ----- 6s
X = 9 batidas
366. (ITA ) Dois pndulos de comprimento L1 e L2 conforme a
figura, oscilam de tal modo que os dois bulbos de encontram
sempre que so decorridos 6 perodos do pndulo menor e 4
perodos do pndulo maior. A relao L2/L1 deve ser:
101

e = 4 rad/s, chega-se funo dada.


2
d) Verdadeira. A acelerao mxima dada por amx = A.
Substituindo-se pelos valores dados, obtemos a =
3
2
= 16 cm/s .
e) Falsa. Nesse trecho a velocidade aumenta, logo a energia
cintica tambm aumenta.

a) 9/4
b) 3/2
c) 2
d) 4/9
e) 2/

368. (Mack-SP) Um menino na beira de um lago observou uma


rolha que flutuava na superfcie da gua, completando uma
oscilao vertical a cada 2s, devido ocorrncia de ondas. Esse
menino estimou como sendo 3 m a distncia entre duas cristas
consecutivas. Com essas observaes, o menino concluiu que
a velocidade de propagao dessas ondas era de:
a) 0,5 m/s.
b) 1,0 m/s.
c) 1,5 m/s.
d) 3,0 m/s.
e) 6,0 m/s.

RESPOSTA: A
COMENTRIO: 6T1 = 4T2 --- 6. 2L1/g = 4. 2L2/g --- L2/L1 =
2
6/4 --- (L2/L1) = 36 / 16 --- L2 / L1 = 9 / 4
367. (UFBA) A figura abaixo representa a posio ocupada, no
instante t, por uma partcula que descreve um movimento
circular uniforme com velocidade angular = 4 rad/s, numa
circunferncia de raio R = cm. A figura representa tambm a
posio da projeo da partcula sobre o eixo Ox, paralelo ao
dimetro OO, contidos ambos os eixos no plano da
circunferncia.

RESPOSTA: C
COMENTRIO: So dados T = 2s, tempo de uma oscilao
completa, e =
= 3 m, distncia entre duas cristas consecutivas. Logo:

3
v 1,5 m / s
2

369. Uma partcula em movimento harmnico simples oscila com


freqncia de 10 Hz entre os pontos L e -L de uma reta. No
instante t1 a partcula est no ponto 3.L/2 caminhando em
direo a valores inferiores, e atinge o ponto - 2.L/2 no
instante t2. O tempo gasto nesse deslocamento :
a) 0,021 s
b) 0,029 s
c) 0,15 s
d) 0,21 s
e) 0,29 s
RESPOSTA: B
COMENTRIO: A equao da posio de um MHS x = A.cos
(wt + 0) onde A = L a amplitude; w = 2f = 20 rads/s a
pulsao e 0 a fase inicial. Para 0 = 0, teremos: (1) 3 . L2 = L.
cs 20t cos 20t = 3./2 20t = 3/4 t2 = 3/80.
O intervalo de tempo t = (3/80) (1/120) = (9 2)/240 =
0,029 s.
Observao: o MHS pode ser analisado como a projeo de
um movimento circular com velocidade angular constante w =
2f. As posies correspondem a projees do raio.

Em relao ao movimento da projeo sobre o eixo Ox,


correto afirmar:
a) O movimento harmnico simples, e sua amplitude igual
a 2 cm.
b) O perodo do movimento igual a 0,5s.
c) A funo horria da velocidade escalar instantnea v =
2
4 sen (4t), sendo a fase inicial igual a zero.
d) No ponto de inverso x = cm, a acelerao escalar
mxima e igual a 163 cm/s2.
e) Ao se deslocar de x = cm at x = 0, a energia cintica da
partcula diminui.

FSICA II
370.

RESPOSTA: F V V V F
COMENTRIO: a) Falsa. O movimento harmnico simples,
mas a amplitude igual ao raio, portanto, igual a cm.
b) Verdadeira. Da expresso = 2 , temos T = 2 .
T

Sendo = 4 rad/s, T = 0,5s.


c) Verdadeira. A funo da velocidade no MHS
v = -A sem t para 0 = 0. Substituindo-se A = cm
102

(Fuvest 2012)

Uma fibra tica um guia de luz, flexvel e transparente,


cilndrico, feito de slica ou polmero, de dimetro no muito
maior que o de um fio de cabelo, usado para transmitir sinais
luminosos a grandes distncias, com baixas perdas de
intensidade. A fibra tica constituda de um ncleo, por onde
a luz se propaga e de um revestimento, como esquematizado
na figura acima (corte longitudinal). Sendo o ndice de refrao
do ncleo 1,60 e o do revestimento, 1,45, o menor valor do
ngulo de incidncia do feixe luminoso, para que toda a luz
incidente permanea no ncleo, , aproximadamente,
Note e adote
(graus) sen
25
0,42
30
0,50
45
0,71
50
0,77
55
0,82
60
0,87
65
0,91

A tabela a seguir relaciona o ndice de refrao para sete


8
meios materiais diferentes. Se necessrio, adote c = 3.10
m/s.

cos
0,91
0,87
0,71
0,64
0,57
0,50
0,42

Com base nessa tabela, correto afirmar que:


a) a velocidade da luz no se altera quando muda de meio.
b) a velocidade da luz no vidro (crown) a mesma que no
vidro (flint).
c) o ar o meio onde a luz apresenta maior velocidade.
d) o vidro (flint) o meio onde a luz viaja mais rpido do que
no leo.
e) na gua a luz viaja mais rpido do que no lcool etlico.

n1 sen 1 n2 sen 2
a) 45.
b) 50.
c) 55.
d) 60.
e) 65.

RESPOSTA: E
COMENTRIO: De n =

RESPOSTA: E
COMENTRIO: Basta calcularmos o ngulo limite, que o
ngulo de incidncia ( ) no meio mais refringente (ncleo)
que provoca uma emergncia rasante (90) no meio menos
refringente (revestimento).
Dados: nncleo = 1,60; nrevest = 1,45.
Aplicando a lei de Snell:

nncleo sen nrevest sen90 sen

nresvest

nncleo

1,45
sen 0,91.
1,60
Consultando a tabela dada: = 65.

371. (UFPB) Em 1621, o cientista holands Willebrord van Roijen


SNELL (1591 -1626) investigou o fenmeno fsico da
propagao da luz em diversos meios, e estabeleceu baseado
na evidncia experimental, a lei que levou o seu nome Lei de
Snell ou Lei da Refrao. Considere esta lei aplicada seguinte
situao: O ndice de refrao absoluto (n) de um meio
material (conforme a figura) definido como sendo a razo
entre a velocidade da luz no meio 1 e a velocidade da luz no
meio 2.

c
, conclumos que no meio de menor
v

ndice de refrao a velocidade de propagao da luz maior.


Sendo ngua = 1,33 e nlcool = 1,36, resulta que na gua a
velocidade de propagao da luz maior do que no lcool
etlico.
372. (UTFPR 2011) Antes de serem usados em joias, os diamantes
passam pelo processo de lapidao, no qual se cortam as
laterais da pedra que passam a ter muitas faces. A luz branca
incidente no diamante pode sofrer decomposio e mostrar as
cores do arco-ris. Quando ocorre essa decomposio, o
diamante tem comportamento similar a um(a):
a) lente.
b) espelho plano.
c) espelho cncavo.
d) espelho convexo.
e) prisma ptico.
RESPOSTA: E
COMENTRIO: A luz branca ao atravessar um prisma sofre
decomposio (disperso) em suas cores componentes.
373. (IFSP 2011) Os fenmenos luminosos so estudados h muito
tempo. A luz, como qualquer onda eletromagntica, tem
grandes aplicaes na engenharia e na medicina, entre outras
reas. Quando a luz atinge uma superfcie, um ou mais
fenmenos podem ocorrer, como a reflexo, refrao, difuso
e absoro.
A seguir so feitas as seguintes afirmativas:
I. Quando olhamos uma moeda dentro de um recipiente com
gua, sabemos que ela no se encontra na posio vista
aparentemente, por causa do fenmeno da reflexo, que
desvia os raios luminosos.
II. Para acendermos um palito de fsforo por meio de raios
solares, podemos usar lentes do tipo convergentes.
III. Toda onda eletromagntica, como a luz, pode se propagar
no vcuo.
IV. Colocando-se um objeto entre dois espelhos planos e
paralelos, obtm-se um nmero infinito de imagens.
103

para os objetos comuns na superfcie terrestre est na faixa de


0,8 m a 0,9 m . Nesse intervalo, a diferena de refletncia

So corretas apenas
a) I e II.
b) II e IV.
c) I, II e III.
d) I, II e IV.
e) II, III e IV.
RESPOSTA: E
COMENTRIO: I. Incorreta. A moeda no est na posio vista
aparentemente devido ao fenmeno da refrao, que desvia
os raios luminosos.
II. Correta. Podemos acender o palito de fsforo colocando
sua (dele) cabea no foco, ponto de encontro dos raios solares
refratados pela lente convergente.
III. Correta.
IV. Correta. O nmero de imagens (n) fornecidas pela
associao de dois espelhos planos dado por:

tambm maior, aumentando a probabilidade de se


identificar corretamente o objeto observado. verdade que
nesse intervalo a refletncia da gua nula, porm a
probabilidade de encontrar gua praticamente nula.
375. (ITA 2011) Um hemisfrio de vidro macio de raio de 10 cm e
ndice de refrao n = 3/2 tem sua face plana apoiada sobre
uma parede, como ilustra a figura.

360
1 , sendo o o ngulo formado entre os

espelhos. Se os espelhos so colocados paralelamente entre si,


= 0. Ento n tende para infinito.
374. (ENEM 2011) O processo de interpretao de imagens
capturadas por sensores instalados a bordo de satlites que
imageiam determinadas faixas ou bandas do espectro de
radiao eletromagntica (REM) baseia-se na interao dessa
radiao com os objetos presentes sobre a superfcie
terrestre. Uma das formas de avaliar essa interao por meio
da quantidade de energia por meio da quantidade de
energia refletida pelos objetos. A relao entre a refletncia
de um dado objeto e o comprimento de onda da REM
conhecida como curva de comportamento espectral ou
assinatura espectral do objeto, como mostrado na figura, para
objetos comuns na superfcie terrestre.

Um feixe colimado de luz de 1 cm de dimetro incide sobre a


face esfrica, centrado na direo do eixo de simetria do
hemisfrio. Valendo-se das aproximaes de ngulos
pequenos, sen e tg , o dimetro do crculo de
luz que se forma sobre a superfcie da parede de
a) 1 cm.

2
cm.
3
1
c) cm.
2
1
d) cm.
3
1
e)
cm.
10
b)

RESPOSTA: B
COMENTRIO:

De acordo com as curvas de assinatura espectral apresentadas


na figura, para que se obtenha a melhor discriminao dos
alvos mostrados, convm selecionar a banda correspondente
a que comprimento de onda em micrmetros (m) ?
a) 0,4 a 0,5.
b) 0,5 a 0,6.
c) 0,6 a 0,7.
d) 0,7 a 0,8.
e) 0,8 a 0,9.
Dados: nar = 1; nvidro =

RESPOSTA: E
COMENTRIO: O grfico nos mostra que a maior refletncia
104

3
; R = 10 cm; A b = 0,5 cm.
2

Aplicando a lei de Snell na figura dada, temos:

normal no ponto de incidncia. A opo que melhor


representa a relao entre esses ngulos
a)

nar sen i nvidro sen r .


c)
d)
b)

Mas i e r so ngulos pequenos. Ento, de acordo com o


enunciado, podemos escrever:
nar i = nvidro r

nar R nvidro R2
A

1 0,5

3 d

2 2

RESPOSTA: A
COMENTRIO: Como nada foi dado a respeito das grandezas
referentes a essas radiaes, necessrio que se tenha
memorizado suas propriedades. A tabela abaixo fornece a
ordem do espectro visvel da luz branca e os comportamentos
das grandezas referentes s radiaes componentes. A seta
indica o sentido crescente da grandeza.

2
cm.
3

376. (UEL 2011) Um raio de luz parcialmente refletido e


parcialmente refratado na superfcie de um lago. Sabendo-se
que o raio de luz incidente faz um ngulo de 55 em relao
superfcie da gua, quais so os ngulos de reflexo e de
refrao, respectivamente?
Dado: ndice de refrao da gua: 1,33.
a) 180 e 360.
b) 55 e 65.
c) 1 e 1,33.
d) 35 e 25,5.
e) 35 e 35.
RESPOSTA: D
COMENTRIO: O ngulo de incidncia (i) de reflexo (i) e de
refrao (r) so todos medidos em relao norma
superfcie.
Ento:

A figura a seguir representa o comportamento dos trs raios,


de acordo com a tabela: menor desvio para o vermelho e
maior desvio para o violeta.

i 90 55 i 35.
O ngulo de reflexo igual ao de incidncia:

i' i 35.
Adotando sen 35 = 0,57 e aplicando a lei de Snell, vem:
sen i ngua
sen 35 1,33
0,57

sen r
nar
sen r
1
sen r
0,57
0,43.
1,33
Consultando uma tabela trigonomtrica, verificamos que r =
25,5.
1,33 sen r

377. (AFA 2011) Trs raios de luz monocromticos correspondendo


s cores vermelho (Vm), amarelo (Am) e violeta (Vi) do
espectro eletromagntico visvel incidem na superfcie de
separao, perfeitamente plana, entre o ar e a gua, fazendo o
mesmo ngulo com essa superfcie, como mostra a figura
abaixo.

Assim:

378. (UDESC 2011) Considere uma lmina de vidro de faces


paralelas imersa no ar. Um raio luminoso propaga-se no ar e
incide em uma das faces da lmina, segundo um ngulo em
relao direo normal ao plano da lmina. O raio
refratado nesta face e refletido na outra face, que
espelhada. O raio refletido novamente refratado na face no
espelhada, voltando a propagar-se no ar. Sendo nAr e nVidro,
respectivamente, os ndices de refrao da luz no ar e no
vidro, o ngulo de refrao que o raio refletido forma no
vidro, com a direo normal ao plano da lmina, ao refratar-se
pela segunda vez, obedece equao:
a) nVidro sen = nAr sen /2
b)

c) sen = cos
d) nVidro sen = nAr sen
Sabe-se que ,

e) nAr sen = nVidro sen

, e so, respectivamente, os ngulos de

refrao, dos raios vermelho, amarelo e violeta, em relao

RESPOSTA: B
COMENTRIO: Observe o trajeto feito pelo raio luminoso:
105

O ar menos refringente que a gua e quando o sentido de


propagao do menos para o mais refringente no ocorre
reflexo total.
II. Correta.
III. Correta.
A figura abaixo justifica II e III. Ela mostra a guia
aproximando-se do peixe, nas posies A1 e A2 e as respectivas
imagens, A1 e A2, vistas pelo peixe, bem como o peixe, P, e as
respectivas imagens vistas pela guia. Ela mostra que, quando
a guia se aproxima, ela v a profundidade aparente do peixe
(imagem) do peixe aumentar, e o peixe v a altura aparente da
guia (imagem) diminuir.
O ngulos 1 e 2 so iguais (alternos internos). Os ngulos 2 e 3
so iguais (reflexo). Os ngulos 3 e 4 so iguais (alternos
internos). Portanto:

nA .sen nV .sen1
, pois 1 4
nA .sen nV .sen4
379. (UEL 2011)
A guia-de-cabea-branca (Haliaeetus
leucocephalus) uma guia nativa da Amrica do Norte que se
alimenta principalmente de peixes. Sua estratgia de pesca a
seguinte: a guia faz um voo horizontal ligeiramente acima da
superfcie da gua. Quando est prxima, ela se inclina
apontando suas garras para a sua presa e, com uma preciso
quase infalvel, afunda suas garras na gua arrebatando sua
refeio.

IV. Incorreta.
As imagens so virtuais.
380. (UEG 2011) O experimento de decomposio (disperso) da
luz solar, realizado por Newton, extraordinariamente
simples, sendo necessrio somente um prisma. Como ilustra a
figura abaixo, ao passar por um prisma, a luz solar, que
branca, se decompe nas cores do arco-ris.

Com base nos conhecimentos sobre reflexo e refrao da luz


e de formao de imagens reais e virtuais, considere as
afirmativas a seguir.
I. A grande distncia, o fenmeno de reflexo interna total
impede que o peixe veja a guia.
II. medida que se aproxima, a guia v a profundidade
aparente do peixe aumentar.
III. medida que a guia se aproxima, o peixe v a altura
aparente da guia diminuir.
IV. Durante a aproximao, as imagens vistas pela guia e pelo
peixe so reais.

Com relao aos fenmenos da luz ao atravessar o prisma,


correto afirmar:
a) Na disperso da luz, a luz monocromtica de maior
frequncia sofrer o menor desvio.
b) Num prisma, a disperso da luz branca menos acentuada
que numa nica superfcie diptrica.
c) A separao da luz branca nas cores do arco-ris possvel
porque cada cor tem um ndice de refrao diferente.
d) Neste experimento, Newton demonstrou que, combinando
dois ou mais prismas, possvel decompor a luz branca, porm
a sua recomposio no possvel.

Assinale a alternativa correta.


a) Somente as afirmativas I e III so corretas.
b) Somente as afirmativas I e IV so corretas.
c) Somente as afirmativas II e III so corretas.
d) Somente as afirmativas I, II e IV so corretas.
e) Somente as afirmativas II, III e IV so corretas.

RESPOSTA: C
COMENTRIO: Quando a luz branca (solar) refrata sob
incidncia oblqua, ela sobre disperso (ou decomposio) em
suas radiaes componentes: vermelha, alaranjada, amarela,
verde, azul, anil e violeta.

RESPOSTA: C
COMENTRIO: I. Incorreta.
106

O maior desvio ocorre para a radiao violeta, que a que


apresenta no interior do prisma: menor velocidade, menor
comprimento de onda, maior frequncia, maior ndice de
refrao.
Obs: no vcuo, todas as radiaes apresentam mesma
velocidade, portanto, o mesmo ndice de refrao.
d)
381. (FGV 2010) Um feixe luminoso de raios paralelos, que se
propaga em um meio ptico homogneo, incide sobre uma
superfcie que separa o primeiro meio de um segundo,
passando a se propagar neste.
Substituindo-se o segundo meio ptico por um vidro fosco e
translcido, e admitindo que os raios de luz nele penetrem,
estes perdem o paralelismo, podendo-se dizer que nessa
situao ocorreu uma
a) reflexo difusa.
b) reflexo regular.
c) refrao difusa.
d) refrao regular.
e) absoro difusa.

e)

RESPOSTA: D
COMENTRIO: Nos materiais naturais, quando ocorre
incidncia oblqua da luz, os raios incidente e refratado esto
em meios diferentes e em quadrantes opostos, definidos pela
superfcie e pela normal a essa superfcie. No metamaterial,
esses raios esto em meios diferentes, mas em quadrantes
adjacentes.

RESPOSTA: C
COMENTRIO: Se os raios penetram no vidro fosco, ocorre
refrao. Se os raios deixam de ser paralelos eles se espalham
(difundem).
382. (ENEM 2010)
Um grupo de cientistas liderado por
pesquisadores do Instituto de Tecnologia da Califrnia
(Caltech), nos Estados Unidos, construiu o primeiro
metamaterial que apresenta valor negativo do ndice de
refrao relativo para a luz visvel. Denomina-se metamaterial
um material ptico artificial, tridimensional, formado por
pequenas estruturas menores do que o comprimento de onda
da luz, o que lhe d propriedades e comportamentos que no
so encontrados em materiais naturais. Esse material tem sido
chamado de canhoto.
Disponvel em: http://inovacaotecnologica.com.br. Acesso em:
28 abr. 2010 (adaptado).
Considerando o comportamento atpico desse metamaterial,
qual a figura que representa a refrao da luz ao passar do ar
para esse meio?

383. (Mackenzie 2010) Um estudante, ao fazer a experincia em


que um feixe de luz monocromtico vai da gua, de ndice de
refrao 1,3, para o ar, de ndice de refrao 1,0, pode concluir
que, para essa onda,
a) o comprimento de onda diminui e a velocidade aumenta.
b) o comprimento de onda e a frequncia da luz diminuem.
c) a frequncia aumenta, mas o comprimento de onda
diminui.
d) a frequncia no se altera e o comprimento de onda
diminui.
e) a frequncia no se altera e o comprimento de onda
aumenta.
RESPOSTA: E
COMENTRIO: Sabemos que ao mudar de meio, a frequncia
do feixe no se altera. O comprimento de onda pode ser
obtido a partir da lei de Snell.

nagua
v

seni nagua

ar ar
ar .
senr
nar
v agua agua
nar
agua

a)

Como

nagua > nar ar > agua.

b)

384. (UECE 2010) A figura a seguir mostra um prisma feito de um


material, cujo ndice de refrao 1,5, localizado na frente de
um espelho plano vertical, em um meio onde o ndice de
refrao igual a 1. Um raio de luz horizontal incide no
prisma.

c)

107

sen1 v1

sen2 v 2

3
1
v 2 3 108
2
2

v2 =

3 108
3

3 3 108
3

3 10 m/s.

v2 =

Sabendo que sen(6 ) 0,104 e sen(9 ) = 0,157, o ngulo de


reflexo no espelho de
o
a) 2 .
o
b) 3 .
o
c) 4 .
o
d) 6 .

s e n 6 30 18 0

s e n 3 0 2
v

386. (UDESC 2010) Um feixe de luz de comprimento de onda igual


-9
a 600 x 10 m, no vcuo, atravessa um bloco de vidro de
ndice de refrao igual a 1,50. A velocidade e o comprimento
de onda da luz no vidro so, respectivamente, iguais a:
8
-9
a) 3,0 x 10 m/s e 600 x 10 m
8
-7
b) 3,0 x 10 m/s e 4,0 x 10 m
8
-9
c) 2,0 x 10 m/s e 400 x 10 m
7
-9
d) 5,0 x 10 m/s e 900 x 10 m
8
-9
e) 2,0 x 10 m/s e 900 x 10 m
RESPOSTA: C
9
COMENTRIO: Dados: = 600 10 m; nvidro = 1,5.
8
Considerando a velocidade da luz no vcuo, c = 3 10 m/s,
temos:

RESPOSTA: B
COMENTRIO: Dados: np = 1,5; nar = 1; sen 6 = 0,104 e sen 9
= 0,157.
A figura a seguir ilustra a situao, mostrando a trajetria do
raio at a reflexo no espelho plano.

. Ento:
v vidro vidro

1,5 =

3 108
v vidro

v vidro

3 108
1,5

vvidro = 2 10

m/s.
1,5 =

600 109
vidro

vidro

600 109
1,5

vidro =

-9

400 10 m.

Na primeira face, a incidncia normal, portanto no h


desvio. Para a segunda face, aplicamos a lei de Snell:
np sen i = nar sen r 1,5 sen 6 = 1 sen r 1,5 (0,104) = sen
r sen r = 0,157 r = 9. A partir da, acompanhando a
figura, conclumos que o ngulo de reflexo no espelho plano
3.

387. (UDESC 2010) Um basto colocado sequencialmente em


trs recipientes com lquidos diferentes. Olhando-se o basto
atravs de cada recipiente, observam-se as imagens I, II e III,
conforme ilustrao a seguir, pois os lquidos so
transparentes. Sendo nAr, nI, nII e nIII os ndices de refrao do
ar, do lquido em I, do lquido em II e do lquido em III,
respectivamente, a relao que est correta :

385. (PUC-RJ 2010) Uma onda eletromagntica se propaga no


vcuo e incide sobre uma superfcie de um cristal fazendo um
o
ngulo de 1 = 60 com a direo normal a superfcie.

a) nAr < nI < nII


b) nII < nAr < nIII
c) nI > nII > nIII
d) nIII > nII > nI
e) nIII < nI < nII

Considerando a velocidade de propagao da onda no vcuo


8
como c = 3 x 10 m/s e sabendo que a onda refratada faz um
o
ngulo de 2 = 30 com a direo normal, podemos dizer que
a velocidade de propagao da onda no cristal em m/s
8
a) 1 10
b)

2 108

c)

3 10

d)

4 108

e)

5 108

RESPOSTA: E
COMENTRIO: A lei de Snell diz que nsen cte , isto , se
o ndice de refrao aumenta, o ngulo diminui e vice versa.
Note que na figura abaixo quando a luz passa do lquido para o
ar o ngulo no se modifica. Ento

nliq nar

RESPOSTA: C
8
COMENTRIO: Dados: 1 = 60; 2 = 30; c = 3 10 m/s.
Aplicando a lei de Snell:

108

Azul
Violeta

1,528
1,532

A partir das informaes e da tabela apresentadas, em relao


a um raio de luz branca proveniente do ar que incide no vidro,
correto afirmar que
a) as cores so percebidas porque o vidro apresenta
aproximadamente o mesmo ndice de refrao para todas
elas.
b) h a predominncia da luz verde porque o ndice de
refrao do vidro para essa cor aproxima-se da mdia dos
ndices para todas as cores.
c) a luz violeta a que sofre menor desvio.
d) a luz vermelha a que sofre maior desvio.
e) a luz azul sofre desvio maior do que a luz vermelha.

Note que na figura abaixo quando a luz passa do lquido para o


ar o ngulo aumenta. Ento: nliq nar

RESPOSTA: E
COMENTRIO:

Da Lei de Snell:

Note que, na figura abaixo, quando a luz passa do lquido para


o ar, o ngulo diminui. Ento: nliq nar

n seni
seni nvidro

senr ar
. Por essa expresso,
senr
nar
nvidro
vemos que a luz que apresenta menor ngulo de refrao (a
que mais desvia) a que apresenta maior ndice de refrao,
no caso o violeta. Alis, os desvios crescem na sequncia
mostrada na figura: Vermelha (Vm), Alaranjada (Al), Amarela
(Am), Verde (Vd), Azul (Az) e Violeta (Vl).

389. (UNESP 2012) Em um experimento didtico de ptica


geomtrica, o professor apresenta aos seus alunos o diagrama
da posio da imagem conjugada por uma lente esfrica
delgada, determinada por sua coordenada p, em funo da
posio do objeto, determinada por sua coordenada p, ambas
medidas em relao ao centro ptico da lente.
388. (PUCRS 2010) Resolver a questo com base nas informaes a
seguir.
O efeito causado pela incidncia da luz solar sobre um vidro,
dando origem a um feixe colorido, conhecido como
disperso da luz branca. Este fenmeno resultado da
refrao da luz ao atravessar meios diferentes, no caso, do ar
para o vidro. Na superfcie de separao entre os dois meios, a
luz sofre um desvio em relao direo original de
propagao desde que incida no vidro em uma direo
diferente da direo normal superfcie.
A tabela a seguir informa os ndices de refrao de um tipo de
vidro para algumas das diferentes cores que compem a luz
branca.
Analise as afirmaes.
Cor
Vermelho
Amarelo
Verde

ndice de refrao do vidro relativo ao ar


1,513
1,517
1,519

I. A convergncia da lente utilizada 5 di.


II. A lente utilizada produz imagens reais de objetos colocados
entre 0 e 10 cm de seu centro ptico.
109

III. A imagem conjugada pela lente a um objeto linear colocado


a 50 cm de seu centro ptico ser invertida e ter

uma cmera formada por uma lente, uma caixa vedada e um


filme, como ilustra, esquematicamente, a figura.

1
da altura
4

do objeto.
Est correto apenas o contido em
a) II.
b) III.
c) I e II.
d) I e III.
e) II e III.
Considere que a distncia focal da lente 55mm e que D e DO
representam, respectivamente, as distncias da lente ao filme
e do objeto lente. Nesse caso, para realizar a
macrofotografia, os valores de D e DO devem ser
a) D = 110mm e DO = 55mm.
b) D = 55mm e DO = 110mm.
c) D = 110mm e DO = 110mm.
d) D = 55mm e DO = 55mm.
e) D = 55mm e DO = 220mm.

RESPOSTA: B
COMENTRIO: Analisando cada uma das afirmativas:
I. (Incorreta). Do grfico dado, tiramos que: para p = 20 cm =
0,2 m p = 20 cm = 0,2 m. Substituindo esses valores na
equao dos pontos conjugados, e lembrando que a
convergncia (V), em dioptria, igual ao inverso da distncia
focal (f), em metro, temos:
1 1 1
1 p' p
p p'

f p p'
f
p p'
p p'

RESPOSTA: C
COMENTRIO: Para que a imagem apresente o mesmo
tamanho que o objeto, devemos posicionar o objeto no ponto
antiprincipal de uma lente convergente, ficando a imagem
com o mesmo tamanho e com a mesma distncia da lente,
comparado ao objeto.

0,2 0,2 0,04

f 0,1 m.
0,2 0,2
0,4
1
1
V
V 10 di.
f 0,1
II. (Incorreta). Analisando o grfico, conclumos que, para
objetos colocados de 0 a 10 cm da lente, a imagem virtual (p
< 0).
III. (Correta). Dado: p = 50 cm = 0,5 m.

Da afirmativa I, a distncia focal da lente f = 0,1 m.


Sendo (A) o aumento linear transversal, h a altura do objeto e
h a altura da imagem, da equao do aumento, vem:
h'
f

h f p
1
h' h.
4
A

h'
0,1
0,1

h 0,1 0,5 0,4

h'
1

h
4

O sinal negativo indica que a imagem invertida.


390. (UFPR 2012) Um datiloscopista munido de uma lupa analisa
uma impresso digital. Sua lupa constituda por uma lente
convergente com distncia focal de 10 cm. Ao utiliz-la, ele v
a imagem virtual da impresso digital aumentada de 10 vezes
em relao ao tamanho real. Com base nesses dados, assinale
a alternativa correta para a distncia que separa a lupa da
impresso digital.
a) 9,0 cm.
b) 20,0 cm.
c) 10,0 cm.
d) 15,0 cm.
e) 5,0 cm.

RESPOSTA: A
COMENTRIO: Aplicando a equao de Gauss, vem:

Y Y0 D D0 x
Considerando que f = 55mm e a equao de conjugao das
lentes esfricas delgadas

1 1 1

, teremos:
f D D0

1 1 1
1
1 1

x 110mm
f D D0
55 x x
D D0 x 110mm
392. (UFU 2011) A tabela abaixo mostra o valor aproximado dos
ndices de refrao de alguns meios, medidos em condies
normais de temperatura e presso, para um feixe de luz
incidente com comprimento de onda de 600 nm

1 1 1
1
1
1
9

p 9cm
f p p'
10 p 10p 10p

Material
Ar
gua (20 C)
Safira
Vidro de altssima disperso
Diamante

391. (UFF 2012) A macrofotografia uma tcnica utilizada para


fotografar pequenos objetos. Uma condio que deve ser
obedecida na realizao dessa tcnica que a imagem do
objeto no filme deve ter o mesmo tamanho do objeto real, ou
seja, imagem e objeto devem estar na razo 1: 1. Suponha
110

ndice de refrao
1,0
1,3
1,7
1,9
2,4

O raio de luz que se propaga inicialmente no diamante incide


com um ngulo i 30 em um meio desconhecido, sendo o

O ar quente sobe, fazendo com que as camadas mais baixas se


tornem mais rarefeitas. Portanto, h aumento da densidade
com a altura da camada. Consequentemente, o ndice de
refrao tambm sofre um aumento, sendo as camadas
inferiores menos refringentes. A passagem de um raio de uma
camada (+) refringente para outra () refringente faz com que
o raio se afaste da normal na trajetria descendente, fazendo
aumentar o ngulo de refrao, at atingir o ngulo limite e a
reflexo total, acontecendo o inverso na trajetria
ascendente.

ngulo de refrao r 45 .
O meio desconhecido :
a) Vidro de altssima disperso
b) Ar
c) gua (20C)
d) Safira
RESPOSTA: D
COMENTRIO: Lei de Snell:

n1.seni n2.senr
2,4.sen30 n2.sen45

2,4 0,5 n2.

2
n2 1,70
2

393. (UFF 2011) O fenmeno da miragem, comum em desertos,


ocorre em locais onde a temperatura do solo alta.
Raios luminosos chegam aos olhos de um observador por dois
caminhos distintos, um dos quais parece proveniente de uma
imagem especular do objeto observado, como se esse
estivesse ao lado de um espelho dgua (semelhante ao da
superfcie de um lago).
Um modelo simplificado para a explicao desse fenmeno
mostrado na figura abaixo.

394. (UFPA 2011) Os ndios amaznicos comumente pescam com


arco e flecha. J na sia e na Austrlia, o peixe arqueiro
captura insetos, os quais ele derruba sobre a gua, acertandoos com jatos disparados de sua boca. Em ambos os casos a
presa e o caador encontram-se em meios diferentes. As
figuras abaixo mostram qual a posio da imagem da presa,
conforme vista pelo caador, em cada situao.

Identifique, em cada caso, em qual dos pontos mostrados, o


caador deve fazer pontaria para maximizar suas chances de
acertar a presa.
a) Homem em A; peixe arqueiro em 1
b) Homem em A; peixe arqueiro em 3
c) Homem em B; peixe arqueiro em 2
d) Homem em C; peixe arqueiro em 1
e) Homem em C; peixe arqueiro em 3

O raio que parece provir da imagem especular sofre refraes


sucessivas em diferentes camadas de ar prximas ao solo.
Esse modelo reflete um raciocnio que envolve a temperatura,
densidade e ndice de refrao de cada uma das camadas.
O texto a seguir, preenchidas suas lacunas, expe esse
raciocnio.
A temperatura do ar ___________________ com a altura da
camada, provocando _________________ da densidade e
_________________ do ndice de refrao; por isso, as
refraes sucessivas do raio descendente fazem o ngulo de
refrao ______________ at que o raio sofra reflexo total,
acontecendo o inverso em sua trajetria ascendente at o
olho do observador.

RESPOSTA: E
COMENTRIO:

Assinale a alternativa que completa corretamente as lacunas.


a) aumenta diminuio aumento diminuir
b) aumenta diminuio diminuio diminuir
c) diminui aumento aumento aumentar
d) diminui aumento diminuio aumentar
e) no varia diminuio diminuio aumentar
RESPOSTA: C
COMENTRIO:

A luz sempre vai do objeto para o observador.


No primeiro caso, o peixe objeto e o homem o observador.
A luz est passando da gua (meio mais refringente) para o ar
(meio menos refringente), afastando-se da normal, de acordo
com a lei de Snell. Por isso o homem deve fazer pontaria em C.
No segundo caso, o inseto objeto e o peixe arqueiro o
observador. A luz est passando do ar (meio menos
refringente) para a gua (meio mais refringente),
aproximando-se da normal, de acordo com a lei de Snell. Por
isso o peixe arqueiro deve fazer pontaria em 3.

O asfalto se aquece, aquecendo as camadas de ar prximas a


ele; quanto mais baixa a camada, maior a sua temperatura.
Por isso a temperatura do ar diminui com a altura da camada.
111

395. (UFLA 2010) O ndice de refrao absoluto da luz em um meio


a relao entre a velocidade da luz no vcuo e a velocidade
da luz nesse meio. Dessa forma, CORRETO afirmar que
a) dependendo do tipo de meio, o ndice de refrao absoluto
pode ser menor que 1.
b) os meios mais refringentes possuem ndices de refrao
absolutos maiores que os meios menos refringentes.
c) o ndice de refrao absoluto de um meio diferente do
vcuo constante e independente da cor da luz.
d) no vcuo, com a ausncia de um meio material, o ndice de
refrao absoluto zero.

397. (UFV 2010) Analise as afirmativas a seguir:


I. Em virtude da refrao na atmosfera terrestre, um
observador na Terra pode ver o Sol mesmo quando esse est
totalmente abaixo da linha do horizonte.
II. Quando a luz passa do ar para a gua, existe um ngulo de
incidncia para o qual ocorre a reflexo total.
III. Quando uma onda sonora de frequncia f passa do ar para
a gua, a sua frequncia se altera.

RESPOSTA: B
COMENTRIO: Mais refringente significa maior ndice de
refrao.

Est CORRETO o que se afirma em:


a) I, II e III.
b) II, apenas.
c) II e III, apenas.
d) I, apenas.

396. (UFMG 2010) Um arco-ris forma-se devido disperso da luz


do Sol em gotas de gua na atmosfera.
Aps incidir sobre gotas de gua na atmosfera, raios de luz so
refratados; em seguida, eles so totalmente refletidos e
novamente refratados.
Sabe-se que o ndice de refrao da gua para a luz azul
maior que para a luz vermelha.
Considerando essas informaes, assinale a alternativa em
que esto mais bem representados os fenmenos que
ocorrem em uma gota de gua e do origem a um arco-ris.

RESPOSTA: D
COMENTRIO: Comentemos cada uma das afirmaes:
(I) Correta: Como a atmosfera transparente, mas no
homognea, os raios solares sofrem desvio ao atravess-la,
fazendo com que o pr do sol seja uma miragem. Como, na
refrao, as radiaes de menor frequncia (vermelha,
amarela e alaranjada) sofrem menor desvio, o observador v a
imagem do Sol com predominncia dessas cores, da, aquele
tom roseado. A Figura ilustra esse fenmeno (de maneira
exagerada).

a)

b)

(II) Errada. O ar menos refringente que a gua. Ao passar do


meio menos para o mais refringente, a luz aproxima da
normal, no ocorrendo reflexo total. Esse fenmeno s
ocorre quando o sentido de propagao da luz do meio mais
para o menos refringente, quando o ngulo de incidncia
maior que o ngulo limite.
(III) Errada. Na refrao no h alterao da frequncia.

c)

398. (UFPR 2010) Descartes desenvolveu uma teoria para explicar


a formao do arco-ris com base nos conceitos da ptica
geomtrica. Ele sups uma gota de gua com forma esfrica e
a incidncia de luz branca conforme mostrado de modo
simplificado na figura.

d)

RESPOSTA: A
COMENTRIO:

De

acordo

com

Lei

de

Snell:

n seni
seni ngota

senr ar
senr
nar
ngota
Como o ndice de refrao da gota maior para a luz azul, essa
radiao apresenta menor ngulo de refrao (ra < rv) ou seja,
sofre maior desvio ao se refratar.

112

COMENTRIO: De acordo com a lei de Snell, quando a luz


passa do meio menos para o mais refringente a luz aproximase da normal e, quando passa do mais para o menor
refringente, a luz afasta-se da normal.
As figuras mostram as duas situaes propostas na questo: n
> 1,4 e n < 1,4. Analisando-as, conclumos que para n > 1,4, o
objeto comporta-se com lente convergente.

O raio incidente sofre refrao ao entrar na gota (ponto A) e


apresenta uma decomposio de cores. Em seguida, esses
raios sofrem reflexo interna dentro da gota (regio B) e saem
para o ar aps passar por uma segunda refrao (regio C).
Posteriormente, com a experincia de Newton com prismas,
foi possvel explicar corretamente a decomposio das cores
da luz branca. A figura no est desenhada em escala e, por
simplicidade, esto representados apenas os raios violeta e
vermelho, mas deve-se considerar que entre eles esto os
raios das outras cores do espectro visvel.
Sobre esse assunto, avalie as seguintes afirmativas:
1. O fenmeno da separao de cores quando a luz sofre
refrao ao passar de um meio para outro chamado de
disperso.
2. Ao sofrer reflexo interna, cada raio apresenta ngulo de
reflexo igual ao seu ngulo de incidncia, ambos medidos em
relao reta normal no ponto de incidncia.
3. Ao refratar na entrada da gota (ponto A na figura), o violeta
apresenta menor desvio, significando que o ndice de refrao
da gua para o violeta menor que para o vermelho.

400. (IFSC 2011) Analise as proposies abaixo:


I. Classificamos as lentes em relao ao seu formato e em
relao ao meio em que elas esto imersas.
II. Quando desejamos concentrar os raios luminosos que vm
do Sol em um nico ponto, podemos utilizar lentes de bordas
grossas desde que elas estejam imersas em um meio de ndice
de refrao maior que o seu.
III. Para que a imagem conjugada por uma lente seja ntida,
devemos levar em considerao a espessura da lente e a
maneira como os raios incidentes chegam a ela.
IV. Lentes esfricas so usadas em instrumentos pticos para
aumentar ou diminuir o tamanho da imagem, devido ao fato
da luz sofrer disperso ao atravess-las.
V. Uma lente convergente possui sempre os raios de curvatura
de suas faces iguais.

Assinale a alternativa correta.


a) Somente a afirmativa 1 verdadeira.
b) Somente a afirmativa 2 verdadeira.
c) Somente as afirmativas 1 e 2 so verdadeiras.
d) Somente as afirmativas 1 e 3 so verdadeiras.
e) Somente as afirmativas 2 e 3 so verdadeiras.
RESPOSTA: C
COMENTRIO: 1. Verdadeira. Disperso o fenmeno que
ocorre quando um feixe de luz policromtica sofre refrao,
com separao das cores componentes.
2. Verdadeira. O ngulo de incidncia igual ao de reflexo
(2 lei da reflexo).
3. Falsa. A radiao violeta que apresenta maior desvio.

Assinale a alternativa correta.


a) Apenas as proposies I, II e IV so verdadeiras.
b) Apenas as proposies I, II e III so verdadeiras.
c) Apenas as proposies II, III e V so verdadeiras.
d) Apenas as proposies II, IV e V so verdadeiras.
e) Apenas as proposies III, IV e V so verdadeiras.

399. (Fuvest 2011) Um objeto decorativo consiste de um bloco de


vidro transparente, de ndice de refrao igual a 1,4, com a
forma de um paraleleppedo, que tem, em seu interior, uma
bolha, aproximadamente esfrica, preenchida com um lquido,
tambm transparente, de ndice de refrao n. A figura a
seguir mostra um perfil do objeto.

Nessas condies, quando a luz visvel incide


perpendicularmente em uma das faces do bloco e atravessa a
bolha, o objeto se comporta, aproximadamente, como
a) uma lente divergente, somente se n > 1,4.
b) uma lente convergente, somente se n > 1,4.
c) uma lente convergente, para qualquer valor de n.
d) uma lente divergente, para qualquer valor de n.
e) se a bolha no existisse, para qualquer valor de n.

RESPOSTA: B
COMENTRIO: I. Correta.
II. Correta.
III. Correta.
IV. Incorreta. Se a lente delgada, praticamente no ocorre
disperso.
V. Incorreta. As faces no tm necessariamente raios iguais,
podendo, inclusive, uma das delas ser plana.
401. (Fuvest 2011) O olho o senhor da astronomia, autor da
cosmografia, conselheiro e corretor de todas as artes humanas
(...). o prncipe das matemticas; suas disciplinas so
intimamente certas; determinou as altitudes e dimenses das
estrelas; descobriu os elementos e seus nveis; permitiu o
anncio de acontecimentos futuros, graas ao curso dos
astros; engendrou a arquitetura, a perspectiva, a divina
pintura (...). O engenho humano lhe deve a descoberta do
fogo, que oferece ao olhar o que as trevas haviam roubado.
Leonardo da Vinci, Tratado da pintura.
Considere as afirmaes abaixo:
I. O excerto de Leonardo da Vinci um exemplo do
humanismo renascentista que valoriza o racionalismo como

RESPOSTA: B
113

instrumento de investigao dos fenmenos naturais e a


aplicao da perspectiva em suas representaes pictricas.
II. Num olho humano com viso perfeita, o cristalino focaliza
exatamente sobre a retina um feixe de luz vindo de um objeto.
Quando o cristalino est em sua forma mais alongada,
possvel focalizar o feixe de luz vindo de um objeto distante.
Quando o cristalino encontra-se em sua forma mais
arredondada, possvel a focalizao de objetos cada vez mais
prximos do olho, at uma distncia mnima.
III. Um dos problemas de viso humana a miopia. No olho
mope, a imagem de um objeto distante forma-se depois da
retina. Para corrigir tal defeito, utiliza-se uma lente
divergente.
Est correto o que se afirma em
a) I, apenas.
b) I e II, apenas.
c) I e III, apenas.
d) II e III, apenas.
e) I, II e III.
RESPOSTA: B
COMENTRIO: I. Correta.
II. Correta.
III. Incorreta. Num olho mope, a imagem de um objeto
distante forma-se antes da retina.

Sobre lentes, espelhos e cmara escura CORRETO afirmar:


a) A lente utilizada para projetar a imagem sobre a tela a
mesma que se utiliza para a correo da miopia.
b) O espelho cncavo utilizado produz uma imagem virtual
direita e maior que o objeto.
c) O espelho cncavo era utilizado para projetar uma imagem
real invertida e menor que o objeto. A funo da lente
convergente era ampliar a imagem.
d) Na cmara lcida a imagem vista pelo observador real
invertida e menor que o objeto.
e) A lente utilizada na cmara escura produz uma imagem com
as mesmas caractersticas de uma lupa.

402. (PUCPR 2010) David Hockney, pintor pop ingls, um dos mais
importantes artistas da atualidade, defende a ideia de que
alguns grandes mestres da pintura no passado teriam
recorrido a dispositivos pticos para projetar sobre as telas as
imagens que pintavam. Hockney procurou saber que recurso
ptico eles poderiam ter usado e descobriu a cmara lcida,
inveno patenteada, em 1807, pelo fsico ingls William Hyde
Wollaston. A cmara lcida um pequeno prisma com quatro
ou cinco faces, uma semiespelhada e outra espelhada, que
permite ao pintor ver sobre a tela ou papel onde faz o esboo
a imagem do objeto que pinta, sua frente.

RESPOSTA: C
COMENTRIO: a) Errada. A lente usada para projees de
imagens (de objetos reais) convergente, e para correo de
miopia utiliza-se lente divergente.
b) Errada. Imagens virtuais no so projetveis.
c) Correta.
d) Errada. As faces dos prismas so espelhos planos,
fornecendo imagens de mesmo tamanho.
e) Errada. A lupa fornece imagem virtual, no podendo ser
projetada.
403. (Mackenzie 2010) A lupa um instrumento ptico conhecido
popularmente por Lente de Aumento, mas tambm
denominada microscpio simples. Ela consiste de uma lente
______________________ de pequena distncia focal e, para
ser utilizada com o seu fim especfico, o objeto a ser
observado por meio dela dever ser colocado sobre o eixo
principal, entre o seu ______________________ e o seu
______________________.
As lacunas so preenchidas corretamente quando se utilizam,
na ordem de leitura, as informaes
a) convergente, centro ptico e foco principal objeto.
b) convergente, ponto antiprincipal objeto e foco principal
objeto.
c) divergente, centro ptico e foco principal objeto.
d) divergente, ponto antiprincipal objeto e foco principal
objeto.
e) convergente, ponto antiprincipal imagem e foco principal
imagem.

Outros recursos pticos tambm eram utilizados, tais como:


lentes, espelhos cncavos e cmara escura, j com
implementos de lentes e espelhos. A cmara escura era usada
por artistas no sculo XVI, como um auxlio para os esboos
nas pinturas, conforme ilustrado a seguir:

RESPOSTA: A
114

COMENTRIO: Observe o esquema:

A imagem da lente serve de objeto para o diptro que altera a


posio da imagem, mas no o seu tamanho. Portanto, com
diptro ou no, a ampliao ser a mesma.

f
pf

405. (PUCSP 2010) O olho humano pode ser entendido como um


sistema ptico composto basicamente por duas lentes
crnea (A) e cristalino (B). Ambas devem ser transparentes e
possuir superfcies lisas e regulares para permitirem a
formao de imagens ntidas. Podemos classificar as lentes
naturais de nossos olhos, A e B, respectivamente, como sendo:

404. (IME 2010)

a) convergente e convergente.
b) convergente e divergente.
c) divergente e divergente.
d) divergente e convergente.
e) divergente e plana.
A figura apresenta, esquematicamente, uma lente
convergente de distncia focal f posicionada no plano de
transio entre o vcuo e um material de ndice de refrao n.
O fator de ampliao (tamanho da imagem dividido pelo
tamanho do objeto) de um objeto muito pequeno (se
comparado com as dimenses da lente) colocado a uma
distncia p da lente :
a)

f
pf

b)

f
npf

RESPOSTA: A
COMENTRIO:

d)

nf
p nf

Em uma pessoa adulta, o globo ocular normal apresenta


vergncia que varia de 51 di a 64 di. Os mais importantes
responsveis por essa vergncia so a crnea, com vergncia
de 43 di, e o cristalino, com vergncia que pode variar de 13 di
a 26 di. Ambos funcionam como lentes convergentes pois so
de bordas finas, com ndice de refrao maior que o do meio.

e)

f
np f

406. (UFSM 2011) Na figura a seguir, so representados um objeto


(O) e a sua imagem (I) formada pelos raios de luz

nf
c)
pf

RESPOSTA: A
COMENTRIO: Se no houvesse o material direita, a imagem
estaria numa posio relativa lente dada pela expresso de
Gauss.

1 1 1

f p p'

p'

1 1 1

p' f p

1 pf

p'
fp

fp
p f

Assinale a alternativa que completa corretamente as lacunas.


A lente em questo _________________, porque , para um
objeto
real,
a
imagem

_________
e
aparece________________ que o objeto.
a) convergente - real - menor
b) convergente - virtual - menor
c) convergente - real - maior

A ampliao provocada pela lente seria:

p'
f

p
p f
115

d) divergente - real - maior


e) divergente - virtual - menor

b) divergente e tem distncia focal 40 cm.


c) convergente e tem distncia focal 15 cm.
d) convergente e tem distncia focal 20 cm.
e) convergente e tem distncia focal 45 cm.

RESPOSTA: A
COMENTRIO: Somente lente convergente conjuga imagem
real para um objeto real.

RESPOSTA: D
COMENTRIO: Como a imagem virtual direita e maior, a
lente convergente.
O aumento linear transversal :

407. (EEWB 2011) Um aluno possui hipermetropia e s consegue


ler se o texto estiver a pelo menos 1,5 m de distncia. Qual
deve ser a distncia focal da lente corretiva para que ele possa
ler se o texto for colocado a 25 cm de seus olhos?
a) 10 cm
b) 20 cm
c) 30 cm
d) 40 cm
RESPOSTA: C
COMENTRIO:

1 1 1
1 1
1

f 25 150
f p p'
1
6
1
5

f 30cm .
f 150 150 150

A
Mas:

f p
f 20 cm.
A

f
f 12

2,5f 30 f

esfrica. Quando um visitante est a 1

1,5f 30

m da porta, esse

dispositivo ptico forma, para o observador, no interior da


residncia, uma imagem trs vezes menor e direita do rosto
do visitante. correto afirmar que a distncia focal e o tipo da
lente que constituem o olho mgico so, respectivamente:
a) 1
b)
c)
d)
e)

m , divergente.
2
1 m , divergente.
4
1 m , convergente.
4
1 m , convergente.
2
1 m , convergente.
4

RESPOSTA: B
COMENTRIO: Dados: p = 1/2 m; A 1/3 m.

RESPOSTA: B
COMENTRIO:

409. (UFTM 2011) As figuras mostram um mesmo texto visto de


duas formas: na figura 1 a olho nu, e na figura 2 com o auxlio
de uma lente esfrica. As medidas nas figuras mostram as
dimenses das letras nas duas situaes.

2,5

410. (UFJF 2011) O olho mgico um dispositivo ptico de


segurana residencial constitudo simplesmente de uma lente

408. (UFPB 2011) Um projetor de slide um dispositivo bastante


usado em salas de aula e/ou em conferncias, para projetar,
sobre uma tela, imagens ampliadas de objetos. Basicamente,
um projetor constitudo por lentes convergentes.
Nesse sentido, considere um projetor formado por apenas
uma lente convergente de distncia focal igual a 10 cm. Nesse
contexto, a ampliao da imagem projetada, em uma tela a 2
m de distncia do projetor, de:
a) 20 vezes
b) 19 vezes
c) 18 vezes
d) 17 vezes
e) 16 vezes

1 1 1
1
1 1
1 1 1


9,5
f p p'
0,1 p 2
p 0,1 2
1
p
.
9,5
p'
2
A

19vezes .
p 1/ 9,5

y ' 10

2,5.
y
4

Da equao do aumento linear transversal:


A

f
f p

f
1
f
2

3f f

1
2

2f

1
2

1
m.
4

Como f < 0, a lente divergente.


411. (Unesp 2010) Escolhido como o Ano Internacional da
Astronomia, 2009 marcou os 400 anos do telescpio
desenvolvido pelo fsico e astrnomo italiano Galileu Galilei.
Tal instrumento ptico constitudo de duas lentes: uma
convergente (objetiva) e outra divergente (ocular). A tabela
indica o perfil de 4 lentes I, II, III e IV que um aluno dispe para
montar um telescpio como o de Galileu.
Lente
Perfil

I
Biconvexa

II
Planocncava

III
Convexocncava

IV
Plano-convexa

Para que o telescpio montado pelo aluno represente


adequadamente um telescpio semelhante ao desenvolvido
por Galileu, ele deve utilizar a lente.
a) I como objetiva e a lente II como ocular.
b) II como objetiva e a lente I como ocular.
c) I como objetiva e a lente IV como ocular.
d) III como objetiva e a lente I como ocular.
e) III como objetiva e a lente IV como ocular

Sabendo que a lente foi posicionada paralelamente folha e a


12 cm dela, pode-se afirmar que ela
a) divergente e tem distncia focal 20 cm.
116

RESPOSTA: A
COMENTRIO: O telescpio, obviamente, usado no ar.
Assim, lentes de bordas finas (bi-convexa, plano-convexa ou
cncavo-convexa) so convergentes e lentes de bordas grossas
(bi-cncava, plano-cncava ou convexo-cncava) so
divergentes.

(3) Quando a imagem real, invertida e menor, o objeto


encontra-se entre A e F.
Falsa. Observe a construo.

412. (UPE 2010) A figura a seguir representa uma lente delgada


convergente. O ponto o o centro ptico, F o foco principal
objeto, f a distncia focal e A o ponto antiprincipal, que
dista em relao ao centro ptico 2f.

(4) Quando o objeto encontra-se entre o foco e o centro


ptico, a imagem real, direita e maior.
Falsa. Observe a construo.

Em referncia ao posicionamento do objeto e respectiva


imagem, analise as proposies que se seguem.
(1) Quando a distncia do objeto ao centro ptico maior que
o dobro da distncia focal, a imagem obtida real, invertida e
menor.
(2) Quando o objeto se encontra sobre o ponto antiprincipal, a
imagem real, invertida e de mesmo tamanho.
(3) Quando a imagem real, invertida e menor, o objeto
encontra-se entre A e F.
(4) Quando o objeto encontra-se entre o foco e o centro
ptico, a imagem real, direita e maior.
(5) Quando a imagem imprpria, o objeto encontra-se na
metade do ponto antiprincipal.

(5) Quando a imagem imprpria, o objeto encontra-se na


metade do ponto antiprincipal.
Verdadeira. Observe a construo.

A soma dos nmeros entre parnteses que corresponde aos


itens errados igual a
a) 15
b) 7
c) 6
d) 8
e) 4
RESPOSTA: B
COMENTRIO: (1) Quando a distncia do objeto ao centro
ptico maior que o dobro da distncia focal, a imagem
obtida real, invertida e menor.
Verdadeira. Observe a construo.

413. (UFRGS 2010) Um objeto delgado, com 10 cm de altura, est


posicionando sobre o eixo central de uma lente esfrica
delegada convergente, cuja distncia focal igual a 25 cm.
Considerando-se que a distncia do objeto lente de 50 cm,
a imagem formada pela lente
a) real e de mesmo tamanho que o objeto.
b) virtual e de mesmo tamanho que o objeto.
c) real e menor que o objeto.
d) virtual e menor que o objeto.
e) virtual e maior que o objeto.

(2) Quando o objeto se encontra sobre o ponto antiprincipal, a


imagem real, invertida e de mesmo tamanho.
Verdadeira. Observe a construo.

RESPOSTA: A
COMENTRIO: Dados: h = 10 cm; f = 25 cm; p = 50 cm.
Da equao de Gauss:

p f
1 1 1
p'
f p p'
pf

p'

50 25
p = 50
50 25

cm. (p > 0 Imagem real)


Calculando o aumento linear transversal (A):
117

p' 50
A = 1 (imagem invertida e do mesmo

p
50

Admita que o vapor dgua apresenta comportamento de gs


ideal. O volume de vapor dgua, em litros, gerado a partir da
fisso de 1 g de urnio, corresponde a:

tamanho).

a) 1,32 10

414. (IME 2010)

b) 2,67 10

c) 3,24 10

d) 7,42 10

RESPOSTA: B
COMENTRIO:
Dados:

R 8 102 atm L/mol ;

A figura apresenta, esquematicamente, uma lente


convergente de distncia focal f posicionada no plano de
transio entre o vcuo e um material de ndice de refrao n.
O fator de ampliao (tamanho da imagem dividido pelo
tamanho do objeto) de um objeto muito pequeno (se
comparado com as dimenses da lente) colocado a uma
distncia p da lente :
a)

f
pf

b)

f
npf

c)

nf
pf

d)

nf
p nf

e)

f
np f

Usando a equao de Clapeyron:


PV

m
mRT 3,6 104 8 10 2 5 102
RT V

M
MP
18 103 30

416. (UFF 2012) Uma quantidade de um gs ideal colocada em


um recipiente de vidro hermeticamente fechado e exposto ao
sol por um certo tempo. Desprezando-se a dilatao do
recipiente, assinale a alternativa que representa
corretamente, de forma esquemtica, os estados inicial (i) e
final (f) do gs em um diagrama PxT (Presso x Temperatura).
a)

1 1 1

p' f p

b)

1 pf

p'
fp

fp
p f

c)

A ampliao provocada pela lente seria:

p'
f

p
p f
d)

A imagem da lente serve de objeto para o diptro que altera a


posio da imagem, mas no o seu tamanho. Portanto, com
diptro ou no, a ampliao ser a mesma.

T 227C 5 102K ;

V 2,67 106 L.

1 1 1

f p p'

M 18 g 18 103 kg ;

P 30 atm .

RESPOSTA: A
COMENTRIO: Se no houvesse o material direita, a imagem
estaria numa posio relativa lente dada pela expresso de
Gauss.

p'

m 3,6 104 kg ;

f
pf
e)

FSICA III

415. (UERJ 2012) Em um reator nuclear, a energia liberada na


fisso de 1 g de urnio utilizada para evaporar a quantidade
4

de 3,6 10 kg de gua a 227C e sob 30 atm, necessria


para movimentar uma turbina geradora de energia eltrica.
118

RESPOSTA: B
COMENTRIO: O enunciado trata de uma transformao
gasosa com volume constante, devido dilatao de o
recipiente ser desconsiderada, ou seja,

espao do reservatrio onde o gs est armazenado. As


colises das molculas entre si e com as paredes do
reservatrio so perfeitamente elsticas. Entre duas colises
sucessivas, as molculas descrevem um MRU. A energia
cintica de translao das molculas diretamente
proporcional temperatura do gs. Com base nessas
informaes, considere as seguintes afirmativas:

P.V
k onde:
T

P: presso do gs;
T: temperatura do gs;
V: volume do gs, que constante;
k : constante.

1. As molculas se deslocam todas em trajetrias paralelas


entre si.
2. Ao colidir com as paredes do reservatrio, a energia cintica
das molculas conservada.
3. A velocidade de deslocamento das molculas aumenta se a
temperatura do gs for aumentada.

P.V
k
k P .T (funo da transformao gasosa)
T
V
k
Como
uma constante, a funo que representa a
V

Assinale a alternativa correta.


a) Somente a afirmativa 1 verdadeira.
b) Somente a afirmativa 2 verdadeira.
c) Somente a afirmativa 3 verdadeira.
d) Somente as afirmativas 1 e 2 so verdadeiras.
e) Somente as afirmativas 2 e 3 so verdadeiras.

transformao gasosa nos mostra que o aumento da


temperatura (T) provocar o aumento da presso (P).
Como o gs exposto ao sol por certo tempo, ele ir receber
calor proveniente do sol, que provocar o aumento de sua
temperatura com o respectivo aumento de sua presso.

RESPOSTA: E
COMENTRIO: 1. Falsa. O movimento das molculas
absurdamente desordenado.
2. Verdadeira. Colises elsticas.
3. Verdadeira. A energia cintica aumenta devido ao aumento
da velocidade.

Diagrama [A]: o valor da presso de i para f se manteve


constante, ou seja, no aumentou como previsto. FALSO!
Diagrama [B]: os valores da presso e da temperatura
aumentaram de i para f como previsto. VERDADEIRO!
Diagrama [C]: o valor da temperatura de i para f se manteve
constante, ou seja, no aumentou como previsto. FALSO!

419.

(UESC 2011)

Diagrama [D]: o valor da temperatura diminuiu de i para f, ou


seja, no aumentou como previsto. FALSO!
Diagrama [E]: alm do valor da temperatura ter diminudo de i
para f, a presso se manteve constante, ou seja, no
aumentaram como previsto. FALSO!

417. (UFPB) Antes de iniciar uma viagem, um motorista cuidadoso


calibra os pneus de seu carro, que esto temperatura
2
ambiente de 27C, com uma presso de 30 lb/pol . Ao final da
viagem, para determinar a temperatura dos pneus, o
motorista mede a presso dos mesmos e descobre que esta
2
aumentou para 32 lb/pol . Se o volume dos pneus permanece
inalterado e se o gs no interior ideal, o motorista
determinou a temperatura dos pneus como sendo:

Considere 4,0mols de um gs ideal, inicialmente a 2,0C, que


descrevem um ciclo, conforme a figura. Sabendo-se que a
constante dos gases R = 0,082atm L/mol.K e 1,0atm =
5
1,0.10 Pa,
a anlise da figura permite afirmar:
a) O sistema apresenta a energia interna mxima no ponto D.
b) A temperatura da isoterma que contm o ponto C igual a
27,0C.
c) O sistema recebe, ao realizar a compresso isotrmica,
86,01J de energia.
d) O trabalho realizado pelo gs, em cada ciclo,
aproximadamente igual a 180,0W/s.
e) O sistema, ao realizar a expanso isobrica, apresenta a
variao da temperatura de 67,0K.

a) 17 C
b) 27 C
c) 37 C
d) 47 C
e) 57 C
RESPOSTA: D
COMENTRIO: A transformao isocrica:
2

p0 p

T0 T

Sendo p0 = 30 bf/pol ; p = 32 bf/pol ; T0 = (27 + 273) K = 300


K, vem:

30 32

T 320 k
300 T
o

Em graus Celsius: T = 320 273 T = 47 C


418. (UFPR 2012) Segundo a teoria cintica, um gs constitudo
por molculas que se movimentam desordenadamente no

RESPOSTA: B
COMENTRIO:

Pela

equao

de

Clapeyron,

temos:

PV 8,2x12
PV nRT T

300K 270 C .
nR 4 x 0,082
420. (Espcex (Aman) 2011) O grfico da presso (P) em funo do
volume (V) no desenho abaixo representa as transformaes
sofridas por um gs ideal. Do ponto A at o ponto B, o gs
sofre uma transformao isotrmica, do ponto B at o ponto
C, sofre uma transformao isobrica e do ponto C at o ponto
119

A, sofre uma transformao isovolumtrica. Considerando TA


,

m
R
M

TB e TC as temperaturas absolutas do gs nos pontos A, B

30 g

e C, respectivamente, pode-se afirmar que:

mol

0,08 atm L

1 atm 4 L
mol K

Mp V
RT

300 K

120

24

m = 5 g.

a)

TA TB e TB TC

b)

TA TB e TB TC

c)

TA TC e TB TA

d)

TA TC e TB TA

e)

TA TB TC

422. (IFSP 2011) No alto de uma montanha a 8 C, um cilindro


munido de um mbolo mvel de peso desprezvel possui 1
litro de ar no seu interior. Ao lev-lo ao p da montanha, cuja
presso de 1 atmosfera, o volume do cilindro se reduz a 900
3
cm e sua temperatura se eleva em 6 C. A presso no alto da
montanha aproximadamente, em atm, de
a) 0,66.
b) 0,77.
c) 0,88.
d) 0,99.
e) 1,08.

RESPOSTA: A
COMENTRIO: Como a evoluo AB isotrmica, TA

RESPOSTA: C
COMENTRIO: Dados: T1 = 8 C = 281 K; V1 = 1 L; P2 = 1 atm; V2
3
= 900 cm = 0,9 L; T2 = T1 + 6 = 287 K.

TB .

Como sabemos PV = nRT. Na evoluo BC, o volume aumenta


e a presso fica constante.
Portanto, a temperatura aumenta:

TB TC .

421. (UERJ 2011) A bola utilizada em uma partida de futebol uma


esfera de dimetro interno igual a 20 cm. Quando cheia, a bola
apresenta, em seu interior, ar sob presso de 1,0 atm e
temperatura de 27 C.
-1 -1
Considere = 3, R = 0,080 atm.L.mol .k e, para o ar,
-1
comportamento de gs ideal e massa molar igual a 30 g.mol .

Considerando o ar com gs ideal, pela equao geral dos gases


ideais:

P1 V1
T1

P2 V2
T2

P1 1
281

1 0,9
287

P1

P1 = 0,88 atm.
423. (UFRGS 2011) Um balo meteorolgico fechado tem volume
de

No interior da bola cheia, a massa de ar, em gramas,


corresponde a:
a) 2,5
b) 5,0
c) 7,5
d) 10,0

50,0 m3 ao nvel do mar, onde a presso atmosfrica

de l,0 l0 Pa e a temperatura de 27 C. Quando o balo


atinge a altitude de 25 km na atmosfera terrestre, a presso e
a temperatura assumem, respectivamente, os valores de
5

5,0 I03 Pa e - 63 C.
Considerando-se que o gs contido no balo se comporta
como um gs ideal, o volume do balo nessa altitude de

RESPOSTA: B
COMENTRIO: OBS: se a presso do ar no interior da bola de
1 atm a bola est vazia ou murcha. Quando se diz que a
bola est sob presso de 1 atm, refere-se presso
manomtrica, ou seja, acima da presso atmosfrica.
Portanto, no caso, a presso no interior da bola de 2 atm. No
entanto, resolvamos com os dados fornecidos.
Dados: D = 20 cm R = 10 cm = 1 dm; p = 1 atm; T = 27 C =
300 K; M = 30 g/mol;

a)

14,0 m3 .

b)

46,7 m3 .

c)

700,0 m3 .

d)

1.428,6 m3 .

e)

2.333,3 m3 .

R = 0,08 atmL/molK; = 3.

RESPOSTA: C
COMENTRIO:

Calculando o volume da bola:

P0 V0 PV
1,0x105 x50 5x103 xV

V 700m3 .
T0
T
300
210

V=

4 3 4
R (3)(1)3 V = 4 dm3 = 4 L.
3
3

Da equao de Clapeyron:

252,9
287

424. (Eewb 2011) Considere a compresso isobrica AB sofrida por


uma amostra de gs perfeito e representada no diagrama
presso x volume, mostrado abaixo.

120

Admita que no estado A, a temperatura do gs perfeito seja


igual a 127C. A temperatura atingida pelo gs ao atingir o
estado B vale:
a) 473K
b) 400C
c) 200F
d) - 73C

426. (UNICAMP 2011) Em abril de 2010, erupes vulcnicas na


Islndia paralisaram aeroportos em vrios pases da Europa.
Alm do risco da falta de visibilidade, as cinzas dos vulces
podem afetar os motores dos avies, pois contm materiais
que se fixam nas ps de sada, causando problemas no
funcionamento do motor a jato.
Uma erupo vulcnica pode ser entendida como resultante
da ascenso do magma que contm gases dissolvidos, a
presses e temperaturas elevadas. Esta mistura apresenta
aspectos diferentes ao longo do percurso, podendo ser
esquematicamente representada pela figura a seguir, onde a
colorao escura indica o magma e os discos de colorao
clara indicam o gs.

RESPOSTA: D
COMENTRIO:

Como

sabemos:

P0 .V0
P.V

n0 .T0
n.T

P0 .2a
P .a
0
n0 .400 n0 .T
T

400
200K 200 273 730 C .
2

425. (UDESC 2011) Uma dada massa gasosa, que est limitada em
um cilindro por um mbolo mvel, sofre as transformaes
representadas pelos seguintes grficos:

Segundo essa figura, pode-se depreender que


a) as exploses nas erupes vulcnicas se devem, na
realidade, expanso de bolhas de gs.
b) a expanso dos gases prximos superfcie se deve
diminuio da temperatura do magma.
c) a ascenso do magma facilitada pelo aumento da presso
sobre o gs, o que dificulta a expanso das bolhas.
d) a densidade aparente do magma prximo cratera do
vulco maior que nas regies mais profundas do vulco, o
que facilita sua subida.
RESPOSTA: A
COMENTRIO: Conforme sugere a figura, medida que as
bolhas sobem, elas sofrem expanso, pois reduz-se a presso
sobre elas.
427. (UFRGS 2010) Considere as afirmaes a seguir, sobre gases
ideais.
I. A constante R presente na equao de estado de gases pV =
nRT tem o mesmo valor para todos os gases ideais.
II. Volumes iguais de gases ideais diferentes, mesma
temperatura e presso, contm o mesmo nmero de
molculas.
III. A energia cintica mdia das molculas de um gs ideal
diretamente proporcional temperatura absoluta do gs.

Assinale a alternativa que contm a correta classificao das


trs transformaes apresentadas acima.
a) I. isovolumtrica / II. isobrica / III. isotrmica.
b) I. isotrmica / II. isobrica / III. isovolumtrica.
c) I. isobrica / II. isovolumtrica / III. isotrmica.
d) I. isovolumtrica / II. isotrmica / III. isobrica.
e) I. isobrica / II. isotrmica / III. isovolumtrica.

Quais esto corretas?


a) Apenas I.
b) Apenas II.
c) Apenas III.
d) Apenas I e II.
e) I, II e III.

RESPOSTA: A
COMENTRIO: I. Se P proporcional a T, o volume constante
evoluo isomtrica, isovolumtrica ou isocrica.
II. Se P constante evoluo isobrica.
III. Se T constante evoluo isotrmica.

RESPOSTA: E
COMENTRIO: Analisando cada uma das afirmaes.
I. Correta. Por isso ela chamada de constante universal.
II. Correta. Da equao de Clapeyron:
121

PV = n R T n

p V
. Se os gases apresentam a mesma
R T

presso, o mesmo volume e a mesma temperatura, eles


contm o mesmo nmero de mols, portanto, o mesmo
nmero de molculas.
III. Correta. exatamente o que afirma a equao de
Boltzmann: ec = k T.

428. (PUCRJ 2010) Seja um mol de um gs ideal a uma temperatura


de 400 K e presso atmosfrica po. Esse gs passa por uma
expanso isobrica at dobrar seu volume. Em seguida, esse
gs passa por uma compresso isotrmica at voltar a seu
volume original. Qual a presso ao final dos dois processos?
a) 0,5 po
b) 1,0 po
c) 2,0 po
d) 5,0 po
e) 10,0 po

V
p1 0
p 2V
p2 V2 p1V1
2 2 1

1 0
T2
T1
T2
T0
T2 2T0
T2 = 4T0.
430. (IFSC 2012) Voc j se perguntou como funciona a geladeira?
De que maneira ela consegue diminuir a temperatura dos
alimentos? Pelo menos sabe, do ponto de vista fsico, explicar
o que acontece? A geladeira uma mquina trmica fria, que
transforma trabalho em calor. Como mquina trmica, ela
respeita um ciclo de transformaes (duas isobricas e duas
adiabticas), como mostra a figura abaixo.

RESPOSTA: C
COMENTRIO: O diagrama a seguir ilustra a situao descrita.
Aplicando a equao geral dos gases:

p0 2 V0
p V
pA VA pB VB

0 0
TB = 2 T0.
TA
TB
T0
TB

p V
p V
pA VA pC VC

0 0 C 0 pC = 2 p0.
TA
TC
T0
2T0

Identifique em qual transformao a temperatura do gs


atinge o seu menor valor. Assinale a alternativa CORRETA.
a) Transformao IV expanso isobrica.
b) Transformao I compresso adiabtica.
c) Transformao II compresso isobrica.
d) Transformao III expanso adiabtica.
e) Transformao III compresso adiabtica.
RESPOSTA: D
COMENTRIO: Da equao de Clapeyron:

p Vn R T T

429. (UFAL 2010) Um gs ideal possui, inicialmente, volume V0 e


encontra-se sob uma presso p0. O gs passa por uma
transformao isotrmica, ao final da qual o seu volume tornase igual a V0/2. Em seguida, o gs passa por uma
transformao isobrica, aps a qual seu volume 2V 0.
Denotando a temperatura absoluta inicial do gs por T0, a sua
temperatura absoluta ao final das duas transformaes igual
a:
a) T0/4
b) T0/2
c) T0
d) 2T0
e) 4T0

p V
.
nR

Essa expresso nos mostra que a temperatura


proporcional ao produto pressoVolume. O
mostra que o mnimo valor desse produto
transformao III, portanto, esse ponto
temperatura atinge o menor valor.

diretamente
grfico nos
no final da
em que a

431. (Udesc 2011) Um gs em uma cmara fechada passa pelo


ciclo termodinmico representado no diagrama p x V da
Figura.

RESPOSTA: E
COMENTRIO: Dados: Estado inicial p = p0; V = V0 e T = T0.
1 Transformao Isotrmica: T1 = T0 e V1 =

V0

V
p1 0
p1V1 p0 V0
2 p0 V0 p1 p p 2p .

0
1
0
T1
T0
T0
T0
2

O trabalho, em joules, realizado durante um ciclo :


a) + 30 J
b) - 90 J
c) + 90 J
d) - 60 J

2 Transformao Isobrica: p2 = p1; V2 = 2 V0.

122

e) - 30 J

RESPOSTA: E
COMENTRIO: Em um ciclo fechado o trabalho
numericamente igual rea da figura. Seu valor negativo
devido ao sentido anti-horrio.

(1,0 0,2) 6,0 2,0 x105


2

1,6x105 J

433. (UFSM 2011) A respeito dos gases que se encontram em


condies nas quais seu comportamento pode ser
considerado ideal, afirma-se que
I. a grandeza que chamada de temperatura proporcional
energia cintica mdia das molculas.
II. a grandeza que chamada de presso a energia que as
molculas do gs transferem s paredes do recipiente que
contm esse gs.
III. a energia interna do gs igual soma das energias
cinticas das molculas desse gs.

3 20
30J
2

432. (UFRGS 2011)

Est(o) correta(s)
a) apenas I.
b) apenas II.
c) apenas III.
d) apenas I e III.
e) I, II e III.

A figura abaixo apresenta o diagrama da

presso p(Pa) em funo do volume

V m3 de um sistema

termodinmico que sofre trs transformaes sucessivas: XY,


YZ e ZX.

RESPOSTA: D
COMENTRIO: I. Correta. A temperatura absoluta
diretamente proporcional a energia cintica mdia das
partculas.
II. Incorreta. Presso no energia.
III. Correta.
434. (UFU 2011) Certa quantidade de gs ideal ocupa inicialmente
um volume V0, presso p0 e temperatura T0. Esse gs se
expande temperatura constante e realiza trabalho sobre o
sistema, o qual representado nos grficos pela rea sob a
curva.
Assinale a alternativa que melhor representa a quantidade de
calor trocada com o meio.
a)

O trabalho total realizado pelo sistema aps as trs


transformaes igual a
a) 0.
b)

1,6 105 J.

c)

2,0 105 J.

d)

3,2 105 J.

e)

4,8 105 J.
b)

RESPOSTA: B
COMENTRIO: Em uma evoluo cclica, o trabalho
numericamente igual rea do ciclo. Se o ciclo horrio, o
trabalho positivo. Se anti-horrio, negativo.

c)

123

d)

d) apenas II e III.
e) I, II e III.
RESPOSTA: D
COMENTRIO: I. Incorreta. A energia interna diretamente
proporcional temperatura absoluta do gs. Como T2
maior que T1, a energia interna em 2 maior que em 1.
II. Correta. A transformao isomtrica, no havendo
realizao de trabalho.
III. Correta. De acordo com a 1 lei da termodinmica:
U Q W .

RESPOSTA: C
COMENTRIO: A Primeira Lei da Termodinmica diz que a
variao da energia interna de um gs a diferena entre o
calor que ele troca com o meio e o trabalho que realiza

Como houve expanso com variao de temperatura (variao


da energia interna U ), o gs recebeu calor (energia

(U Q W) . Quando a temperatura se mantm


constante, a variao da energia interna nula e o calor
trocado igual ao trabalho realizado.
No diagrama P x V, o trabalho numericamente igual rea
compreendida entre a curva representativa do grfico e o eixo
V. Como U 0 , ento Q W

Q ) do meio e realizou trabalho (W).


436. (UFPA 2011) Na madrugada de 12 de julho de 1884, no largo
da S em Belm, o paraense Julio Cezar Ribeiro de Souza
comeou a encher seu dirigvel Santa Maria de Belm, para
validar, na prtica, o sistema de navegao area por ele
inventado. Devido a problemas na produo do hidrognio, o
processo foi suspenso s 11h da manh, antes de se completar
o enchimento do dirigvel. Nesse horrio, a intensa radiao
solar provoca o aquecimento do gs contido no balo.
Assumindo que o hidrognio no balo um gs ideal e que a
partir das 11h tanto a sua presso quanto seu nmero de
moles permanecem constantes, identifique qual dos grficos
abaixo descreve acertadamente a variao do volume V do
balo, com relao variao da temperatura T, aps as 11h.
a)

435. (UFSM 2011) A inveno e a crescente utilizao de mquinas


trmicas, a partir da revoluo industrial, produziram, ao
longo de dois sculos, impactos ecolgicos de propores
globais. Para compreender o funcionamento das mquinas
trmicas, necessrio estudar os processos de expanso e
compresso dos gases no seu interior. Em certas condies,
todos os gases apresentam, aproximadamente, o mesmo
comportamento. Nesse caso, so denominados gases ideais.
Considere o diagrama presso (P) x volume (V) para um gs
ideal, sendo as curvas isotermas.

b)

c)

d)
Analise, ento, as afirmativas:
I. A energia interna do estado 1 maior do que a energia
interna do estado 2.
II. No processo 1 3, o gs no realiza trabalho contra a
vizinhana.
III. No processo 1 2, o gs recebe energia e tambm
fornece energia para a vizinhana.

e)

Est(o) correta(s)
a) apenas I.
b) apenas II.
c) apenas III.
124

est em litro. (1 L = 10 m ).
Calculando o trabalho (Wciclo) em cada ciclo. Como se trata de
um ciclo no sentido horrio, o trabalho realizado positivo,
sendo numericamente igual rea interna do ciclo.

Wciclo " rea" 0,6 0,2 2 1 105 10 3

Wciclo 40 J.
RESPOSTA: C
COMENTRIO: A presso e o nmero de mols permanecem
constantes: trata-se de uma transformao isobrica.
Da equao de Clepeyron:

pV nRT

O trabalho total (W) em 40 ciclos :

W 40 40 1.600 J.

Calculando a potncia do sistema:

nR
T.
p

Por essa expresso, vemos que o volume diretamente


proporcional temperatura absoluta do gs, portanto, a
variao do volume tambm diretamente proporcional
variao da temperatura absoluta. Por isso o grfico uma
reta que passa pela origem.
437. (ITA 2011) A inverso temporal de qual dos processos abaixo
NO violaria a segunda lei de termodinmica?
a) A queda de um objeto de uma altura e subsequente
parada no cho.
b) O movimento de um satlite ao redor da Terra.
c) A freada brusca de um carro em alta velocidade.
d) O esfriamento de um objeto quente num banho de gua
fria.
e) A troca de matria entre as duas estrelas de um sistema
binrio.

P 1.600 W.

439. (ENEM 2011) Um motor s poder realizar trabalho se


receber uma quantidade de energia de outro sistema. No
caso, a energia armazenada no combustvel , em parte,
liberada durante a combusto para que o aparelho possa
funcionar. Quando o motor funciona, parte da energia
convertida ou transformada na combusto no pode ser
utilizada para a realizao de trabalho. Isso significa dizer que
h vazamento da energia em outra forma.
CARVALHO, A. X. Z. Fsica Trmica. Belo Horizonte: Pax, 2009
(adaptado).
De acordo com o texto, as transformaes de energia que
ocorrem durante o funcionamento do motor so decorrentes
de a
a) liberao de calor dentro do motor ser impossvel.
b) realizao de trabalho pelo motor ser incontrolvel.
c) converso integral de calor em trabalho ser impossvel.
d) transformao de energia trmica em cintica ser
impossvel.
e) utilizao de energia potencial do combustvel ser
incontrolvel.

RESPOSTA: B
COMENTRIO: A segunda lei da termodinmica envolve a
transformao de calor em trabalho.
Dos processos dados, o nico que no envolve realizao de
trabalho o movimento de um satlite em rbita, pois se trata
de um sistema conservativo, mesmo quando a rbita no
circular. Assim, no h transformao de calor em trabalho ou
vice-versa, no violando, portanto, a segunda lei da
termodinmica, qualquer que seja o sentido de giro do
satlite.
438. (Epcar (Afa) 2011) O diagrama abaixo representa um ciclo
realizado por um sistema termodinmico constitudo por n
mols de um gs ideal.

W 1.600 J

t
1s

RESPOSTA: C
COMENTRIO: De acordo com a segunda lei da
termodinmica. impossvel uma mquina trmica,
operando em ciclos, converter integralmente calor em
trabalho.
440. (UFT 2011) Equipe de cientistas descobre o primeiro
exoplaneta habitvel
O primeiro exoplaneta habitvel foi encontrado depois de
observaes que duraram 11 anos, utilizando uma mistura de
tcnicas avanadas e telescpios convencionais. A equipe
descobriu mais dois exoplanetas orbitando em volta da estrela
Gliese 581.
O mais interessante dos dois exoplanetas descobertos o
Gliese 581g, com uma massa trs vezes superior da Terra e
um perodo orbital (tempo que o planeta leva para dar uma
volta completa em torno de sua estrela) inferior a 37 dias. O
raio da rbita do Gliese 581g igual 20% do raio da rbita da
Terra, enquanto sua velocidade orbital 50% maior que a
velocidade orbital da Terra. O Gliese 581g est "preso"
estrela, o que significa que um lado do planeta recebe luz
constantemente, enquanto o outro de perptua escurido. A
zona mais habitvel na superfcie do exoplaneta seria a linha
entre a sombra e a luz, com temperaturas caindo em direo
sombra e subindo em direo luz. A temperatura mdia varia
entre -31C e -12C, mas as temperaturas reais podem ser
muito maiores na regio de frente para a estrela (at 70 C) e
muito menores na regio contrria (at -40C). A gravidade no

Sabendo-se que em cada segundo o sistema realiza 40 ciclos


iguais a este, correto afirmar que a(o)
a) potncia desse sistema de 1600 W.
b) trabalho realizado em cada ciclo - 40 J.
c) quantidade de calor trocada pelo gs com o ambiente em
cada ciclo nula.
d) temperatura do gs menor no ponto C.
RESPOSTA: A
COMENTRIO: A frequncia de operao 40 ciclos/s, ou seja,
40 Hz. Notemos ainda que, no eixo das abscissas o volume
125

Gleise 581g semelhante da Terra, o que significa que um


ser humano conseguiria andar sem dificuldades.
Os cientistas acreditam que o nmero de exoplanetas
potencialmente habitveis na Via Lctea pode chegar a 20%,
dada a facilidade com que Gliese 581g foi descoberto. Se
fossem raros, dizem os astrnomos, eles no teriam
encontrado um to rpido e to prximo. No entanto, ainda
vai demorar muito at que o homem consiga sair da Terra e
comece a colonizar outros planetas fora do sistema solar.

441. (UEL 2011) A figura apresenta trs possveis transformaes


de fase de um gs, desde o estado a at o estado c. Na
transformao de a at c, ao longo do caminho curvo do
diagrama PV, o trabalho realizado pelo gs de W = 35J e o
calor absorvido pelo gs Q = 63J. Ao longo do caminho abc,
o trabalho realizado pelo gs de W = 48J.

Texto adaptado de artigo da Revista VEJA, Edio 2185, ano


43, n 40 de 06 de outubro de 2010.
Suponha que uma mquina de Carnot seja construda
utilizando como fonte fria o lado do planeta Gliese 581g que
nunca recebe luz e como fonte quente o lado que sempre
recebe luz. A temperatura da fonte fria Tf = -40C e da fonte
quente Tq = 70C. A cada ciclo a mquina retira da fonte
quente 1000J de calor.

I. Para o caminho abc, a quantidade de calor Q absorvida pelo


gs vale 76J.

Com base na figura, no enunciado e nos conhecimentos sobre


o assunto, considere as afirmativas a seguir.

II. Se a presso Pc =

vale 14J.
III. Se a diferena de energia interna Ud Uc = 15J, a
quantidade de calor Q cedida para o caminho da vale 15J.
IV. Se a diferena de energia interna Ud Uc = 5J, a quantidade
de calor Q cedida para o caminho da vale 23J.

Considerando que a mquina trabalha com um gs ideal, leia


os itens abaixo:
I. A mquina pode ser representada por um ciclo com duas
transformaes adiabticas reversveis e duas transformaes
isotrmicas reversveis.
II. Se o ciclo desta mquina consiste de uma expanso
isotrmica, uma expanso adiabtica, uma compresso
isotrmica e uma compresso adiabtica, respectivamente,
ento ocorre transformao de calor em trabalho til.
III. O rendimento da mquina maior do que 40%.
IV. A cada ciclo uma quantidade de calor maior que 700J
rejeitada para a fonte fria.

Assinale a alternativa correta.


a) Somente as afirmativas I e II so corretas.
b) Somente as afirmativas I e IV so corretas.
c) Somente as afirmativas III e IV so corretas.
d) Somente as afirmativas I, II e III so corretas.
e) Somente as afirmativas II, III e IV so corretas.
RESPOSTA: B
COMENTRIO: Dados: Q ac = - 63 J; Wac = - 35 J; Wabc = - 48 J.
I. Correta.
As variaes de energia interna pelos caminhos ac abc so
iguais pois so os mesmos estados inicial e final.
Usando a 1 lei da termodinmica:

Marque a opo correta:


a) I e III so verdadeiras.
b) I e II so verdadeiras.
c) I e IV so verdadeiras.
d) III e IV so verdadeiras.
e) II e IV so verdadeiras.

Uac Uabc

IV. Falsa.

TQ TF
TQ

QQ QF
QQ

Para as presses temos pc

pb
.
2

Como o trabalho numa transformao isobrica dado pelo


produto da presso pela variao de volume, vem:
Wabc pb Vab e Wcda pc Vcd
Dividindo membro a membro:
Wabc pb Vab
pb Vab
48

Wcda 24 J.
p
Wcda pc Vcd
Wcda
b
Vab

110
0,44 44%
70 180

Qac Wac Qabc Wabc

63 35 Qabc 48 Qabc 76 J.

II. Incorreta.
Nas transformaes bc e da o trabalho nulo, pois elas so
isomtricas. As transformaes ab e cd so isobricas com as
variaes de volume iguais em mdulo (Vcd.=- Vab).

RESPOSTA: B
COMENTRIO: I. Verdadeira: este o ciclo de Carnot;
II. Verdadeira: o ciclo descrito tem sentido horrio. Portanto; o
trabalho positivo;
III. Falso:

1
Pb, o trabalho W para o caminho cda
2

QF
Q
0,44 1 F
QQ
1000

QF
0,56 QF 560J
1000

III. Incorreta.
As variaes de energia interna pelos caminhos cda e adc so
iguais em mdulo, porm de sinais opostos (Uadc = Ucda).
Aplicando novamente a 1 lei da termodinmica:

TEXTO PARA AS PRXIMAS 2 QUESTES:


Analise a figura a seguir e responda.

Uac Ucd Uda

- Uda Ucd Uda

Qac Wac 15 Qda +Wda

63 35 15 Qda +0 Qda 13 J.
IV. Correta.

Uac Ucd Uda

- Uda Ucd Uda

Qac Wac 5 Qda +Wda

63 35 5 Qda +0 Qda 23 J.

126

442. (UEL 2011) Com referncia figura, assinale a alternativa que


apresenta, correta e respectivamente, o valor da quantidade
de calor Q para o caminho cda e o valor da energia interna Ua
Uc.
a) Q = 25J e Ua Uc = 28J
b) Q = 52J e Ua Uc = 82J
c) Q = 57J e Ua Uc = 15
d) Q = 45J e Ua Uc = 15
e) Q = 52J e Ua Uc = 28

1 cal 4,2 J

443. (UFJF 2011)

Cp / Cv 3 / 2 . A

como ideal, monoatmico com

temperatura e presso iniciais do gs so, respectivamente,


T0 26 C e P0 1,0 atm .
Considerando que o volume final da lata tenha sido reduzido
(lata amassada) para 25% de seu valor inicial, em um processo
adiabtico, determine a temperatura e a presso final do gs.
a) 52,0 C e 8,0 atm.
b) 68,0 C e 7,0 atm.
c) 58,0 C e 6,0 atm.
d) 46,0 C e 9,0 atm.
e) 42,0 C e 10,0 atm.

RESPOSTA: E
COMENTRIO: O gabarito oficial d como resposta a
afirmativa (E), porm, nem clculos seriam necessrios para
verificar que ela falsa, pois o produto presso volume no
estado a maior que no estado c, sendo ento Ta > Tc e,
consequentemente, Ua > Uc, portanto Ua Uc > 0. Alm disso,
est faltando unidade nas afirmativas C, D e E.
Mas, com muito boa vontade, vamos aos clculos.

Uac Uca Qca Wca Uda

Uma lata de spray, com volume inicial

V0 400 ml , contm um gs que podemos considerar

63 35
RESPOSTA:

Uac 28 J.

A questo foi classificada como de dificuldade ELEVADA e


RUIM por no apresentar nenhuma opo correta.
Vamos, ento, a uma soluo fictcia, puramente matemtica:

Com os dados do enunciado no possvel calcular a


quantidade de calor fornecida na transformao cda. Para
chegar resposta fornecida pela banca examinadora, temos
que adotar uma hiptese da afirmao II da questo anterior,
que considera que a presso em c metade da presso em b.

Dados: V0 = 400 mL; 3 2 ; P0 = 1 atm; T0 = 26 C = 299


K; V = 0,25 V0 = 0,25(400) = 100 mL.
Aplicando a equao de uma transformao adiabtica para as
situaes final e inicial:

pb

pc 2 .

PV P0 V0

P 100 2 1 400 2

P 102 2 202 2

20
P 103 203 P 23
10

Usaremos, ento, essa hiptese para calcular Wcda.


Nas transformaes bc e da o trabalho nulo, pois elas so
isomtricas. As transformaes ab e cd so isobricas com as
variaes de volume iguais em mdulo (Vcd = Vab).
Como o trabalho numa transformao isobrica dado pelo
produto da presso pela variao de volume, temos:
Wabc pb Vab e Wcda pc Vcd

Aplicando a equao geral para os estados final e inicial:

Dividindo membro a membro:

T 325 C.

Wabc pb Vab

Wcda pc Vcd

pb Vab
48

Wcda pb
Vab
2

P 8 atm.

PV P0 V0

T
T0

8 100
T

1 400
299

T 598 K

Wcda 24 J.

444. (UFLA 2010) O diagrama PV mostrado a seguir ilustra dois


processos termodinmicos: 1 ABC e 2 ADC, em que um gs
ideal levado de um estado A para outro C. considerando V2 =
2V1 e P2 = 4P1, CORRETO afirmar:

Uac Qcda Wcda 28 Qcda 24


Qcda 52 J.

TEXTO PARA A PRXIMA QUESTO:


Dados:
2

Acelerao da gravidade: g 10 m/s


3

a) O trabalho realizado pelo gs ao longo do processo ADC


maior do que o trabalho realizado ao longo do processo ABC.
b) A energia interna do gs maior no estado B.
c) O trabalho realizado pelo gs ao longo do processo ABC 4
P1V1.
d) A razo TA/TB, em que TA e TB representam as temperaturas
do gs nos estados A e B, 1/8.

Densidade da gua: a 1,0 g/cm 1000 kg/m


8

Velocidade da luz no vcuo: c 3,0 10 m/s


5

Presso atmosfrica: Patm 1,0 10 N/m

1 litro 1 dm3 103 m3


1 ano - luz 9,461 1015 m
Calor especfico da gua: ca 1 cal/gC 4000 J/KgC
1 eV 1,6 1019 J

RESPOSTA: C
COMENTRIO: a) Errada. Observe os grficos abaixo

127

447. (UECE 2010) Pode-se afirmar corretamente que a energia


interna de um sistema constitudo por um gs ideal
a) diminui em uma expanso isotrmica.
b) aumenta em uma expanso adiabtica.
c) diminui em uma expanso livre.
d) aumenta em uma expanso isobrica.
RESPOSTA: D
COMENTRIO: Numa expanso isobrica AB (VB > VA),
temos:

b) Errada. A energia interna diretamente proporcional


temperatura que por sua vez diretamente proporcional ao
produto PV.

VA VB

. Sendo VB > VA TB > TA.


TA
TB
Como a energia interna diretamente proporcional
temperatura absoluta, a energia interna aumenta.

PA .VA P1.V1
PB .VB P2 .V1 4P1.V1
PC .VC P1.V2 P1.2V1 2P1.V1
PD .VD P2 .V2 4P1.2V1 8P1.V1

448. (UPE 2010) No diagrama PV, a seguir, est representada uma


srie de processos termodinmicos. No processo ab, 250 J de
calor so fornecidos ao sistema, e, no processo bd, 600 J de
calor so fornecidos ao sistema.

TB 4T1 ; TC 2T1 ; TD 8T1


c) Certa. Calculemos a rea do segundo grfico mostrado na
letra a.

WABC V2 V1 .P2 2V1 V1 .4P1 4P1V1

d) Errada.

TA
T
1
1
TB 4T1 4

445. (FGVRJ 2010) Ao realizar um trabalho de 80 mil calorias, um


sistema termodinmico recebeu 60 mil calorias.
Pode-se afirmar que, nesse processo, a energia interna desse
sistema
a) aumentou 20 mil calorias.
b) diminuiu 20 mil calorias.
c) aumentou 60 mil calorias.
d) diminuiu 80 mil calorias.
e) se conservou.

Analise as afirmaes que se seguem.


I. O trabalho realizado no processo ab nulo.
II. A variao de energia interna no processo ab 320 J.
III. A variao de energia interna no processo abd 610 J.
IV. A variao de energia interna no processo acd 560 J.

RESPOSTA: B
COMENTRIO: Dados: W = 80.000 cal; Q = 60.000 cal.
Da primeira lei da termodinmica:
U = Q W U = 60.000 80.000 U = 20.000 cal.
O sinal () indica que a energia interna diminuiu.

CORRETO afirmar que apenas as(a) afirmaes(o)


a) II e IV esto corretas.
b) IV est correta.
c) I e III esto corretas.
d) III e IV esto corretas.
e) II e III esto corretas.

446. (Ufc 2010) Dois sistemas termodinmicos completamente


isolados esto separados entre si por uma parede diatrmica
(que permite a passagem de energia), impermevel (que no
permite o fluxo de partculas) e fixa. No equilbrio
termodinmico, tais sistemas so caracterizados por
apresentarem:
a) mesma energia e mesma temperatura.
b) diferentes energias e mesma temperatura.
c) mesma energia e diferentes temperaturas.
d) energia igual a zero e mesma temperatura.
e) diferentes energias e diferentes temperaturas.

RESPOSTA: C
COMENTRIO: Processo AB:

Qab 250J
Processo isomtrico Wab 0

U Q W Uab 250 0 250J

RESPOSTA: B
COMENTRIO: Os sistemas em questo podem trocar apenas
energia. Neste caso, o equilbrio termodinmico
ocorrer quando as temperaturas de ambos os sistemas forem
idnticas. As energias no
necessariamente so iguais. Portanto, a alternativa B est
correta.

Processo BD:

Qbd 600J
Processo

isobrico
3

Wbd p.V 8 10 3 10 240J


U Q W Ubd 600 240 360J
4

128

Processo ABD:

VA VB
.

TA
TB

Uabd Uab Ubd 250 360 610J


Processo ACD:

O grfico uma reta que passa pela origem, sendo o volume


diretamente proporcional temperatura: V = k T.

A variao da energia interna entre dois estados no depende


da evoluo. Portanto:

No entanto, com os valores dados:

Uacd Uabd 610J

1
3
.

300 500

A relao entre volume e temperatura nesse grfico :

449. (UFV 2010)


A figura a seguir ilustra um processo
termodinmico em um gs. Sabendo que durante o processo
ABC a variao da energia interna do gs foi igual a U e que o
trabalho realizado pelo gs no processo BC foi igual a W, ento
a quantidade de calor transferida ao gs no processo ABC foi:

V 1 T 300

2
200

T
2 , que no apresenta
100

relao de proporcionalidade.
Alm disso, a unidade de temperatura no eixo das abscissas
est grafada em letra minscula (k).
A nica maneira de contornar a situao considerar que
esteja sendo bombeado gs no recipiente, aumentando a
massa gasosa.
Assim:

pV
, considerando R = 8 J/molK, vem:
RT
120(1)
nA =
nA = 0,048 mol.
8,3(300)
120(3)
NB =
nB = 0,087 mol.
8,3(500)
Sendo n =

a) U + VA (PA PC) + W
b) U + PA (VB VA) W
c) U + VC (PA PC) + W
d) U + PA (VB VA) + W

Porm, o mais provvel que a banca examinadora tenha


cometido um deslize ao apresentar o grfico.
Vamos soluo esperada:

RESPOSTA: D
COMENTRIO: Dados: variao da energia intena: U; trabalho
realizado no trecho BC: WBC = W
De acordo com 1 lei da termodinmica:
Q = U + WAB + WBC
Q = U + PA (VB VA) + W
450. (UNEMAT 2010) O grfico abaixo mostra a variao da energia
interna de um gs ideal que sofreu uma transformao
2
presso constante de P = 120 N/m . A quantidade de calor
recebida pelo gs durante o processo foi de 800 joules.

Sendo W o trabalho realizado, temos:


W = P V = 120(3 1) W = 240 J.
Sendo o calor recebido Q = 800 J, aplicando a 1 lei da
termodinmica:
U = Q W = 800 240 = 560 J.
451. (UFU 2010) Um botijo de cozinha contm gs sob alta
presso. Ao abrirmos esse botijo, percebemos que o gs
escapa rapidamente para a atmosfera. Como esse processo
muito rpido, podemos consider-lo como um processo
adiabtico.
Considerando que a primeira lei da termodinmica dada por
U = Q - W, onde U a variao da energia interna do gs, Q
a energia transferida na forma de calor e W o trabalho
realizado pelo gs, correto afirmar que:
a) A presso do gs aumentou e a temperatura diminuiu.
b) O trabalho realizado pelo gs foi positivo e a temperatura
do gs no variou.
c) O trabalho realizado pelo gs foi positivo e a temperatura
do gs diminuiu.
d) A presso do gs aumentou e o trabalho realizado foi
negativo.

Com os dados, pode-se dizer que a variao da energia interna


que este gs sofreu foi de:
a) 560 joules.
b) 260 joules.
c) 300 joules.
d) 480 joules.
e) 580 joules.

RESPOSTA: C
COMENTRIO: Ao abrirmos o botijo, o gs sofreu expanso
realizando trabalho contra o meio (W > 0)

RESPOSTA: A
COMENTRIO:Obs: se a massa de gs constante, essa
questo est furada, pois o grfico est incoerente com o
enunciado. Para uma transformao isobrica, de acordo com
a lei geral dos gases:

Como o calor trocado foi nulo (Q = 0), a primeira lei da


termodinmica nos d:
U = Q W U = W.
129

Dado: considere que as temperaturas em graus centgrados,


TC, e Kelvin, TK, se relacionam atravs da expresso TC = TK
273.
a) 127 C
b) 177 C
c) 227 C
d) 277 C
e) 377 C

Se a variao da energia interna foi negativa (U < 0) o gs


sofre resfriamento, ou seja, a temperatura do gs diminuiu.
452. (UPE 2010) O diagrama PV para uma determinada amostra de
gs est representado na figura a seguir. Se o sistema levado
do estado a para o estado b, ao longo do percurso acb,
fornece-se a ele uma quantidade de calor igual a 100 cal, e ele
realiza um trabalho de 40 cal. Se, por meio do percurso adb, o
calor fornecido de 72 cal, ento o trabalho realizado vale em
cal:

RESPOSTA: A
COMENTRIO: Dados: T1 = 27 C = 300 K; Q1 = 40 kJ; W = 10 kJ.
O rendimento () desse motor :
=

W 10

0,25 .
Q1 40

Aplicando esse rendimento ao ciclo de Carnot:


= 1

T2

T1

T2
1
T1

T1 =

T2
1

T1 =

300
300

400 K T1 = 400 273


1 0,25 0,75

a) 28
b) 60
c) 12
d) 40
e) 24

T1 = 127 C.
455. (ITA 2010) Uma mquina trmica opera segundo o ciclo JKLMJ
mostrado no diagrama T-S da figura.

RESPOSTA: C
COMENTRIO: A variao da energia interna de um gs s
depende das energias internas inicial e final, no dependendo
da evoluo gasosa.

Uacb Uadb Qacb Wac Wcb


Qadc Wad Wdb
Wac Wdb 0 evolues isomtricas
Qacb Wcb Qadc Wad
100 40 = 72 - Wad

Wad = 12cal
Pode-se afirmar que
a) processo JK corresponde a uma compresso isotrmica.
b) o trabalho realizado pela mquina em um ciclo W = (T 2
T1)(S2 S1).

453. (CFTMG 2010) Um processo cclico de Carnot possui um


rendimento de 50%.
Uma mquina real, que opera sob as mesmas condies
trmicas desse ciclo, apresentar um rendimento trmico r, tal
que
a) r 50%.
b) r = 50%.
c) r > 50%.
d) r < 50%.

T
c) o rendimento da maquina dado por 1 2 .
T1

d) durante o processo LM, uma quantidade de calor QLM =


T1(S2 S1) absorvida pelo sistema.
e) outra mquina trmica que opere entre T2 e T1 poderia
eventualmente possuir um rendimento maior que a desta.

RESPOSTA: D
COMENTRIO: A termodinmica estabelece que nenhuma
mquina operando em ciclos fornece rendimento maior que a
mquina ideal de Carnot.Portanto, r < 50%.

RESPOSTA: B
COMENTRIO: No ciclo temos as seguintes transformaes:
JK: expanso isotrmica. Se a entropia aumenta, o sistema
recebe calor e realiza trabalho;
KL: resfriamento adiabtico. A temperatura diminui sem variar
a entropia, logo no h troca de calor;
LM: compresso isotrmica. A entropia diminui, o sistema
perde calor e recebe trabalho;
MJ: aquecimento adiabtico. A temperatura aumenta sem
variar a entropia.
Nota-se, ento, que se trata de um ciclo de Carnot, com

454. (UFAL 2010) A cada ciclo de funcionamento, o motor de um


certo automvel retira 40 kJ do compartimento da fonte
quente, onde se d a queima do combustvel, e realiza 10 kJ
de trabalho. Sabendo que parte do calor retirado da fonte
quente dispensado para o ambiente (fonte fria) a uma
temperatura de 27 C, qual seria a temperatura no
compartimento da fonte quente se esse motor operasse
segundo o ciclo de Carnot?

rendimento: 1

130

T1
T2

Calculemos o trabalho realizado no ciclo, lembrando que a


variao da entropia :
S =

Carnot e a razo T2/T1 das temperaturas em que opera a


mquina.

Q
, onde Q o calor trocado na transformao.
T

O ciclo de Carnot um ciclo termodinmico especial, pois uma


mquina trmica que opera de acordo com este ciclo entre
duas temperaturas T1 e T2, com T1 maior do que T2, obtm o
mximo rendimento possvel. O rendimento r de uma
mquina trmica definido como a razo entre o trabalho
lquido que o fluido da mquina executa e o calor que absorve
do reservatrio temperatura T1.

A transformao JK isotrmica, portanto a variao da


energia interna nula. Da 1 lei da termodinmica (
U Q W ). Ento:
0 = QJK WJK
WJK = QJK. (equao 1)
Mas: SJK =

QJK
QJK SJ SK T2
T2

QJK = (S2 S1)T2 . Substituindo nessa expresso a equao (1),


obtemos:
WJK = (S2 S1)T2.
Seguindo esse mesmo raciocnio para a transformao LM,
que tambm isotrmica, mas uma compresso, vem:
WLM = (S1 S2)T1 WLM = (S2 S1)T1.
Nas transformaes KL e MJ o sistema no troca calor.
Novamente, pela 1 lei da termodinmica:
UKL = WKL e UMJ = WMJ.
Como UMJ = UKL WMJ = WKL.
O trabalho no ciclo o somatrio desses trabalhos, ou seja:
Wciclo = WJK + WKL + WLM + WMJ
Wciclo = (S2 S1)T2 + WKL (S2 S1)T1 WKL
Wciclo = (S2 S1)T2 (S2 S1)T1
Wciclo = (S2 S1) (T2 T1).

Pode-se concluir, pelo grfico e pelas leis da termodinmica,


que o rendimento da mquina de Carnot aumenta quando a
razo T2/T1 diminui,

a) alcanando 100% quando T2 vale 0 C.


b) alcanando 100% quando T1 muito maior do que T2.
c) alcanando 100% quando a diferena entre T1 e T2 muito
pequena.
d) mas s alcana 100% porque representa o ciclo ideal.
e) mas nunca alcana 100%.

456. (UDESC 2010) No diagrama p x V a seguir, est representado o


ciclo termodinmico da mquina de Carnot, considerada ideal
porque tem o maior rendimento entre as mquinas trmicas.
O sistema recebe calor da fonte quente temperatura T1 e
transfere calor para a fonte fria temperatura T2.

RESPOSTA: E
COMENTRIO: A 2 Lei da Termodinmica afirma que
nenhuma mquina trmica, operando em ciclos entre uma
fonte quente, temperatura T1, e uma fonte fria,
temperatura T2, consegue transformar integralmente calor em
trabalho. Portanto o rendimento nunca pode chegar a 100%,
sendo no mximo, igual ao da mquina de Carnot.
De fato, analisando o grfico, vemos que o rendimento seria
igual a 100% quando a razo

T2
fosse nula, ou seja:
T1

T2
0 T2 0 . A fonte fria teria que estar a 0 K, o que
T1
um absurdo. Portanto o rendimento r sempre menor que
100%.
Com relao s transformaes termodinmicas que
constituem esse ciclo, correto afirmar que o sistema passa
por uma:
a) expanso adiabtica entre os estados b e d (b d).
b) expanso isovolumtica entre os estados b e c (b c).
c) compresso isobrica entre os estados c e d (c d).
d) expanso isotrmica entre os estados a e b (a b).
e) compresso isotrmica entre os estados d e a (d a).

458. (UFRGS 2010)


Assinale a alternativa que preenche
corretamente as lacunas do texto a seguir, na ordem em que
aparecem.
A figura a seguir representa simplificadamente o diagrama pV,
sendo p dada em atm e V dado em I, para um ciclo de uma
mquina trmica que opera com um gs ideal. Considere que,
durante o percurso ABCD, o nmero de partculas do gs
permanece constante, e que, para esse gs, a razo entre o
calor especfico a presso constante (cP) e o calor especfico a
volume constante (cv) cp/cv = 5/3.

RESPOSTA: D
COMENTRIO: D) expanso isotrmica entre os estados a e b
(a b).
Correta, pois a temperatura mantm-se constante.
457. (PUCRS 2010) Para responder a questo, considere o texto e o
grfico, o qual relaciona o rendimento de uma mquina de

131

cP

5
cV

3
para o processo CD : pC VC 9,5 1 9,5 e

cP

5
p V c V 3 2 3 9,52
D D

Esses clculos mostram que os processos AB e CD so, com


boa aproximao, adiabticos.
459. (Ufla 2010) O ciclo da Carnot constitudo de duas
transformaes isotrmicas a temperaturas T1 e T2 e duas
transformaes adiabticas. Considere o diagrama P x V a
seguir e o sentido do ciclo ABCDA. CORRETO afirmar:

As etapas A B e C D do ciclo representado na figura so


processos............... . Sendo assim,............... troca de ................
entre a mquina trmica e o ambiente.
a) isotrmicos h
trabalho
b) isotrmicos no h trabalho
c) adiabticos no h calor
d) adiabticos h
calor
e) adiabticos no h trabalho

a) As variaes da energia interna U nos processos BC e DA


so nulas.
b) As variaes da energia interna U nos processos AB e CD
so nulas.
c) a temperatura associada ao processo isotrmico AB
menor do que a temperatura associada ao processo
isotrmico CD.
d) Ao final do ciclo ABCDA, o trabalho realizado igual
variao da energia interna U de ciclo.

RESPOSTA: C
COMENTRIO: Os processos AB e CD no so isotrmicos,
pois, caso o fossem, o produto p V seria constante em cada
um deles.
Constatando:
pA VA = 2 atm.L e pB VB = 3 atm.L pA VA pB VB;
pC VC = 9,5 atm.L e pD VD = 6 atm.L pC VC pD VD
Analisando as opes, considerando que uma delas correta,
por excluso, temos que admitir que os processos so
adiabticos. Ento, no h troca de calor com o meio
ambiente, chegando-se facilmente opo correta. Da a
questo ter sido classificada como de baixa dificuldade
Porm, no basta no ser isotrmico para ser adiabtico. Para
a confirmao, temos que verificar se vlida a expresso do

RESPOSTA: B
COMENTRIO: Estes processos so isotrmicos, portanto no
h variao de temperatura.
460. (UECE 2010) No diagrama P-V a seguir, quatro processos
termodinmicos cclicos executados por um gs, com seus
respectivos estados iniciais, esto representados. O processo
no qual o trabalho resultante, realizado pelo gs menor o

cP

cV

k , sendo k uma constante,


processo adiabtico p V

para cada um deles.


Essa verificao torna-se difcil, muito trabalhosa, sem usar
uma calculadora (cientfica).
Sendo

cP 5
, temos (usando calculadora):
cV 3

para o processo AB :

p V
A

cP
cV

1 2 3,175
5
3

cP

5
p V c V 3 13 3
B B

a) I.
b) J.
c) K.
d) L.
RESPOSTA: C
COMENTRIO: O trabalho (W) realizado numa transformao
cclica numericamente igual rea interna do ciclo. A rea

132

interna dos ciclos I, J e L corresponde de 4 quadrculos. A


rea do ciclo K menor que a de 4 quadrculos.
Podemos tambm efetuar os clculos:
WI = 1 4 = 4 J;
WJ = 2 2 = 4 J;
2
WK = 3,14 1 = 3,14 J;
WL = 2 2 = 4 J.

COMENTRIO: Dados: PV 5 cte ; V2 =

P2 V2 5 P1V1 5

461. (PUCRS 2010) O ciclo Otto um ciclo termodinmico


constitudo por dois processos adiabticos e dois processos
isovolumtricos, como mostra o grfico que segue.

Num motor que opera segundo este ciclo, um pisto


inicialmente na posio correspondente ao mximo volume,
estado 1, comprime o ar at que atinja o volume mnimo,
estado 2. Ento ocorre a combusto, resultando em um sbito
aumento da presso enquanto o volume permanece
constante, levando o ar ao estado 3. O processo que segue a
ejeo de potncia quando o ar expande adiabaticamente
para o estado 4. No processo final, calor transferido para a
vizinhana e o ciclo completado.
A partir das informaes obtidas pela anlise do grfico
representativo do ciclo Otto e de acordo com as leis da
termodinmica, correto afirmar que:
a) o calor lquido trocado no ciclo nulo, visto que a
temperatura final igual temperatura inicial.
b) o sistema realiza um trabalho lquido nulo durante o ciclo,
pois o volume final igual ao volume inicial.
c) o trabalho realizado no processo de compresso adiabtica
maior do que o realizado no processo de expanso
adiabtica.
d) o sistema absorve calor durante a compresso adiabtica e
rejeita calor durante a expanso adiabtica.
e) a variao da energia interna no ciclo zero, porque o
estado final igual ao estado inicial.
RESPOSTA: E
COMENTRIO: A variao de energia interna entre dois
estados, para um sistema gasoso diretamente proporcional a
variao de sua temperatura absoluta entre esses dois
estados. No caso das transformaes cclicas, a temperatura
final sempre igual inicial, portanto a variao de energia
interna nula.

P2 V1

P1 V2

7
P2
P
32 5 2 25
P1
P1

1
V1 .
32

P2 V1

P1 1

1
32

P2
7
2 P2 = 128 P1.
P1
463. (UFU 2010) Em relao Primeira e Segunda Lei da
Termodinmica, correto afirmar que:
a) Na expanso isotrmica de um gs ideal monoatmico, a
temperatura permanece constante e, de acordo com a
primeira lei da termodinmica, a variao da energia nula.
Desse modo, o calor absorvido convertido completamente
em trabalho. Entretanto, pode-se afirmar que a segunda lei da
termodinmica no violada porque o sistema no est
isolado.
b) Na expanso isotrmica de um gs ideal monoatmico, a
temperatura permanece constante e, de acordo com a
primeira lei da termodinmica, a variao da energia nula.
Desse modo, o calor absorvido convertido completamente
em trabalho e pode-se afirmar que a segunda lei da
termodinmica violada, uma vez que esse um sistema
isolado.
c) Na expanso adiabtica de um gs ideal monoatmico, a
temperatura permanece constante e, de acordo com a
primeira lei da termodinmica, a variao da energia nula.
Desse modo, o calor absorvido convertido completamente
em trabalho e, considerando que esse no um sistema
isolado, pode-se afirmar que a segunda lei da termodinmica
violada.
d) Na expanso isotrmica de um gs ideal monoatmico, a
temperatura permanece constante e, de acordo com a
segunda lei da termodinmica, a variao da energia nula.
Desse modo, o calor absorvido convertido completamente
em trabalho. Entretanto, pode-se afirmar que a primeira lei da
termodinmica no violada, porque o sistema no est
isolado.
RESPOSTA: A
COMENTRIO: Se a expanso isotrmica a energia interna
no varia. Sendo o sistema no termicamente isolado, todo
calor recebido pelo gs transformado em trabalho.
MATEMTICA I
464. 1. (Uesc 2011) Sendo m o nmero de anagramas da palavra
UESC e n o nmero de anagramas do nmero 2011, o valor do

m
15

m-n

que

m P4 4! 24

determinante da matriz M

462. (PUCRJ 2010) Uma quantidade de ar sofre uma compresso


7/5
adiabtica, ou seja pV = constante, onde p a presso e V o
volume do gs. O volume diminui por um fator de 1/32
durante essa compresso. De quanto variou a presso?
a) Diminuiu 16 vezes.
b) Aumentou 32 vezes.
c) Aumentou 64 vezes.
d) Aumentou 128 vezes.
e) Diminuiu 32 vezes.

a) 216
b) 36
c) 72
d) 108
e) 216
RESPOSTA: D
COMENTRIO:

n P4(2)

RESPOSTA: D
133

Temos

4!
12.
2!

2sen 2x sen(x ) 2cos2x sen x


A

.
2

sen x
1
sen x
1

12 24 12
24

.
15 24 12 15 12

Desse modo, M
Portanto,

detM

24 12
15 12

465. (Udesc 2011)

24 12 15 12 9 12 108.
Da,

Sejam A a matriz quadrada de ordem 2

2cos2x sen x
AT
1
sen x

2sen 2x sen(x )

definida por A
2

e f a

sen(x)
1

e, assim,

funo definida por f(x) det(A A ) . O grfico da

4cos2x 0
A AT
.
2
0

funo f , para x , , :

Ento,

f(x) | det(A A T ) |

4cos2x 0
0

| 8cos2x | .

Portanto,
a)

1 cos x 1 1 cos 2x 1
8 8cos 2x 8
0 | 8cos 2x | 8
0 f(x) 8,
isto , a imagem de f o intervalo [0, 8].
Por outro lado, sabendo que o perodo (P) de uma funo da

b)

forma g(x) acos(mx), em que {a, m}

2
,
|m|

temos que o

perodo

da

, dado por

funo

2
3,14. Assim, o grfico da funo f, para
|2|
x [, ], o da alternativa (E).
c)

466. (Epcar (Afa) 2011) Sendo

4 a

2 0

1 1 b

1 0

d)

1 1 3

70 , o valor de

2 c

7 1 0 b 3c
a) 280
b) 0
c) 70
d) 210
RESPOSTA: D
COMENTRIO: A nova matriz foi obtida de A da seguinte
forma:

e)
RESPOSTA: E
COMENTRIO: Temos que

1. Foram trocadas as posies das colunas 1 e 3, (o


determinante fica multiplicado por -1).
134

2. A nova quarta linha foi multiplicada por 3 (o determinante


fica multiplicado por 3).
3. Somou-se a terceira linha com a quarta linha, originando
uma nova quarta linha (determinante no se altera).

2 18 54
1
a 1 23
(1)3 2 3 48 192 (a 1 ) 23 12
72
1 0
0

Logo, o novo determinante ser (-1).3.70 = - 210.


467. (Eewb 2011)

O determinante da matriz A 4x4 onde os

469. (Fgv 2010) Uma matriz 4 x 4 que admite inversa

elementos da primeira linha so 4, 3, 5 e 1; os elementos da


segunda linha so 0, 3, 0 e 2; os da terceira linha so 2, 7, 0 e 0
e os da quarta linha, 8, 6, 10 e 2,
a) - 5
b) 0
c) 5
d) 15

a)

b)

RESPOSTA: B
COMENTRIO:

c)

Todo o determinante com filas paralelas e proporcionais vale


zero.
d)
468. (Ita 2010) Sobre os elementos da matriz

x1 x 2
y y
2
A 1
0 0

1 0

x3
y3
0
0

x4
y 4
M4x4 ( )
1

e)

sabe-se que (x1, x2, x3, x4) e (y1, y2, y3, y4) so duas progresses
geomtricas de razo 3 e 4 e de soma 80 e 255,
1
1
respectivamente, Ento, det(A ) e o elemento (A )23 valem,
respectivamente,

1
e 12.
72
1
b)
e -12.
72
1
c)
e 12.
72
1
1
d)
e
.
72 12
1
1
e)
e
.
72
12

RESPOSTA: E
COMENTRIO: a) No admite inversa, pois a linhas 1 e 3 so
proporcionais e seu determinante vale zero.
b) No admite inversa, pois a terceira linha uma combinao
linear das duas primeiras. Seu determinante tambm zero
c) No admite inversa, pois as linhas da matriz so
proporcionais, seu determinante vale zero.
d) No admite inversa, pois a terceira linha igual ao dobro da
segunda menos a primeira, seu determinante vale zero.
e) Seu determinante 36416 (diferente de zero). Logo,
admite inversa.

a)

470. (Uece 2010) Se n um nmero inteiro positivo e X a matiz

1 0 0
1 2 0 , ento o valor do determinante da matriz Y = Xn

1 1 3

RESPOSTA: C
COMENTRIO: x1 + 3x1 + 9x1 + 27x1 = 80 40 x1 = 80 x1 = 2
y1 + 4y1+ 16y1 + 64y1 = 255 85y1 = 255 y1 = 3

2 6 18 54
3 12 48 192
e det(A ) = 1.(-1)4+1.,
ento A =
0 0 0
1

0
1 0 0
6 18 54

12 48
0

1 2 3 4

4 3 2 1
2 4 6 8

5 6 7 8
1 2 3 4

1 4 5 16
2 6 8 20

5 6 11 8
1 1 1 1

2 2 2 2
3 3 3 3

4 4 4 4
1 2 3 4

5 6 7 8
9 10 11 12

13 14 15 16
4
-1 2 3

8
1 -6 7
9 10 - 11 12

13 14 15 - 16

a) 2
n
b) 3
n
c) 6
n
d) 9
RESPOSTA: C
COMENTRIO: O determinante da matriz dada 1.2.3 = 6
n
n
Ento, o determinante de x ser 6 .
471. (Mackenzie 2010)

Dadas as matrizes A = (aij)3x3 tal que

aij 10,se i j
e B = (bij)3x3 tal que

aij 0,se i j

2 = -72
1

o valor de det(AB)
135

bij 3,se i j

bij 0,se i j

a) 27 x 10
3
b) 9 x 10
2
c) 27 x 10
2
2
d) 3 x 10
4
e) 27 x 10

474. (Uerj 2012)


Uma famlia comprou gua mineral em
embalagens de 20 L, de 10 L e de 2 L. Ao todo, foram
comprados 94 L de gua, com o custo total de R$65,00 .
Veja na tabela os preos da gua por embalagem:
Volume da embalagem
(L)
20
10
2

RESPOSTA: A
COMENTRIO:

10

A 0
0

B 0
0

10 0 det( A) 10 3
0 10
0

Nessa compra, o nmero de embalagens de 10 L corresponde


ao dobro do nmero de embalagens de 20 L, e a quantidade
de embalagens de 2 L corresponde a n.
O valor de n um divisor de:
a) 32
b) 65
c) 77
d) 81

0 0

3 0 det( B) 3 3
0 3
3

det(A.B) = det(A).det(B) = 10 .3 = 27.10

Preo
(R$)
10,00
6,00
3,00

472. Se A uma matriz 2 x 2 inversvel que satisfaz A = 2A, ento o


determinante de A ser:
a) 0
b) 1
c) 2
d) 3
e) 4

RESPOSTA: C
COMENTRIO: Sejam x, y e z, respectivamente, os nmeros
de embalagens de 20 L, 10 L e 2 L.
Do enunciado e da tabela, obtemos

20x 10y 2z 94

10x 6y 3z 65
y 2x

RESPOSTA: E
2
COMENTRIO: Se A = 2A, temos:
2
det (A ) = det (2A)
2
de A . det A = 2 . det A
2
(det A) = 4 . det A
Sendo det A 0 , pois a invertvel, vem det A = 4.

473. Se

det p
x

- 2a

det 2p x
3x
a) 0
b) 4
c) 8
d) 12
e) 16

q
r 1 , ento o valor
y z
- 2b
- 2c

2q y
2r z igual a

3y
3z

20x z 47

22x 3z 65
y 2x

60x 3z 141

.
22x 3z 65
y 2x

Adicionando as duas primeiras equaes do ltimo sistema,


vem: 38x 76 x 2.

do

Logo, da segunda equao do sistema, encontramos

3z 65 22x 3z 65 22 2 z 7.
Portanto, como z n 7 e 77 7 11, segue que n um
divisor de 77.
475. (Fatec 2011) Sejam a e b nmeros reais tais que o sistema,
nas incgnitas x e y,
3

x.cosa y.sen a sen

admita uma nica soluo.


7

x.cosb y.sen b cos

Nessas condies, pode-se afirmar que, sendo k um nmero


inteiro,

RESPOSTA: D
COMENRIO:

a)

b a k. .
2

b)

b a k..

c)

b a k.

2
.
3

k.r.
2

2
e) b a
k.
.
2
3
d)

ba

RESPOSTA: B
136

COMENTRIO:

cosa sena
cosb senb

478. (Mackenzie

2011)

kx 4ky 0
,k

3x ky 8

cosa.senb cosb.sena 0

Relativas

ao

sistema

, considere as afirmaes I, II e III

abaixo.

sen(b a) 0

I. Apresenta soluo nica para, exatamente, dois valores


distintos de k.
II. Apresenta mais de 1 soluo para um nico valor de k.
III. impossvel para um nico valor de k.

b a 0 k.
b a k.

2x 2y 2z 2 0

476. (G1 - ifsc 2011) O sistema 2x y 3z 6 possvel


kx y 5z 9

Dessa forma,
a) somente I est correta.
b) somente II e III esto corretas.
c) somente I e III esto corretas.
d) somente III est correta.
e) I, II e III esto corretas.

e determinado, quando o valor de k for:


a) k 3 .
b) k 5 .
c) k 3 .

RESPOSTA: B
COMENTRIO:

d) k 5 .

k 4k

e) k 0 .

Resposta:
[D]
O determinante dos coeficientes dever ser diferente de zero.

0 k 2 12k 0 k 0 e k 12

(o

sistema possui soluo nica)


Se

temos

0 0 0
8
x e y pode ser qualquer real, logo o sistema

3
3 0 10 6k 4 2k 6 20 0 4k 20 k3x
5 8

2 2 2

Se

12

temos

12x 48y 0(: 4)


3x 12y 0

(sistema impossvel)

3x 12y 8
3x 12y 8

477.
14. (G1 - epcar (Cpcar) 2011) Certo dia, Isabela e Ana
Beatriz saram para vender pastis na praia. Elas tinham juntas 460
pastis. No final do dia, verificou-se que Isabela conseguiu vender

3
5
dos pastis que levara e Ana Beatriz dos pastis que levara.
5
8

I) Falsa. Possui soluo nica para infinitos valores de k.


II) Verdadeira, se k = 0 o sistema apresenta infinitas solues.
III) Verdadeira, impossvel se k = 12

Ao final do dia, o nmero de pastis que restou para Ana Beatriz


era a metade do nmero de pastis que restou para Isabela.
Se Ana Beatriz, levou x pastis para vender, ento, a soma dos
algarismos de x
a) 6
b) 7
c) 8
d) 9

479. (Epcar (Afa) 2011) Trs amigos Samuel, Vitria e Jlia, foram
a uma lanchonete.

RESPOSTA: B
COMENTRIO: Isabela tinha y pastis e Ana Beatriz tinha
x pastis, ento: x + y = 460.

Considerando-se que cada um dos trs pagou o valor exato do


que consumiu, correto afirmar que
a) o guaran custou o dobro da esfirra.
b) os trs amigos, juntos, consumiram 16 reais.
c) cada esfirra custou 2 reais.
d) Jlia pagou 8 reais pelo que consumiu.

Samuel tomou 1 guaran, comeu 2 esfirras e pagou 5 reais.


Vitria tomou 2 guarans, comeu 1 esfirra e pagou 4 reais.
Jlia tomou 2 guarans, comeu 2 esfirras e pagou k reais.

3y
2y
, restando-lhe
.
5
5
5x
3x
Ana Beatriz vendeu
, restando-lhe
.
8
8
3x 1 2y
15

y
x.
Portanto,
8 2 5
8
15x
Fazendo x
460 x 160 .
8
Isabela vendeu

RESPOSTA: C
COMENTRIO: Vamos considerar x o preo do guaran e y o
preo da esfirra.

x 2y 5

4x 2y 8

Somando os algarismos, temos: 1 + 6 + 0 = 7.

Somando as equaes, temos:


- 3x = - 3
x = 1 e y = 2.
Logo, o preo de cada esfirra de R$2,00.

137

480. (Unesp 2011) Uma famlia fez uma pesquisa de mercado, nas
lojas de eletrodomsticos, procura de trs produtos que
desejava adquirir: uma TV, um freezer e uma churrasqueira.
Em trs das lojas pesquisadas, os preos de cada um dos
produtos eram coincidentes entre si, mas nenhuma das lojas
tinha os trs produtos simultaneamente para a venda. A loja A
vendia a churrasqueira e o freezer por R$ 1.288,00. A loja B
vendia a TV e o freezer por R$ 3.698,00 e a loja C vendia a
churrasqueira e a TV por R$ 2.588,00.
A famlia acabou comprando a TV, o freezer e a churrasqueira
nestas trs lojas. O valor total pago, em reais, pelos trs
produtos foi de
a) 3.767,00.
b) 3.777,00.
c) 3.787,00.
d) 3.797,00.
e) 3.807,00.

b) se

Somando as equaes, temos: 2.(x + y + z) = 7574. Logo, x + y +


z = 3.787.
481. (Uesc 2011) Uma empresa turstica pretende alugar alguns
nibus para levar 260 pessoas em excurso.
Para minimizar a despesa com esse aluguel, foi feita uma
pesquisa de preos junto a uma empresa de transportes que,
para o perodo desejado, disponibilizou 5 nibus de 40 lugares
e 8 nibus de 50 lugares, mas apenas 6 motoristas.
Sabendo-se que o aluguel do nibus maior custa

abc 0 ento

os elementos

nesta

ordem, tambm em progresso aritmtica.


c)

a2 0 ou c 0 , mas a2 bc 0

d)

a2 0 ou c = 0, mas a2 bc 0

e) E) pelo menos dois elementos no conjunto

a,b,c so

diferentes de zero.
RESPOSTA: B

ax by 1 ,

cx ay 1
multiplicando a primeira equao por -1 e somando com a
segunda, temos (a-c) . x + (b a) . y = 0. Com isso, conclumos
que para o sistema ser possvel e indeterminado deveremos
ter a = c e b = a, portanto, (a.b,c) formam uma P.A de razo
zero.
Se a = b = c = 0, o sistema ser impossvel.
COMENTRIO>

RESPOSTA: C
COMENTRIO: Sendo, x o preo da TV, y o preo do freezer e z
o preo da churrasqueira, podemos escrever o sistema:

y z 1288

x y 3698
x z 2588

a,b,c .
a,b,c esto,

a) a ser o nico termo no nulo no conjunto

Considerando

sistema

483. (Unicamp 2011) Recentemente, um rgo governamental de


pesquisa divulgou que, entre 2006 e 2009, cerca de 5,2
milhes de brasileiros saram da condio de indigncia. Nesse
mesmo perodo, 8,2 milhes de brasileiros deixaram a
condio de pobreza. Observe que a faixa de pobreza inclui os
indigentes.
O grfico a seguir mostra os percentuais da populao
brasileira enquadrados nessas duas categorias, em 2006 e
2009.

R$2000,00 , e o aluguel do nibus menor,


R$1300,00 , pode-se concluir que a menor despesa com
aluguel de nibus, nessa empresa de transportes, ser, em
reais, igual a
a) R$8500,00

R$9200,00
R$9900,00
d) R$10600,00
e) R$11900,00
b)
c)

RESPOSTA: B
COMENTRIO: Seja o par

Aps determinar a populao brasileira em 2006 e em 2009,


resolvendo um sistema linear, verifica-se que
a) o nmero de brasileiros indigentes passou de 19,0 milhes,
em 2006, para 13,3 milhes, em 2009.
b) 12,9 milhes de brasileiros eram indigentes em 2009.
c) 18,5 milhes de brasileiros eram indigentes em 2006.
d) entre 2006 e 2009, o total de brasileiros includos nas faixas
de pobreza e de indigncia passou de 36% para 28% da
populao.
RESPOSTA: C
COMENTRIO: Se x e y, respectivamente, denotam a

(x, y), em que x e y

representam, respectivamente, o nmero de nibus de 50


lugares e o nmero de nibus de 40 lugares.
Como sero disponibilizados 6 motoristas e 260 pessoas
sero
transportadas,
temos
que

(x, y) {(5, 1), (4, 2), (3, 3), (2, 4)}.

populao brasileira, em milhes, em 2006 e 2009, ento:

Portanto, a menor despesa com aluguel obtida em (2, 4) ,

26%x 21%y 8,2

10%x 7%y 5,2

ou seja, 2 2000 4 1300 R$ 9.200,00.


482. (Upe 2011) Os elementos

26%x 21%y 8,2

30%x 21%y 15,6

x 185
.

y 190

Portanto, 10% 185 milhes 18,5 milhes de brasileiros


eram indigentes em 2006.

a,b,c , todos reais e positivos,

esto, nesta ordem, em progresso geomtrica. Sabendo que

ax by 1
possvel e indeterminado, correto afirmar

cx ay 1

484. (Ufrgs 2011)


Rasgou-se uma das fichas onde foram
registrados o consumo e a despesa correspondente de trs
mesas de uma lanchonete, como indicado abaixo.

que necessariamente
138

c) 9
d) 14
e) 19
RESPOSTA: D
COMENTRIO: Para que
indeterminado, deve-se ter

sistema

Nessa lanchonete, os sucos tm um preo nico, e os


sanduches tambm. O valor da despesa da mesa 3
a) R$5,50 .

2 1 5
a 4 e b 10.
a 2 b

R$6,00
c) R$6,40 .
d) R$7,00
e) R$7,20 .

Por conseguinte, a b 4 10 14.

b)

RESPOSTA: A
COMENTRIO: Sejam x e y, respectivamente, o preo de um
suco e o preo de um sanduche.
De acordo com o consumo e a despesa de cada mesa, temos
que

2x 3y 14
.

4x 5y 25

Considerando

sistema

5x 3y 4z 3

analise as afirmativas abaixo e


15x 9y 8z 6
20x 12y 16z 12

conclua.
a) O sistema impossvel.
b) O sistema possvel e indeterminado.
c) O sistema possvel e determinado.
d) O sistema admite como soluo nica x = 4, y = 8, z = -11
e) O sistema admite como soluo, para qualquer valor de x a
terna (x, x, 5x)

60n 3v 180
, temos: n = 13

125n 5v 625
488. (Pucsp 2011) Vtor e Valentina possuem uma caderneta de
poupana conjunta. Sabendo que cada um deles dispe de
certa quantia para, numa mesma data, aplicar nessa
caderneta, considere as seguintes afirmaes:
- se apenas Vtor depositar nessa caderneta a quarta parte da
quantia de que dispe, o seu saldo duplicar;
- se apenas Valentina depositar nessa caderneta a metade da
quantia que tem, o seu saldo triplicar;
- se ambos depositarem ao mesmo tempo as respectivas
fraes das quantias que tm, mencionadas nos itens
anteriores, o saldo ser acrescido de R$4947,00 .

Resposta: F V F F F.
COMENTRIO:

5x 3y 4z 3

Escalonando o sistema, temos:


4z 3
0000

Nessas condies, se nessa data no foi feito qualquer saque


de tal conta, correto afirmar que
a) Valentina tem R$6590,00 .

Como o nmero de equaes menor que o nmero de


incgnitas, conclui-se que o sistema possvel e
indeterminado.

b) Vtor tem R$5498,00 .


c) Vtor tem R$260,00 a mais que Valentina.
d) o saldo inicial da caderneta era R$1649,00 .

(F) o sistema possvel e indeterminado;


(V) o sistema possvel e indeterminado;
(F) o sistema possvel e indeterminado;
(F) basta substituir o terno ordenado no sistema escalonado;
(F) basta substituir o terno ordenado no sistema escalonado.
486. (Espcex (Aman) 2011)

RESPOSTA: A
COMENTRIO: Sejam n nmero de parcelas e v o valor de cada
parcela, ento:
n.v = (n - 3).(v + 60) ou n.v = (n - 5) .(v + 125).
Desenvolvendo as equaes e resolvendo o sistema

da mesa 3 R$ 5,50.
2011)

possvel

487. (Fuvest 2011) Uma geladeira vendida em n parcelas iguais,


sem juros. Caso se queira adquirir o produto, pagando-se 3 ou
5 parcelas a menos, ainda sem juros, o valor de cada parcela
deve ser acrescido de R$ 60,00 ou de R$ 125,00,
respectivamente. Com base nessas informaes, conclui-se
que o valor de n igual a
a) 13
b) 14
c) 15
d) 16
e) 17

Subtraindo a primeira equao da segunda, obtemos


2x 2y 11 x y 5,50, ou seja, o valor da despesa

485. (Upe

seja

e) o saldo inicial da caderneta era R$1554,00 .


RESPOSTA: D
COMENTRIO: S = saldo da poupana
x = valor de Vtor
y = valor de Valentina

Para que o sistema linear

2x y 5
seja possvel e indeterminado, o valor de

ax 2y b

x
S 2S

y S 3S
2

a b :
a) 1
b) 4
139

x
4 S

y 2S

Em um grande banco, os nmeros de todas as contas so


formados por algarismos de 0 a 9, na forma abcdef xy,

Portanto,

em que a sequncia (abcdef) representa, nessa ordem, os

x x
4947
4 2
S 2.S 4947

algarismos do nmero da conta e x e y, nessa ordem,


representam os dgitos verificadores.
Para obter os dgitos x e y, o sistema de processamento de

S 1649

dados do banco constri as seguintes matrizes:

1 2 1
x
(a b)

A 0 1 0 B y C (c d)
0 2 1
z
(e f )

489. (Unicamp simulado 2011) Uma empresa deve enlatar uma


mistura de amendoim, castanha de caju e castanha-do-par.
Sabe-se que o quilo de amendoim custa R$5,00, o quilo de
castanha de caju, R$20,00 e o quilo de castanha-do-par,
R$16,00. Cada lata deve conter meio quilo da mistura e o
custo total dos ingredientes de cada lata deve ser de R$5,75.
Alm disso, a quantidade de castanha de caju em cada lata
deve ser igual a um tero da soma das outras duas.

Os valores de x e y so obtidos pelo resultado da operao


matricial A B C, desprezando-se o valor de z. Assim, os
dgitos verificadores correspondentes conta corrente de
nmero 356281 so
a) 34
b) 41
c) 49
d) 51
e) 54

Nesse caso, as quantidades de cada ingrediente por lata so


a) 270 g de amendoim, 125 g de castanha de caju e 105 de
castanha-do-par.
b) 270 g de amendoim, 172,5 g de castanha de caju e 57,5 g de
castanha-do-par.
c) 250 g de amendoim, 125 g de castanha de caju e 125 g de
castanha-do-par.
d) 228 g de amendoim, 100 g de castanha de caju e 72 g de
castanha-do-par.

RESPOSTA: E
COMENTRIO: Para o nmero 356281 a matriz C dada
por

RESPOSTA: C
COMENTRIO: x a quantidade de amendoim
y a quantidade de castanha de caju
z a quantidade de castanha-do-par.

3 5 2
C 6 2 4 .
8 1 7

x y z 0,5

5 x 20 y 16 z 5,75 , resolvendo o sistema temos:

x 3y z 0

Logo,

1 2 1 x 2
A B C 0 1 0 y 4
0 2 1 z 7
x 2y z 2

y 4
2y z 7

x 5

y 4.
z 1

x = 0,25kg = 250g
y = 0,125kg = 125g
z = 0,125kg = 125g
490. (G1 - cftmg 2011) Em um determinado ms, o salrio de uma
funcionaria excedeu em R$600,00 as horas extras. Se ela
recebeu um total de
salrio foi de

R$880,00 , ento, o valor de seu

R$460,00
b) R$540,00
c) R$660,00
d) R$740,00
a)

RESPOSTA: D
COMENTRIO: Sejam x = salrio e y = horas extras.

x - y 600

x y 880
2x 1480
x 740
491. (Espcex (Aman) 2011) Os nmeros das contas bancrias ou
dos registros de identidade costumam ser seguidos por um ou
dois dgitos, denominados dgitos verificadores, que servem
para conferir sua validade e prevenir erros de digitao.

Portanto, os dgitos verificadores correspondentes conta


corrente de nmero 356281 so 54.

x 2y 3z a

492. (Ita 2011) O sistema y 2z b


3x y 5cz 0

a) possvel, a,b,c .
7b
b) possvel quando a
ou c 1.
3
c) impossvel quando c 1, a,b .
7b
d) impossvel quando a
, c .
3
140

e) possvel quando c 1 e a

494. (G1 - cftmg 2011) Um restaurante serve um prato especial


com dois tipos de comida A e B, cujas quantidades de
carboidratos e gorduras por poro encontram-se indicadas na
tabela abaixo.

7b
.
3

RESPOSTA: B
COMENTRIO:

D= 0

2 5c 12 9 2 5c 5

COMIDAS
A
B

3 1 5c

de pores A e B, respectivamente, usadas pelo nutricionista,


ento, a soluo desse problema um par ordenado que
pertence ao grfico da funo
a) y 3x 1

x 2y 3z a

, multiplicando a primeira equao por -1


y 2z b
3x y 5z 0

y 5x 6
c) y 4x
d) y x 2
b)

e somando com a segunda temos:

x 2y 3z a

y 2z b
, multiplicando a segunda equao por

0 2y 14z 3a

RESPOSTA: B
COMENTRIO: De acordo com o problema, temos:

sete e somando com a terceira temos:

20x 5y 60

2x y 8

x 2y 3z a

0 y 2z b se c = 1 e a = 7b/3 o sistema ser


0 0 0 7b 3a

Resolvendo o sistema, temos x = 2 e y = 4, que verificam a


equao do item [B].

possvel e indeterminado e se c = 1 e a 7b/3

493. (Unesp 2011) Uma pessoa necessita de 5 mg de vitamina E


por semana, a serem obtidos com a ingesto de dois
complementos alimentares e . Cada pacote desses
complementos fornece, respectivamente, 1 mg e 0,25 mg de
vitamina E. Essa pessoa dispe de exatamente R$47,00
semanais para gastar com os complementos, sendo que cada
pacote de custa R$5,00 e de R$4,00 .
O nmero mnimo de pacotes do complemento alimentar
que essa pessoa deve ingerir semanalmente, para garantir os 5
mg de vitamina E ao custo fixado para o mesmo perodo, de:
a) 3.

495. (Ufu 2011) Por causa de hbitos alimentares inadequados, um


cardiologista nota que os seus pacientes com hipertenso so
cada vez mais jovens e fazem uso de medicamentos cada vez
mais cedo. Suponha que Pedro, Mrcia e Joo sejam
pacientes, com faixas etrias bem distintas e que utilizam um
mesmo hipertensivo em comprimidos. Sabe-se que Joo utiliza
comprimidos de 2 mg, Mrcia de 4 mg e Pedro de 10 mg. Alm
disso, mensalmente, Pedro toma o triplo de comprimidos de
Mrcia e os trs consomem 130 comprimidos, totalizando 780
miligramas da droga.
Com base nestas informaes, correto afirmar que Mrcia,
mensalmente, ingere
a) 50 comprimidos
b) 20 comprimidos
c) 60 comprimidos
d) 30 comprimidos

5
16
c) 5,5 .
3
d) 6 .
4
b)

RESPOSTA: B
COMENTRIO: Sejam j, m e p, respectivamente, o nmero
de comprimidos que Joo, Mrcia e Pedro tomam
mensalmente. Logo, temos:

e) 8.
RESPOSTA: A
COMENTRIO:

Sejam

y,

respectivamente,

2j 4m 10p 780
m 20
j 17m 390

j 50 .
j m p 130
j 4m 130
p 3m
p 60

as

quantidades de pacotes dos complementos e que sero


ingeridos.

x 0,25y 5

5x 4y 47

16x 4y 80
.

5x 4y 47

Adicionando-se

as

duas

equaes,

vem

que

11x 33 x 3.
Portanto, devero ser ingeridos 3 pacotes do complemento

GORDURAS (g)
2
1

O nutricionista prepara esse prato de forma que contenha


60g de carboidrato e 8 g de gordura. Se x e y so os nmeros

Se -5c + 5 0 c 1 o sistema ser possvel e determinado.


Se c = 1, temos:

O sistema ser impossvel.

CARBOIDRATOS (g)
20
5

496. (G1 - ifsc 2011) Um cinema recebeu R$663,00 (seiscentos e


sessenta e trs reais) pela venda de ingressos (entrada),
durante uma nica sesso.
Nessa sesso, o nmero de ingressos vendidos para adultos foi
o triplo do nmero de ingressos vendidos para crianas. O
ingresso para adulto custava R$12,00 (doze reais) e o das
crianas R$3,00 (trs reais). Considere que x seja o nmero

141

a b c 888

ab c
c b 198

de ingressos vendidos para os adultos e y, o nmero de


ingressos vendidos para as crianas.

Resolvendo o sistema, temos: a = 444, b = 123 e c = 321.


Fazendo (a c) 90, temos (444 321) 90 = 33.
498. (Fgv 2010) Em um quadrado mgico, como o indicado na
figura, a soma dos nmeros em cada linha, em cada coluna e
em cada diagonal assume o mesmo valor.
A
18
25

24
C
E

B
D
21

Se as letras A, B, C, D e E representam nmeros, ento D + E


igual a
a) 43.
b) 44.
c) 45.
d) 46.
e) 47.
Assinale a alternativa que expressa corretamente a equao
que permite determinar o nmero de ingressos vendidos para
crianas, bem como para os adultos.

RESPOSTA: D
COMENTRIO:

x y 3
12x 3y 663

a)

x 3y
x y 663

b)

x y 3
x y 663

c)

x 3y
12x 3y 663

d)

43 + A = 21 + A + C c = 22
43 + A = A + B B = 19
25 + B + C = 66
B + D + 21= 66 D = 26
25 + E + 21 = 66 E = 20
Logo, D + E = 26 + 20 = 46

x 3y
3x 12y 663

e)

RESPOSTA: D
COMENTRIO: Nmero de adultos: x = 3y;
Nmero de crianas: y;

499. (Ibmecrj 2010) Seja o sistema linear nas incgnitas x, y e z


De acordo com o enunciado, temos:

x 3y

12x

3y 663

x y kz 1

2
2x k z 1
x y 2z 0

497. (G1 - epcar (Cpcar) 2011) Considere trs nmeros naturais a,


b e c, nessa ordem. A soma desses nmeros 888, a diferena
entre o primeiro e o segundo igual ao terceiro. O terceiro
deles excede o segundo em 198
O valor da diferena entre o primeiro e o terceiro tal que
excede 90 em
a) 23
b) 33
c) 43
d) 53

Assinale a afirmativa correta:


a) para k = 1, possui mais de uma soluo.
b) para k = 3, no possui soluo.
c) para k = 2, possui infinitas solues.
d) para k = 2, no possui soluo.
e) para k = 2, possui uma nica soluo.
RESPOSTA: D
COMENTRIO:

1 1

2 0 k 2 0 2k 4 0 k 2

RESPOSTA: B
COMENTRIO: Escrevendo o sistema de acordo com o texto:

1 1

142

x y 2200

y z 2100 somando as equaes, temos:


x z 2500

Fazendo k = 2, temos:

x y 2z 1

2 x 4 z 1
x y 2z 0

2x + 2y + 2x = 6800 x + y + z = 3400
Observado a primeira e terceira equaes, entendemos que
para k -2 o sistema impossvel.

2x k! y 2
1 k! x 21y 3

500. (Fgv 2010) Para que o sistema linear

de soluo (x, y) no seja possvel e determinado, o parmetro


k IN tem de ser igual a
a) 2.
b) 3.
c) 4.
d) 5.
e) 6.

503. (Enem 2 aplicao 2010) Algumas pesquisas esto sendo


desenvolvidas para se obter arroz e feijo com maiores teores
de ferro e zinco e tolerantes seca. Em mdia, para cada 100 g
de arroz cozido, o teor de ferro de 1,5 mg e o de zinco de
2,0 mg. Para 100 g de feijo, de 7 mg o teor de ferro e de 3
mg o de zinco. Sabe-se que as necessidades dirias dos dois
micronutrientes para uma pessoa adulta de
aproximadamente 12,25 mg de ferro e 10 mg de zinco.
Disponvel em: http://www.embrapa.br. Acesso em: 29 abr.
2010 (adaptado).
Considere que uma pessoa adulta deseja satisfazer suas
necessidades dirias de ferro e zinco ingerindo apenas arroz e
feijo. Suponha que seu organismo absorva completamente
todos os micronutrientes oriundos desses alimentos.
Na situao descrita, que quantidade a pessoa deveria comer
diariamente de arroz e feijo, respectivamente?
a) 58 g e 456 g
b) 200 g e 200 g
c) 350 g e 100 g
d) 375 g e 500 g
e) 400 g e 89 g

RESPOSTA: B
COMENTRIO:

k!

(1 k)! 21

0 k!.(1 k!) 42 0

k! k! 42 0
2

k! 7(no convm)
k! 6 k 3

501. (Pucrj 2010) Maria comprou duas bicicletas por um total de


R$ 670,00. Vendeu uma das bicicletas com lucro de 10% e a
outra com prejuzo de 5%. No total, ela ganhou R$ 7,00. Quais
foram os preos de compra?
a) R$ 370,00 e R$ 300,00
b) R$ 270,00 e R$ 400,00
c) R$ 277,00 e R$ 400,00
d) R$ 200,00 e R$ 470,00
e) R$ 377,00 e R$ 293,00

RESPOSTA: C
COMENTRIO: Sejam a e f, respectivamente, os nmeros de
pores de 100 gramas de arroz e de feijo que devero ser
ingeridas.
De acordo com o enunciado, obtemos o sistema

1,5a 7f 12,25

2a 3f 10

6a 28f 49

6a 9f 30

a 3,5
.

f 1

RESPOSTA: B
COMENTRIO:

Portanto, as quantidades de arroz e feijo que devero ser


ingeridas so, respectivamente, 3,5 100 350 g e

x y 670
x 270 e y 400

0,1x _ 0,05 y 7

1 100 100 g.

502. (Pucpr 2010) Como est aproximando-se o trmino do


desconto do IPI para a linha branca dos eletrodomsticos, uma
determinada loja de departamentos, para vender uma
geladeira, uma mquina de lavar e uma secadora, props a
seguinte oferta: a geladeira e a mquina de lavar custam
juntas R$ 2.200,00; a mquina de lavar e a secadora, R$
2.100,00; a geladeira e a secadora, R$ 2.500,00.

504. (G1 - cftmg 2010) Uma loja de ferramentas apresentou os


seguintes pacotes promocionais para chaves de fenda e de
boca:
pacote 1
preo: R$ 31,00

Quanto pagar um cliente que comprar os trs produtos


anunciados?
a) R$ 2.266,00
b) R$ 6.800,00
c) R$ 3.200,00
d) R$ 3.400,00
e) R$ 4.800,00

pacote 2
preo: R$ 44,00

RESPOSTA: D
COMENTRIO: x o preo da geladeira
y o preo da mquina de lavar
z o preo da secadora

143

Nessa promoo, o preo de uma chave de boca somado ao


de uma chave de fenda, em reais, igual a
a) 17
b) 21
c) 22
d) 34

x 2 y 3z 100

y 2 z 35

y 3

z 2

RESPOSTA: A
COMENTRIO: X o preo de cada chave de fenda e y o
preo de cada chave de boca. Considerando as figuras, temos:

Resolvendo o sistema temos z = 10, y = 15 e x = 40


Resposta: 40 retiradas de apenas um copo

3x y 31 3
9x 3y 93

2x 3y 44
2x 3y 44
Somando as equaes, temos x= 7 e y = 10.
7 + 10 = 17
505. (Uerj 2010) Um conjunto de 100 copos descartveis, dispostos
em um suporte, ser usado em uma festa.

506. (Fgv 2010) No incio de dezembro de certo ano, uma loja tinha
um estoque de calcas e camisas no valor total de R$ 140
000,00, sendo R$ 80,00 o valor (preo de venda) de cada cala
R$ 50,00 (preo de venda) o de cada camisa.
Ao longo do ms, foram vendidos 30% do nmero de calas
em estoque e 40% do nmero de camisas em estoque,
gerando uma receita de R$ 52 000,00.
Com relao ao estoque inicial, a diferena (em valor absoluto)
entre o nmero de calcas e o de camisas :
a) 1450
b) 1500
c) 1550
d) 1600
e) 1650
RESPOSTA: B
COMENTRIO: X = nmero de camisas (50)
Y = nmero de calas (80)

5 x 8 y 14000(3)
15x 24 y 42000

20
x

24
y

52000

20 x 24 y 52000
Resolvendo, temos x = 2000 e y = 500
x y = 2000 = 500 = 1500

Considere, agora, as seguintes informaes:


sempre se tenta retirar apenas 1 copo de cada vez desse
suporte;
quando se tenta retirar 1 copo, e exatamente 2 saem juntos,
1 deles desperdiado;
quando se tenta retirar 1 copo, e exatamente 3 saem juntos,
2 deles so desperdiados;
quando se tenta retirar 1 copo, nunca saem 4 ou mais de 4
juntos;
foram retirados todos os copos desse suporte, havendo
desperdcio de 35% deles.
a razo entre o nmero de vezes em que foram retirados
exatamente 2 copos juntos e o nmero de vezes em que
foram retirados exatamente 3 juntos foi de

507. (G1 - cps 2010) Um tcnico em edificaes realizou um


trabalho em y dias, fazendo x horas por dia. Se trabalhasse
duas horas a mais por dia, teria terminado o servio dois dias
antes, e se trabalhasse quatro horas a mais por dia, teria
terminado o servio trs dias antes. Os valores de x e y so tais
que
a) y o dobro de x.
b) y 1,5 de x.
c) a sua diferena 3.
d) a sua soma 9.
e) x a tera parte de y.

RESPOSTA: B
COMENTRIO: De acordo com o texto temos:

x y x 2 y 2 x y xy 2x 2y - 4 2x 2y 4.
x y x 4 y 3 x y xy 3x 4y 12 3x 4y 12

3
.
2

O nmero de vezes em que apenas 1 copo foi retirado do


suporte igual a:
a) 30
b) 35
c) 40
d) 45
RESPOSTA: C
COMENTRIO: x retiradas de 1 copo
y retiradas de 2 copos y copos desperdiados
z retiradas de 3 copos 2z copos desperdiados

Resolvendo um sistema com as equaes acima, temos:

2x 2y 4(2)
4x 4y 8

3x

4y

12

3x 4y 12
Temos: x = 4 e y = 6
Logo, y 1,5 x
508. (Fgv 2010) O sistema linear abaixo, nas incgnitas x e y:

x 3y m

2x py 2
Ser impossvel quando:

Ento temos o seguinte sistema:


144

a) Nunca
b) p 6 e m = 1
c) p 6 e m 1
d) p = 6 e m = 1
e) p = 6 e m 1

milhes de habitantes, um modelo matemtico capaz de


aproximar o nmero de habitantes (P), em milhes, a cada ano
(t), a partir de 1970, dado por:

P(t) 280 190 e0,019(t 1970)

RESPOSTA: E
COMENTRIO: Se D = 0 SPI ou SI

Baseado nesse modelo, e tomando a aproximao para o


logaritmo natural

0 p 6 0 p 6

2 p

14
In 1,9
95

Fazendo p = -6, temos:

x 3y m

2 x 6 y 2

a populao brasileira ser 90% da suposta populao de


estabilizao aproximadamente no ano de:
a) 2065.
b) 2070.
c) 2075.
d) 2080.
e) 2085.

Resolvendo temos 0 = -2m + 2


Logo, o sistema ser SI quando 2m + 2 for diferente de zero,
ou seja, quando m 1.
509. (Uece 2010) Se x, y e z constitui a soluo do sistema linear

RESPOSTA: B
COMENTRIO: Para que a populao brasileira seja 90% da
suposta populao de estabilizao, deveremos ter

x y z 1

x 2y 3z 2
x 4y 5z 4

0,9 280 280 190 e0,019(t 1970) e0,019(t 1970)

14
95

14
95
0,019(t 1970) 1,9

ento o produto x. y. z igual a


a) 4.
b) 8.
c) 2.
d) 6.

n e0,019(t 1970) n

t 1970

1,900
0,019

t 2070.

RESPOSTA: A
COMENTRIO: Escalonando os sistemas temos:

512. (Uff 2011) O ndice de Theil, um indicador usado para medir


desigualdades econmicas de uma populao, definido por
x = 2, y = 1 e z = -2

M
T n A ,
MG

Logo, x.y.z = -4

sendo

MATEMTICA II

MA

510. (Fuvest 2010) Tendo em vista as aproximaes log10 2 0,30,


log10 3 0,48, ento o maior nmero inteiro n, satisfazendo
n
418
10 12 , igual a
a) 424
b) 437
c) 443
d) 451
e) 460
RESPOSTA: D
n
418
COMENTRIO: 10 12
n
418
log10 log12 n log10

x x
1 N
xi 1 2 N
N i1
N

MG N xi N x1 x 2
i1

xN

xN ,

respectivamente, as mdias aritmtica e geomtrica das


rendas x1, x2, , xN (consideradas todas positivas e
medidas com uma mesma unidade monetria) de cada um dos
N indivduos da populao.
Com base nessas informaes, assinale a afirmativa incorreta.
a) T n(MA ) n(MG ).

MA
xi

418.log(2 .3) n
418(log2 + log3) n 418(2log2 + log3)
n 418(2.0,30+0,48) n 451,44
Logo o maior inteiro 451.
2

b) b) n

0 para todo xi 0, i 1,

, N.

xi
MA para todo i = 1, ..., N.
N
d) Se x1 x2 xN, ento T 0.
c)

511. (Unesp 2012) Em 2010, o Instituto Brasileiro de Geografia e


Estatstica (IBGE) realizou o ltimo censo populacional
brasileiro, que mostrou que o pas possua cerca de 190
milhes de habitantes. Supondo que a taxa de crescimento
populacional do nosso pas no se altere para o prximo
sculo, e que a populao se estabilizar em torno de 280

e)

145

MA
1 N MA 1 MA
n

n
n

N i1 xi N x1
x2

M
n A
xN

RESPOSTA: B
COMENTRIO: Devemos mostrar que existe pelo menos um

M
xi de modo que n A
xi

x.log2 = log 10 log2


x.0,3 = 1 0,3
0,3x = 0,7
X = 2,3333333....

0.

Com efeito, sem perda de generalidade, podemos supor que


x1 x2 xN. Assim, x1 MA xN e, portanto,

M
MA
1 n A 0.
xN
xN
513. (Fuvest 2011) Seja x > 0 tal que a sequncia a1 = log2x, a2 =
log4(4x), a3 = log8(8x) forme, nessa ordem, uma progresso
aritmtica. Ento, a1 + a2 + a3 igual a

515. (Ufrgs 2010) Sabendo-se que os nmeros 1 + log a, 2 + log b, 3


+ log c formam uma progresso aritmtica de razo r,
correto afirmar que os nmeros a, b, c formam uma
r-1
a) progresso geomtrica de razo 10 .
r
b) progresso geomtrica de razo 10 -1.
c) progresso geomtrica de razo log r.
d) progresso aritmtica de razo 1 + log r.
1+log r
e) progresso aritmtica de razo 10
.
RESPOSTA: A
COMENTRIO: r = 2 + logb (loga + 1)
r = 1 + logb loga
r 1 = log(b/a)
r-1
b/a = 10

13
2
15
b)
2
17
c)
2
19
d)
2
21
e)
2
a)

r = 3 + logc (logb + 2)
r = 1 + logc logb
r 1 = log(c/b)
r-1
c/b = 10
logo a,b e c formam uma P.G de razo 10

RESPOSTA: B
COMENTRIO: Como a1 , a2 ,a3 uma P.A, temos:
a1 + a3 = 2. a2

log2 x log8 8x 2.log4 4x


Escrevendo tudo na base 2, temos:

log2 x

516. (Fgv 2010) Considere o grfico das funes reais f(x) = 2 log x
e g(x) = log 2x, nos seus respectivos domnios de validade.
A respeito dos grficos de f e g, correto afirmar que
a) no se interceptam.
b) se interceptam em apenas um ponto.
c) se interceptam em apenas dois pontos.
d) se interceptam em apenas trs pontos.
e) se interceptam em infinitos pontos.

log2 (8x) log2 (8x)

log2 8
log2 4

RESPOSTA: B
COMENTRIO: Igualando as funes, temos:

Aplicando as propriedades dos logaritmos:

2logx = log(2x)
2
logx = log(2x)
2
x 2x = 0
x.(x 2 ) = 0

log2 8 log2 x 2. log2 4 log2 x)


log2 x

3
2

Calculando os logaritmos e desenvolvendo a equao, temos:

x = 0 (no satisfaz a condio de existncia) e x = 2.


Logo, as funes se interceptam em apenas um ponto de
abscissa x = 2.

log2x = 3 x = 8
Fazendo x = 8, temos:

517. (Mackenzie 2010) Considerando a soluo (x, y) do sistema

log4 x log2 y 5
x
, com x 1, o valor de logx

log
x

log
y

0
2
4
y

a1 = log28 = 3
a2 = log432 =

5
2

a) 1
b) 4
c) 1

a3 = lgo864 = 2
Somando a1 + a2 + a3 = 3 +

r1

5
15
+2=
2
2

1
2
1
e)
4
d)

514. (G1 - cftmg 2010) Considerando a equao 2 = 5 e que log2 =


0,3, o valor mais prximo de x
a) 2,2
b) 2,3
c) 2,4
d) 2,5

RESPOSTA: C
COMENTRIO: Mudando os logaritmos para a base 2, temos:

RESPOSTA: B
x
COMENTRIO: 2 = 5 (aplicando o logaritmo dos dois lados,
temos)
X
Log2 = log (10/2)
146

1
1
2 log 2 x log 2 y 5
2 log 2 x log 2 y 5

1
log 2 x log 2 y 0 ( x 2)

2.log 2 x log 2 y 0
2

a) 4.
b) 4 2 .
c) 6.
d) 5 3 .
e)

6 3.

5
1
, log 2 x 5 x 4 e log 2 4 log 2 y 5 log 2 y 4 y 16
RESPOSTA: C
2
2
COMENTRIO: Considere a figura abaixo.

Logo log x

x
4
1
log 4
log 4 1
y
16
4

MATEMTICA III
518. (Fuvest 2012) Em um tetraedro regular de lado a, a distncia
entre os pontos mdios de duas arestas no adjacentes igual
a
a) a 3
b) a 2

a 3
2
a 2
d)
2
a 2
e)
4
c)

Como P e S so, respectivamente, os pontos mdios das


arestas AV e AB do tetraedro, segue que PS base
mdia
do
tringulo
isto
,
AVB,

PS

VB
VB 2 PS.
2

Por outro lado, como o tetraedro regular, temos que as


quatro faces do tetraedro so tringulos equilteros e,

RESPOSTA: D
COMENTRIO:

portanto, PS SR RQ QP.
Alm disso, como a superfcie total do tetraedro 9 3, vem

(2 PS)2 3
3
9 3 PS . Da, o permetro
4
2
3
pedido 4 PS 4 6.
2
que 4

2
a 3
3a2 a2
a
2
d2

d
d

4
4
2
2

519. (Ufrgs 2011)

2.a2
a 2
d
4
2

520. (Espcex (Aman) 2011) Na figura abaixo, est representado um


slido geomtrico de 9 faces, obtido a partir de um cubo e
uma pirmide. Sabendo que todas as arestas desse slido tm
medida , ento as medidas da altura (distncia do ponto V
face ABCD) e da superfcie total desse slido so,
respectivamente,

A superfcie total do tetraedro regular

representado na figura abaixo 9 3 . Os vrtices do


quadriltero PQRS so os pontos mdios de arestas do
tetraedro, como indica a figura.

2 2
e
2

a)

b)

O permetro do quadriltero
147

2 2

e
2

( 3 4)

( 3 5)

3 2
e
2

3
5

c)

d)

e
2

e)

e
2

( 3 5)

3
4

RESPOSTA: B
COMENTRIO: Considere a figura abaixo, em que O o
centro da base da pirmide.
A aresta da base da pirmide tem a mesma medida do raio da
circunferncia.
Logo,
temos

1 6.r 2 . 3
.h .r 2 .6 3 h 12
3 4

2
.
2

Como VE EF , segue que OE

Aplicando o Teorema de Pitgoras no tringulo


obtemos
2

VO VE OE VO

VOE,

2
.
2

522. (Insper 2011) Dois faras do antigo Egito mandaram construir


seus tmulos, ambos na forma de pirmides quadrangulares
regulares, num mesmo terreno plano, com os centros de suas
bases distando 120m As duas pirmides tm o mesmo volume,
mas a rea da base de uma delas o dobro da rea da base da
outra. Se a pirmide mais alta tem 100m de altura, ento a
distncia entre os vrtices das duas pirmides, em metros,
igual a
a) 100.
b) 120.
c) 130.
d) 150.
e) 160.
RESPOSTA: C
COMENTRIO: Sejam A1 e A 2 , as reas das bases das

Desse modo, a distncia do ponto V face ABCD

2 2
2

.
2
2

pirmides.
Como os volumes so iguais, temos

1
1
A1 O1V1 A 2 O2 V2 A1 O1V1 A 2 O2 V2.
3
3

A superfcie total do slido dada por

4 (VEF) 5 (ABCD) 4

Dado que A1 2 A2, vem

2 A2 O1V1 A2 O2 V2 O2 V2 2 O1V1.

( 3 5).

Assim, a pirmide mais alta tem a base menor e, portanto,

521. (Ifsp 2011) A base de uma pirmide hexagonal regular est


inscrita em um crculo que a base de um cilindro reto de

O2 V2 100 m O1V1 50 m.

altura 6 3 cm. Se esses slidos tm o mesmo volume, ento


a medida, em centmetros, da altura da pirmide
a) 9
b) 12
c) 15
d) 18
e) 24
RESPOSTA: B
COMENTRIO:

148

Como o terreno plano, segue que V1P O1P, sendo P o

a)

p da perpendicular baixada de V1 sobre O2 V2. Da,

V1P O1O2 120 m e V2P 100 50 50 m.

b)

Finalmente, aplicando o Teorema de Pitgoras no tringulo


V1PV2, obtemos

c)

V1V2 502 1202 V1V2 130 m.

d)

523. (Ita 2010) Sejam A, B, C e D os vrtices de um tetraedro


regular cujas arestas medem 1 cm. Se M e o ponto mdio do

e)

segmento AB e N e o ponto mdio do segmento CD , ento


2
a rea do triangulo MND, em cm , e igual a

RESPOSTA: D
COMENTRIO: O baricentro divide a mediana na razo 2 para
1
Seco transversal = quadrado (maior) destacado

2
a)
.
6
b)

2
.
8

c)

3
.
8

d)

3
.
8

e)

3
.
9

5
9
4
9
1
3
2
9
1
9

Asec 2k
A
4
4
sec Asec
Abase 3k
1
9
9
1
2
rea pedida Asec
2
9

RESPOSTA: B
COMENTRIO:
MD = MC =

3
2

ND =
525. (Ufmg 2010) Em uma indstria de velas, a parafina
armazenada em caixas cbicas, cujo lado mede a.
Depois de derretida, a parafina derramada em moldes em
formato de pirmides de base quadrada, cuja altura e cuja

2
3

1 ( MN ) 2
2
2

aresta da base medem, cada uma,

2
MN =
2
Logo a rea pedida

A=

a
.
2

Considerando-se essas informaes, CORRETO afirmar que,


com a parafina armazenada em apenas uma dessas caixas,
enche-se um total de
a) 6 moldes.
b) 8 moldes.
c) 24 moldes.
d) 32 moldes.

1 21
2
2
=
cm
2 2 2 8
A

RESPOSTA: C
3
COMENTRIO: Volume do cubo = a

Volume da pirmide =

Nmero de moldes =

C
N
D
524. (Fuvest 2010) Uma pirmide tem como base um quadrado de
lado 1, e cada uma de suas faces laterais um tringulo
equiltero. Ento, a rea do quadrado, que tem como vrtices
os baricentros de cada uma das faces laterais, igual a
149

1 a a a3
.
3 2 2 24
Volume do cubo
a3
3 24
Volume da pirmide a
24

VI = VII

526. (Ueg 2012)


Em uma festa, um garom, para servir
refrigerante, utilizou uma jarra no formato de um cilindro
circular reto. Durante o seu trabalho, percebeu que com a
jarra completamente cheia conseguia encher oito copos de
300ml cada. Considerando-se que a altura da jarra de 30cm,
ento a rea interna da base dessa jarra, em cm,
a) 10
b) 30
c) 60
d) 80

.62.h .82.4
h
h

64.4
36
7,11 cm

528. (Uftm 2011) Um paraleleppedo reto-retngulo, de volume V1,


e um cilindro circular reto, de raio R = 0,5 m e volume V2, tm
a mesma altura h = 4 m.

RESPOSTA: A
COMENTRIO:

Se

V1 2
, ento a medida x da aresta da base do
V2

paraleleppedo igual a
a) 5 2.

Ab rea da base

5 2
.
2
2
.
c)
2
2
.
d)
4
10
.
e)
4
b)

1mL 1cm3
Volume da jarra 8 30mL 2400mL 2400cm3
Ab .30 2400
Ab 80cm2
527. (Ufpr 2012) As duas latas na figura abaixo possuem
internamente o formato de cilindros circulares retos, com as
alturas e dimetros da base indicados. Sabendo que ambas as
latas tm o mesmo volume, qual o valor aproximado da altura
h?

RESPOSTA: C
COMENTRIO:

x2 .h
2

.(0,5) .h

a) 5 cm.
b) 6 cm.
c) 6,25 cm.
d) 7,11 cm.
e) 8,43 cm.

2
1
2
x2 x

2
2

529. (Uesc 2011) Um reservatrio com formato de um cilindro


circular reto de altura H, completamente vazio, comea a ser
abastecido de gua a uma razo de k litros por minuto, ficando
completamente cheio em T horas.
Dentre os grficos, o que melhor representa h(t), nvel da gua
no reservatrio a cada instante t,

RESPOSTA: D
COMENTRIO:
150

a) 6
b) 5
c) 7
d) 8
e) 9

a)

RESPOSTA: C
COMENTRIO: Se Vt denota a capacidade total do tanque,
ento:
b)

Vt abc 80 200 50 800.000cm3 .


Assim, a metade da capacidade do tanque corresponde a:

Vt 800000

400.000cm3 .
2
2
Da, se Vb a capacidade do balde, temos:
c)

Vb r 2h 3,1 202 50 62.000cm3 .


Por conseguinte, o nmero de vezes que o tratador dever
buscar gua no reservatrio :

d)

e)
RESPOSTA: E
COMENTRIO: Sabendo que a vazo a razo entre o volume

531. (Ufrgs 2011) Um tipo de descarga de gua para vaso sanitrio


formado por um cilindro com altura de 2 m e dimetro
interno de 8 cm.
Ento, dos valores abaixo, o mais prximo da capacidade do
cilindro
a) 7L.
b) 8L.
c) 9L.
d) 10L.
e) 11L.

RESPOSTA: D
COMENTRIO: Se a altura do cilindro mede 2 m 20dm e

r2 h
t
h(t) k 2 .
e o tempo, segue que k
t
r

o dimetro 8cm 0,8dm, ento a capacidade do cilindro


dada
por

Portanto, como h e t so diretamente proporcionais, segue


que o grfico que melhor representa h(t) o da alternativa

0,8
3

20 3,14 0,16 20 10,048dm 10 L.


2

(E).
530. (Ufu 2011) Durante uma feira de exposio de animais, um
tratador de cavalos encarregado de levar gua a alguns
animais em uma baia. colocado um tanque vazio na baia na
forma de um paraleleppedo retangular com a = 80 cm, b = 2
m e c = 50 cm, conforme ilustra a figura. O tratador transporta
gua de um reservatrio para o tanque, em um balde de
formato cilndrico com base de 40 cm de dimetro e 50 cm de
altura. Estima-se que a cada vez que vai ao reservatrio, ele
enche o balde e, no caminho, derrame 5% de seu contedo.
Para que o nvel de gua no tanque atinja a metade de sua
capacidade, o nmero mnimo de vezes que o tratador dever
buscar gua no reservatrio igual a

400.000
7.
0,95 62000

532. (G1 - ifal 2011) Arquimedes, para achar o volume de um


objeto de forma irregular, mergulhou-o num tanque cilndrico
circular reto contendo gua. O nvel da gua subiu 10 cm sem
transbordar. Se o dimetro do tanque 20 cm, ento o
volume do objeto :

(Utilize = 3,1).
a) 1.000
151

b) 2.000
c) 3.000
d) 4.000
e) 5.000

Portanto, como a massa o produto do volume pela


densidade, segue que:

8900 24 2700 36 310.800kg 310,8 ton.


535. (Insper 2011) Um quadrado de lados medindo 1cm sofre
uma rotao completa em torno de um eixo paralelo a um de
seus lados. A distncia desse eixo a um dos vrtices do
quadrado xcm como mostra a figura.

RESPOSTA: A
COMENTRIO: O volume do objeto dado por
2

20
10 1.000 cm3 .
2
533. (Epcar (Afa) 2011) Uma vincola armazena o vinho produzido
em um tanque cilndrico (reto) com sua capacidade mxima
ocupada. Esse vinho ser distribudo igualmente em barris
idnticos tambm cilndricos (retos) e vendidos para vrios
mercados de uma cidade.
Sabe-se que cada mercado receber 2 barris de vinho, com

1
da altura do tanque e com dimetro da base
5

altura igual a
igual a

O grfico que melhor representa a rea total S do slido


2
gerado por essa rotao, em cm em funo de x para x 0,

1
do dimetro da base do tanque. Nessas condies, a
4

quantidade x de mercados que recebero os barris (com sua


capacidade mxima ocupada) tal que x pertence ao intervalo
a) 0 < x < 20
b) 20 x < 40
c) 40 x < 60
d) 60 x < 80
e) 80 x < 100

a)

RESPOSTA: C
COMENTRIO: Considerando VT o volume do tanque e VB o
volume do barril:

VT r 2 .h
2

r h .R2 .h VT
VB = .

80
80
4 5

b)

Portanto, x = 80 barris e 40 mercados.


534. (Cesgranrio 2011) Um slido totalmente macio composto
pela unio de dois cilindros circulares retos de mesmo
dimetro. As densidades do cilindro menor e do cilindro maior
3
3
valem, respectivamente, 8.900 kg/m e 2.700 kg/m

c)

d)
Considerando-se = 3, a massa desse slido, em toneladas,
vale
a) 97,2
b) 114,5
c) 213,6
d) 310,8
e) 320,4
e)
RESPOSTA: D
COMENTRIO:

2 2 24 m
2

volume

do

e o do maior

cilindro

menor

RESPOSTA: E
COMENTRIO: Atravs da rotao do quadrado em torno do
eixo, obtemos o seguinte slido.

2 3 36 m .
2

152

sobrar como espao vazio. Adotando 3,14 como aproximao


para p , correto afirmar que a capacidade volumtrica desse
espao vazio :
a) inferior capacidade de um cilindro.
b) maior que a capacidade de um cilindro mas menor que a
capacidade de dois cilindros.
c) maior que a capacidade de dois cilindros mas menor que a
capacidade de trs cilindros.
d) maior que a capacidade de trs cilindros mas menor que a
capacidade de quatro cilindros.
e) maior que a capacidade de quatro cilindros.

2 (x 1) 1 2 x 1 2 (2x 1)cm2 .

RESPOSTA: D
3
COMENTRIO: Volume do continer:= 10.8.6= 480m
2
3
Volume de um cilindro = 3,14 .1 . 10 = 31,40m
3
Espao vazio = 480 12.31,40 = 103,2 m
3
Volume de 3 cilindros = 3. 31,40 = 94,20 m
3
Volume de 4 cilindros= 4. 31,40 = 125,60 m

Logo, a rea total do slido

94,20 < 103,2 < 125,60

A rea da base do slido dada por

[(x 1)2 x2 ] (2x 1)cm2 .


A soma das reas laterais externa e interna

S(x) 2 (2x 1) 2 (2x 1) 8 x 4 cm2 .


Observando que a rea total dada por uma funo afim,
basta calcular

S(0) 8 0 4 4
e

S(1) 8 1 4 12
para deduzir que o grfico que melhor representa a rea total
S do slido o da alternativa (E).

538. (Enem 2 aplicao 2010) O administrador de uma cidade,


implantando uma poltica de reutilizao de materiais
descartados, aproveitou milhares de tambores cilndricos
dispensados por empresas da regio e montou kits com seis
tambores para o abastecimento de gua em casas de famlias
de baixa renda, conforme a figura seguinte. Alm disso, cada
famlia envolvida com o programa ir pagar somente R$ 2,50
por metro cbico utilizado.

536. (Uece 2010) Um fabricante de latas de alumnio com a forma


de cilindro circular reto vai alterar as dimenses das latas
fabricadas de forma que volume seja preservado. Se a medida
do raio da base das novas latas o dobro da medida do raio da
base das antigas, ento a medida da nova altura
a) a metade da medida da altura das latas antigas.
b) um tero da medida da altura das latas antigas.
c) um quarto da medida da altura das latas antigas.
d) dois teros da medida da altura das latas antigas.

Uma famlia que utilizar 12 vezes a capacidade total do kit em


um ms pagar a quantia de
(considere 3)
a) R$ 86,40.
b) R$ 21,60.
c) R$ 8,64.
d) R$ 7,20.
e) R$ 1,80.

RESPOSTA: C
COMENTRIO:
1
2
H

RESPOSTA: B
COMENTRIO: Como 40cm 0,4 m, segue que o volume

R
2

de

2R

tambor

dado

por

0,4
3
r2 h 3
1 0,12 m .
2

V1= .R .H e V2 = .(2R) .h
Como V1 = V2 temos:
2
2
.R .H = .(2R) .h
H = 4h
H/4 = h
Portanto a nova altura ser da anterior.
537. (Ufc 2010) Em um continer de 10 m de comprimento, 8 m de
largura e 6 m de altura, podemos facilmente empilhar 12
cilindros de 1 m de raio e 10 m de altura cada, bastando displos horizontalmente, em trs camadas de quatro cilindros
cada. Porm, ao faz-lo, um certo volume do continer

um

Assim,

volume

de

gua

contido

em

um

kit

6 0,12 0,72 m .
3

Por conseguinte, o valor a ser pago por uma famlia que


utilizar 12 vezes a capacidade total do kit em um ms de

2,5 12 0,72 R$ 21,60.


539. (Cesgranrio 2010)

153

RESPOSTA: B
COMENTRIO:
Sejam r1 2cm e h1 13,5cm, respectivamente, o raio
da base e a altura do cilindro cujo rtulo custa R$ 0,60.
Se V1 e A

denotam, respectivamente, a capacidade e a

V1 22 13,5 54 cm3

rea do rtulo, ento


Uma placa metlica quadrada dobrada de modo a formar
um cilindro (sem fundo e sem tampa), como ilustrado.
O volume no interior desse cilindro 18 litros. Ao ter sua
temperatura aumentada de 40 C, a placa dilata de forma que
2
sua rea aumenta de 72 mm . Considerando-se = 3, o
coeficiente de dilatao linear do material do qual a placa
1
constituda vale, em C ,
-6
a) 5,0 10
-6
b) 2,5 10
-7
c) 5,0 10
-7
d) 2,5 10
-8
e) 5,0 10

2 2 13,5 54 cm .

Sejam r2 e h2 , respectivamente, o raio da base e a altura da


nova embalagem. Como h2 2 r2 e as capacidades das
embalagens
so
iguais,
temos
que

V1 V2 54 r22 2r2 r2 3 27 3.
Alm

disso,

rea

lateral

da

nova

embalagem

2 3 6 36 cm .
2

Supondo que o custo da embalagem seja diretamente


proporcional rea lateral da mesma, obtemos

RESPOSTA: B
COMENTRIO:
2r

0,6
, sendo k a constante de
54
proporcionalidade e c1 o custo da primeira embalagem.
c1 k A

2r

2r

Portanto,

-4

c2 k A

0,6
36 R$ 0,40
54

c 2 36 2

, ou seja, o valor que o fabricante dever


c1 54 3

18L = 18dm e 72mm = 72.10 dm


2
.r .2.r = 18 (fazendo = 3)
3
2
18 r = 18 r = 1dm logo a rea do quadrado 36dm

pagar por esse rtulo de R$ 0,40, pois haver uma


reduo de c1 c 2 c1

A A i .2t , onde o coeficient e de dilatao linear.

2
1
c1 c1
3
3

na superfcie da

embalagem coberta pelo rtulo.

-4-

72.10 = 36.2.40 =2,5.10

-6 O -1

540. (Enem 2 aplicao 2010) Um fabricante de creme de leite


comercializa seu produto em embalagens cilndricas de
dimetro da base medindo 4 cm e altura 13,5 cm. O rtulo de
cada uma custa R$ 0,60. Esse fabricante comercializar o
referido produto em embalagens ainda cilndricas de mesma
capacidade, mas com a medida do dimetro da base igual da
altura.

541. (Enem 2010) Alguns testes de preferncia por bebedouros de


gua foram realizados com bovinos, envolvendo trs tipos de
bebedouros, de formatos e tamanhos diferentes. Os
bebedouros 1 e 2 tm a forma de um tronco de cone circular
reto, de altura igual a 60 cm, e dimetro da base superior igual
a 120 cm e 60 cm, respectivamente. O bebedouro 3 um
semicilindro, com 30 cm de altura, 100 cm de comprimento e
60 cm de largura. Os trs recipientes esto ilustrados na
figura.

Levando-se em considerao exclusivamente o gasto com o


rtulo, o valor que o fabricante dever pagar por esse rtulo
de
a) R$ 0,20, pois haver uma reduo de

2
na superfcie da
3

embalagem coberta pelo rtulo.


b) R$ 0,40, pois haver uma reduo de

1
na superfcie da
3

embalagem coberta pelo rtulo.


c) R$ 0,60, pois no haver alterao na capacidade da
embalagem.
d) R$ 0,80, pois haver um aumento de

1
na superfcie da
3

embalagem coberta pelo rtulo.


e) R$ 1,00, pois haver um aumento de

2
na superfcie
3

Considerando que nenhum dos recipientes tenha tampa, qual


das figuras a seguir representa uma planificao para o
bebedouro 3?

da embalagem coberta pelo rtulo.


154

Poucos dias aps o trmino da construo da cisterna, quando


ela ainda estava totalmente vazia, choveu dois dias seguidos, o
que deixou o Sr. Joo muito feliz e ele pde observar que:
2

no primeiro dia, o ndice pluviomtrico foi de 36 mm/m , o


que fez o nvel da gua na cisterna atingir a marca de 72 cm;
2
no segundo dia, o ndice foi de 30 mm/m .

a)

Considere que:
no foi retirada gua da cisterna nesse perodo;
no interior da cisterna entrou apenas a gua da chuva;
o ndice pluviomtrico e a altura da gua na cisterna so
grandezas diretamente proporcionais.

b)

Sendo assim, o Sr. Joo determinou que o volume de gua


captado e armazenado na cisterna aps esses dois dias de
chuva , em litros,

c)

Lembre que:
3
1 m = 1 000L
a) 980.
b) 1 860.
c) 2 100.
d) 3 030.
e) 3 960.

d)

RESPOSTA: E
COMENTRIO: 36-----------72
30 -----------x
x = 60cm
Altura total em 2 dias 72 + 60 = 132 cm = 1,32 m
2

Volume: V = .2 .1,32
2
V = 3.1 .1.32
3
V = 3,96m
V= 3960 L

e)
RESPOSTA: E
COMENTRIO: A superfcie do bebedouro 3 constituda por
dois semicrculos e por um retngulo.
542. (G1 - cps 2010) Para evitar o desperdcio de gua potvel em
sua casa, o Sr. Joo construiu um sistema de captao de gua
de chuva. Essa gua ser armazenada em uma cisterna
cilndrica cujas dimenses internas so trs metros de altura e
dois metros de dimetro, conforme esquema na figura.

543. (Enem 2 aplicao 2010) Uma fbrica de tubos acondiciona


tubos cilndricos menores dentro de outros tubos cilndricos. A
figura mostra uma situao em que quatro tubos cilndricos
esto acondicionados perfeitamente em um tubo com raio
maior

Suponha que voc seja o operador da mquina que produzir


os tubos maiores em que sero colocados, sem ajustes ou
folgas, quatro tubos cilndricos internos. Se o raio da base de
cada um dos cilindros menores for igual a 6 cm, a mquina por
voc operada dever ser ajustada para produzir tubos
maiores, com raio da base igual a
a) 12 cm

Volume de um cilindro
2
V = . r . h,
em que r o raio da base e h a altura
Adote: = 3

12 2cm
c) 24 2cm
b)

d) 6 1

155

2 cm

e) 12 1

2 cm

RESPOSTA: D
COMENTRIO: Considere a figura, em que O o centro da
base do cilindro cujo raio queremos calcular.

545. (Enem 2 aplicao 2010) Certa marca de suco vendida no


mercado em embalagens tradicionais de forma cilndrica.
Relanando a marca, o fabricante ps venda embalagens
menores, reduzindo a embalagem tradicional tera parte de
sua capacidade.
Por questes operacionais, a fbrica que fornece as
embalagens manteve a mesma forma, porm reduziu
metade o valor do raio da base da embalagem tradicional na
construo da nova embalagem. Para atender solicitao de
reduo da capacidade, aps a reduo no raio, foi necessrio
determinar a altura da nova embalagem.
Que expresso relaciona a medida da altura da nova
embalagem de suco (a) com a altura da embalagem tradicional
(h)?

h
12
h
b) a
6
2h
c) a
3
4h
d) a
3
4h
e) a
9
a)

O lado do quadrado ABCD igual ao dimetro da base dos


cilindros menores. Logo, AB 2 6 12cm. Alm disso,

BD
segue
,
2
AB 2 12 2
OB

6 2 cm.
2
2

como

OB

que

Portanto, o raio da base do cilindro maior dado por

OQ OB BQ 6 2 6 6( 2 1)cm.

RESPOSTA: D
COMENTRIO: Sejam v e v ', respectivamente, a capacidade
da embalagem tradicional e a capacidade da nova embalagem.
Portanto, de acordo com o enunciado, temos

544. (Enem 2010) Dona Maria, diarista na casa da famlia Teixeira,


precisa fazer caf para servir as vinte pessoas que se
encontram numa reunio na sala. Para fazer o caf, Dona
Maria dispe de uma leiteira cilndrica e copinhos plsticos,
tambm cilndricos.

Com o objetivo de no desperdiar caf, a diarista deseja


colocar a quantidade mnima de gua na leiteira para encher
os vinte copinhos pela metade. Para que isso ocorra, Dona
Maria dever
a) encher a leiteira at a metade, pois ela tem um volume 20
vezes maior que o volume do copo.
b) encher a leiteira toda de gua, pois ela tem um volume 20
vezes maior que o volume do copo.
c) encher a leiteira toda de gua, pois ela tem um volume 10
vezes maior que o volume do copo.
d) encher duas leiteiras de gua, pois ela tem um volume 10
vezes maior que o volume do copo.
e) encher cinco leiteiras de gua, pois ela tem um volume 10
vezes maior que o volume do copo.

v'

RESPOSTA: D
COMENTRIO:

Volume da leiteira = .4 .20 = 320 cm


2

Volume do concreto V. Logo:


V = Volume do cilindro maior volume do cilindro menor

1
2
20. 16 cm = 160
2

cm3

1
1
4h
r
v a r2 h a
.
3
2
3
3

546. (Enem 2010) Para construir uma manilha de esgoto, um


cilindro com 2 m de dimetro e 4 m de altura (de espessura
desprezvel), foi envolvido homogeneamente por uma camada
de concreto, contendo 20 cm de espessura.
Supondo que cada metro cbico de concreto custe R$ 10,00 e
tomando 3,1 como valor aproximado de , ento o preo
dessa manilha igual a
a) R$ 230,40.
b) R$ 124,00.
c) R$104,16.
d) R$ 54,56.
e) R$ 49,60.

RESPOSTA: A
2
3
COMENTRIO: Volume do copinho = .2 .4 = 16 cm
Volume de 20 copinhos pela metade =

V = .(1,2) .4 - .1 .4
V = 1,76.3,1

156

V= 5,456m
Logo, o preo da manilha ser 5,456 . 10 = R$ 54,56
547. (Unemat 2010) Para projetar um reservatrio cilndrico de
3
volume 81 m , dispe-se de uma rea circular de 6 m de
dimetro. Considerando = 3,14, a altura dever ser de:
a) 6 m
b) 9 m
c) 12 m
d)

81
m
6

e) 3m
RESPOSTA: B
2
COMENTRIO: .3 .h 81 h 9m

Volume ( m )
Espcie I
Espcie II

Massa (toneladas)

3.3 .12.0,06=19,44
2
2.4 .10.0,06 = 19,2

0,77.19,44 = 14,96
0,78.19,2 = 14,97

549. (Enem 2 aplicao 2010) Joo tem uma loja onde fabrica e
vende moedas de chocolate com dimetro de 4 cm e preo de
R$ 1,50 a unidade. Pedro vai a essa loja e, aps comer vrias
moedas de chocolate, sugere ao Joo que ele faa moedas
com 8 cm de dimetro e mesma espessura e cobre R$ 3,00 a
unidade.
Considerando que o preo da moeda depende apenas da
quantidade de chocolate, Joo
a) aceita a proposta de Pedro, pois, se dobra o dimetro, o
preo tambm deve dobrar.
b) rejeita a proposta de Pedro, pois o preo correto seria R$
12,00.
c) rejeita a proposta de Pedro, pois o preo correto seria R$
7,50.
d) rejeita a proposta de Pedro, pois o preo correto seria R$
6,00.
e) rejeita a proposta de Pedro, pois o preo correto seria R$
4,50.

RESPOSTA: D
COMENTRIO: Sejam r e h, respectivamente, o raio e a
espessura das moedas de chocolate fabricadas atualmente.
Logo, o volume V de chocolate de uma moeda

548. (Enem 2010) No manejo sustentvel de florestas, preciso


muitas vezes obter o volume da tora que pode ser obtida a
partir de uma rvore. Para isso, existe um mtodo prtico, em
que se mede a circunferncia da rvore altura do peito de
um homem (1,30 m), conforme indicado na figura. A essa
medida denomina-se "rodo" da rvore. O quadro a seguir
indica a frmula para se cubar, ou seja, obter o volume da tora
3
em m a partir da medida do rodo e da altura da rvore.

V r 2 h.
De acordo com a sugesto de Pedro, o volume V ' de
chocolate empregado na fabricao de uma moeda com
8cm
de
dimetro
seria

V ' (2r)2 h 4 r 2 h 4V.


V

Supondo que o preo p


proporcional

ao

da moeda seja diretamente

volume de
p k V R$ 1,50, em que

chocolate, segue que


k a constante de

proporcionalidade. Assim, o preo p ' da moeda sugerida por


Pedro

deveria

ser

de

p' k V' k 4V 4 1,50 R$ 6,00.

Um tcnico em manejo florestal recebeu a misso de cubar,


abater e transportar cinco toras de madeira, de duas espcies
diferentes, sendo
3 toras da espcie I, com 3 m de rodo, 12 m de comprimento
3
e densidade 0,77 toneladas/m ;
2 toras da espcie II, com 4 m de rodo, 10 m de
3
comprimento e densidade 0,78 toneladas/m .

550. (G1 - cftmg 2010) Um aluno gira um retngulo em torno do


eixo que contm um de seus lados e calcula o volume V do
slido obtido. Depois, ele traa a diagonal do retngulo e o
separa em dois tringulos,como mostra a figura.

Aps realizar seus clculos, o tcnico solicitou que enviassem


caminhes para transportar uma carga de, aproximadamente,
a) 29,9 toneladas.
b) 31,1 toneladas.
c) 32,4 toneladas.
d) 35,3 toneladas.
e) 41,8 toneladas.

Ao girar cada um dos tringulos, em torno do mesmo eixo de


rotao, os volumes dos slidos obtidos so

1
2
Ve V
3
3
1
3
b) V e V
4
4
1
4
c) V e V
5
5
a)

RESPOSTA: A
COMENTRIO:

157

d)

1
1
2
2
rcon
hcon rcil
hcil (2R)2 hcon R2 hcil
3
3
3
hcon hcil.
4

1
5
Ve V
6
6

RESPOSTA: A
COMENTRIO:

552. (Ufpb 2011) A prefeitura de certo municpio realizou um


processo de licitao para a construo de 100 cisternas de
placas de cimento para famlias da zona rural do municpio.
Esse sistema de armazenamento de gua muito simples, de
baixo custo e no poluente. A empreiteira vencedora
2
estipulou o preo de 40 reais por m construdo, tomando por
base a rea externa da cisterna. O modelo de cisterna pedido
no processo tem a forma de um cilindro com uma cobertura
em forma de cone, conforme a figura abaixo.

V .R2 .h
1
1
.R2 .h .V
3
3
V 2
V2 V V1 V V
3 3
V1

551. (Unicamp 2011) Depois de encher de areia um molde


cilndrico, uma criana virou-o sobre uma superfcie
horizontal. Aps a retirada do molde, a areia escorreu,
formando um cone cuja base tinha raio igual ao dobro do raio
da base do cilindro.

Considerando que a construo da base das cisternas deve


estar includa nos custos, correto afirmar que o valor, em
reais, a ser gasto pela prefeitura na construo das 100
cisternas ser, no mximo, de:
Use: = 3,14
a) 100.960
b) 125.600
c) 140.880
d) 202.888
e) 213.520
RESPOSTA: E
COMENTRIO:

A altura do cone formado pela areia era igual a

3
da altura do cilindro.
4
1
b) da altura do cilindro.
2
2
c) da altura do cilindro.
3
1
d) da altura do cilindro.
3
a)

rea de uma cisterna = rea da sup. lateral do cone + rea da


superfcie lateral do cilindro + rea do crculo.
2
rea da Cisterna = .2.2,5 + 2. .2.2 + .2
2
rea da cisterna = 17.m
2
rea de 100 cisternas 1700.m
Valor das cisternas 40.1700.3,14 = 213.520 reais.

RESPOSTA: A
COMENTRIO: Como o volume de areia o mesmo, segue
que:

158

553. (Upe 2011) Ao se planificar um cone reto, sua superfcie


lateral igual a um quarto de um crculo com rea igual a 12
. Nessas condies, a rea de sua base igual a
a)
b) 2
c) 3
d) 4
e) 5
RESPOSTA: C
COMENTRIO: Considerando 12 como sendo a rea da
superfcie lateral, r o raio da base e g sua geratriz temos:

.r.g 12 r.g 12
2 .r

g 4.r
g
2
Logo, r.4r. 12 r 3 .

Como a rea do tringulo do tringulo ABC S, segue que

(ABC)
2

Portanto, a rea da base ser A =

. 3 3 .

r
2S
Sr
.
2

Portanto, o volume pedido dado por

1
1
1
r 2 x r 2 ( x) r 2 (x x)
3
3
3
1
r 2
3

554. (Insper 2011) Considere o slido gerado pela rotao


completa do tringulo acutngulo ABC de rea S em torno de
um eixo que passa pelo lado BC que tem comprimento .

O volume desse slido igual a


a)

4 S 2
.
3

1
2S

4S2
.
3

555. (Uece 2010) A superfcie lateral de um cone circular reto,


quando planificada, torna-se um setor circular de 12 cm de
raio com um ngulo central de 120 graus. A medida, em
centmetros quadrados, da rea da base deste cone
a) 144 .
b) 72 .
c) 36 .
d) 16 .

2 S 2
.
3
4 S
.
c)
3
2 S
.
d)
3
S
.
e)
3

RESPOSTA: D
COMENTRIO:

b)

RESPOSTA: A
COMENTRIO: Uma rotao completa do tringulo ABC em

2 .R

torno da reta suporte do lado BC gera o slido abaixo,


constitudo de dois cones.

2 .12
3

R4
A .4 2
a 16cm 2
556. (Enem 2 aplicao 2010) Um arquiteto est fazendo um
projeto de iluminao de ambiente e necessita saber a altura
que dever instalar a luminria ilustrada na figura
159

d)

e)
Sabendo-se que a luminria dever iluminar uma rea circular
2
de 28,26m , considerando 3,14, a altura h ser igual a
a) 3 m.
b) 4 m.
c) 5 m.
d) 9 m.
e) 16 m.

RESPOSTA: B
COMENTRIO:

252 7

180
5
7
2 .R
.10 R 7 e g = 10 (raio do setor)
5
o

252 =

RESPOSTA: B
2

COMENTRIO: Se a rea a ser iluminada mede 28,26 m e r

o
raio
da
rea circular
iluminada,
ento

28,26
r 3 m.
3,14
Portanto, como g 5 m e r 3 m, segue que h 4 m.
r 2 28,26 r

557. (Fgv 2010) A figura indica a planificao da lateral de um cone


circular reto:

558. (G1 - cftsc 2010) Considere as sentenas abaixo:

3 7
- 13.
1 2

I O determinante da matriz A

II As retas r: 2x + y = 4 e s: 2y + 4x = 9 so paralelas.
III Um copo de sorvete possui forma cnica com raio da base
medindo 3 cm e altura 7 cm. Sabendo que a frmula para
calcular o volume desse copo dada por V

O cone a que se refere tal planificao

1 2
r h , ento
3

seu volume ser de 14 cm.


Considerando as proposies apresentadas, assinale a
alternativa correta:
a) Apenas as proposies I e III so verdadeiras.
b) Apenas as proposies II e III so verdadeiras.
c) Apenas as proposies I e II so verdadeiras.
d) Apenas a proposio II verdadeira.
e) Apenas a proposio III verdadeira.

a)

RESPOSTA: C
COMENTRIO: I (verdadeira), DET(A) = 3.(-2) 1.7 = -13
b)

2
2
1
II (verdadeira)
logo r e s so paralelas
4
ms
2
2
mr

c)

III (falsa) V =

160

1
.3 2.7 21
3

559. (Ufpr 2010) Com base nos estudos de geometria, identifique


as afirmativas a seguir como verdadeiras (V) ou falsas (F).

RESPOSTA: A
COMENTRIO: Considere a figura abaixo.

(
) Dois ngulos so opostos pelo vrtice se, e somente se,
os lados de um deles so as respectivas semirretas opostas aos
lados do outro.
(

) A razo entre dois ngulos suplementares igual a

2
.O
7

complemento do menor vale 140 graus.


( ) A hipotenusa de um tringulo retngulo issceles que gira
em torno de um dos catetos, gerando um slido cujo volume

cm3 , 2 cm. , 2 cm.

( ) Se trs retas so, duas a duas, reversas e no paralelas a


um mesmo plano, ento, por qualquer ponto de uma das
retas, passa uma reta que se apoia nas outras duas.
(
) Se um polgono regular possui, a partir de um dos seus
vrtices, tantas diagonais quantas so as diagonais de um
hexgono, ento esse polgono um dodecgono.

Sabendo que a rea do setor circular VAB 3 cm , segue


que
2

VA 120
3 VA 3cm.
360

Assinale a alternativa que apresenta a sequncia correta, de


cima para baixo.
a) V - F - V - F - V.
b) F - V - F - V - F.
c) F - V - V - F - V.
d) V - V - V - V - V.
e) V - F - F - F - F.

O comprimento do arco AB dado por

VA 2 3 2 cm.
AVB
3
Desse modo, como o comprimento do arco AB corresponde
ao comprimento da base cone, obtemos

RESPOSTA: A
COMENTRIO: (verdadeiro) definio de ngulos opostos pelo
vrtice.
o
o
(falsa) 2x + 7x = 180 x = 20 e 2x = 40 . O complemento de
o
O
40 (menor) 50
(Verdadeiro)

2 r 2 r 1cm,
em que r o raio da base do cone.
Portanto, a rea total do cone

3 r 2 3 12 4 cm2 .
Como VA a geratriz do cone, temos que
2

h2 VA r 2 h2 32 12 h 2 2 cm,
V=

112.1

3
3

sendo h a altura do cone.


Por conseguinte, temos que o volume desse cone mede

(falso) definio de retas reversas.


(verdadeiro) d =

6.(6 3)
9
2
n 3 9 n 12

560. (Ita 2012) A superfcie lateral de um cone circular reto um


2
setor circular de 120 e rea igual a 3 cm . A rea total e o
2
3
volume deste cone medem, em cm e cm , respectivamente
a) 4 e

2 2
3

b) 4 e

2
3

1
1
2 2
r 2 h 12 2 2
cm3 .
3
3
3
561. (Uff 2011) Para ser aprovada pela FIFA, uma bola de futebol
deve passar por vrios testes. Um deles visa garantir a
esfericidade da bola: o seu dimetro medido em dezesseis
pontos diferentes e, ento, a mdia aritmtica desses valores
calculada. Para passar nesse teste, a variao de cada uma
das dezesseis medidas do dimetro da bola com relao
mdia deve ser no mximo 1,5%. Nesse teste, as variaes
medidas na Jabulani, bola oficial da Copa do Mundo de 2010,
no ultrapassaram 1%.

c) 4 e 2
d) 3 e

2 2
3

e) e 2 2
161

Se o dimetro de uma bola tem aumento de 1%, ento o seu


volume aumenta x %.
Dessa forma, correto afirmar que
a) x [5,6).
b) x [2,3).
c) x = 1.
d) x [3,4).
e) x [4,5).

563. (Espm 2011) Um reservatrio de gua constitudo por uma


esfera metlica oca de 4 m de dimetro, sustentada por
colunas metlicas inclinadas de 60 com o plano horizontal e
soldadas esfera ao longo do seu crculo equatorial, como
mostra o esquema abaixo.

RESPOSTA: D
COMENTRIO: O volume (V) de uma esfera, em funo do
seu dimetro (D), dado por

3
D .
6

Se o dimetro tem aumento de 1%, ento o volume dessa


esfera passa a valer

V ' (1,01 D)3 1,030301 D3 1,030301 V.


6
6

Sendo 3 1,73 , a altura h da esfera em relao ao solo


aproximadamente igual a:
a) 2,40 m
b) 2,80 m
c) 3,20 m
d) 3,40 m
e) 3,60 m

Portanto,
x%

1,030301 V V
0,030301 V
100%
3,03% [3, 4).
V
V

RESPOSTA: C
COMENTRIO: Considere a figura abaixo.

562. (Fuvest 2011) A esfera , de centro O e raio r > 0, tangente


ao plano . O plano paralelo a e contm O. Nessas
condies, o volume da pirmide que tem como base um
hexgono regular inscrito na interseco de com e, como
vrtice, um ponto em , igual a

3r 3
4
5 3r 3
b)
16
a)

c)

3 3r 3
8

Queremos calcular h PO' OO' OP.

7 3r 3
d)
16
e)

3r
2

Temos

que

O' A

AD 10

5m
2
2

4
2 m O'C.
2
Logo, AC O' A O'C 5 2 3 m.
OB

RESPOSTA: E
COMENTRIO:

Do

tringulo

ABC,

vem

que

BC BC 3 tg60 3 3 3 1,73 5,19 m.


tgBAC
AC
Portanto, h 5,19 2 3,19 3,20 m.

V=

1
A b .h
3

V=

1 6.r 3 3
.
.r
3
4

V=

r3 3
2

564. (Ufsm 2011) Um fabricante decidiu produzir luminrias no


formato de uma semiesfera com raio de 20 cm. A parte
interior, onde ser alojada a lmpada, receber uma pintura
metalizada que custa R$ 40,00 o metro quadrado; j a parte
externa da luminria receber uma pintura convencial que
custa R$ 10,00 o metro quadrado. Desconsiderando a
espessura da luminria e adotando o valor de = 3,14 o custo,
em reais, da pintura de cada luminria
a) 3,14.
b) 6,28.
c) 12,56.
d) 18,84.
e) 25,12.
162

RESPOSTA: C
COMENTRIO:

O dimetro da base e a altura do cilindro medem, cada um,


4dm, e o volume de uma esfera de raio r

rea de cada uma das partes (interna e externa):

A 2.3,14.(0,2)2 0,2512

4 3
r .
3

Dentre as opes a seguir, o valor mais prximo da capacidade


do reservatrio, em litros,
a) 50.
b) 60.
c) 70.
d) 80.
e) 90.

Logo, o valor total ser:


0,2512( 40 + 10 ) = R$ 12,56.
565. (Pucsp 2011) Um arteso dispe de um bloco macio de
resina, com a forma de um paraleleppedo retngulo de base
quadrada e cuja altura mede 20 cm. Ele pretende usar toda a
resina desse bloco para confeccionar contas esfricas que
sero usadas na montagem de 180 colares. Se cada conta tiver
um 1 cm de dimetro e na montagem de cada colar forem
usadas 50 contas, ento, considerando o volume do cordo
utilizado desprezvel e a aproximao = 3, a rea total da
superfcie do bloco de resina, em centmetros quadrados
a) 1250.
b) 1480.
c) 1650.
d) 1720.
e) 1850.

RESPOSTA: D
COMENTRIO: V= V(cilindro) + V(esfera)

4
V .2 2.4 .2 3
3
32
V 16
3
V

80
(fazendo = 3)
3
3

V= 80dm = 80L

RESPOSTA: C
COMENTRIO:

Nmero de esferas = 180 50 9000


3

Volume total das esferas = 9000

4
1
4500cm3
3
2

(considerando 3 )
Volume do bloco = x x 20
Logo,

20x 2 4500
x 2 225
x 15cm

567. (Ufpr 2010) Para testar a eficincia de um tratamento contra


o cncer, foi selecionado um paciente que possua um tumor
de formato esfrico, com raio de 3 cm. Aps o incio do
tratamento, constatou-se, atravs de tomografias, que o raio
desse tumor diminuiu a uma taxa de 2 mm por ms. Caso essa
taxa de reduo se mantenha, qual dos valores abaixo se
aproxima mais do percentual do volume do tumor original que
restar aps 5 meses de tratamento?
a) 29,6%
b) 30,0%
c) 30,4%
d) 30,8%
e) 31,4%

566. (Ufrgs 2010) Um reservatrio tem forma de um cilindro


circular reto com duas semiesferas acopladas em suas
extremidades, conforme representado na figura a seguir.

RESPOSTA: A
COMENTRIO: Seja R raio do tumor e x o nmero de meses.
Logo R(x) = 3 0,2x, aps 5 meses o raio ser:
R(5) = 3 0,2 . 5 = 2 cm

163

Volume inicial =
Volume final =

4 .3 3
36
3

c)

17
.
2

d) 9.

4 .2 3 32

3
3

e)

32
3 29,6%
P=
36

19
.
2

RESPOSTA: E
COMENTRIO: Considere a figura abaixo.

568. (Ufpa 2011) Uma rasa um paneiro utilizado na venda de


frutos de aa. Um tpico exemplar tem forma de um tronco de
cone, com dimetro de base 28cm, dimetro de boca 34cm e
altura 27cm. Podemos afirmar, utilizando = 3,14, que a
capacidade da rasa, em litros, aproximadamente
a) 18
b) 20
c) 22
d) 24
e) 26
RESPOSTA: B
COMENTRIO: O raio da base mede r
raio de boca R

28
14cm e o
2

34
17cm.
2

Como as pirmides VPGQ e VABC so semelhantes,

Portanto, como a altura do paneiro mede h 27cm, segue


que a capacidade da rasa dada por

temos que

3,14
h (R2 R r r 2 )
27 (172 17 14 142 )
3
3
3,14 9 723

sendo k a razo de semelhana.


Desse

VG
VB

6 2
k,
9 3
modo,

[VPGQ] 2
8
[VPGQ]
[VABC].
[VABC] 3
27

20.431,98cm3
20 L.

O volume da pirmide VABC dado por

1 AB BC

VB
3
2
1 33

9
3 2
27

.
2

569. (Ufrgs 2011) Na figura abaixo, esto representados um cubo


de aresta 3 e uma pirmide triangular de altura 9. Os pontos A,
B e C so vrtices da pirmide e do cubo, e V pertence ao
prolongamento de BG.

[VABC]

Portanto, o volume pedido

[VABC] [VPGQ] [VABC]

8
[VABC]
27

19
[VABC]
27
19 27

27 2
19

.
2

O volume comum aos dois slidos


a)

570. (Espcex (Aman) 2011)


A figura abaixo representa a
planificao de um tronco de cone reto com a indicao das
medidas dos raios das circunferncias das bases e da geratriz.
A medida da altura desse tronco de cone

15
.
2

b) 8.

164

1
2
3
b)
4
5
c)
6
7
d)
8
a)

RESPOSTA: D
COMENTRIO: Seja g uma geratriz do cone emerso e G uma
a) 13 cm
b) 12 cm
c) 11 cm
d) 10 cm
e) 9 cm

geratriz do slido. Segue que

RESPOSTA: B
COMENTRIO: Considere a figura abaixo.

Assim, se v o volume emerso e V o volume do slido,


temos

g 1
k,
G 2
com k sendo a constante de proporcionalidade.

v
v 1
1
V
k3 v .
V
V 2
8
8
Seja Vs o volume submerso.

Vs V v V

V 7V

.
8
8

Portanto, a razo pedida

Sabemos

que

OP 6cm,

O'Q 11cm

7V
Vs
7
8 .
V
V
8
e

PQ 13cm.
Logo, como OP O'P', segue que
P'Q O'Q O'P' 11 6 5cm.

572. (Udesc 2011) Considere um tronco de pirmide regular, cujas


bases so quadrados com lados medindo 4cm e 1cm Se o
3
volume deste tronco 35cm ento a altura da pirmide que
deu origem ao tronco :
a) 5cm

5
cm
3
20
c)
cm
3
b)

Aplicando o Teorema de Pitgoras no tringulo P'PQ,


encontramos
2

PP' PQ P'Q PP' 132 52 12cm,

d) 20cm
e) 30cm

que a altura procurada.


RESPOSTA: C
COMENTRIO: Considere a figura.

571. (Uerj 2011) Um slido com a forma de um cone circular reto,


constitudo de material homogneo, flutua em um lquido,
conforme a ilustrao abaixo.

Se todas as geratrizes desse slido forem divididas ao meio


pelo nvel do lquido, a razo entre o volume submerso e o
volume do slido ser igual a:
Como a pirmide menor e a maior so semelhantes, vem que
165

e) V

v h
1
1

,
V H
64
4

RESPOSTA: E
COMENTRIO:

sendo v o volume da pirmide menor e V o volume da


pirmide que deu origem ao tronco.

V(EFGHI) b

p
a

Alm disso, como o volume do tronco 35cm , temos

V v 35 V

V
320 3
35 V
cm .
64
9

573. (Pucrs 2010) O metrnomo um relgio que mede o tempo


musical (andamento). O metrnomo mecnico consiste num
pndulo oscilante, com a base fixada em uma caixa com a
forma aproximada de um tronco de pirmide, como mostra a
foto.

p
V(EFGHI) b
V(EFGHI) =
64
p
4b
p
63
p
Logo V = p
=
64 64

Portanto,

1 2
320
20
4 H
H
cm.
3
9
3

63
p
64

574. (Uerj 2010) A figura abaixo representa um recipiente cnico


com soluo aquosa de hipoclorito de sdio a 27%. O nvel
desse lquido tem 12 cm de altura.

Para o preparo de um desinfetante, diluiu-se a soluo inicial


com gua, at completar o recipiente, obtendo-se a soluo
aquosa do hipoclorito de sdio a 8%.
Esse recipiente tem altura H, em centmetros, equivalente a
a) 16
b) 18
c) 20
d) 22

Na representao a seguir, a o lado da base maior, b o lado


da base menor e V o volume do tronco de pirmide
ABCDEFGH. Se a = 4b e p o volume total da pirmide ABCDI,
ento:

RESPOSTA: B
COMENTRIO: A soluo inicial ocupa um volume igual a

1 2
r 12 cm3 , em que r o raio do cone menor definido
3
pelo nvel do lquido. O recipiente tem volume igual a

1 2
R H cm3 , em que R o raio do recipiente e H a
3
sua altura.
Como os cones so semelhantes, segue que:

r 12
12R

r
.
R H
H
Por outro lado, do enunciado vem:
2

1
1
12R
2
27% r 2 12 8% R2 H 27
12 8 R H
3
3
H

3
p
4
3
p
b) V
16
15
p
c) V
16
15
p
d) V
64
a) V

H3

33 123

23
3 12
H
2
H 18cm.

166

575. (Ita 2012) Um cone circular reto de altura 1 cm e geratriz

2 3
interceptado por um plano paralelo sua base, sendo
3
determinado, assim, um novo cone. Para que este novo cone

3
3
AO'
v AO'
1
243


AO' cm.

V AO
3
1
9

13

243

tenha o mesmo volume de um cubo de aresta

cm,

necessrio que a distncia do plano base do cone original


seja, em cm, igual a
a)
b)
c)
d)
e)

Portanto, o resultado pedido

O'O AO AO' 1

1
4
1
3
1
2
2
3
3
4

RESPOSTA: D
COMENTRIO: Seja v o volume do cone determinado pelo
plano.
Sabendo que o volume desse cone igual ao volume do cubo
13

243

de aresta

cm, obtemos
3

1 3

v
cm3 .

243
243

Considere a figura abaixo.

Como AO 1cm e AB

2 3
cm, do Teorema de
3

Pitgoras aplicado no tringulo AOB segue que


3

2 3
4
1
2
OB
1 1 .
3
3
3
2

O volume do cone maior dado por

2
1
1
1

OB AO 1 cm3 .
3
3
3
9

Da, como os cones so semelhantes, vem


167

1 2
cm.
3 3

S-ar putea să vă placă și